You are on page 1of 375

Maths Extension 2

Zndedltion
by

S.K. Patel
------J

L~, ~~-et\1'\'c~
X\"
...
-.- --~-- -,

~,..,

;..,
')

Copyright S.K. Patel and Pascal Press


First Edition published by Pascal Press April 1989
Revised Edition published October 1989
Second Edition published January 1991
Reprinted 200 I, 2003, 2004

ISBN I 87531 226 9

Pascal Press
PO Box 250
Glebe NSW 2037
(02) 8585 4044 '

., ' .

www.pascalpress.com.~u

Printed in Singapore by SNP SPrint Pte Ltd

COPYING OF THIS BOOK BY EDUCATIONAL INSTITUTIONS


A purchasing educational institution may only photocopy pages within this book in accordance with
the Australian Copyright Act 1968 (the Act) and provided the educational institution (or body that
administers it) has given a remuneration notice to the Copyright Agency Limited (CAL) under the Act.

It is mandatory that ALL photocopies are recorded by the institution for CAL survey purposes.
For details of the CAL licence for educational institutions, contact:
Copyright Agency Limited
Level 19, 157 Liverpool St
Sydney, NSW, 2000
Telephone: (02) 9394 7600
Email: info@copyright.com.au
COPYING BY INDIVIDUALS OR NON-EDUCATIONAL INSTITUTIONS
Except as permitted under the Act (for example for fair dealing for the purposes of study, research,
criticism or review) no part of this book may be reproduced, stored in a retrieval system, or transmitted in any form by any means, without the prior written approval of the publisher. All enquiries should
be made to the publisher at the address above.

PREFACE
As always, my aim in this book has been to impart the knowledge of mathematics
to students who will need it in their later academic and professional work.
I would like to thank and acknowledge the help and contribution of the following
individuals:
- Roger Myers of the Banks town Technical College for his many thoughtful
suggestions
- Judy Faulkner who typed the manuscript in the required time
- all my colleagues in the Technical Colleges and many other school teachers
for their useful suggestions.
I sincerely hope that this new edition will be found to be even more useful than the
previous one.
Suresh Patel

./

CONTENTS
CHAPTER 1

CURVE SKETCHING
1.1
1.2
1.3
1.4
1.5
1.6
1.7
1.8
1.9
1.10

CHAPTER 2

Basic Curves
Method for Sketching the Curves
Graphs of Basic Functions
Curves with Vertical and Horizontal Asymptotes
Trigonometric Graphs
Exponential Curves
Drawing Graphs by Composition of Ordinates
Implicit Differentiation and Sketching Curves
Applications
Miscellaneous Curves Sketching
Exercise: l:A
Exercise: l:B

1
2
3
8
11
14
15
24
26
30
40
43

INTEGRATION
2.1 Standard Integrals
2.2 Change of Variable: Substitution
Exercise 2A
2.3 Integration by Part
Exercise 2B
2.4 Trigonometric Integrals
2.5 Use of t = tan (X/2)
Exercise 2C
2.6 Reduction Formulas
Exercise 20
2.7 Trigonometric Substitution
Exercise 2E
2.8 Integration of Rational Functions
Exercise 2F
2.9 Method of Partial Fractions
Exercise 2G
2.10 Completing the Square (Integration)
Exercise 2H
2.11 Integration: Special Properties
Exercise 2I
Exercise 2J

49
51
53
54
57
58
61
63
64

66
67
68
68
72

73
77
78
80
81
85
86

CHAPTER 3

VOLUMES
3.1 Formulas for Volumes
Exercise 3A
3.2 Volumes: Shell Method
3.3 Volumes: Washer Method
Exercise 3B
3.4 Worked Examples (Miscellaneous)
Exercise 3C

CHAPTER 4

88
97
99
103

104
105
109

COMPLEX NUMBERS
4.1 Introduction
4.2 Operations with Complex Numbers
Exercise 4A
4.3 Complex Plane (Argand Diagram)
Exercise 4B
4.4 Multiplication and Division of Complex Numbers
Using Trigonometry
Exercise 4C
4.5 Powers of Complex Numbers: De Moivre's Theorem
Exercise 4D
4.6 Roots of Complex Numbers
Exercise 4E
4.7 De Moivre's Theorem and its Applications
Exercise 4F
4.8 Square Roots of a Complex Number
Exercise 4G
4.9 Properties of Conjugate Complex Numbers
Exercise 4H
4.10 The Complex Roots of Unity
Exercise 41
4.11 Miscellaneous: Factorisation over the Complex Field
Exercise 4J
4.12 Geometric Representation of Complex Numbers
Exercise 4K
4.13 Product and Quotient: Rotation
.Exercise 4L
4.14 Locus Problems with the Complex Variable z
Exercise 4M
4.15 Miscellaneous Locus Problems (Including Inequations)
Exercise 4N
Exercise 40

112
113
116
118
121
122
124
125
126
127
130
131
134
135
136
137
138
139
141
143
145
146
149
150
152
153
154
155
159
160

CHAPTER 5

POLYNOMIALS
5.1 Introduction
Exercise SA
5.2 Zeros of a Polynomial/Multiple Roots
Exercise SB
5.3 Polynomial over the Complex Field
Exercise SC
5.4 Relation between Roots and Coefficients of P(x) = 0
Exercise 50
5.5 Miscellaneous (Worked Examples)
Exercise SE

CHAPTER 6

CONIC SECTIONS
6.1 Introduction
6.2 Ellipse (e < 1): (Focus and Directrix Definition)
Exercise 6A
6.3 The Hyperbola
Exercise 6B
6.4 Shape of the Conics
6.5 ParaiJ~.etric Equations of the Circle and the Ellipse
6.6 Equations of chord, Tangent and Normal to the Ellipse
6. 7 Parametric Equations of the Hyperbola
6.8 A Special (Rectangular) Hyperbola xy = c2
Exercise 6C
6.9 Miscellaneous Problems on Conics
Exercise 60
6.10 Tangents and the Chord of Contact
6.11 Geometric properties of the Ellipse
6.12 Geometric Properties of the Hyperbola
6.13 Properties of the Rectangular Hyperbola
6.14 Geometric Properties of Rectangular Hyperbola xy = c2
Exercise 6E

CHAPTER 7

164
167
168
172
173
177
178
183
185
187

191
192
195
196
198
199
201
203
205
206
208
209
214
222
224
227
228
229
232

ELEMENTARY PARTICLE DYNAMICS


7.1 Introduction
7.2 Laws of Motion - Force
Exercise 7A

234
234
239

CHAPTER 8

MOTION PROBLEMS IN TWO DIMENSIONS


8.1 Introduction
8.2 Simple Harmonic Motion (Revision)
Exercise 8A
8.3 Motion of a Projectile (Revision)
Exercise 8B
8.4 Resisted Motion: Other Laws of Motion
Exercise 8C

CHAPTER 9

240
241
244
246
249
252
259

CIRCULAR MOTION
9.1 Introduction
9.2 Angular Velocity: Period
9.3 Circular Motion: Tangential Velocity
Exercise 9A
9.4 Acceleration of a Particle Rotating in a Circle
9.5 Uniform Circular Motion
Exercise 9B
9.6 Conical Pendulum
Exercise 9C
9.7 Banked Tracks
Exercise 9D
9.8 Components of Acceleration (Variable Angular Velocity
Exercise 9E
9.9 Miscellaneous (Worked Examples on Circular Motion)
Exercise 9F

265
266
267
269
270
271
272
273
275
276
278
279
281
282
285

CHAPTER 10 HARDER 3 UNIT TOPICS


10.1 Harder Trigonometry
Exercise lOA
10.2 3 Unit Co-ordinate Geometry: Circles (Harder Problems)
Exercise lOB
10.3 Plane Geometry: Circles (Harder Problems)
Exercise lOC
10.4 Inequalities
Exercise lOD
10.5 Method of Mathematical Induction
Exercise lOE
10.6 Properties of the Integrals
Exercise lOP
Appendix 1
Appendix 2

Answers

288
291
292
2%
297
300
305
307
308
310
312
315
316
317
318

Chapter 1: Curve Sketching


INTRODUCTION
A structure, no matter how complicated, is composed of basic building materials. In
mathematics, the study of complicated functions such as f(x) = -1 (ex + e-x), log x e x
2
sin-l ex, etc. can be facilitated by sketching the graphs of these functions. But how
can we best accomplish this? Firstly,by identifying them by the basic curves (listed
below) and then using a number of systematic steps to produce reasonably good
graphs. The objective is to draw a quick and neat sketch of the curve showing all
the essential features (see section 1.2). Any thought of drawing the graph by brute
force i.e. plotting scores of points must be abandoned, as it would most probably miss
the essential features, such as asymptotes and the critical points.

1.1

Basic Curves

The following basic curves are well known to students. (See Chapter 1 for the graphs
of the basic functions.)
Function
1.

Linear: ax + by + c
y = mx + b

2.

Quadratic: y

+ bx + c

= ax

Cubic: y

4.

Quartic: y = ax

6.

+ bx
4

.. bx

k
Y=x,

x + y
(x - a)

2
2

A straight line

= ax 2

3.

5.

=0

+ ex + d
3

+ ex

A cubic curve

+ dx + e

= r

=0

or

7.

Exponential: y = ax , ex

8.

Logarithmic: y

9.

Trigonometric: y = sinx, cosx, tanx

10.

Inverse Tng: y

11.

Type: y

= x 1/n

An exponential curve

= loga x ,
.-1
= sm
x,

, n

A quartic curve
A rectangular hyperbola

xiO

+ 2gx + 2fy + c

+ (y - b)

A parabola

= 2,

loge x

-1

A Iogarithmrc curve

-1

cos x, tan x
3

A sine curve, cosine


curve, tangent curve
An inverse sine curve etc.

1.2

Method for Sketching the Curves

A sketch of the curve shou1d convey the general shape of the curve, showing the
following information.
1.

Intercepts on the axes


x

2.

=0

gives the y-intercept, y

=0

gives the x-intercept.

Symmetry
The curve is symmetric about the
(a) y-axis if f(x) = f(-x)
(b) origin 0 if f(-x)

3.

= -f(x)

Asymptotes
If y = f(x) is a rational function of the form f(x) =
_
(a) vertical asymptotes are g1ven by h(x) = 0

~~x~
x

, where g(x) and h(x) are


polynomials, then the:

(b) horizontal asymptotes are given by ;i: I co I f(x) if it exists, say y

= c.

2
Example: The vertical asymptotes of the curve y = 2X + 1 are given by
2
X - 1
2
x - 1 = 0 , i.e. x = .! 1 and the horizontal asymptotes are given by
2
Lim 2/ + 1 = 2 and y = Lim 2x + 1 _
.
_
Y =X +co. 2
x~-oo
2
- 2 'I.e. Y- 2
X
- 1
X
- I
4.

Critical points

=0

(a) Relative maxima:

f' (x)

(b) Relative minima:

f' (x) = 0 , f"(x)

(c) Vertical tangent at x

=a

, f"(x)

<0
>0

if f' (a) is undefined

(d) Point of inflection at x = a, if f"(a) = 0 and f"(x) changes sign as x


increases through x = a.

5.

Rising and falling curves


(a) f' (x)
(b) f' (x)

6.

7.

>0
<0

for the increasing function (rising curve)


for the decreasing function (falling curve)

Concavity
(a) f"(x)

<0

indicates where the grapl:l is concave down

(b) f"(x)

>0

indicates where the graph is concave up

Zones (regions) of exclusion:


The domain in which the graph of y = f (x) does not exist (i.e. y is undefmed) is
an important consideration, because we don't have to worry about the graph in
this interval.
~
Example:
y = f(x) = V 9 - /
The graph of this function is real if -3 ~ x ~ 3. It does not exist for lx I
The curve IS the upper half of the circle x 2 + / = '3.

> 3.

j.

1.3
1.

Graphs of Basic Functions


The graph of a linear function y

= mx

a straight line, with gradient m and


y-intercept b.
x = b represents a vertical line.

js

y =c

2.

represents a horizontal line.

The graph of a quadratic function

= ax 2

. a para bo Ia.
1s
Its vertex lies on the axis of symmetry

x = -

+ bx + c

2a

If a

>

If a

< 0,

0, y has a minimum at x
.

b
=-2a

y h as a max1mum at x = -

b
2a

Discriminant /J. = b 2 - 4ac .


If A

<

the curve does not intersect

0 ,

the x-axis,
If

/J.

If IJ.

0,

>

the curve touches the x-axis.

O, the curve intersects the x-axis


b

at x =- 2a

Examples:
(a)

(c)

Sketch the curves:

= x 2 - 4x + 3
y = x2 + x + 1
y

/:;

2a

(b)

y = -x 2 + 2x

tanS

4
Solution:

y =

(a)

>

- ~X

0, A = 16 - 12

>

The intersection with x-axis is given by y = 0


(x - 3) (x - 1) = 0

:.

A(l, 0), B(3, 0)

When x

0 ,

v (2,

=; 3

Vertex at x = 2 , y = -1
(b)

y = -x

a<

-1)

y
V(l,l)

+ 2x

y = 0 gives x (x - 2) = 0
0 (0, 0) and

A (2, 0)

Vertex (1, 1)
(c)

= X2

a>

+ X + 1
A=l-~<

0,

No intersection with x-axis


Vertex (- l

3
2,4 )

Some points on the graph


A(l, 3),

B(O, 1), C(-1, 1)

The equation of the form x = ay

+ by + c

represents a parabola with its axis horizontal.


Vertex on ax1s of symmetry dt y = For a

>

0 and a

< 0,

b
2a

the shapes

are as shown.
If A

<

0, the graph does not intersect


X

y-axis.

Example:
Sketch the curve
Solution:
a = l > 0.
Vertex:
y =

= y2

y
+ 2y

-2a

x = 0 gives y (y + 2) = 0
(0, 0)

and

(0, -2)

-22

= -1, V(-1, -1)

i.e.

-1

GRAPHS OF POLYNOMIAL FUNCTIONS

3.

For sketching the graph of the polynomial function we need reasonably


sufficient information, such as:
The function y = f(x) is an even function if f(x)

a.

= f{-x).

The graph of an even function is symmetric about Y-:axis.


b.

= f(x)

The function y

is odd if f{-x)

= -f(-x)

and the graph is

symmetric about the origin.


The intersection of the curve y = f(x) with

c.

the x-axis is given by y = 0 This is not always simple, but worth


trying.

- the y-axis is given by x = 0.


d.

The behaviour of the curve as x -

e.

The nature of the turning points, i.e. a maximum or a

co

minimum
Concavity and points of inflexion

f.

i.e. f" (x)

=0

Example: (I)

Sketch the curves y = x

and y = -x

3
y=x

Solution:

f (x) = x 3 and f (-x) = -x 3


The graph of y = x

is symmetric about the origin

0 (0, 0) is a point on the curve


As x X

~
dx

co,

_,.-oo,

y + co
y - - co

3x 2

But x = 0 is not a turning point, since

~ does not change its sign in passing

through 0
Some points: A (1, 1) , B (2, 8) This is reasonably sufficient information to sketch
the curve.

6
Example: (2)
y = (x

-t

Sketch the curve:

2) (x - 1) (x_- 3)

y = 0 gives A(-2, 0), B(l, 0), C(3, 0)


= 0 gives D(O, 6)
---,....-1:----~-~lc-----:::---:Jf:::-~x

l'

y -+ ~ as x-+ ..,
y--a.as x-+-~
Other points x
X

Example: (3)

y = (x +

= 2,
= -1

y
1

= -4

y :: 8

Sketch the curve:

2) (x - 2)

y = 0 gives A(-2, 0), B(2, 0)


The double zero x = 2 indicates
that the x-axis touches the
curve at x = 2
~2

Y-i'.:!:.a.asx~;t""

x "'

gives y = 8

Other points;
C(-1, 9)1 D(l, 3), E(J, 5)

Example: (4)

y = (x

-t 2) (x

Sketch the curve:


+ 1) (x - 2) (x - 3)

The sketching is similar to that


of Example 2, noting thdt y ....
dS

X~-

oo

oo

X=O,y~l2

(I, I 2) is another point

7
4.

= x 1/2 dnd y =x 1/3


Draw a sketch of y = .JX

Graphs of y

EXAMPLE: (1)

by analysing the
behaviour of the function near x =0

SOLUTION:
The function
Also f(x)

f(x) =

,jx

exists and is continuous for

~ 0 for x ~ 0 and f'(x) = -~


2

for x

>0

V X

The point x = 0 is a critical point as f' (0) does not exist.


The curve y = {x has a vertical tangent at x = 0

Since f' (x) > 0 for x > 0, f(x) is an increasing function


and f(O) = 0 is the absolute minimum, but f(x) has no
absolute maximum.
I -3/2
It may be noted that since f" (x) = - ii x
< 0 for x > 0,
the curve is concave down.
This is sufficient information for drawing a reasonable graph of y
Note: y

vx

is the upper half of the parabola /

Graph of y

= .(X.

= x.

EXAMPLE: (2)
Draw a sketch of y = x 1/3
SOLUTION:
Y =x

f (x)

1/3

' dx

= x l/ 3

f(-x) = (-x)

I -2/3

=3 x

fr

' dx2 =

exists for all real x

1/3 .

=-x

1/3

f' (x) does not exist for


tangent, at x = 0.

= - f(x),

so the curve has point symmetry about the origin.

x = 0, so the curve has a vertical

Also f' (x) > 0 for all x, x I 0, so it is an increasing


curve. x = 0 is a critical point.
f (x) ~

oo

f (x) -t -

oo

as x

-t oo

as x __,. -

oo

f"(x) > 0 for x > 0 and f"(x) < 0 for


x < 0, so the origin is an inflection point.
Also f"(x) > 0 for x > 0
tells us that the curve is concave down,
while f"(x) < 0 tells us that the curve
is concave up for x < 0. A few simple
points are 0, A(l,l), B(-I,-1).

-J

= ,fX

1.4

Curves with Vertical and Horizontal Asymptotes

Example: (1)
2

Sketch the curve y =

y has same sign as x, hence curve in first and


third quadrant
As

As x

-IX>'
- -co,

y -

a+ (from above)

y -+ 0- (from below)

The x-axis and y-axis are the asymptotes of the

curve.
The asymptotes are very useful in sketching the
curves. This curve is a rectangular hyperbola rectangular because the asymptotes are perpendicular.

Example: (2)
I

Sketch the curve y = x + x


1

y=x+x'

xl- 0

Near x = 0 , the term

x.

dominates, i.e. x is negligible


I

compared to x
:. The curve y = x + .! behaves
X

like hyperbola y = .! near x = 0


X

As x-+ "" y - x i.e. the


curve approaches the line y = x
The graph is symmetric about
the origin as f( -x) = -f{x)
The turning points are:

~ dx -

1
---:z
X

p (1, 2)

Q(-1,-2)

The curve does not intersect either axis


Other points: (2, 2.5) , (-2, -2.5)
This is reasonable enough information to sketch this curve

Example: (3)

2
--2X-

Sketch the curve y

-------"-0-+--i'i

Solution:
2
y = -2-

X-

\ I

The function is undefined at x = 0, i.e. y-axis is


the asymptote
X '"'

2 ' y = 0 A (2, 0)

> 2,

When x

>

0 ,

< 2,

<

CRITICAL
POINTS

Near x = 0 , the graph behaves more like


As x x -+

~
dx

y -+ 0 + from above

oo ,
-oo,

y -+ 0

=
P (4,

from below
0 gives x

2>

=4

is a maximum turning point.

The first sketch shows the important features, the


second shows a complete graph
5

Other points: (:-1, -3), (-2, -1}, (-3, -9), (1, -0


y

Example: (4)
X

Y = (x - 2) (x + 2)
Solution:

The curve is undefined at x = + 2


x = 0, y =

o,

0(0, 0) is a point on

the curve.
The Jines x = 2 and x = -2
are the asymptotes.
As x-+ "" y - I+ from above
x -

- oo

I+ from above

y -

This is so because the numerator


is greater than the denominator
when x is large.
For - 2

<x <2

<

0 and !h.E'

curve is asymptotic to x =

10

It would be instructive for you to study the sketches of the following curves and
reasoning out the shape and critical behaviour of each of them.

Example: (5)
X

y = (x + 2) (x -

What are the asymptotes?


Why is the RIGHT (TOP) branch

I
-2'

below?
Why is the

LEFT (TOP)

branch -

y = 1 from

.:-

---~.

__ I _ - -

approaching y = 1 from

-1

0
I

above?
Useful points are at
X

= 0,

= -1.

= 2,

= 4,

= -3

Example: (6)
(x + 1) (x- 1)

Y = (x + 2) (x - ~)

What are the asymptotes?


As x -

~ oo ,

/!

- - - -- -- - ,_ - -2

The useful points:


X

= 0, I, 1.5, -1, -1.5

Example: (7)
y =

x (x + 2)

2
x - I

Questions:
a
b

Asymptotes?
After x = I, as x - oo
what does y approach?

Before x = -1, as x - -oo,


how does y approach 1?

d.

As x - + 1, what does
y- ? -

y =I

to which line

does the curve approach?

j~
,_-- -----

-1 I

11

1.5
1.

Trigonometric Graphs

y = a sin bx

=a

The amplitude

The period, p =

b2n

f (x) = sin x
f(-x) = -sinx

:. f(x) = sinx is an odd


function.

2.

y = Acos bx
Amplitude = A
. d
2n
reno
p =b

f (x) = cos x is an even


function.

3. y = tan x
y -

oo

as x -

(2n - I)

The asymptotes are at

x = (2n - I) ~ ,
X

n = 0 , .:!:_1, .:!:_2,,

f(x) = tanx is an odd


function.

If.

(a) y = cosec x ,

period 2n

cosec x is an odd function


cosecx- "" at x = nn
(b) y = secx ,

(n

=Q.,.:!:

1,.:!: 2, )

period 2n

sec x is an even function

sec x (c) y

oo

at x = (2n + 1)

lT

2,

= cot x is an odd function,

=nn,

(n = 0, .:!:. 1 , )

The sketches are on the next page.

(n=0,_:!:1,

period n, with asymptotes at

12
y

!\

1\
I

I
I

n:

\
y

:X

----+-o---------4-n---------+l-2n~_.
-1

-n

z1
I

:nn/2
I
I

-1

\!

=cosecx

I
I
I

I
I
I

y = secx
y

y =cot x

lx I "'
IYI

lx I "'

. -n/2
y

. -1
= sm

-1

=COS

13
Sketch the foHowing curves:

Example: (1)
(a)
(c)

(b) y = Icosx

sin(x + y) =
2
y = sin x

I, Ix I ~

2u

(d ) y=cossm
( . -1 x )

y
Solution:
(a)

sin(x + y) =
:.

x + y

= nu

+ (-l)n ~ is the

general solution. [n = O, .:!:_1, .:!:. 2, ]


Now x + y = K

represents a series of

paraHel lines.
(b)

y = lcosx I , -2u.(: x" 2u


The easiest way to draw the sketch
of any absolute function is to draw
X

the curve of ordinary function and


then draw the REFLECTION of the
graph below the x-axis

k)

y = sin x
y

. 2
= sm
x

0 for all real x

The greatest value of y is


Some points:
(0, 0) ' (

*'

0.5) ' ( ~ ' l)

( ~u , 0.5) , ( u, 0) etc.
(d)

. -J x )
y = cos (sm

lxl ~ 1

Here

We simplify the function by replacing


. -J
-1 r----7_
sm x by cos { 1 - x'
. -1 x )
y =cos (sm

cos~os- 1 M)

y=~
Graph of y = cos (sin- 1 x) is a
semi-circle, above x-axis, radius 1,
centre 0.

B
(-1, 0)

A
0

(1, 0)

'~
117

14

Exponential Curves

1.6

Sketch the following curves:

y
(a)

=e X

>

As

0 for all x
X~

co, y - co

x--,

y-.. 0

=e X

Some points:
(0, 1), (1, e = 2.7), (2, 7.4), (-1, 0.3)

/
(b)

1.
2.
3.

4.

2
y =e-x
y > 0 for all x
A (0, 1)
f(x) =f(-x), even function. The graph
is symmetric about the x-axis
As x-co, y - 0
2
~ = -2x e-x gives a maximum at
X :

5.

0, y

=1

-I

d2

~= -2[1- 2x ]e-x = 0 gives inflexion


dx
at x

= 0.7

and x

=-0.7,

= 0.6

(c) y = x e-x

1.

y
y

2.

3.

>0
<0

As
As

for x
for x

xX-

"" y - 0
-co, y _....,

~ = e-x(l-

x)

=0

-1

= 0.4

x = 1, y = e
4.

>0
<0
X

gives
(Maximum)

Some points
(0, 0), (1, 0.4), (2, 0.26), (-1, -2.7)

15

1.7

Drawing Graphs by Composition of Ordinates

(a)

Addition or Subtraction of Ordinates

EXAMPLE: (l) Sketch the curve y

=1 +

SOLUTION: We separately sketch y

=1

2 sin (2x), - 11 ~ x ,< 11.


and y = 2 sin 2x.
2

I
I

=1 +

- -

- -

2sin 2x I
I

- - -

-1 - .

11

-111
\

' _ .,

\
/

' 11

I(
I I

4
-2

' . _..,"-..:.,.::~

= 2sin 2x

Algebraic addition of ordinates gives us the equation


y

= y 1 + y 2 = 1 + 2 sin 2x

(1)

By graphical addition we obtain the graph of y


by the heavy line.

Note that the amplitude of y

= 2 sin 2x

=1 +

2 sin 2x ' - 11 ~

is 2 and its

per~od

is

X~

11 ' shown

=n

Addition of ordinates must be done at all key points, such as end-points, at the
intersections with the axes and the critical points (Maxima, Minima) of both graphs.
It is useful to note that the graph of y = 1 + 2 sin 2x is obtained by translating the
graph of y = 2 sin 2x one unit upwards, thus preserving all the characteristics of
y = 2 sin 2x.

EXAMPLE: (2)
Sketch the graph of the function f(x)

= 2sinx

+ x,

0~ x~

211.

= 2sinx and y = x
2
The sine graph has an amplitude of 2 units and a period of 211. y = xis a straight
2
line through the origin. We separately sketch these graphs and by graphical addition
at key points x = O, ~ , 11 , ~11 , n , obtain the composite graph of y = x + 2 sin x,

SOLUTION:

Let y

as shown by the solid line.


(cont.)

16
y

,/
6

"

y:

=x

+ 2sin x

,"B
/

I
I

//

....._, I

y = 2sin x

n/2

3n/2

2n

Y2

1.57

3.14

4.71

6.28

y1

-2

y1+y2

3.57

3.14

2.71

6.28

Note that the points of intersection of the required curve y = x + 2 sin x and
y =x are where the curve y = 2sinx cuts the x-axis. So, 0, A(n,n), (B(2n, 2n)
are the points on the graph.
Cr itica1 Points:

x + 2 sin x

The stationary points are given by l + 2 cos x


i.e. cos x

1 + 2cos x

=-

2n
=T

f"(2n)
3

2 sin x

2n 2n
3 , 3 + {3)
4n 4n
0(3 , T- fj)

<0

=0

=-2
and x

4n
T

are stationary points.

and

So C (

is a maximum point.

c (2.1,

and

is a minimum point.

D (4.2, 2.5)

3.8)

We have sufficient information to sketch the graph.


2
Note:
= 0 gives the point of inflection, i.e. sin x = 0 and sin x changes sign
as x increases through n
dx

!!.f

:. x

=n

is the point of inflection A ( n, n)

17
(b)

Drawing Graphs by Reflecting Functions in Co-ordinate Axes

A function y = - f(x) can be graphed by reflecting the graph of y = f(x) in the


x-axis. The graph of y =c - f(x) may then be obtained by a suitable translation.

Sketch the following curves

EXAMPLE:

(a)

= -sin x

(b) y

=1 -

sin x

y = 1

SOLUTION:

We draw the graph of y = sinx (dotted line), then


reflect it in the x-axis to obtain the graph of
y = - sinx and finaUy shift it vertically upwards by
1 unit.

---.;:;.:::.,--I

1/

, 1
I y
-sinx \ 1
is shown by the heavy
,
. 1
\
.I
'
- -4,-,-.....::::.W:::-......:.f...---..&---.-;,.~.o x

The graph of y = l - sinx


line.
The key points to consider for plotting are
X

= 0,

n
2,

1T ,

11

3n
2 , 2lf

Graph of y

= IHx) I from

the graph of y

11

311

.. 2

......'

''"

-1

(c)

11

= sinx

= f(x)

Sii)Ce I f(x) I ~o for all x, all we have to do is to draw the reflection of any part of
the curve y = f (x) below the x-axis, in the x-axis, leaving unchanged those parts
above the x-axis.

EXAMPLE:

Sketch the curves: (a)


(c)

=X2 -

(b)
2

y = 2 + Ix -

y =

I x 2 - 41

41

SOLUTION:

(a)

=y 1 = / - 4

is a (complete)

parabola. (The part below the x-axis


is shown by a dotted line)
(b)

Reflecting only the part of the


parabola below the x-axis, we obtain
the graph of y

(c)

= y 2 = Ix 2 - 4 I

Shifting Y 2 , 2 units vertically


upwards, we sketch the graph of
y =y

=2 +

I x 2 - 41

-2\

'\

''

-2

''

,,

I
I

yl

18
Graph of y = f(,.. x) by initially graphing y = f(x)

(d)

y = f(-x) is the reflection of the graph of


y = f (x) in the y-axis.
EXAMPLE:

Sketch y

(b)

Sketch y = log (-x)

(c)

Sketch y = log

= loge

Note: The graph of y


y :

= Fx

(a)

log

X , X )

e
= log (-x) , x

I xl

lx I is the graph

y = log (-x)
e

<0

i.e. both branches make the graph of


Y

=loge I xl.

y = log lxl

(e)

= f(x)

Graph o.f y

= f(x

- a) by graphing y

=f(x

- a) is obtained by shifting the graph of y = f{x) to the right

The graph of

>0

by a units, if a

and to the left by

Ia I

units if a

< 0.

EXAMPLE:

(a) y = cos(x - 1)

Sketch the curves

(b)

- 1)

=cos(x

= cosx

\
'

= loge (x

+ 1)

19
(f)

Graphing the functions by multiplication of ordinates

EXAMPLE:

Sketch y

= x e-~

We separately sketch the curves y

-x

and y = y

-x \

=e

'\

0,

As X~

Q)'

X eX)

''

y = y y =xe
from F1g. 1.
1
2
When x < 0, y = x e-x < 0
X )

=e
2
We derive the properties of the graph

-x

,,

=y 1 =x

,"

y =X

"

,/

xex +0;

-1

As x"'"-"" xe-x-+-oo
2
dv
-x
) d y
-x
~ = e (1 - x , - - = -e (2 - x) , the
2
dx
stationary point x = 1 is a maximum.
Some useful points: (0,0), (1,0.37), (2,0.3),
(-1, -2.7).

x = 2 is the point of
inflection

The graph can now be sketched (solid line).

Graphing the functions by division of


ordinates

(g)

"
~y=x

f(x)

y = grx) by division of ordinates.

Gr~phing

/
/
/

logex
EXAMPLE:
Sketch y = - x separately sketching
y = logex and y = x

by
/

E)

_... -- ---

,. ,.., .... ~y =loge x

....

P(e, 0.37)

"

log x

--f-

"

SOLUTION:

y =

exists only for x

>0

-I

A few points:
X

0.5

Critical Points:

2
0.7

0.35 0.37

~~

Asymptote at x = 0.

0.32

As x

_!_ (x !
2
x
X

~ddx = 0 ~ log e x =

1.6

+ oo ,

- loge x)

~ x = e is the critical point.

Using the graph of y = log x , we find:


e
2
Since x > 0, ~~ has the same sign as the numerator
For x

< e,

1 - log x
_
e
1
x = e .., P (e, e )

>0

y _. 0.

and for x

> e,

1 - log x

< 0,

1ogex
The sketch of y = - x - is shown by the solid line.

=I

- logex

so the maximum point is at

20
(h)

Graph of [f(x)]n , n

> 1 by

graphing f(x)
1

Let y = g(x) =[f(x)f, then g'(x) = n. f'(x). [f(x)f- , so all the stationary points
and the x-intercepts of y = f(x) are the stationary points of g(x). The following
properties of [f (x) ]n may be useful.
lf(x)l

> 1,

Ig(x) I > lf(x) I, e.g. x > 2 ~ x 2 > 4 ~ x 2 > x


2
then 0 < lg(x) I < I f(x) I, e.g. 0 < x < ~ ~ x < x

then

1.

If

2.

If 0

3.

[f(x)]n

4.

If n is odd: (a) [f(x)]n

>0

for f(x)

> 0,

e.g. (x - 2)

>x

- 2 for x

>2

(b) [f(x)]n

<0

for f(x)

< 0,

e.g. (x - 2)3

<x

- 2 for x

<2

< lf(x) I < 1,

~ 0 for all x, if n is even, e.g. (x 2 -

4)

>0

for all x

Points of Intersection are useful and obtained by solving, g (x)

5.

= f (x).

Sketch y = (x - 1)
2
2
2
Le y = g(x) = (x - 1) , f(x) = x - 1
EXAMPLE: (1)
f' (x)

(1)

=0

and f(x)

2x = 0 and x

=0

give

- 1=0

'

.\

= O, !: 1 are the stationary points.


2
g'(x) = 4x(x - 1), g"(x) = 4(3i- 1)
g"(O) < O, g"(l) > O, g"(-1) > 0
So, x

''

So, the maximum occurs at x = 0 and

minimum occurs at x

= !:

............. .

'

,_

"/

,
/

(2)

g(x) = g(-x), so g(x) is symmetric about

(3)
(4)

g (x) ~ 0 for all x, g(x)-. co as x ... !: co


.
.
(x 2 - 1)2 = x 2 - 1 gives X=.!. 1, + l/'[ =.!. 1.4
.
Pomts
o f mtersect10n:

(5)

Additional points: (0, 1), (.!.1,0), (!: 2, 1), (.!. 2, 9)

the y-axis.

2
Sketch y = sin x
2
period of g(x) = sin x = .!. (1 - cos 2x) is n.
2
f(x) = sinx.
~ 0 for all x. g'(x) = 2sinxcosx
= 0, f'(x) = 0 give sinx = O, cosx = 0

EXAMPLE: ( 2)
The
Let
g(x)
f(x)

= 0, n,

3n

..:n, 2, 2
2
Observe that 5in x < Isin xI except at the stationary points.
The graph of g(x) can now be sketched with the points (Fig. 1).
So stationary pomt5 are at x

(0,0),

<-?, 1), (n,O),

(~ 1), (2n,O).

1T

2n

21

(i)

Graphs of the Reciprocal Functions

By using the properties of the reciprocal function y =

r/xr , in relation to the known graph

of y = f (x) , we can draw the sketch of y = f ~x) .

To sketch y

= f ~x)

>0

, consider the following:

, then

!y >

<0,

0 and if y

a.

If y

b.

Solve the equation y =

c.

The asymptotes of

d.

Behaviour of the function as x -+ + ""

e.

Simple points at x = 0 ,

!y

!y

i.e. y = + I
-

then

!y < o.

!y

As y increases

decreases.

for the common points of intersection

are given by y = 0

I , etc.

Example: (I)

Sketch y
x

>0

= loge x

and y

= log

for y = logex

I
Y = logx

y = logex is a known graph


1

y - - - logex
as follows.

(base, e, omitted for convenience)


I

I.

The asymptote for y = - - 1ogx

2.

The points of intersections are given by solving:

3.

x =e

and

x = e-l = 0.37

2.7

As y = log x increases from 1


1

As x - I
X-+

5.

is at x = I

i.e.

y = - - - decreases.
1ogx
4.

--

y = logx

is shown by solid line and can be sketched

I
, y =-1ogx

1-, y -

-oo.

(both positive)

-""

and

Some useful points (0.1, -0.4) , (0.9, -9.5) ,


(1.1, 10.4).

22
Example: (2)
y =

Sketch the curve

2""--:-

x - 4

Solution:
4 is a parabola as shown in the

.1'

figure. (broken line)


The graphs of both functions

= - 2- I.-

= x2 -

-1

2
and y = x - 4

o.

are both positive or both negative.


These two meet at points given by
X

= - 2-1-

-4

x - 4

X=~

+1

~,fj

15'

The graph of y = - - 1 2
X

has vertical asymptotes at x = ~ 2 and horizontal asymptotes as

k.

the x-axis. Also x = 0 , y = -

This is reasonably sufficient information to sketch the

graph. (SOLID LINE)

y
Example: (3) Sketch the curves

3
y = x + I and y = - 3 - x + 1
3

Draw the graph: y = x

2.

1
Asymptote of y = - - at x = -1
3
X + 1

3.

4.

As x -

=0 ,

=1

is a common point.

"" y 1.
y = -3-x + 1

>0

As y

<0

and increases,

>

!y <

and increases,

Two graphs intersect at x = 0 and


X

= -1.2

~/11

'\

and decreases.
7.

I
=-3--

0 for

and decreases.
6.

1.

As y

Solution:

.5.

y :

I
I
I
I

I
I
I

Solve: y

X
X

=0

=+ 1
, -1.2

23
(j)

VtW

Graph of

by graphing f(x)

By notmg the followmg, the sketch of y


I.

g(x)

2.

g (x)

3.

(a)

{Hx)

< f (x)

If f (x)

> 1,

(b)

fiW = f(x)

if f(x)

=1

(c)

J'iW > f (x)

if 0

flit;) so f'(x)

=0

4.

Jf(x) exists only for

~ 0.

2
2
e.g. Jx - 4 :::> x

=0

.JX < x

e.g.

< f(x) < I,

e.g.

if x

{X>

~4 ~

>,

2 or x

-2

>I

if 0

<x <I

gives the location of stationary points and

gives the positions of the vertical tangents.

EXAMPLE:

(a)

can be developed.

0 for all x in the domain of g (x).

~
f (x)

f(x)

= g(x) = v'fW

Sketch the curve


3
y = f(x) = x - 4x

and hence sketch


(b)

= ./f (x)
1.2

\
-3
Solution
(a)

_.,

'

3
The graph of y = f(x) = x - 4x = x(x - 2) (x + 2) is as shown in the diagram

(broken line).
(b)

Let g (x) =

Mx> .

The domain of g(x) is given by f(x)


is -2

x ~ 0 or x

(i)

g (x) ~ 0

(iii)

g(x)

>f(x)

(i;)

for

0. From the graph of y

= f(x),

this

? 2. Remembering that:

{ff;) ~ f (x)
< f(x) < I ,

g (x) =
0

for

f (x)

We take the square root of the ordinates of y

? I

= f (x)

in the domain of g (x).

The.graph of y = g(x) is shown by the solid line. Verify that the maximum
turning point of f(x) and hence g(x) occurs at x

= 1.15.

24

1.8

Implicit Differentiation and Sketching Curves


y =x

Consider the function


For y

=x2

~
dx
+

+ y

= 4, where

we have two ways of finding


(a)

2
2
and the relation x + y = 4

2x.

~ from x 2

But to find

y is not defined explicitly in terms of x,

We differentiate x + y

(b)

= 4

= 4

as it is, term by term.

Solve for y
y =

{;7

j 4-x 2

or y = -

-x
~--
dx-~

2x + 2y

or ~ = --,-.c;..x=~

~
dx

dx~

= 0

=-y

=-y

It is not always possible to solve explicitly for

y, as in the example x

y - 3axy = 0

For these types of functions, called the implicit functions, we find the derivative by
differentiating each term and then solving for

Example:

(a)

3
3
x + y - 3xy = 0, find

(b)

Differentiate with respect to x


3x

dx (3y
dy
dx =

3/

~-

- 3x) = 3y - 3x
2

3 (x

- X

+ y) =

x siny = 2, find

Differentiate with respect to x

X CO<; y

dx +

~ =-~
dx

x cosy

_ tan y
X

Silly :

25
Graphs of y

= l(x)

EXAMPLE:

1.

Vx (x

y = .:!:

We have

= x(x - 1) (x - 2)

Sketch /

- 1) (x - 2)

The graph of g(x) = x (x - J) (x - 2) shows


that g(x)

>/ 0

for 0 ~ x ~ 1 or x ~ 2

This 1s the domam of f(x)


2.

= Vx (x

- 1) (x - 2)

Relation (1) shows that the required curve is


symmetric dbout the x-ax1s.
loop m the intervdl 0

y ... .:!:
3.

(1)

,< x ,<

Hence there is a
1. As x +

oo,

co.

3
2
= x - 3x + 2x

y
dv

d,c = 3x

2y

- 6x + 2 gives the turning point

at x = 0.42.

4.

=2

5:

are the vertical tangents.

IntersectiOn with the axes:


X

= 0, 1, 2

EXAMPLE:

Sketch y

y = .! (x - 3)

We have

v'x'=J

(1)

The curve 1s symmetric about the x-axis.

2.

Intersection with the x-ax1s:

x = I, x = 3

The domain 1s x ~ 1. There is a loop between


x = 1 and x = 3, due to symmetry.

4.

As x -+

5.

2Y

co,

y ..,. .!

2 = (x - J) (x - 3) 2

1.

3.

is undefined dt x = 0, 1, 2, so x = 0, x = 1,

co

= 2 (x - I) (x - 3) + (x - 3) 2
= (x - 3) (3x - 5)

5
The possible cnucal points are x = 3 or 3 .
x = I is the vertical tangent.

26

1.9

Applications

By using the graphs of functions, we can:

~~

~I

(a)

Solve an inequality such as cos 2x

or 21 xI - I x - 21

(b)

Find the number of solutions of an equation, such as 2 sin x = x or x = e-x


f (x )

and by application of Newton's formula x 2 = x 1


to any required degree of accuracy.
(c)

1
-l'{i(}>
, fmd

a particular root

Solve physical and engineering problems involving equations which are either
impossible or extremely difficult to solve.

EXAMPLE: (1)

211 X

= sin 2x

The sketch shows the graph of y


1
are obtained by solving sin 2x = 2

and y

=~

The intersections P, Q, R, S

=> 2x = ~6 , 511 , 1311 , ..!.Z2rgiving x _ ..2!. 5n


6

Hence the solution of the inequality sin 2x

>, ~

1311 1711
- 12' 12 ' I 2 ' 12

is given by

x ~ :; or \ ~11 ~ x ~ \7 11
2

EXAMPLE: (2) Solve for x: 2lx I - Ix - 21 ~ 2


To solve this, we draw the graph cf y = 2lxl-lx- 21 as follows:
(i)

For x

(ii)

For 0

< x < 2,

(iii) For

x~

2, y
O, y

= 2x
y

- (x - 2)

= 2x

=x

-- (x - 2)

= 3x

2
= -2x -- (x-2) = -x-2

- 2

Now draw the line y = 2.


This line intersects the graph of
y = 2lx I -

Ix

- 21 at P and Q.

At P, y3 = 2 =9

-X

-2 = 2 ==>

= -4

-4

At Q, y 2 = 2 =9 3x- 2 = 2 ==> x = ~ = 1~
Hence the solution of 2lx I - I x - 21 ~ 2 is
given by x

- 4 or x

-2 A

~ 1~

Note that the sharp corners are located where

lx I = 0 and

Ix -

21 = 0.

27
Find the stationary points of the function y = f (x) = (x + l)(x + 4 >

EXAMPLE: (3)

Sketch the graph and find the domain and range of f(x).

Using the graph:

(a)

Solve the inequality (x + 1)x (x + 4) ~ 10

(b)

Shade the region R between the line y = 10 and the curve


y =

(c)

(x + 1) {x + 4)

Find the area of this region R.

Find the volume of revolution when the region R is rotated about the x-axis.

SOLUTION:
f(x)=(x+0(x+4)

4
f(x) = x + 5 + -

(l)

f' (x) = 1 -

+5x+4

...

2X

f"(x) = 8 3

(2)

. (3)

f' (x) = 0

- 4 = O, the

stationary points are where x = .!2.


f"(i)

>0

and f"(-2)

<0

A (2, 9) is the minimum and


B (-2, l) is the maximum point on
the curve.
(i)

Vertical asymptote at x = 0.

(ii)

Intercepts: y = 0 gives the x-intercepts x = -I, -4.

(iii) As

I xI ~co, using (l) f(x) .... x + 5 (oblique asymptote).

The graph can now be sketched. From the graph, we find that the domain is all real
X and the range is y ~ I or y 9 9.
(a)

To solve the inequality (x +O (x + 4 ) >,.. 10, draw the line


X

y = 10. The line

cuts the graph at P and Q. Since x > 0 for P and Q, (x + 1) (x + 4) >,..lOx


2
:. x - 5x + 4 9 0 (x - I) (x - 4) > 0 ~ x 9 4 or 0 < x ~ 1
(b)

Area of the shaded region between the line


y

= f (x) = x + 5 + '!. is given by


b
X
f4
4
A= a (y -y )dx=
[IO-(x+5+x)dx
2
1
1

= 10 and the curve


2
4
= [5x-y-4logex]
1

A= 5(4- I)- 1 06-1)- 4(log4 -Jogl) = 15- 7.5- 81oge2 = 7.5- 81oge2 sq.u.

(cont.)

28
(c)

Volume

=n

ab (y 2 - Y 2) dx
1
2

=n

x) ]

[ 100 - (x + 5 + 4 2 dx

2
16 40
(67 - x - lOx - - - - ) dx

16
=n [67x - Tx - 5x 2 + x
- 40 logex] 41
16
=n [67(4- 1)- 31 (64- 1)- 5(16- 1) + <-;;-= (93 - 40 loge 4) n c.u.

16)- 40loge4]

EXAMPLE: (4) Determine the number of roots of the equation 2sinx - x = 0 and
find the positive root to 2 decimal places by using Newton's formula
f(x )
x2 = xl -

'fi'<xJ

SOLUTION:
The graphs of y = 2sinx and y

=x

intersect at three points where


2sinx

=x

i.e. 2sinx - x = 0
At 0,

At P, x

=0

1.9 and at Q, x

So, the equation 2 sinx - x

-1.9

=0

y = 2sinx

has

3 roots.
Let f(x)

= 2sinx

then f'(x)

- x and x

=2cosx

Substituting f(l.9)
x

=x 1 -

f(~ 1)
~

= 1.9,

- 1.

= 0.18141,

= 19

f'(l.9)

18141

+ _18323

= 1.8323

= 180

in the formula

. I paces.
I
to 2 d ec1ma

29
EXAMPLE: (5)

A rod of length L is hung from a pivot A, attached

o~r----r------~

to the rim of a horizontal circular disc of radius r.


The system rotates with a constant angular velocity w
about the vertical axis OD. It can be shown that
2
(r + L sin 9 ) w = g tan 9 , where g is the acceleration
due to gravity.
Show by a graphical method that, if 0 ~ 9

,< ; ,

then

there is just one value of 9 which satisfies this

relation.
y

SOLUTION:
We draw the graphs of
Yi = w2 (r + Lsin9) and y
y

is of the form y

where A
y

= r w2 > 0

=A
1
, B

= gtan9.

= L w2 > 0

is obtained by drawing

A + Bsin 9

+ B sin 9

B sin 9 and then

1
shifting it vertically up by A units.

B
A

g tan 9

.I .JI
1

B . 9
sm

~/

I
I

It is seen that these graphs intersect at only one

= 9 1 , so there is just one value


satisfies the given relation.

,point P, where 9
of

e-. ~hich

9J

'IT

30

1.1 0 Miscellaneous Curve Sketching


In this section we sketch the functions involving combinations of trigonometric,
exponential, rational and irrational functions. Several or all of the techniques of
section I. 2
may be required. A sketch of a function should show the following
features:
1.

Intercepts

2.

Symmetry

3.

Asymptotes

4.

Critical points

5.

Concavity

6.

Rise and fall

Do not attempt a scale drawing, unless one is stipulated.


EXAMPLE: (1)

x (x + 1)

Sketch y

X -

SOLUTION:
(i)

(ii)

Intercepts:

..

y :: 0 gives the x-intercepts


x = 0 and x = -1.
x = 0 gives y = 0

:I

Symmetry:

. I

B
/

I
: I

,'

:: I/ "

f(-x) I f(x), - f(x)


No symmetry.

,,

"

/I

2 ":

(iii) Asymptotes:
Vertical asymptote: x

'../"'

.....

=1

2
We write y = x + 2 + - - (by long
division), as x ~ co,
y+x+2
. y = x + 2 is the
oblique asymptote.

y --2 - X - I

---- ----

X - 1

2.4

We work out the rough


shape of the curve by
separately sketching
2
y 1 = x + 2 and y 2 =x-:-y
(hyperbola) and then usin
y=yl+y2.
(iv)

Critical points:
2
f' (x) = x - 2x

21

'

I 1

(x - I)

The critical points are given by f (x) = 0.


2
x - 2x - 1 - 0 gives x = I .:!: 2,~ -0.4, 2.4
f"(x) is too complicated, so we use the signs of f' (x).

Now (x - I) 2 > 0 for all x,


x I I, so by drawing a parabola given by the numerator N(x) = x 2 - 2x - I, we find:
For x < -0.4 or x > 2.4, f' (x) > 0
i.e. f (x) is increasing
For -0.4 < x < I, I < x < 2.4, f' (x) < 0 i.e. f (x) is decreasing
Also the points A(-0.4, 0.2), B(2.4, 5.8) are respectively, the maximum and the
miminum points. This is sufficient information for the sketch (solid line).

31
Sketch the curve y = 2cosx

EXAMPLE: (2)

cos 2x, 0 ~ x {. 2n

SOLUTION:

The period of f(x) = 2cosx is 2n and the period of g(x) = cos2x is n, so the
function y = 2cosx + cos 2x is periodic with period 2n. A few key points are
necessary to guide us correctly along the curve. These are:
(0, 3),

<-,

3
-1), (n, -1), ( ; , -1}, (2n, 3)

Both f(x) and g (x) are continuous curves for all x.


To find the critical points, we have:

= - 2 sin x - 2 sin 2x

= - 2 sin x (1 + 2 cos x)

The critical points are given by sin x = 0 or cos x = 2n


3
These are (0, 3), (n, -1}, (2n, 3) and (3,-

z>

4n

(3,-

~
3
z>

It is unnecessary to check the nature of these points as we have sufficient points to


sketch the curve.

..
.

I
I I

..

,'

'
I

-1

-2

We can also sketch this curve by the addition of ordinates of the curves
f (x) = 2 cos x

7n

and g(x) = cos 2x at the selected key points x = 0, ; ,

4 , 2n This is shown in the figure.

3
, : ,

.... '

32
EXAMPLE: (3) Sketch the curve Y = 1 + sin x
SOLUTION:
(i)

(ii)

Intercepts:
No x-intercept.
The y-intercept

(0, I)

IS

-t----

-i f(x), -f(x), No symmetry

f(-x)

-n

(iii) Asymptotes:
Vertical asymptotes are where sin x = -I
x
(iv)

= 2n n

31T
0 +
2 , n = , _1

Critical Points: f' (x) =


f' (x) = 0 when

COS X

= 0 , giving

'

f(~)

(vii) y

>0

>0

3n

1T

1T

7n

31T

21T

2
l ln

,-2222>

sin x I - I

= - 1.

3n iT 5n
'- 2' 2 ' 2' .. where we exclude

2
f"(x) = sinx. (l + sinx) + 2~osx (l + sinx) cosx
(l + sin x)
2
.
x -_ sinx(l + sinx) + 2cos x , now usmg
cos 2x
f "()
(l + sin x) 3
.
. x , f"(x) __ . 2- sinx
1 + Sin
an d cance 11 Ing
(l + sin x) 2
,'. f" (x)

(vi)

(x

=>

-cosx
. )2
(l + smx

those points given by sin x


v)

, -+ 2 , .

-2

x ) (l
= (l - sin

x)
+ sm

for all real x, and the curve is concave up.

(All critical points on the line y

for all x (sin x

= ~)

-i - J) and the curve IS periodic with a period of 2n.


~ < x < 32n. The curve is as shown in

The pnncipal branch is in the interval the figure.

(viii) A few simple points (0, 1) and (n, J) are useful in sketching.

EXAMPLE: (4) Sketch the curve y

= ..,....--.....:....,+ sin x

by imtially sketching y = 1 + sin x.

I
I

SOLUTION:

I
I

After sketching y = sin x, we shift the curve


1 unit vertically up.
The asymptotes are where l + sin x = 0.
These points are easily seen to be
n
3n
x= ... ,-2 ' 2 ' ' "
The maxima of y = 1 + sin x are the minima
of the reciprocal curve y =
are at x = -

+l sin x

I
I

-Tf\

These

3n

n 5n
2 , 2 , 2 , ...

The graph of y

=1

+ Sin X

-!!
2 ....

....._.,

can now be sketched. (Heavy line)

A few simple points such as (0, 1) , (n, 1) are useful.

/ 0

:3
,.
X
:2_.: 2n
.. : .y =sin x
. -- - .... -- ..; .. - ..... .

'Tl.

--~-----------------------------------------------

33
EXAMPLE: (5)

=x

Sketch /

(1 - x )

SOLUTION:
(i)

Intercepts:
y = 0 gives the x-intercepts

(ii)

= 0,

=-

-1

! I
x = 0 gives y = 0
Symmetry:
-~

+
X

"1 -

X-

-J ~ X~ 1

So the curve is symmetric about


the x-axis.

/1 - l

Let y = f (x) = x
We
shall eventually combine the
graph of y = f (x) with the
reflection in the x-axis, i.e.
y

=-

to obtain the

the required graph of

/ = x2

(1 - x2)
Now f(-x) = - f(x), so y = f(x) is point symmetric about 0. Putting these
facts together, the curve is symmetric about both axes as well as the origin.

(i-ii) No asymptotes.
(iv)

f'(x) =

~-

x2 (1 - x2rl/2 = 1 - 2x2

/1- x2

The critical points are given by 1 - 2x

=0

X :/:

1, -1

=!

12

r---7

VI - x- > O, the sign of f' (x) depends on the sign of the numerator
2
N (x) = 1 - 2x (A parabola) This graph tells us that:

Since

(a)

f(

(b)

f(

(c)

The curve is rising for

~2

) is the minimum, f(-

.!.:.. )
v2

- ~, ( -~ v2

0.7)

~ ) is the maximum, f (-1-) = -21


V'i

v 2

-I ~

~ V"[ and

VL

Jz <

<~

as f' (x)

>0

and falling for

~ 1

Also, f' (x) does not exist at x


= _:!: 1.

=!

1, so there are vertical tangents at

This is sufficient information to sketch the curve which is a double loop, as shown
in the diagram.

34
= (x + 1) (x + 2)

Sketch /

EXAMPLE: (6)

SOLUTION:

.:!:

We have: y

J<x + l)x (x + 2)

The curve is symmetric about the x-axis.

/
/

(i)

Intercepts:

"c

x J 0, so, no y~intercept
y = 0 gives x = - 1, x = - 2
(ii)

Symmetry:
L et f ( X)

.
I

_ ,Ax + 1) (x + 2)

- t

f\

f(x) J f(- x)
No symmetry about the y-axis.
(iii) Asymptote at

= 0.

We work out the rough shape by initially


2
.
sketchmg
g (x) = x + 3x
x + 2 =x + 3 + ~
x
(shown by the dashed line).
y = ~g(x) is easily sketched by noting that
~ g(x) for g(x) ~ 1, Vi[X> > g(x) for g(x)
Hence the graph of y = .:!: JgW can be sketched.

vgw

(iv)

(x + 1) ( x + 2)

Domam: y ex1sts for

0 ,

< 1.

x I 0.

l,

Multiply by
then x (x + I) (x + 2) > 0
The domain is x > 0 or - 2 ~ x .$. - 1.
Since the curve is symmetric about the x-axis,
there is a loop in the interval - 2 .$. x .$. -I.
(v)

Critical Points:
x

Differentiating /

2 ~
Y dx

+ 3x + 2
X

x (2x + 3) - (x
X

+ 3x + 2)

w.r.t. x:
2
x - 2
-2-

-~-~
dx 2

x J 0, y I 0 i.e. x I I, - 2.
2x y
Vertical tangents at x = 0, x = - 1, x = - 2

~ = 0 gives x = .:!: .fi as the critical points, i.e.

A (1.4, 2.4),

B (1.4, -2.4),

C (-1.4, 0.4), D (-1.4, -0.4).


The curve being symmetric about the x-axis, we have altogether 4 critical
points. For obvious reasons the maxima and minima occur at both point'

x =

fi

and x = -

.fi.

There are vertical tangents at


sketch the curve.

x = 0, -1, -2.

This is sufficient information to

35
EXAMPLE: (7)

Sketch y = e

1/x

SOLUTION:
y

>0

(i)

f (x)

1/x

----------------

for all real x, x i 0.


f (- x)

i f(x), - f (x)

No symmetry
(ii)

=I 0

y ~1

The graph does not intersect


the axes.
(iii)

As x + .:!:

oo,

y + e

=I

The horizontal asymptote: y = 1.


(iv) Critical Points:

= -

for either x
(v)

el/x

<

0 for all x, x i 0. So the function is strictly decreasing

>0

or x

< 0.

Behaviour near x = 0
As x + o+, y e"' -+ oo, so the y-axis is a vertical asymptote for the right
branch of the curve (x > 0).
As x

(vi) f"(x)

+ 0-, y + e-"'

=~
X

~ 0

for the left branch (x

el/x + el/x - ;
X

= ~.

< 0)

el/x. (2 +

~)

el/x (2x + l)
X

>- ~ ,

f"(x)

>0

f"(x)

< 0 for x < - 21

for

x :/. 0

~ is a point of inflection with the curve concave down for x < - 21 and
1
concave up for x > 2 but x i 0.
So x = -

This is sufficient information for a sketch of y = el/x.

36
y

EXAMPLE: (8)
Sketch y

= ex cos X

0~

for

X,<

2n
y

= e-x cos x

3n

2'1T
X

SOLUTION:
(i)

Intersections:
= 01 y = 1
.
= 0 gtves
cos x

y
(ii)

=0

=2n , 23 n

Symmetry:
y

f(x) /. f(-x), -f(x)


No symmetry about the y-axis or 0.
(iii) No asymptotes.
(iv)

Critical points:
dv

= - e-X cos x

-x .
- e sm x

= -e-x(cosx + sinx)

dv

= 0,

when cosx

= e-X (cosx

sinx

=0

tanx

+ smx + smx - cosx)

dx

= -1

3n 7n
= 4,
4

= 2e-X smx

When x

d v
=43n , -=-t
> 0,

soy has a minimum at x

= 43n ,

so y has a maximum at

= 47n , Q (7n
4 ,

dx

When x

d2
Also ~
dx
(v)

= 47n ,
=0

dx

< 0,

~ sinx

=0

~ The points of inflection at x

It is useful to draw the basic graphs y = e-x and y


From these two we find that e-x cosx
for

TT

P(

>0

for

3n
)
4 , -0.07.

= O,

n,

0.003).
2n.

= cosx.

< X< ;

and ex cosx

<0

3TT

2 < x <2 .

The simple points are (0, 1), ( ;, 0), ( 11, -e-n ) i.e. (n, -0.04), ( 3; , 0) and
( 2n, e- 2n )

= (2n,

0.002)

The sketch is as shown in the diagram.

37

EXAMPLE: (9)

--I--- -

Sketch y = log (1 + cosx) ,


e
- 2n ~ x .$. 2n

=0
+

I
I

= 0,

i.e.
-2n

X=-2-2

I
I

-3n

Also x = 0 ~ y = loge 4.
(ii)

3n

t_
I
I

when cosx

'

~0~~-

Intercepts:
y

I
I
I
I

SOLUTION:
(i)

f (x) = f (-x), so the curve is


symmetric about the y-axis.

(iii) Asymptotes:
y ~ -oo when 1 + cos x + 0
The asymptotes are x = ~ n
(iv) ~
dx

2 (-sin x)
1 +COS X

x = O, ~ 2n
_

the critical points are given by sin x

'

(:. cos x

f. -1, we exclude x = ~ n)

( 1 + cos x) cos x + sin x (sin x)


(1 +cos x)

For x = 0, ~ 2n ,

2
4
dx

< 0,

(1 +cos x)
(1 +cos x)

so the critical points are maximum points.

These are (0, loge4), (2n, log 4), (-2n, loge4).


2
.
Ot h er pmnts
are ( -+ 2n

=0

O), ( -+ 2
3n

0)

This is sufficient information to draw the sketch.

- 2
+ cosx

38
EXAMPLE: (IO)
Sketch the curve y =

(x - 1)2
3

(x + I)

SOLUTION:
(i)

Intercepts:
x = 0, y = I and y = 0 gives x =

(ii)

f(x) J f-x), -f(x), so, no symmetry

(iii) Asymptotes:
As x ~ - I+, y ~ oo + and as
x~-1-,

y~oo-

x = - I is the asymptote.
As x ~ + oo , y + 0 +
As x ~ -oo, y -+ 0-

-1

Horizontal

I
I

asymptote y = 0.
(iv)

Critical Points:
3

f'(x) = 2(x-l}(x+l) - ~(X+I) (x-1)


(x + 1)
2
2 (x - 1) (x + I) - 3 (x - I )

(x + 1) 4

f, (x) -_ (x - I) ( 5 - x) , so t h e
4

(x + I)

critical points are (1, 0), (5,


Since (x + 1)
signs of

is always

2~)

> 0,

the

f' (x) depend on the

numerator

N = (x - 1) (5 - x).

The graph of

N (x)

is a parabola as shown in the figure.

This graph indicates that f(J) is the minimum and f(5) is the maximum point.
The sketch can now be drawn with this information.

39
y

EXAMPLE: (ll)
(logex)

Sketch y

SOLUTION:
(i)

Intersections
x J 0, so, no y-intersection

=0 =>

=0 =>

logex

=I

(logex)2

(ii)

f (x)

exists for

>0

f(x) ? 0 for all x


(iii) f(x)
(iv)

-j: f(-x), -f(x). No symmetry.

Asymptotes:
Vertical asymptote x

=0

As x + .,, y-+ 0
y

=0

is the horizontal asymptote. Verify this by taking x

= 1~g ~

f(e10)

=e 10 ,

e
(v)

Critical Points:
_ x (21ogx) 1/x - (logx) 2
dx 2

21ogx - (logx) 2
2

~ddx = 0 =>
:. x

=I

log x
e

=0

and x

or log x
e

=e =7.I+

For 0

< x < 1, ~ < 0

for

<x <e2

, ~
dx

=2

are the critical points

, so y is decreasing

> O,

y is increasing

x = 1 is the minimum point


x =e

log x (2 - log x)
e
e

2 . h
.
.
1s t e maximum pomt.

This is sufficient information to sketch the function.

whence

40

Exercise 1A
1.

2.

3.

Sketch the functions y :: f (x) and y = g (x), hence by the addition of ordinates,
sketch the graph of y =f(x) + g(x):
(a)

f(x)

(c)

f (x)

9.

(d)

f (x)

=cos x

(d)

f (x)

=x

, g (x)

Sketch the function y

= f (x),

f(x) = cosx

=sinx, g(x) =cosx


=x , g (x) =~

f(x) = cosx ,

=x

= f(x),

hence by translation, sketch

sketch f(x - 1)

g (x)

Sketch

f (x)

- 2 , g (x)

=[f(x) ]

=i

f(x)

hence sketch
(b)

y = g (x),

=x

= f (-x):

f(x)

hence sketch

+ 2

= f(x

+ a):

=ex , sketch f(x + I)


y = f (x) g (x) (any other
=x
y =

, g(x)

= loge x

f(x)
grxJ

=sinx

(b)

f(x)

(b)

f (x) = logex

(any other information


g(x)

=x

, given:

-4

Sketch

y = f (x) and hence

(a)

= x3

f (x)

(b)

=cos x

Sketch y = f (x) and


may be used):

=x

= -X

hence by reflection in the y-axis, sketch y


1
f(x) = siri x
(c) f(x) = ex

Sketch y = f (x) and y =g (x),


informaation may be used):
f (x)

f (x) = sin x , g (x) = cos x


1
f (x) = x , g (x)

(b)

Sketch the function

(a)

8.

f(x)

(b)

(c)

(a) f (x)

7.

(b)

Vx

f(x) = x , g(x) =

(a)

6.

, g (x)

(a)

(a)

5.

= Vx
=cos x

, g(x)

Sketch separately the functions y = f (x) and y :: g (x), hence by the subtraction
of ordinates, sketch the graph of y = f(x) - g(x):

(a)

4.

=x
=x

(b)

= VtW :
f (x) = sin x

(c)

f(x) ::

2 (4 -

2)

Sketch the following functions by the most appropriate techniques, noting


intercepts, asymptotes, critical points, behaviour near x = 0, when x -+.:!:. oo, etc:

H-

(a)

f (x)

=e-x sin x

(b)

f (x)

(d)

f(x)::x~x--21)

(e)

f(x) =

sin (x - l)

;x~x- -/)

=x 2 e-x

(c)

f(x)

(f)

4
f(x) = ~
X

2 x 2 (2 + x)
(g) y ::
2- X

(h)

f(x)

= (x

- 2)

+ 1

41
10.

(a) Show that x _ 2 - x

~3

+ 3

(x

~x:) (/ + 3)

3i- 7

(b) Find the vertical and horizontal asymptotes of the curve y = f (x) = (x _ ) (x + )
3
2
(c) Find the turning points and determine their nature.
(d) Sketch the curve.
(e) Using the sketch, solve the inequality 0
(f)

11.

< (x

<

Find the area between the curve and the x-axis between their points of
intersection.

Solve the following inequalities by graphical methods:


(a) Sketch separately the $raphs of y = 2lx the inequality 2lx - J I ~ lx + 21
(b) Sketch the function f(x)
1
cos2x ~ 2 .

12.

7
?x ) (x + 3)
2

Sketch the curve /

= x(x

= cos2x

- 2)

, 0~ x

II

and y

=I x

+ 21, hence solve

2n, hence solve the inequality

(a) Find the area of the loop in the exact form.


(b) Find the volume of the solid when the region inside the loop is rotated
about the x-axis.
13.

The chord AB subtends an angle a at the centre


0 of the circle, where 0 < a < 1T
(a) Show that the perimeter p of the minor
segment APB is given by p = 2r sin

(~) + r a

(b) If p is half of the circumference, prove that


2 sin ( ~ )

=n

- a

(c) Find a, in this case, by separately sketching the graphs of y


and y = 1T - a ' 0 < a <1T.
Give your answer to 1 decimal place.

= 2sin (;)

(d) Using one step of Newton's method, find the value of 9 to 2 decimal
places.
(Hint: f(9) = 2sin (;)+a-n)
14.

By sketching the graphs of y

=l

+ 2

approximate root of the equation f(x)

and

= x3

y = ! on the same axes, find the


X

+ 2x - l = 0. Hence, using Newton's

method, find to 2 decimal places a better root of f(x)

= 0.

42
15.

3x + 2

(a) Sketch the function y : f(x) : ( 2x _ 1) (x + J)


and the points of intersection with the axes.
(b)

Express

, showing the asymptotes

f( x ) m
. part1a
. 1 f ract1ons,
.
.
2 + 3)
1.e.
( 2x 3x
_ I)+ (x

(c) Shade the region whose area is given by the integral

A
= 2X":-r

J:

B
+ X"+3

t (x) dx. Find

this area.
16.

By sketching the graphs of y : e-x and y

=x

on the same axes, find to one

decimal place the approximate root of f(x) : x - e-x = 0. Hence by using


Newton's method, find to 2 decimal places a better root of f(x) : 0.

43

Exercise: 1B
Sketch the following curves, showing the CRITICAL points and labelling the axes
carefully. All logarithms are to the base e.

1.

2.

(a)

IX -

(c)

x- lx

(e)

lx + 1 I + \x - II

(a)

I Yl

21

(b)

\2x- 31

(d)

IX I

X-

(b)

lxl -

(e)

IXI + IYI
IY - 1 I

(d)

(g)

IX

(c)

3.

YI

IYI =

(f)

IYI = lx + II
IX I + 1
IY I

(h)

= IX

21 +

For all real values of x, the symbol [x] denotes 'the greatest integer not
exceeding x'.
Sketch the graph of y

/.j..

Given that

< a < b,

rI

evaluate

X-

[x], lxl ~ 3

draw the sketch of y

=I x -

aj for

lx I ~ b. Hence

a! dx

-b

5.

= max

(x, 1 - x) where max (a, b) denotes the greater of the two numbers

a and b, i.e.
max (a, b)

a if a
b if

6.

7.

(a)

(b)

x - [x] , where [x] has the meaning given in Exercise (3)

(a)

(c)

/.j.x

IY I =

l.j.

[x] ,

>b
<b

-2 ..$,

..$,

(b)

(d)

44

8.

9.

10.

11.

=x 3 -

(a)

(c)

y=(x

(e)

(a)

-y - X - 2

(c)

2x

- 4x + 4

-O(x

=(x 2

- 1)

+1)

(g)

1
Y =(x - 2) (x + 3)

(i)

(x + 2) (x - 3)
Y =(x + 1) (x - 2)

(a)

x (x + 3)
Y =(x + 2) (x - 2)

(a)

(c)

(e)

=41 (x

(b)

="""'i("':i:'"2

{d)

=x:~

- 2)

(x

2x - 4

(x - 2)

(d)

(x + 1)2

(e)

(c)

y=x(x-1)

(b)

(f)

1
y =-2-X
+ 1

(h)

X
=- 2
--

+ 2

(b)

2
X
- I
y=-3X

- 4

=--x:;:--3
2

=x3
X

=x (x

+ 1
- l)

=4x (x

- 2)

(b)

(d)

(f)

=9x

(x

=x:-1

2
2 -~
2
4 - y

- 4)

- 1)

45

12.

(a)

2
y = sin (2x) , (-11 ~ x~ 11)

(b)

2
y = cos (2x) , (-11 ~ x ~ 11 )

(c)

sin (x + y) = 0

(d)

cos(x + y) = 0

(e)

y = lsinxf

(f)

y = sinJx

(g)

sin x
y =-X

(h)

. ( . -1 x )
y=smsm

(i)

. -1 ( smx
. )
y =Sin

(j)

. (cos-1 x )
y = sm

(k)

. -1 (COSX)
y = Sin

(1)

2
y = /cos x

y =

(m)

(b)

y = xe

(d)

X- 1
y=-X
e

13. (a)

14.

15.

y=e
e

'

lXI ~ 211

2
X
X

-X
+ e
2

J,

Jxl ~ 211

M.

(c)

y =

(e)

y = [1-x]
- - ex
1 + X

(f)

y = e

(a)

y =log Jxl

(b)

y = loge (x

(c)

y = xlogx

(d)

X
Y = logx

(e)

(f)

y =

(a)

2
2
X + 2y - I = 2(x + 2y)

(b)

2
X (y - 1) = y

(c)

(x + 3) (y - 2) = 1

(d)

(x + 2) (y + I)

rx

= logx

-X

COS

llo~x

- 1)

I
=1

46

MISCELLANEOUS

16.

Determine the domain and the range of the function


(a)

Find the nature of the turning points and

(b)

Hence sketch the curve

= loge (x
x

17.

A function is defined by

f (x) = e

e
(a)

Find

(i)

lim

y = loge (x - x )

2
- x )

-x

- e .
x
-x
+

f (x)

x-+oo
(ii)

X-

-oo

State the domain and the range of the function.

(c)

Determine the stationary points and their nature and hence sketch the curve

(d)

19.

f (x)

(b)

18.

lim

= f (x)

Determine the inverse function

A function is defined by

y =

1
(x) and draw the sketch of

2
f (x) = xsin-l (x )

(a)

State the domain and the range of

(b)

Show that

(c)

Find

(d)

Describe the behaviour of

(e)

Hence sketch the curve

f (-x)

f (x)

= - f (x)

f' (x)
x

=0

(b)

= I(x

(d)

y =

((x) near

and

=1

y = f (x)

Sketch each of the following:


(a)

y = (x - 1) (x - 2)

(c)

(e)

Y = (x - 1) (x - 2)

= Ix

- 1 I (x - 2)

- 1) (x - 2)

(x - 1) lx -

21

y =

(x).

47

20.

A function f (x) = logex

X )

21.

(a)

Show that the curve y .= f (x) has a maximum at x = e and y =

(b)

Discuss the behaviour of f (x) near x

(c)

Sketch the curve y = f (x), by considering the points at

=0

and when x -+X

.!.e

co

= 1, 2, 3,

e'1

10

2x - 8

Sketch the graph of f ( x ) = (x _ 3) (x + 1) , showing the points of intersection


with the axes, the turning points and asymptotes.

22.

Sketch the graph of the function y

=x

- x + 1

2
(x - 2)

with axes, the turning points and asymptotes.

23.

2x

Sketch the graph of y = 2 - X

24.

Prove that the curve y

maximum turning point at (2,

25.

= x 2 ex

has a minimum turning point at (0, 0) and a

2e

> Hence sketch the curve.

x2

Sketch the curve y = - - - , showing the vertical and horizontal asymptotes,


2
X

turning points and inflexions.

26.

Sketch the curves y

2
= 10 x e-x 12

and y

10 ex '

on the same axes, showing

the turning and inflexion points.

27.

2x

Sketch the curve y = - - 2

, showing that it has a minimum at A (-1 ,-1) and

+ X

a maximum at B (1, 1) and inflexion at 0 .

, showmg the pomts of mtersectton

48

28.

Sketch the curve /

= (x +

1
( 2 _ x) , showing the vertical and horizontal

asymptotes as well as the turning points.


29.

Sketch the following curve, showing the critical features:


y = x2- lx

30.

I.

Sketch the curve y

2
x
= - - - , showing the vertical and horizontal asymptotes.
2
X

(Hint: x =

o,

y = 0 is a point, a lonely point detached from the rest of the

graph.)

31.

32.

The following curves are of the type y


2

(a)

y = (x - 1) (x - 2)

(c)

(e)

= (x - I) (x - 2) (x - 3)

=X- X

= f (x)

Sketch them.

(b)

y2 = (x - 2)3

(d)

= /

(x + 2)

Using the properties of the reciprocal functions, sketch the graphs of the following
pairs of functions on the same set of axes.
(a)

y = X - 2 and y = i("':"2'

(b)

2
I
y = X - 4 and y =-2-X - 4

(c)

1
2
y = 9 - X and y = - - 2
9- X

(d)

3
y =X

(e)

33.

- I

(f)

Y=lxt-2 and y=~

Use CALCULUS

1
and y = -3--

-1

y = (x - 1) (x - 2) and y = (x _ 1) (x _ 2)

to find the turning points, the sign of

~ etc. to sketch the

following curves:
(a)

y =x

+ I

(b)

(c)

y = -2--

x + 4

x-

.
34. Fmd the turmng
pomts
o f t he curve y = I

2I - 3I an dh ence s ketc h.1t.


X

35. Find the turning points of the curve y = 8x -

4
and hence sketch it.
2

49

CHAPTER 2 INTEGRATION

Almost any function

==

I (x)

can be differentiated, but unfortunately, the reverse

process of integration is not only difficult, there is no systematic method for integration.
Even worse there are functions which can never be integrated, for example:

e-x

sin x

-2x

So first we have a standard Jist of integrals and then Jearn a few more tricks.

NOTE: All logarithms in this chapter are to the base e.

2.1

Standard Integrals

I] =

J.

d rxn +
dx l""'j)""';T

2.

11
1
d [<ax + b) + ]
a (n + I)
dx

3.

d
dx

4.

d [cos (a~ + b)]


dx

5.

d
dx

6.

d [ax+bl
e
:: e ax + b
dx
a

7.

d
dx

pin(a~

[tan(a~

+ b)]

:: (ax + b)n

==

==

cos(ax +b)

-sin (ax + b)

+ b)] = sec2(ax +b)

e f (x)]

= f'(x)ef(x)

n+l

-J (
J

(ax + b)n dx

-+

J sec 2 (ax

-1.

==

(ax + b)n + 1
+ C,
a (n + I)

cos ax + b) d x

-+

-+

1-

=n-:;:1

xn dx

sin (ax + b) dx

c'

cos (ax + b)

==

tan (ax + b)

==

ax + b
eax + b dx

==

1\

sin (axa + b) + C

==

+ b) dx

+C

Jf'( xe) f(x) dx=e f(x) + C

+C

1-

-1.

50

8.

d
-d (log x)

loge x + C

9.

J t;{:/

-+-

dx = loge f(x) + C

ax + b
J -1

dx =.!.log (ax +b)+ C


a
e

u.

-J

12.
a

+ x

. -J

=Sill

I
+ x

2 dx =

aI tan-1

Jx 2 -a
I 2 dx = _I_ log
[ ~lc
2a
+ cij

2a

13.

a +C

x2 -a 2

for x
14.

15.

a - x

12:2) _

dx oge x + vxL:!:. a" -~


x

16.

2a

d 1oge [a+x]
dx
a-:lZ

-d (a ) = a

17. dx(cotx)
d

18. dx (sec x)
d

19. dx (cosec x)

_a

Ja

-4-

sec x tan x

ax dx

Jsecxdx

log

21.

Jcosecx dx

- log

(sec x
e

I
I

tan x)

= lo:x a
e

J cosec 2 x dx
sec x tan x dx
cosec x cot x dx

The following two formulas should be noted:


20.

< a,

-cosec x

-cosec x cot x

2 1 2 dx = _1_ log [a + x] + C
2a
e a - x
- x

for x

-J

log a

> a, a > 0

(cosec x + cot x)

C
+

a>

C '

-cot x + C
sec x

-cosec x

>

51

2.2

Change of Variable: Substitution

We are easily tempted to apply a known method to new situations.


2
x dx

J
Now

=;

+ C

[sin~x]

d
dx

Consider

=
X

. 2

Slll X COSX

2
then

J2x ex 2 dx ,

But given

.
J

2
sm x dx

, so why not

'

= -sin3-

+ C

. 2
not sm x.

~
dx

2x e

how do we proceed to find such integrals?

The method of substitution, leading to a change of variable, is used to solve such problems.
The symbol I will be used to represent the integral in each question.
WORKED EXAMPLES

I.

2
dx

2x ex

Solution:
Let

Solution:

= X2

du
dx

Let

= x2

du
dx

2x

J2x ex

- 4
2x

2
dx

2
2x and x - 4 in terms of u

Substitute for

feu

du
dx

Jeu

du

ru

~+ c

e
Thus

2
+

f f(x)dx

In either case

f)u

dx

Jg(u)du
f g(u)du

by using either

is easy to integrate.

du
dx

dx

-1/2
u
du
+

u = r$(x) or

9(u)

52

Integrate the following:


(3)

-1
~

J~ dx

Solution:

(4)

f.

Sln-1 X

(3)

Ju. dudx
=

du
dx

dx

d
X

. -1

Sin

Judu
u

21 (sin-1 x )2
(4)

J X loge X

Let u

dx

1 - x2

(5)

+ C

Jexdx

Let u = e

e2x + 1

du
dx

du
dx2. dx

+ 1

J- 2du- u

+ 1

-1
tan u +

-1 X
tan (e ) +
(5)

Jxlog
dx
x

du
Let u = 1ogx , dx

.!.X dx

lo!x

J~ .

+- Watch this: -+ dx
X

du

loge u +

log (log x) +
e
e

= xl
= du

53

Exercise 2A

(change of variable)

Find the indefinite integrals:


I.

2.

(a)

J 2x ( /

(c)

J 4~

(e)

Jx

(g)

J x dx
~

(a)

Jcos 2 x

(c)

(e)

+ 1)

3 dx

(b)

f 2x (I - x2) 4 dx

(d)

J 4x

dx
X + I

v'i-7 dx

(f)

J2x d\
I + X

(b)

Jsin 3 x cosx

Jtan 4 x sec 2 x dx

(d)

fcot 4 x cosec 2 x dx

J -si~x

(f)

sinx dx

dx

(g)

Jsec 5 x

(a)

fx2 e

(c)

J e sm

(a)

J~

(c)

tanx dx

(h)

dx

IX ./I - rx

Jo

cosx)

Jo

tanx)

dx

dx

(b)

x dx

(d)

J ex dx

logx dx

(b)

Jl (logx) 2 dx

dx
x (iogx) 3

(d)

. -1

4.

dx
(x 4 + 1)2

(h)

COS X

3.

sinx dx
2
sec x dx

e-X dx
I + e2x

x (I

dx
logx) 3

54

5.

I.
I

(a)

2 sinx

(c)

6.

ecos 2x dx

SlnXCOSX

(b)

-sinx ~x
I +cos X

(d)

cosx dx

Evaluate:

I~

(a)

(c)

(e)

2.3

2x (x

- 3)

3 dx

(b)

Tl/4

3
sin x cosx dx

x dx

(d)

0~

I~

eX
e

(f)

dx

Irr/

+ I

dx
xlog x

cosx dx

0 (I + sinx) 2

Integration by Parts

The reverse process of the product rule

d
dx (uv)

dv
u dx

du
+ v dx

is the method of

integration by parts.
From the above,
dv
udx

or,

I
I

d
dx (lv)

dv dx
u dx

uv

u dv

uv

I
- I

du
vdx

v du dx
dx

.. (I)

v du

... (2)

WORKED EXAMPLES

Let

du
. dx
Using the formula

x ex dx

uv' dx = uv -

x ex x ex -

v u' dx

ex I dx

ex + C

dv
dx

u = x,

=I

= e

Je

dx

= eX

55

2.

Jx cosx dx
U

u'

=X

dv
dx

=1 ,

Jlogx dx

3.

= COS X

Jcosx dx

and let

sinx

du
dx

Jx cosx dx

=X

f x logx dx ,

u or v can lead to a disastrous situation.

~~

if you choose u = x ,

not easy to integrate. Again in

Jx cosx dx = uv - Ju'v dx = ;

x . cosx dx, if

cosx +

= logx , then v = logx dx is

= cosx

~ Jx2 sinx dx ,

dv
, dx

=x

, then v

a situation worse than

Jx cosx dx
4.

Jexcosx dx

Let

X.

JX
e smx d x

Jex sinx dx

We integrate

ex (-cosx) -

()
1
by parts again

Jex (-cosx) dx

-ex cosx + I

where I

Then from ( 1)
X .

e smx
2I

ex (sinx + cosx) ,

Je

=ex

dv
, dx

u' =ex , v

Jex cosx dx
uv - Jvu' dx
e smx -

=I

x logx - x + C

A bad selection of

For example in

= x'

dv
, dx

uv -

x sinx + cosx + C

Warning:

= log x

Jv u' dx
(log x) x - Jx ~ dx

Jsinx dx

x sinx -

:. Jlogx dx

Jvu' dx

uv -

flogx dx as f (logx) 1 dx

Write

divide by 2

x cos x d x = 1 e x(.
sm x + cos x) +

= cosx
= sinx

= 2x

56

It is important to note that the given integral may occur while integrating it by

Note:

parts, but actually this occurrence helps to find the solution, as seen above.

or x

Integrals with
example to find:

Il

(5)

require repeated use of the 'integration by parts' method. For

2
dv
x , dx = cosx

cosx dx

2
x sinx Again

u 1 = 2x , v = sin x

2x sinx dx

x sinx dx

x (-cosx) -

I (-cosx) dx

-x cosx + sin x

=x

, v1

= sin x

u 1 = I , v = -cosx

2
x sinx- 2(-xcosx + sinx) + C
2
x sinx + 2xcosx- 2sinx + C
(6)

Find

Write

-1

I sin
=
=

x d)(

1
(sin x) 1 dx

uv . -1

X Sin

Now for

Let u =

v u 1 dx

X -

x dx

v 1-x~

.. (I)

2
du
Use u = I - x , dx = - 2x

+-

-~I~
Iu-

dv
x , dx =

I
I
U=T2,V=X

x dx

-~

. -1

Sin

du
or xdx =- T

[It's easier to substitute

112

for x dx , than x itself]


du

-Vu

- vr:7
. -1

X Sin

[from (I) ]

So it seems there is no end to the number of tricks you may be required to play in
Integration. But that is what makes it so fascinating!

57

Exercise 28
Integrate the following:

I
I

X C-x dX

(b)

x cos 2x dx

(b)

2
sec x dx

(b)

1.

(a)

2.

(a)

3.

(a)

Jx

4.

(a)

J x 2 logx dx

(b)

5.

(a)

cos-1 x dx

(b)

6.

(a)

log(x

7.

(a)

Jx

(b)

8.

9.

I
I
I

(c)

2
x sjnx dx

(c)

2
x sin x dx

(c)

J x cosec 2 x dx

(c)

f rx

logex dx

I
I

(b)

- 1) dx

I
I

x 2 e X dx

tan-1 x dx

(c)

e-2x cos3x dx

(c)

2
tan x dx

(Hint:

2
tan x

2
x cos x dx

(Hint:

2
2cos x

(a)

J~dx

(Hint:

Write

(b)

J~dx

ex sinx dx
2
x cosx dx

(logx)

I
I

dx

x tan-l x dx

x (loge x)

dx

2
sec x-1)

= 1 + cos 2x)

I<~l).

1 dx)

Evaluate the following definite integrals:

(a)

(d)

10. (a)

r ' 2 x cosx dx
0

(b)

x ex dx
0

( ' 2 sin-1 2x dx
0

(e) r'2 excosx dx


0

Use the substitution

y =

J2

. 9 to find

Sin

Find

1:

. -1
Sin

:i

v'J:Y"

d
y

(f)

e xlog x dx
1
e
n/2

2
x sin x dx

0
r'3
0

(b)

(c)

~dy

58

Use integration by parts to show that

11. (a)

cos-1

(b)

6 ) dx

(X

Show that

-1

X COS

6 -

11

tan- 1 u du

6n.

(X)

4 - 2 loge2

0
12. Evaluate:

r'3

(a)

I;

(b)

x sec 2 x dx

Ie

(d)

(e)

log x dx
e

I~

(j)

(i)

-t
te dt

(I)

(k)

log2

2
x cos x dx

2
1
(Hint: cos x = (I

2.4

Trigonometric Integrals

. 2
Using sm x

Io

= 2I (I

I
~I
I.
I
I

= 2I (I

(I + cos 2x) dx

I (
sin 2x)
2x+-2-

dX

(I - cos x) sinx dx

-(1 - u ) du

-u +

-cosx +

3
3 cos x

t dt

cos 2x) , we find:

(x _ sin 2x )

Sin X

. -1

cos 2x) )

2X

t e- 2 t dt

Sin

Io

(I - cos 2x) dx

Sin

dx
xlogex

Powers of sinx, cosx, tanx

2
cos x

- cos 2x) and

11/4

(m)

dx

-1
u tan u du

logex

x cosx dx

(f)

x logex dx

(h)

x sin 2x dx

11

r'4

(g)

r
r - 4-

(c)

log 10 x dx

+ C

cosx

du
dx

-sinx

59

4.

3
cos x dx

(I - sin x) cosx dx

sinx = u
du
dx = cosx

.
1 sm
. 3x +
smx-

I[l- 2cos2x +~(I+ cos4x)] dx

COS

(I - 2cos 2x + cos 2x) dx

~ [~
6

x - sin 2x +

sin 4x] + C

u = sinx

. 2X

COS X Sin

I
u

u2 (I - 2 u2

2u

COS X

Itanx dx

u7

I
I

dX

du
dx =

where

u = sin x

sinx
cosx dx = -log cosx + C
=

log secx

tan x tanx dx

f (sec 2 x -

I
I

u du

.!.

tan2x

I) tanx dx

2
tanx sec x dx -

COSX

3--5-+T

8.

5 X Sin
. 2Xd X

7.

u = tanx

tanx dx

log cos x + C
log cosx

60

I
I

2
2
tan x.tan xdx

2
2
(sec x - l) tan x dx

2
2
sec x tan x dx -

2
Iu du-

31 tan 3 x
10.

2
tan x dx

2
I<sec x-l)dx

=tanx

for the

first integral
- tan x + x + C

I
I

secx (secx + tanx) dx


secx + tanx

secx dx

2
sec x + secx tanx dx
tanx + secx

f'(x) dx
J fTxl

f(x)

=tanx

log (tanx + secx) + C

2
2
Jsec x (l + tan x) dx

u = tanx

2
J (l + u ) du
3
u
u + T + C ,
where u = tan x
3
tanx + .!. tan x + C
All the even powers of sec x and cosec x are integrated as above (odd powers by
integration by parts).

12.

_ J _ (cosec x + cot x) cosec x dx


cosec x + cot x
- loge (cosec x + cot x) + C

J cosec x dx

Jsecx sec 2 x dx

I
I

secx tanx

2
secx (sec x - I) dx

3
secx tanx
sec x dx + log (secx + tanx)
3
sec x dx , then
I
1
sec x tan x +
log (sec x + tanx) + C

dx = log f(x)

Integrate by parts

tanx secx tanx dx

secx tanx

Solve for
3
sec dx

f' (x)

f TIXf

+ sec x

61

2.5

Use oft = tan(x/2)

Rational Express1on

sinx and cosx

111

Jacosxd:bsinx

Expression of the form


.

2
1

dt
I
2 X
dx = 2 sec 2
2t

Also sinx

can only bt> integratt>d by special

SUbStltUtlOil t = tan
We have

+C

2
(I + t ), so dx

= 2dt 2
I +t

cosx

I +t 2 '

WORKED EXAMPLES

(1)

J3

Find:

1
+ 2cosx

dx

2 dt

-.--::7"

J_2.5!!.._i
5 +

2
./5

v0
(2)

Find:

t
tan-1 [ {5
+ C

X]

tan-1 2

-1

tan

./5

dx

J sinx + cosx
2 dt

1:7
2 dt

J1+2t-i
J 2 dt
2 - (1 - t)

(cont. next page)

62

J-2 d~

y = 1 - t

2- y

J-;L

- 2

n: ~I

2
~ log
1

71

.f2- tan~
1 + /i.- tan~

1-

log

.f2
(3)

,1-t-/21
1-t+Vi'

log

J2 sinS
- cos9

+C

d9

Jd~

U = 2 - cos9

loge U

dU
d9

loge (2 - cos9) + C
(4 )

J3

COSX

+ 2cosx

x
I
3
2 (3 + 2cosx) - 2

Arrange numerator as
I

(3 + 2cosx) -

3 + 2cosx

dx

J[ 4 - 3

+ icosx]

I x3 -2
- t an-I

dx

l-tan

2".f5

-J

2 -./5 tan

7'5
[tan~]

15

~2 )

+ C

using example (I)

.
sm 9

63

Exercise 2C

Integrate the following:


TT /4

l.

2.
TT/4

3.

4.

tan x dx
0

6
J sin

7.

512

8.

x cosx dx

4
cos x dx

J0

.2 2 d
Jsm xcos x x
5

J
Jsin52x

. 2 X dX

COS X Sin

dx

COS X

10.

Jcosec 3 x dx

4
11. J cosec x dx

12.

Jcos 3 2x dx

3
13. J tan 2x dx

14.

16.

3
.4
J Sin X COS X

3
J cot x dx

9.

15. J cos

213

5
x sin x dx

17.

Jc~s4 x
Sin

19

21.

23.

18.

dx

co~x

dx

n/3
0
1 + sinx

dx

dx
sinx - cosx

J0

.3
f sm /

dX

dx

COS X

TT/2

25

3
sin 2x dx

3 + 5cosx

dx

20.

22.

24.

J 2 sinxcosx

~in x

1TT /2

dx

d
-,.2---"'x:.:..__+ cosx
dx

64

2.6

Reduction Formulas

A reduction formula is one that reduces the index (usually an mteger) of the integrand.
By repeated applications the index is reduced to either one or zero. By using the method
of integration by parts, we can establish the following reduction formulas:
I.

2.

Jcos" x dx

-n

3.

f tan" x dx

n-1
X
tan
n - 1

sin

nI Sln. n-1 X

dx = -

cos

n-1

I
I.
I

sm

.
n - 1
x. sm x + - n

n-2

x dx

cos n-2 x dx

tan n-2 x dx

Solution:
Let In

I.
I

(n - I)
--n-

COS X

I
sin

sin" x dx , so that In_ 2

n-2

x dx

Using integration by parts:


.
dx
sm n-1 x. smx
. n-1
dv
. ]
[ 1et u = sm
x , dx = sm x

sin n-1 x d
dx (-cosx) dx

sin n-1 x (- cos x ) -

I( ) .
I

n - 1 sm n-2 x cos x (- cos x ) dx


2
2
sinn- (I - sin x) dx

- cosx sinn-l x + (n - 1)

- cosx sinn-l x + (n - I) In_

- (n - 1) In

Solve for In , then:


I

sinn-l x cosx +

!!....:....!
n

I
n-2'

where I

11

sin" x dx

You can see why these formulae are called the 'REDUCTION formulas'. The index n is
reduced to n - 2 or n - 1. For the example above, starting with, say, n = 10, we shall have
n = 8, 6, 4, 2, 0, which means you only have to evaluate
odd, then you have to evaluate only
For

tanxdx, we write

sin x dx =

sin x dx, etc. Proof of

fdx

If n is originally

cos" x dx is similar.

2
2
2
2
In= Jtan"xdx = Itan"- x(sec x-I)dx = Jtan"- x sec xdx- In_
tan

n-1

--n:l - In-2

65

WORKED EXAMPLES

5
=-t sin x cosx +

Jsin6 x dx

1.

sin x cosx +

.
- 21 smx
cosx

J dx =
6
Jsin xdx

14
12

1 I

- 61.5
Sln X

COSX

5(1.3
6 - ij Sin

- 61.5
Sln X

COSX

24

. 5 X COSX
- 61Sln

24

X COSX

31)

+ ij

5Sln
.3X

COSX

851 2

5Sln
.3 X

COSX

.
g5 (
-1
2 SlnX

COSX

211)
O

5
3
6
Jsin xdx = -tsin xcosx- 2!sin xcosx- 1!sinxcosx + 1!x
Since [sinm x cos

n/2

J0

x}

n/2
0

6
sin x dx = 1 [x}

... 0, we have

n/2

5n
32

i , the easier method is:

So, if the limits are 0 and

n/2
6
sin x dx

J0
14 =

3 1
2

ii

n/2

J0

[-

ii

sin 6 x dx

5
1
6 sin x cosx

ii
3

Tf '

J0

sin 8 x dx

65 14 = 5614

Tf

Tf

ii

7
8 16

. 65

n/2
2.

2 1o
6

3i

ii

Tf

Tf

35n
256

n/2
You must show that [sinm x cosk x}

= 0. Also remember you can not use this easier


0

method for limits other than 0 and


method.

n
Jn/3 6
2 . For 0 sin x, we must use the long

66

Exercise 20
Find a formula finding In in terms of In-l : (for examples 1-4)
1.

3.

n x
x e dx

n e-2x dX

4. I

Find a formula reducing In to In_

: (for examples 5-9)

Jcot 6 x dx

5.

Jcot" x dx

6.

I
n

Jcos" x dx ,

7.

I
n

Jsin" x dx

, hence find (a) r ' 2 sin 8 x dx


0

8.

I
n

Jtan" x dx

, hence find (a)

J(sin-1 x) n

9.

10.

, hence find

6
hence find (a) Jcos xdx

(b)

Jn/2 cos8 x dx
0

dx , hence find

Jtan 5 dx

(b)

r'4 sin 4 x dx
0

(b) r ' 4 tan 8 x dx

(sin- 1 x) 3 dx

(Hint: x

= sinU)

o xn

Show that Un

(1 + x)

1/2

dx , hence

-1

evaluate
11.

If In =
I

u3
mx

Je

n
tan x dx , show that

emx tann-1 x

n- 1

12.

Find

13.

If I

Jx4 ex

dx , using the reduction formula developed in Exercise 1.

Jsec" x dx
sec

-n:l

, prove that

n-2

x tanx
n - 1

n- 2
--I
n - 1
n-2

r ' 4 sec 6 dx
0
n
-x
x e dx = -e-x . x n + n

Hence, evaluate
14.

Prove:

Hence, find:

Jx3 e-x

dx

x n-1 e-x dx

67

2.7

Trigonometric Substitution

/"=>2

ll"aL+XL

Integrals involving expressions like


substitution.

x=atane,

x = a sinS ,

etc. are simplified by

x = a cosa

etc.

WORKED EXAMPLES

1.

J /: 2 1

Find

+ x

dx
d9

a tanS ,

dx

a sec a d9
a seeS

JseeS

Fig. I

d9

loge (sec a + tan 9) +


log (

1:2?

~)

.;;:z-;-;!
+ x ) + C
a
log (x + .;;;z-:-;;z ) - log a
e

log (

~)

log (x +
2.

Find

J ~ 2dx

x -a

Let

2 ,

>a >0
a sec9tan9

a seeS tanS d9

2
1 + tan 9

(sec a - 1)

Jsec a d9
log (seeS + tanS) + K

x jx2
- I)
a2
log (x
/x27)
e
log ( -

[Note: log a

x = asec9

I v~/

is a constant]

+
+

68

Exercise 2E

=a

Use x

F-2
x

1.

4.

Ia2 - x2 ,

sin 9 for

=a

tan9 for

2.

dx

5.

J?dx

dx
J

10.

13.

-l

/25- x

fi J 2 dx
x

dx

X~

sec9 for

2
J x dx

2
x dx

6.

J/xQ

dx

dx

2
J x dx

9.

J~

dx

11.

dx
x2)3/2

12.

X
d
~X

15.

14.
4
17.

I{.6)3

2.8

=a

8.

dx

dx

16.

19.

3.

Xi
X 2 9-

/25- x
J /a

and

to find:

7.

/a2 + x2

20.

;:G
{4

J)2_ 4 dx

18.

dx
x./:7-

J9dx

JJT7.
2 dx
X

f~dx

21.

f.[;.2:9

x2 + 3 dx

Integration of Rational Functions

TYPE 1: We shall consider the rational function


We divide
P{x)

a~{:) b where P{x) is a polynomial.

P{x) by ax + b by long division and write

Q{x) + ~b where Q{x) is a polynomial and R is a constant.


ax +

69
EXAMPLES: Integrate the following:
(a)

B= ~ ~

dx

(b)

xx~ 2 dx

x - 1
'"'X""'7T
dx

(c)

(d)

SOLUTION:
1

(a)

Divide 2x - 1 by 2x

2x + 1 f2i(':"l
2x + 1

~dx
2x + 1

Jo-2x:ldx
= x - loge (2x + I) + C

When the denominator is linear, we can divide by inspection. For example,


2x - 1
2x + 1 - 2
2x + I =
2x + 1
(b)

2
J ~dx
2
J[x 2x

X -

+ 4 + x _2

_ __2_
2x + 1

X-

dx

x + x + 4x + 8 log (x - 2) + C
3
(c)

3
Observe that x - 1 can be factorised as (x - I) (x 2 + x + I)
J

L.:.l_
X - I

J<i

dx =

+ x + l)dx

+ 2x + 4

)x3
2
3
x - 2x
2
2x
2
2x - 4x
4x
4x - 8

-'-&

3
x - 1 by x - 1 and get the same result.

Of course, we can divide


(d)

To make division easier, let U = 2x + 1

16
3

16 J 1
1

16
-I

16

1
- 16
-

J1

[u

(U

- 3U

2
U

- 3U

3U- 1

Also

1
3 --) dU

3U - log e U]

[! - J
3

dU

3
+

loge 3

~~

= 2 _,. dx =

when x=O, U=l


X : 1, U : 3

--r - -3U

_,

:.

3
(U - I) dU

u-

x=~

70

TYPE D: Integral of the form

P(x)
ax

dx

+ bx + c

If the degree of P(x) ~ 2 we divide P(x) by ax


P(x)
ax

in the form

Q(x) +
ax

+ bx + c

:x

i.e.

ax

+ bx + c

+ bx + c and express

+ D
+ bx + c

We shall only consider the cases where


2

ax

cannot be factorised.

+ bx + c is irreducible over the real field,

Method of completing the square and the

following integrals are required.

1.
2.

3.

f
f
f

dx
X

+ a

1
-1
-tan
a

-a

- X

2a

loge

[~]
x +a

2a

loge

c,
[~]+
a-x

dx
a

dx
2

c,

>

I <

lxl

lx

EXAMPLES:
(I)

f
x

fu
tan

dx
- 4x + 5

The quadratic

log (x

2x- 3

- 4x + 5
- 4x + 5)

U
dU
dx

1 ~ dx

The derivative of

dx

We write
dx

= (x - 2) 2
= x- 2

- 4x + 5

where

- 4x + 5
2x- 4

- 4x + 5 is irreducible

square.

-1
tan (x - 2) + C
(2)

over the real field, so we complete the

dU
2
+ 1

2x - 3

= 2x

dx
- 4x + 5

-1
+ tan (x - 2) + C,

using example (l ).

+ 1

= u2

+ 1

dU

- 4x + 5 is
- 4 + 1

2x - 4

71

(3)

I
I
2

dx
+ 6x + 4

The quadratic x
:. x

+ 6x + 4

+ 6x + 4 is irreducible
(x + 3)

- 5

dU
2
u - 5

1
V5

log

where

[u-v'5J
U + v5
+

- -1- 1og [x+3-V5J + C


-2./5
X+3+/5
(4)

I
= 21

2x

I
4

2 v'55

dx
+ 7x + 13

2x
2x

u2

+ 7x + 13 cannot be factorised.
+ 7x + 13 = 2

dU
55
+16

-1 (
tan

=2
u )
\/55/
4

1/55

7) +

-2- t an-1 (4x


- +-

\/55

~ x + 1i]

rx

~)2 + ~~

= 2 [u

c
where U

= x + 47

2
+

~~

72

Exercise 2F
Find the integrals:
2.

J1 ~4x dx

4.

J2x1 - - 2x3 dx

6.

J3:~ 1 dx

dx
+ x + 1

8.

dx
+ 2ax + b

10.

4x + 3
dx
+ 2x + 3

12.

I.

3.

J__
dx
X+ J

5.

7.

6x
3x + 2 dx

ax + b dx
ex+ d

x
9.

J2
x

11.

J
x

13.

J 2 .x dx
x

J
9x

3dx
- 6x + 2

J 2 x dx
x

14.

dx
- x + 1

- x + 1

JX~ + X + J dx
X -X+ J

+ 2x - 2

Evaluate the following in the exact form:

15.

r
2

J 7.

19.

I:
I:

dx
2
X - 6x + 10

16.

(x + 1) dx
2
X +X+ 1

18.

3 dx
2
X +X- 2

20.

r
r

dx

2
J X + 2x- 1

2x + J
dx
2
-1 X + 6x + 10

r
a

3
_x_ dx
J - X

(1

< a < b)

73

2.9

Method of Partial Fractions

It is easy to add two fractions.

1.

c
+

ad+ be
bd
x-2-x+3
(x - 3) (x - 2)

2.

(x - 3) (x - 2)

1
The reverse process of separating a fraction such as (x _ ) (x _ )
3
2

is not so simple.

We shall now study a method of splitting the general rational function

~~~)

into its

partial fractions.
If the degree of

P(x)

A(x), we first divide

P(x) by A(x), and express

as the sum of a polynomial and a rational function


R (x) < the degree of A (x)
The method of decomposing a rational function

~ ~x~

~~~~

~~~~

where the degree of

x
into

partial fractions can be

stated as follows:
I.

Factorise the denominator.


I
I
For example: - - - =
2
3
x - I
(x - I) (x + x + I)

2.

A theorem on polynomials ensures that every polynomial R (x) with real coefficients
can be factorised into the product of powers of either linear terms of the form (x- a)n or
2
irreducible quadratics of the form (x + bx + c)m .

We are only required to study cases where

n = I

and

i.e. the factors are not

repeated.
3.

In each partial fraction, the degree of the numerator


denominator.

< the

degree of the

74

Example: (1)
Find

2x - 1
(x - 2) (x - 3)

dx

by partial fractions.

Solution:
Let

2x - 1
(x - 2) (x - 3)

- - , where A and B are constants


X- 3

Multiply both sides by (x - 2) (x - 3)

2x - 1

A (x - 3)

B (x - 2)

x (A

B) -

3A - 2B

This is an identity, so by comparing coefficients of similar terms:


A + B = 2
Solve these:

and

=-3

3A + 2B = 1

B =5

I[

2x- 1

J(x - 2) (x - 3)

__::_L
X - 2

dx

x -53]

dx

- 3log (x - 2) + -5log (x - 3)

5
log (x - J)
e (x - 2)3

Example: (2)

Find

(x

x +
+ 2) (x - 2)

dx

Solution:
Here we have an irreducible quadratic factor x
2x
(x - 2)

A
x - 2

<l + 2)

Multiply both sides by (x - 2) (x


Then

2x

A(x

+ 2)

2 , so we write

Bx + C
x2 + 2

2)

Bx (x - 2) + C (x - 2)

2
x (A + B) + x (- 2B + C) + (2A - 2C)
- 2B + C

=2

and

2A- 2C

=3
(cont. next page)

75

Solve:

A =

J(x -

2x + 3
2) (x

6'

d
+ 2)

-6'

?.6

J-

?.6

Jx-2
-- dx -

1
- 2 dx

.!. J _ 7x

X-

- 2 dx

X2 + 2

?.6

J2 x - dx X + 2

7
(
)
7
( 2
6 log x - 2 - T2 log x

.!.3

J 2-X

2) - -1- tan-1 - X

v'2

JVi

dx

+ 2

Example: (J)

(x - 2) :x + 1)2

dx

Solution:
We have a repeated factor (x + 1) , 2 times. We write:

(x - 2) (x + 1)

A
x - 2

A (x

--

+ 1)2 +

+~

+ 1,

\X

B (x

+ 1)

C
(x + l)2

(x - 2)

C (x - 2)

Here it is easier to use suitable values of x


Put

= -1

= -3C

= 2,

= A-

J (x

0 ,

1
- 2) (x +

02

dx

or

9A,

3
1

9
1

B=-9
1

~ - 9 rx+n -

9 log (x
1

=-

A=

2B- 2C,

J [.!.9
1

- 2) -

x-2

9 log i(""""+'"1

log (x + 1)
1

lx+n

(x + 1)

2Jdx

+3rx+n
+

76

Example: (4)

x3

~1

dx

Solution:
x

We have

2
- 1 = (x - 1) (x + x + 1)

1
-3-x - 1

Multiply by (x - 1) (x
Then

A(x

Put

Bx + C

Bx (x - 1) + C (x - 1)

1,

3A = 1

o,

A - C = 1 or

Hence

+ x + 1

+ x + 1)

+ x + 1) +

X = -1,

or

A=3

or

A +2B- 2C =

1
dx= 1
- x_3
3
1

2
= -3
B = -3

1 dx-.!J
x+2
x - 1
3
2
X +X+

1
1
3 log (x - 1) I ,
3

f2
X

X+ 2

+ X + 1

J21 (2;

dx

Iog(x

where

+ 1) +

dx

+ X + 1

.!f2x+l
2
2
X + X +

log (x - 1) -

log (x

A (x) d

J P\xr

To integrate

1.

A (x) ) P (x) , then divide

2.

If deg. A (x)

<

J(x + 2)
/ 2 + 43

dx

t VJ/2

+ x + 1) -

.fj
3 .

-1
tan

deg. P (x) , then factorise

A (x) by

P (x) completely.

(b)

For (x - a)0

write

If ax

Warning:

+ bx + c

~ + _B__ + +

Do not use

or

(x - a) 2

C
(x - a) 0

Ax+ B
2
ax + bx + c
B twice; all constants must be different.

occurs, then write

v'3

P (x)

For each linear factor (x - a) , write the corresponding term


,

2x + 1

(a)

\X - a,

3.

If tan-lf(x + ~)]

+X+ 1) +

SUMMARY:

If degree of

dx

A
x - a

77

Exercise 2G
Use the method of PARTIAL FRACTIONS to find:

1.

3.

5.

+X

(x + 1)

9.

x (x

4)

~ 1)2

dx

(x - 2) (x \

8.

3
x 2+ 2
x (x

2) (x + 3) dx

dx
1)

2x :x
(x + 1)

(x - 2)

13.

(x + 2)

15.

2 2x + 3
dx
- 7x + 12

dx

(x - 2)

10.

dx

x dx

11.

~x2

4.

3
x 2 + 5 dx

7.

x + 2
(x - I)

+ 2
(x - 1) (x - 2) dx

2.

(x - 1) lx - 2) dx

J -f-

12.

- 1

~X

(x

2
x
(x - 2) (x

14.

+ 4)

~Hint:

dx

1
+ x + 2)

~y

2
(x

dx

dx 2
+ 1) (x

let x

+ 2)

= y only for decomposing ]

2 ) into partial fractions

+ I)l(y +

Evaluate the following:

16.

18.

20

4
f3

dx
x

2x dx
2
+ 2) (x + 4)
(
x
0
ll/2
cosS dS
2
sin s + 5sinS + 6

fo

y = sinS)

(Hint:

22.

- 3x

17.

1
1 +

rx

dx

(Let U = v'x>

19.

f1

dx
x

(x + 2)

n/4
cosS dS
2 + sinS

(Hint: 2 + sinS = y)
21.

(x2

1)2

dx

2
2
(Hint: (x 2 - 1) 2 = (x + 1) (x - 1) )

78

2.1 0 Completing the Square (Integration)


Integrand of the type

involving the irreducible quadratic

ax

+ bx + c

over the real

field is easily found by completing the square and then using one of the following:

1. f x2 :x a2

3.

i . tan-l(i)

==

dx

::log (x + J. x2 :_ a2)

2.

==

./.T7

Example: (1)

Find

dx

- 4x + 5

Solution:
x

= (x

- 4x + 5

:.

- 2)

= U2

+ 1

= tan-! U

Example: (2)

dx

~x-i

Solution:
4x- x

- (x

- 4x + 4 - 4)

4 - (x - 2)
4 -

u2

dU

= x - 2

. -1

u
2

. -1

(x - 2)
-2-

Sin

Sin

c
+

+ 1

U =

= tan

X -

(x - 2) +

79

2x + 3
dx
2
/ x + 2x + 3

Example: (3)
Solution:
2

2x + 3 = (x + 1)

Also

d
2
dx (x + 2x + 3)

u2

2 =

2x + 2

Rearranging:
I

J2x

Jx

2 + I

2x + 3

(2x + 2) dx

Vx

2x + 3

For 1 , let
1

For 1 , let
2

dx

v'1x + 1)

dz
dx

2x + 3

2x + 2

X +

f .tz I ~

I =

dz

Vz

2 /x

dU

log (U +

2x + 3

Example: (4)

vlJZ':2i

log (x + 1 +

/x 2

2x + 3) +

2
dx

- 4x + 6

Divide out
+

2 4x - 6
x - 4x + 6

2(2x - 4)
2
x - 4x + 6

= x + 2log (x 2 - 4x + 6) + 2

= x + 2log (x 2 - 4x + 6) +

dX

d
2
dx (x - 4x + 6) = 2x - 4

2
2
(x - 2) + 2

- dU
-2
U + 2

../i tan 1 (x

- 2)

V2

=X-

80

Exercise 2H
Find:
1.

2.

J X2 + 2x + 4

+ 1
dx
x + 2x + 4

4.

7.

3.

5.

6.

8.

10.

dx

lx

dx

11.
2

13.

x dx

r lx
IV)
0

17.

dx
2 + 2x + 2

Evaluate:

dx
2 + 4x + 5
X
X + 12
-2-+ 9
X

r hx7
dx

dx
2x

x + 5

x dx

x dx

J
J

Jx 4 - 3x 2 + 1
(1 - 2x) dx

2
/ x + 2x + 3

Show that:

(
16.

dx
/x 2 + 2x + 4

/ 6x- x 2

dx

15.

9.

+ X +

/1 - 2x - x
14.

dx

x dx
2
/x + 2x + 4

/2- x- x
12.

2x

dx

J x2 ;x
f 2 x2

dx

= loge

If : vA I

81

18.

19.

I rr-:;_

X + 1

dx

{~

[ Hint: I

1
- x
./(I - x) (I + x)

=J

;-;-:;

20.

f/

1 - x

dx

dx

dx

x : 2

2.11 Integration: Special Properties


Use of the following formulas simplifies the work and saves time.

1.

I
f

a f(x) dx

2.

f (x) dx =

J0

a f (a - x) dx

-a

3.

a f(x) dx ,
0

if f(x) = f(-x) , i.e. f(x) is an even function.

a
f(x) dx = 0 ,

if f(x)

= -f(-x),

i.e. f(x) is an odd function.

-a

Proof:

Put a-x= U ,

1.

when x

=0

, U =a

x=a,
RHS

LHS

a f(a- x) dx =
0

JOa f(x) dx

RHS.

J
a

U=O

f(U) (-dU) = -

J
a

f(U) du =

a f(U) du
Q

82

2.

The graph of an even function is symmetrical

about the y-axis, i.e.


f(x) dx

f(x)

f (-x)

Area PABQ

-a

2 x Area MOBQ

f(x)dx

Thus for an even function

J-a

=2

a f (x) dx

fa f (x) dx
0

-a -x

Fig. 2

3.

The graph of an odd function has a point

symmetry about 0.
Area OBA = - Area ODC

a
0 (as an integral)

f(x)dx

-a

Thus for an odd function


fa f(x) dx

= 0

-a

Fig. 3

Note:
Though as a pure integral

f a f (x) dx = 0,

if

f (x) is odd, we need to be careful if the

-a
area bounded by the curve y = f (x) and the x-axis is required.
A = A

+ A

-a

f(x) dx

a f (x) dx

= 2

I f a f (x) dx I
0

Then we use:

83
WORKED EXAMPLES

Evaluate:
2

J.

J-2

2
x dx

2.

fn

sinx dx

-1T

Solution:

1.

2
2
. an even f unct1on
.
x 2 1s
as f (-x) : (-x) : x

f(x) :

16

::; 3
2.

f(x) : sinx and f(-x) : sin(-x)

-sinx
y

f (x) is an odd function.

1T

sin x dx : 0

-1T

The area etween the curve and

Note:

x-axis from x : -n to x
given by
A = 2

3.

J:

sinx dx =

Use the result


n/2
(a)

J0

J:

=n

is

2[-cosx]~

f(x) dx =

.
. sm x
dx
smx + cosx

Fig. 4

J:

f(a - x) dx to evaluate

(b)

Solution:
(a)

=
=

Jn/2
0

r'2
0

sinx
dx
sinx + cosx

( 1) '

sin (n/2 - x)
sin ( n/2 - x) + cos (n/2 - x)

Replace x by 2 - x

cosx dx
cosx + sinx

(2)

(cont. next page)

84

= Jn/2

21

Add (1) and (2):

sinx
+ cosx d

X
+ COSX

SlnX

:: 2
n

ii
2

(b)

x 2 (2 - x) 1/2 dx

J 2 (2 -

x) 2 [2 - (2 - x) ] 1/2 dx

=J

2
1/2
(4 - 4x + x ) x
dx

r~

l3

x3/2 -

x5/2

~ x7/2]
7

128/2
105

1+.

Using the property of odd and even functions, evaluate:

t:

(a)

x3

(b)

tan x dx

-1+-- dx
+ 1

Solution:
(a)

f(x)
f(-x)

= tan

(b)

f(-x)

1
-1

[tan (-x) ] 5
[-tanx]

f(x)

f(x)

+ 1

3
(-x}
4
(-x) +

-X

- tan x

..

f(-x)

-f(x)

..

f(x) is an odd function

is an odd function
5

tan x dx

...

r
-2

- f (x)

- 4 - - dx
X
+ 1

-1+--

+ 1

85

Exercise 21
1.

Using the properties of the odd and the even functions evaluate:

(a)

ll/4

3 dx

f-n/4

(b)

-2

COSX

1
(c)

(e)

(d)

1
(x

x tanx dx

f-1

5
X
- 4- X
+ J

(f)

+ x sinx) dx

-1

(g)

(x cos x - 100 x

(a)

(c)

(e)

fon/2

TI/2
JO

2-

Ja

dx

sin x cos x dx

f (a - x) dx

and evaluate

dx

(b)

J,(01

(d)

x (1 - x)

dx

+ lcosx

Tl

X (

a f (x) dx
0

n/2

f-n/2

(h)

+ 2) dx

Prove that

2.

dx

ll/2

)
X

COS

2
cos x dx

10 100 x (I - x)

99

dx

3.
(a)

4.

Using the properties of odd/even functions, evaluate:

+ x

+ smx d

-1

} + X

(a)

Show that if

(b)

If

(b)

f (x) is even and

f_:

x ,

and b

f (x) dx

Ja

sinx dx
2
0 a+ b cos x
1f

f' (x) e;{ists, then

f (x) is odd and continuous for all

explain why

f (x) dx

>0
>0

f' (x) is odd.

>a >0

use a sketch to

86

Exercise 2J

(REVISION)

Use any suitable method. Some integrals can be found in more than one way.
Integrate the following:

I.

3.

I-x
.;x:-'2

I e

5.

2x
dx , using u = e

9.

II.

I
J

4.

6.

15.

17.

2 2
b sin x)

25.

2
a co/x

Jx 2 1ogx dx

x v'f'"+i( dx

14.

16.

dx

18.

+ 4x

5
20.

dx

22.

3x + 2
4x

4x

dx
5 + 4sinx

2 2
b sin x

dx

2
(Hint: U = x )

dx

I + e; dx
I - e

(Hint: t = ex)

dx

f Vt-:7
f
2

dx

+X +

dx

12.

f X~
f
J
f

5
sin(x ) dx

2
(Hint: Divide by cos x)
8.

/X

+ 3x - x

23.

x 2 sinx dx

4x

21.

~dx

10.

x)

dx
I +ex

19.

JtfJ

dx
2(1

r
f

- I

13.

I:
I

sin 2x
dx
2
2
b sin x

Ja 2

- I

(Hint: t = a
7.

2.

dx

dx

24.

5
26.

_e_ _ dx
I + e2x

I
f
f
f

xsin- 1x dx
dx
X

4x - 5

dx
y{x

4x + 5

3x + 2
/4) + 4x

dx
+

3sinx + 2cosx
dx
3cosx + 2sinx

[Hint: 3sinx + 2cosx


A Ocosx + 2sinx) + B(-3sinx + 2cosx)]

87

27.

29.

31.

I/
I
I

2
(cos2x = 1 - 2sin x,
2
divide by cos x)

dx
2 - 3cos2x

30.

/x2- X+ 1 dx

32.

2x + 5
dx
2
X - X- 2

34.

= t)

36.

J -X -- 3
X

(ex = t)

38.

35.

f (x 2 + 3) (x 2 + 1) ,

37.

f + 3ex + 2e 2x
cosx dx
f (I + sin x) (2 + sin x)

2x dx
dx

(x

cos

v'X

40.

2
(Hint: X = t )

dx

3
X
dx
2
X - 3x + 2
dx

I
I

(Hint: divide
2
by cos x)

dx
(3cosx + 2sinx)

cosx
5- 3cosx dx

33.

39.

I
I
I

28.

(Hint: multiply
by .ja"+X)

a-x dx

[x - x

dx
O+X+X

2 + x3)

= x(l

(Factorise)

dx
sinx + sin 2x
(Multiply by sinx, then t

41. Show that

n/ 2

n/2

n
n - I
sin a da = - n

sin

n-2

42. Apply successively the result of exercise 41 to show that


n/2

(n - 1) (n - 3) 4.2
n (n - 2) ... 3.1

sinna da

if n is odd
n

(n - 1) (n - 3) 3.1
n (n - 2) ... 4.2

and

if n is even.

43. Establish the reduction formula (using integration by parts)

xnsin bx dx = - x: cosbx +
n/4

hence evaluate:

n-1

cos bx dx ,

2
x sin 2x dx

44. Establish the formula (using integration by parts)

1
xn+ 1 . -1
xn sin- x dx = n + Sin x 1

Hence find

n+T

x sin-l x dx
0
45. Establish the reduction formula

cot

dx

= -

[Hint: cotnx

cotn-l x
n - 1

Jcot n-2 x dx

= cotn- 2 x (cosec2x - 1)]

= cosx)

a da , and hence show that

n/ 2 . 4
3n
sin ada =
16
0
.

- x)(l + x)]

xn + 1

dx + C

88

CHAPTER 3 VOLUMES
3.1

Formulas for Volumes


y

In elementary calculus, we have the


following two formulas for calculating
the volumes of revolution.
1.

The volume of revolution


generated by the region bounded

by the curve y = f (x) , above


the x-axis, between x = a and

l:!.x

Fig. I

x = b is given by:
y
V =

ny

dx

(Fig. 1)

2.

The volume of revolution generated

l:!.y

---------

f(y)

by the region bounded by the


curve x

= f (y),

between y

and

y = b is given by:

nx

dy

(Fig. 2)

=a

Fig. 2

In many cases, it is necessary to calculate volumes whose boundaries are not surf.1ces of
revolution and hence the two formulas stated above can not be used. For example we
cannot find the volume of a pyramid or a doughnut shaped solid by these two formulas.

Fig. 3: Pyramid

Fig. 4-: Doughnut

89

In what follows, the formulas for volumes would be derived intuitively by means of
simple examples.

The volume of a solid of uniform


cross-section A and height h is
h

given by
V =A. h

Fig. 5
If h is very smaJJ, we have an

element of volume given by


ll V =

where

A llz
llz

Let us calculate the volume of the


solid (pyramid) shown in the
figure.

We slice

the \l.ho!t'

pyramid by

n planes para1Jel to the base of


pyramid.
Let the distance between two
successive planes be ll z.
We find the area A(z) of one of
the cross-sections at a distance z

Fig. 7

from the vertex V. Quite clearly


A (z) is a function of

We express

z.

A(z) in terms of the sides

a and b of the base. From the two similar

triangles shown in the diagram, we have:


X

Similarly

i.e.

ahz]
y

bz

(l)

90

A (z)

area of the rectangular cross-section at P


X

y
zb

za
abz

[from relation (I) ]

h
2

The volume of the pyramid is


V = f. !:J. V = f. A (z) !:J.z

As n -+ ""

f. 6. V -+- V

and

f. A (z)!:J.z =

A(z)dz

h A (z)dz
0

(I)

Now from solid geometry, we know that the volume of the pyramid is given by
V:

B.h

where B = area of the base = ab


The formula V

~ Jb A(z)dz

a
stattment can be mJ.ck:

used above is quite general and the following general

(see diagram above)

The volume of a solid whose cross-sectional ~rea is a continuous function A (z), is


given by:
V

Ib

A (z)dz

where

plane).

The limits of integration are chosen to include the entire volume. To evaluate

is the distance of the cross-section from the pre-determined point (or a

V, we must express A (z) and dz in terms of a single variable.

91
We note that the general formula for
V =

the volume

Jab A (z) dz

includes
y

the volumes of revolution given by

2
ny dx

In the diagram:
Z

::

A(z)

A(x)

area of the disc at a distance


x from the origin

n/
v

Hence

A(z)dz

Fig. 8

ny dx.

a
Note that the volume of revolution given by the last formula is also known as a DISC
d

method.
An Important Method

Finding lengths from the given diagram is of particular interest in calculating the
required volume.

An elegant method is presented below and you will be asked to

derive the same result by using similar triangles.


Example: (I)

Find the length y in terms of h, from


the diagram which shows an isosceles
trapezium;

r--- ~~-I~ ~--~y~----~

L----~~

Solution:

12

We observe that:

Fig. 9

when h

12

when h

Since y is always proportional to h,


y is a linear function of h

and

T
5

mh+b

Substituting the given values


b =4

m =

----rT
I

f - - - - ' - y - - - \ - - -,'

j_

I
I

12

Fig. 10

92

Example: (2)
Using the intercept properties of parallel lines, prove the above result. (Fig. I 0)

Examp!e: (3)
Find the volume of the
block

shown in Fig. 3.

Solution:
We consider the

block to be made up

of horizontal laminas of length 1,


width w and thickness dh

l
.5
= 1 <-r
.5

Hence, V

wldh

12

7h

r:

+ 3)

2~

4 [14h
2.5

(14h
3

---:r

8h

<-r

+ 4) dh

w = 7h
T
=

... 6.5h ... 7.5) dh


2

6.5h
+ -2- ... 7.5h

].5
0

= 283 to 3 significant figures


The students familiar with the prismoidal formula

v =

...

can verify the result.

6 (A B 4M)

10

( 10 ... 3) (124)
2
2

= 52

i (12

4)

v =

X
X

12

12

120

+ 120 ... 208

= 1133.33

8h

Fig. 11

...

93
Example: (If)
The base of a certain solid is the
ClrC1eX 2
0

+ y2

= 4

Each plane section of this solid cut


out by a plane perpendicular to the
y-axis is an equilateral. triangle with
one side in the base of the solid.
Find the volume.
Solution:
A typical slice is an equilateral
triangle PBC.
BC
A

Fig. 12
2x

Area of !1PCB

= ~ PC PB sin 60"
,fJ

= 2 .2x.2x.T
=

,flx

I
=I

v =

b A(x) dy
a
2
13x dy

I2 ~(If- y 2)dy
-2

2 Io2 Vl(lf - y2) dy

2/i [4y-

~f
3 0

+y

:If

The limits are the ends


of the diameter.
(Owing to symmetry)

94

Example: (.5)

Find the volumes generated when the areas bounded by the given curves and lines are
rotated about the x-axis.
(a)

(b)

cosx, '

(c)

2x- x

I '

2'

0'

Solutions:

Since the rotation is about the


the formula V =
(a)

y =

Ja

x-axi~,

we use

n y dx

p (2, 3)

X+

:. The limits are a = - I, b = 2


V = n I2 y 2 dx = n 12 (x 2 + 2x + I) dx
-1
I

11

-1

rt\3 2 ]2
+X

+X

Fig. 13

-J

9n

Verify: V

(b)

2
nr h
volume of a cone = - 3

cosx , a = 0 , b = n

2
I
cos x = (I + cos2x)

~n

11

9n

J;

(I + cos2x) dx

n [x + I sin 2x]

11
X

95

(c)

= 2x - x

y = 0

Solve these: x = 0

11

J0

(4x

11[ 4x3

- 4x

- x4 +

3
1611

x =2

or

+ x ) dx

x5] 2
5

F1g. l 5

TI
Example: (6)
Find the volume generated when the area bounded by the curve y = x
x

=0

(a)

, y = 2 and

is rotated
(b)

about the y-axis

about the line y = 2

Solution:
(a)

A typical strip is shown and


it will generate a volume

2
ll.V = 11X fl.y

JO2 11X 2 dy ,

= 11

y dy

11

2
X =y

[~1:

211

Fig. 16

96

(b)

The typical strip has a radius

= y1 -

=2

- x

The volume generated when the


region OABO is rotated about the

=2

line y

, is given by:

!22r dx

=1r J

liJ./202

(2 - x 2 ) 2 dx

li (4

=1r J

- 4x

=
Note:

1T

x5]fi0

r4x - 4x3 +
3
5

l'

32Jin
15

The rotation is not about the x-axis, hence we can not use
V

f
a

n (y

+ x ) dx

2
2
- y ) dx
1
2

yl

=2

y2

=X

2
Fig. 17

97

Exercise 3A
For Exercises 1-10 find the volumes of revolution when the areas bounded by the given
curves and lines are rotated about the x-axis.

rx,

1.

2.

y=x

=0

+1,

= 0,

-x,

=0

=2
X =2
X

J.

y=x

4.

y = sinx ,

5.

y = cosx

6.

y=tanx,

7.

y=x

8.

y = sinx cosx,

y = 0.

9.

y=logx,

=1 ,

=2

lO.

= 0,

=1

X= 0,

=n
n

x=2

-4,

= 21 (e x

-x

+ e

) ,

= 0,

=0

x=;;

For Exercises 11-15 find the volumes of revolution when the areas bounded by the given
curves and Jines are rotated about the y-axis.

11.

x+y=4,

12.

y =

JJ.

X = 4- y

14.

y = Jogex,

15.

y = sm

rx.
2

. -1
X '

y = 0.

y = 2

X = 0,

X= 4

X= 0
X= J

X = 0,

X = 2
X

= 1

(Hint

= eY)

(Hint x = siny)

98

16.

The base of a certain solid is the circle x

+ y

= 9.

If each plane section of

the solid cut by a plane perpendicular to the x-axis is


(a) a square with one edge in the base of the solid,

find the volume of the

solid.
(b) an equilateral triangle with one side in the base of the solid,

find the

volume.
(c) a semi-circle with its diameter in the base of the solid,

find the volume.

(d) an isosceles right-angled triangle with the shorter side in the base,

find

the volume.
(e) an isosceles right-angled triangle with its hypotenuse in the base,

find the

volume.

17.

+ ~ =
2
a
b

Find the volume of revolution when the region inside the ellipse x
is rotated around the x-axis.

18.

A rugby ball has a volume that is the same as the volume generated by

rotating the region inside the ellipse ~

~ = l

about the x-axis.

Find the volume of the ball.

19.

A solid has a base in the shape of an ellipse whose major axis is 12 units and

minor axis 8 units.

If each section perpendicular to the major axis is an

isosceles triangle with altitude 12 units, show that the volume of the solid is
14411.
20.

The base of a solid is the circle x


dicular to the

+ /

= 8x and every plane section perpen-

x-axis is a rectangle whose height is one third of the distance

of the plane of the section from the origin. Show that the volume of the solid
is

6411

99

Volumes: Shell Method

3.2

Section

In

3.1,

the volume of revolution was found by rotating the rectangular

strips perpendicular to the axis of revolution. The elementary volume was a disc.
If the rectangular strip is parallel to the axis of revolution, a cylindrical shell is
generated. A shell is a solid contained between two parallel concentric surfaces.
To find the element of volume contained in a shell of inner radius, r = x and outer
radius, R

=x

n (R

AV

n y (x

+ 8x , length

- r )y

+ 2xAx + Ax

2 n xy Ax + n y Ax

As llx is very small, (llx)


Hence

AV

y, we have

2
- x )
2
is negligible.

2nxy . Ax

Fig. 18

To find the volume when a region bounded b


the curve y = f (x) , a ~ x ~ b ,
and

the x-axis, is rotated about the

y-axis, we divide the volume into thin


cylindrical shells. The cross-section of
one typical shell is shown in the diagram.
The element of volume is given by
AV
V

2nxy Ax

= Jb 2nxydx,
a

where limits are chosen to include the

Ax

entire volume.

Fig. 19

100

Example: (1)
The region bounded by the parabola
y

= x2 ,

the x-axis and the line x

=2
y

is rotated about the y-axis. Find the


resulting volume by using the shell
method.

A(2, 4)

Solution:

The volume swept out by the typical


strip (height y, breadth Ax) at a
p

distance x from 0 , is given by


AV

2 nx y Ax

The limits of integration are a = 0 to b = 2

The required volume is

v =

Fig. 20

211 xy dx

8ll

=X

2nx 3 dx

The method is very general and each example requires a good deal of pre-planning and
modification, so let us make it general.
Element of volume

AV

Inner circumference x Height x Thickness

Four usual cases are summarised below.


In each case the region R is rotated about a specified axis.

101

(a)

y =

b
a 2 n x y dx

gives the volume generated when


the region R

bounded by

y = f(x) , x =a, X= b

y
y =0

is rotated about the y-axis.

X
0

Ax

b
Fig. 21

(b)

y
2ny. x. dy

gives volume when the region


bounded by the curve X = f(y)

-----

Ay
X

lines Y =a' y = b' X= 0


is rotated about the x-axis.

Fig. 22

(c)

The region between two curves


y

= f(x) and Y2 = g(x) is

y2

The volume is given by

vy

~
I

rotated about the y-axis.

2nx(y 1 - y )dx
2
0

Fig. 23
(d)

The region R bounded by the


curve y = f(x) ,

a~ x ~ b , and

above the x-axis is rotated about

the line x = c , where c

>b

the volume of revolution is given


b

by:

V =

Ja

2n (c - x) y dx

Ax

Fig. 24

102

Example: (2)
Find the volume of the solid formed by rotating the circle x

+ y

= 4 about the

line x = 4

Solution:
y
x
r=4-x
-------

-2

H
t:.x

Fig. 25
X

Volume of cylindrical shell at P (x, y)


t:.V

2nrh t.x

4-

2y

(PA)

(PQ)

Using the shell method for finding the


volume, we have:

V =

2nrh dx

-2
2

nJ-2

4TI

(4 - x) . 2 .

~ dx

1-22 (4 - x) .;::7- dx
2

4nJ 41:7
-2

dx-

-2

Fig. 26

4Tir
-2

~ dx

The second integral is zero, since the function f(x) = x


(See Fig. 26) For the first integral, let x = 2 sin 9

/:'"7 is an odd function.

103
I
2
1/4- 4sin S (2cosSdS)

n/2

16n

f
f

-TT/2
2
64n n/ cos 2 e dS
-TT/2
n/2
(I + cos2S)dS
32n
-n/~
n/2
32nJS + 2 sin 29] -n/ 2
32TT

3.3

Volumes: Washer Method

A washer is a small hollow cylinder


(with small height). If the radii of two
concentric circles are R and r, and the

I !J.y

height of the washer !J.y ' then the


volume of the washer is
!J. V

= 11 (R 2 -

r ) !J.y

Fig. 27
y

Example: (I)

Use the washer method of finding the


volume generated when the region
bounded by the parabola y = x
and the lines x

=2

and y = 0

is

rotated about the y-axis.


Solution:

A typical strip at P (x, y) in the

given region will sweep out a washer


of volume !J.V

= TT

2
2
(R - r ) !J.y

=n

2
(4 - x ) !J.y

:. Required volume V

11

2
(4 - x ) dy

=(

11

(4- y)dy'

= 11

4y -

[r=x,

J0

11

~]:

[16 - 8)
811

R=2)
Fig. 28

= X2

104

Exercise 38

In Exercises 1-.5, find the volumes generated when the region bounded by the given
curves and lines is rotated about the x-axis.
1.

2.

y =X

y =X

+ 1

4.

+ 2'
2
y =1- X
'
2
X = 3y- Y

.5.

y=x+2,

3.

y = .5

'

y =X+ 4
y = 1- X

'

X=0
y = 4'

X= 0

In Exercises 6-10, find the volumes when the region bounded by the given curves and
lines is rotated about the y-axis.
6.
7.

y = X'
2
y =X

8.

y =X

'
'

X = 2'

y =0

X = 2'

y =0
y =0

X = 2'
y = 3x- x

9.

y =X'

10.

(Hint:

Jxsinx dx , integrate by parts)

11.

The region bounded by y = -1 , y = e

= sinx

y = 0'

0~ X~

1T

2x

, x = 0 and x = 2 is revolved about

the line y = -1 Find the volume of the resulting solid.


12.

2
2
A circle of radius 2 is given by x + y = 4 and is revolved about the line
x = .5. Using the washer method, prove that the resulting volume (called a
TORUS) is 40n 2
(Hint: Simplify your work by remembering that
2

J-2 Q

dx = area of a semi-circle = 2n )

105

3.4

Worked Examples (Miscellaneous)

EXAMPLE: (1)

The triangle with vertices

A (2,2),

B (2,4)

and

C (4,4)

is rotated

about (i) the x-axis, (ii) the y-axis, (iii) the line x = 8.
Find the volume generated in each case.

y
I

SOLUTION

4
(i)

Equation of

AC

is

y =x

Equation of

BC

is

y = 4

The element of volume, l:i V = 11 (y

=4

=x
1

2 - y 2 ) /:ix
2
1

and

f4
2
2
2
- y ) dx = 11
(16 - x ) dx
1
2
2

fa

11 (y

v = 11

~6x _ x: J~

(ii)

V =

11 (x

- x ) dy ,
1

6
= 11 [ ;

4J ~

16 - (

II Y
I
I

- 8)]

T
r

Fig. 30

= PR = 8 - 2 = 6, r 1
4

V =

J2 11 (r 2
4

f2
= 11

_..
X

3211

(iii)

Qv:p

/:iy

4) dy

= 11[

/:ix

Fig. 29

4~ 11

r</ _

= 11

---------

2
Substitute in the formula

V =

- r

2
) dy
1

(y

= x)

4
(-28 + 16y _

6411
-3-

l> dy

11 [36 - (8 - y) ] dy

= 11

= QR = 8 -- x = 8 -

3 4

28y + 8/ -

La.

8 X
Fig. 31

106

EXAMPLE (2)
The area bounded by the curve y = x

+ 2 and the line

y = 2x + 5 is revolved about

the x-axis.
Find the volume.

SOLUTION:

=X

= 2x

+ 2

(l)

(2)

Solve (l) and (2) for the intersections.


x 2 + 2 = 2x + 5
2
x - 2x - 3 = 0

=0

(x - 3) (x + l)

x = 3 or x = -1
The element of volume when the region ACBA is
rotated about the x-axis is given by:
2
2
I!.V = n (PM - QM ) l!.x

(washer)

Fig. 32

= n (y 2 - Yl ) l!.x

We substitute y

=x

1!. V = n [ (2x + 5)
= n (4x

y 2 = 2x + 5

+ 2,

- (x

+ 2) ]

+ 20x + 25 - x

= n (20x - x

f
=f

:. Volume V =

n(y

- 4x

- 4)

+ 21)

2
2
- y ) dx
1
2

3
n (20x - x

+ 21) dx

-1

= n [lOx

x5
5

3
1

+ 2lx]_

li 90 - T
243 + 63) - (lO + "5l
= n~
21)

576n
-5-

107

EXAMPLE: (3)

The cross-sections of a certain solid by planes perpendicular to the


2
x-axis are circles extending from the curve y = x to the curve
2
y = 2 - x The solid lies between the points of intersection of
these curves. Find the volume of this solid.

SOLUTION:
y

The curves y = 2 and

... (1)

2
y =X

... (2)

intersect at A (-1, 1) and B (I, 1).


The element of volume at distance x from
the origin is given by:
ll V =

where r

11

(disc, radius r)

/lx
I

PQ =

(PM - QM) =

(y 2 - y 1 )

PM
QM
!:N

Fig. 33
2

11

11

/lx

(y 2 - YI)

/lx
y 2 - Yt

11

J(

1
(1 - 2x

= 2- X 2 -X 2 = 2(1

+ x ) dx

-1

211

1611

15

(1 - 2x

+ x ) dx

-X )

(owing to the symmetry)

108
2
The region bounded by curve y = 2x - x and the line y = x is rotated

EXAMPLE: (4)

about the y-axis. Find the volume by the method of cylindrical shells.
y

SOLUTION:
The curve y

=2x

the line

=X

We use:

- x

and
intersect at 0 (0,0) and A (1, 1)

2n

x(y2- yl) dx

(1)

PR = 2x - x

f
f

:. V

2n

2n

x (2x - x

(x

Fig. 34

= QR = x

- x) dx
.

3
- x ) dx

x3 _ x4] 1

2n [ 3

2n

(j- i)

l!

6
EXAMPLE: (5)

2
The area bounded by the curve y = x
and the line x = 1 is revolved about
the line x = 3. Find the volume

----f;; _

_ _ _ _ _ _ _g_e_ne_r_a_t_ed_,_b_y_t_he_m_e_t_h_o_d_o_f_s_h_e_ll_s_.

J~: !L __

SOLUTION
3

V = b 2n (c - x) h dx

a
=

= 3,

h = PQ = 2y = 2yx

4n (3 - x)

41!

.Jx dx

(3xl/2 - x3/2) dx
0
4 n [ 2 x3/2 _ ! x5/2] 1
5
0
2
41! (2 - 5)

32n
)

Fig. 35

109

REVISION

1.

Exercise 3C

VOLUMES

(a) On a number plane, shade the region R representing the inequality


(x - 4)

(b) Show that the volume of a right cylindrical shell of height H with inner
and outer radii r and r + I:J.r respectively is given by 2n r H I:J.r
(c) The region R in (a) above is rotated about the y-axis, generating a solid of

revolution called a

torus.

volume of this torus is 32n


2.

By using the shell method, prove that the


2

A solid figure has a semi-circular base of radius 4 units. The cross-sections at


right angles to the semi-diameter of this base are semi-ellipses.

If the semi-

minor axis of each ellipse is } of the semi-major axis, prove that the volume
of the solid is given by V

=41T

f0

2
(16 - x ) dx and hence find the volume of

the solid
3.

The area common to the curves y = x

and y

= x is revolved about the

y-axis. Find the volume generated by using:


(a) the disc method (washer)
(b) the cylindrical shells.
4.

The co-ordinates of the vertices of a triangle PQR are (0, 4) , (2, 2) and
(-2, 2) respectively. The region PQR is rotated about the
(a) x-axis

(b) y-axis

(c) line x = 4

Find the volume of revolution generated in each case.

5.

A certain solid has a semi-circular base of radius 4 units.

The cross-sections

at right angles to the x-axis are triangles with one leg in the base.

If the

heights of these triangles are bounded by the arc of the parabola y

= 16

show that the volume of the solid is given by:


V

J0

(16 - x 2) 3/2 dx and hence find V

2
- x ,

110

6.

An electronic valve used in a computer is in the shape of a solid that has a


circular base.

At a height z above the horizontal base, the horizontal cross-

section is a circle of radius R given by the relation


R

f(z)

ih

If the height of the solid is h, find the volume of this solid.

7.

A frustum of a right circular cone


is shown in the diagram.
R- r

(a) Prove that x = r + (-h-)z


h

(b) Prove that the volume of the


frustum is given by
V =

nh

T (R

+ Rr + r )

Fig. 36
8.

A very large waste container is shown in the diagram.


The top face of the container is a rectangle of sides 3 m and 4 m
respectively and the bottom is a rectangle of sides 2m and 3m
respectively. If the height of the container is 1.5 m, find the volume of this
container by integration.

).5

I
I
I

I
I

I]- - - -

~----~----'

Fig. 37

111

9.

The area bounded by the parabola

= 4ax

and the line

x = a

is rotated

about the line x = a. Find the volume generated, by using the Shell method.
10.

II

Find the volume of the torus obtained by rotating the area bounded by the
2
2
2
circle x + y = a
about the line x = c, (c > a). Use the method of (a)
cylindrical shells and (b) disc (washer method).
2
The area bounded by the parabola y = 4ax and the line

=a

is rotated

about the y-axis. Find the volume by two methods.


I 2.

A ring of altitude 2h is generated by revolving


about the y-axis the area of the segment bounded
2
2
by the circle x + / = a
and the chord of

length 2h that is parallel to the y-axis.

13.

By using the method of shells, show that the


. .
4nh 3
Fig. 38
volume IS g1ven by - 3
x2
2
The area enclosed by the ellipse
+
= I is rotated about the line x = 8.
25
By using the method of shells, find the volume generated.

14.

The ellipse in exercise (13) is revolved about the vertical line through

15.

The triangle

the vertex A (5,0). Find the volume.


ABC

formed by the points

A (a,O), B (-a,O), C (O,a) is revolved

about the line x = 2a. Find the volume generated by disc (washer) method.
16.

The cross sections of a certain solid by planes perpendicular to the x-axis are
2
circles with diameters extending from the curve
y = x
to the curve
2

y = 8 - x

The solid lies between the points of intersection of these two

curves. Find the volume of this solid.


2
[Hint: radius r is given by 2r = (8 - x ) 2
where A = ll r ]
17.

The area bounded by the curve y = x


about the x-axis.

+ I

=8

- 2x

and V =

2
A (x) dx,

-2

and the line y

Find the volume of revolution.

=3 -

x is revolved

(Hint: use the washer

method)
18.

The region bounded by the curves y = 3x -

and y = x is rotated about the

y-axis. Find the volume by the shell method.


19.

The triangle with vertices

(a, a), (a, 2a), (2a, 2a) is rotated (a) about the x-axis

20.

The area bounded by the curve /

(b) about the y-axis. Find the volume generated in each case.
= 4x

and the line x = I is revolved about

the line x = 2. Find the volume generated.

112

CHAPTER 4 COMPLEX NUMBERS


4.1

Introduction
- Necessity is the mother of invention.

Invented Number Systems: Complex Numbers


Imagination and art of invention were required several times in extending our number
system from the counting numbers.

1.

The first invented system J: the set of all integers as developed from the
counting numbers. In this system we can solve equations such as x .:!:. 2 = O,
in general x + b = 0, where b is any integer.

2.

The second invented system Q: the set of all rational numbers p/q as
developed from the integers. In this system we can solve equations such as
2x - 3 = 0, in general ax + b = O, where a and b are rational numbers.

3.

The th1rd invented system R: the set of all real numbers x as developed
from the rational numbers. In this system, we can solve not only the types
2
x + a = 0, ax + b = 0, but in addition all quadratic equations ax + bx + c = 0,
2
a ~ 0 and t:. = b - 4ac ~ 0. The roots are not real if 1:!. < 0. The simple
2
2
quadratic equation x + I = 0 or x + x + I = 0 is impossible to solve with
the above mentioned three number systems. There is no real number that
2
satisfies the equation x + I = 0, since x = r-f does not exist in the real
number system R.

A new kind of number has to be invented to handle the roots which are not real. The
[ i =

symbol

;-:II

is used with the understanding that

= -I

i is called

2
the imaginary number. The roots of x = -1 can now be written as
2
2
x = -1 = i
-> x =- .:!:. i
Again the roots of x 2 + 2x + 3 = 0 can be given as
X= -

.:!:_

f8

-1

.:!:_

= -1.:!:.

Iii

Now we come to the fourth invented system, the set of all complex numbers of the
form

We shall use a single pronumeral z to define a complex number

x + iy, i.e.
X+

iy

113

x is called the real part and


y is called the imaginary part of the complex number z.
It is important to note that the imaginary part of a complex number is not imaginary!
It is i that is called the imaginary number.
The history of imaginary numbers is very fascinating. The earlier mathematicians
thought that such numbers had no practical use (hence the term 'imaginary'), yet
complex numbers are of great importance in fields such as Electronics.
A complex number can also be defined by an ordered pair of numbers (x, y), without
ever mentioning the imaginary number i, but in this book we shall only work with
the binary form z = x + iy. However, the computer requires the form (x, y) for
multiplication of two complex numbers.

4.2

Operations with Complex Numbers

We have the following definitions:


a + bi

c + di

if and only if a

=c

1.

Equality:

2..

Addition:

(a + bi) + (c + di)

3.

Multiplication:

(a + bi) (c + di) = (ac - bd) + (ad + be) i

and b

= d.

(a +c) + (b + d) i

By using these definitions, we can verify that the complex numbers satisfy all the
laws of algebra and hence the complex numbers form a field (denoted by C).

3 - 5i, then x

= 3,

If x + iy

EXAMPLE: (2)

Find the sum of 2 + 3i and 3 + 2i

SOLUTION:

(2 + 3 i) + (3 + 2 i) = (2 + 3) i

EXAMPLE: (3)

Find the product of 2 + 5i and 3 + 6i

SOLUTION:

(2 + 5i) (3 + 6i)

= -5

EXAMPLE: (l)

5 + 5i

(2x3- 5x6) + (2x6 + 5x3)i

=-24

27i

If you ignore the definition of multiplication and expand (2 + 5 i) (3 + 6i), i.e.

6 + 15i + 12i + 30i

and put

= -1,

then the result is

-24 + 27i, and so you

now discover the secret of strange definitions!

Identity elements:
By definition:
and

(a + bi) + (0 + Oi)

(a + 0) + (b + O)i

a + bi

(a + b i) (1 + 0 i) = (a 1 - b 0) + (a 0 + b 1) i = a + b i

Thus, the complex number 0 + 0. i, written as z = 0, is the additive identity and


1 + 0. i written as z = 1, is the multiplicative identity for the set C of complex
numbers.

114

Additive inverse:
We define the additive inverse of the complex number a + ib to be a number x + iy
such that:
(a + bi) + (x + iy) = 0 + 0. i
(a + x) + (b + y) i = 0 + 0 i
By definition of two equal complex numbers, we have:
a + x = 0

and

b +y =0

giving

x = -a, y = -b

Hence the additive inverse of z = a + bi


now find the difference: (subtraction)

-a - bi, i.e. -z. Using this, we can

is

(a+ bi)- (c + id) =(a+ bi) + (-c- di) =(a- c)+ (b- d)i.
Multiplicative inverse:
The multiplicative inverse of the complex number a + bi f. 0 is defined to be a
number x + iy such that (a + bi) (x + i y) = l + 0. i. By definition of equality,
and

ax - by = l
bx + ay = 0

We solve these equations for x and y, then


a
b
and y = - - - - , hence the multiplicative inverse of z = a + bi
2
2
2
2
a +b
a +b
a
-b
_1
is the complex number z-1 = [ ~
~
Note that z. z-1 = z I = I, z"
0.
a +b
a +b
We can now divide a complex number z by aflother complex number w, but shall
use a method used in ex. (6} .below, i.e. complex conjugates. We can easily verify that
the complex numbers obey all the rules of Algebra and hence they form a complex
field C. Further, if b = 0, then a + bi reduces to the real number a, hence the
set of real numbers R is a subset of the complex numbers C. If a = 0, b f. 0, then
a + bi reduces to bi, and we call bi a purely imaginary number.
x =

z:

Note that a real number can be written in the form a + bi, for example 2 = 2 + 0. i.
Similarly a purely imaginary number such as 2i can also be written as 0 + 2i.
Calculations with the complex numbers do not require any special rules; wherever i
occurs we replace it by -1. Further,
4
2
4
5
3
2
i = -i, i = i x i = (-1) (-I} = 1, i = i i = i
etc.

Complex conjugates:
If two complex numbers differ only in the sign of their imaginary parts, each is
called the conjugate of the other. Thus a + b i and a - i b are the conjugate
complex numbers. Notation
is used for the conjugate of z, i.e.
= a - ib.
Since z + z = (a + bi) + (a - bi) = 2a, the sum of two conjugate complex numbers is
a real number.

EXAMPLE: (4)

Find the sum and difference of

SOLUTION:

(3 + 2 i) + (3 - 2 i) = 6

and

3 + 2 i and

3 - 2i

(3 + 2 i) - (3 - 2 i) = 4 i

115

zz

2
2
Again the product
= (a + bi) (a - bi) = a + b , hence the product of two
conjugate complex numbers is a non-negative real number.
(3 + 2 i) (3 - 2 i) = 9 + 4 = I 3

EXAMPLE: (5)
QUOTIENT:

a+ bi .
We can simplify the quotient c + di , I.e. divide a + bi by c + di by using
a+ bi
the following procedure: c + di

(ac + bd) + (be - ad) i


c2 + d2

c- di
c- di

ac+bd
be-ad.
-2--2 + -2--2 I
c +d
c +d
EXAMPLE: (6)

Divide 3 + 4i by 2 + i
3 + 4i

SOLUTION:

3 + 4i

6 + 8i - 3i - 4i
4 + I

(3 + 4 i)
(2 - i)
= (2 + i) "(2:1}

2+1

10 + 5i
--5-

2+1

2 + i

EXAMPLE: (7)

If

SOLUTION:

.
.
5 + 6i
5 + 6i
2 - 3i
28 - 3i
Wnte x + 1Y = 2 + 3i = 2 + 3i 2- 3i = --13-

(x + iy) (2 + 3i) = 5 + 6i, find x and y.

28
.
I d .
.
Equatmg rea an 1magmary parts, x = TI

y =-

TI3

Alternatively, expanding and equating real and imaginary parts, we find:


and

5]

2x - 3y =
3x + 2y = 6 '

EXAMPLE: (8)

28
x=n'

solving these:

Expand (a) (I + i)
a+ ib.

3
y=-n

and (b) (I - i)

and simplify in the form

SOLUTION:

(a)

(I + i)

+ 4i + 6i

+ 4i

+ i

+ 4i - 6 - 4i +

i.

= -4
and

(b)

(I - i)

5
4
2
3
- 5i + !Oi - !Oi + 5i - i

- 5i - 10 + !Di + 5 - i
=

It is easier to expand by
writing z = I + i
2
z4 =
z 2 = (I + i) 2 (I + i)
= (2i) (2i) = -4
5
2 2
. . I
S1m11ar y z = z z z
=(-2i}(-2i)0-i)
= -4 (I - i) etc.

-4 + 4i

EXAMPLE: (9)

Express (2 - 3if

SOLUTION:

<2 - 31

.)-1

in the form a + ib

I
= ~

2 + 3i
2 + 3i
2
3 .
2 + 3i = -13- = T3 + TI I

116

Exercise 4A
Perform the indicated operations and express the answers in the form
2.

(5

2i)

(3

2i)

4.

(3 + 2i)

(3

2i)

6.

(5

i) (3

8.

(4

3i) 2

l.

(3 + 2i) + (2 -

3.

(-3 - 4i) - (12 -

5.

(3

7.

3 i (2 -

9.

(l

11.

i (2

13.

r:-i

2i
14. 3 +
5 + 2i

15.

3 - 2i
5i

16.

3i)
5i)

4i) (2 + i)

i)

i )2

10. (l
(2

+ i)

4i)

12. (-4 i) (2 i)

i)

3if

a + ib:

+ 2i

.3

Find x and y in each of the following:


17.

3x +

2iy
iy)

12 + 5i

18. (2 -

3i) + (x + 2iy)

2i

20. (x + i y) ( 3 + 4 i) = 2

19.

(x

21.

Expand: (2 + i) 3 and answer in the form a + ib.

22.

If x + i y

23.

If

5- 4i

5i

= 5 (cos 60 - i sin 60), find x and y in the surd form and hence
2
1
express (a) (x + iy >
in the form a + i b
(b) - -.X- IY

(a)

24.

=2

+ i,

3z + 4

evaluate:
2
(b) z - 2z + 3

(c)

2z - l
2z + l

(d)

(z - 1) (z

2 + z + 1)

If z = x + iy, express each of the following in the form a + ib:


(a)

25.

-z

(b)

-z

(c)

Solve the following equations for


(a)

(l + i)z

(c)

2 - i

2
l + i +
z =

r:l

z + l
Z-1

(d)

z; express answers in the form

a + ib:

(b)

2z
(I - i) z
T+i + 3- 2i =

(d)

Z-1

z + 3

2 - 3i

117

26.

Solve the following equations for


(a) z 2 + z + I = 0

27.

28.

(b) z

z; express answers in the form a + i b:

- 2z + 4 = 0

(c)

2z

- 3z + 2 = 0

(d) z + ..!.

Find the quadratic equations with wots given below:


(a)

i, -i

(d)

3 + i,

(b)

+ i,

+ 3i

(c)

I - i

2 + 3i, 2- 3i

2 + i, 2 + i

(e)

Solve the following pairs of equations for z and w where z and w are
complex numbers. Express answers in the form a + ib.
(a)

(c)

z + iw

2 + 3i

z- iw

2 - 3i

(b)

(d)

(2 + i)z + (2 - i)w
(2 - i) z + (2 + i) w

2z + w

I +i

z-w

I - i

z +(I - i) w

2i

w + (I - i) z

29.

Given z = 2 + i, evaluate the following in the form a+ ib:


(f) z 4
(e) z 3
I
I
(c)
(b) z 2
(d) z 2 + 2
(a)
2
z
z
z
2 z2)
2
4
3
z and
z
(Hint: z
z
= z

30.

What is thqfallacy in the following:

V-3 V-i2

1<-3> . <-12)

6 ?

What is the correct answer?


31.

Prove the associative law for multiplication of complex numbers:


(z

32.

. z )z
= z (z . z )
2 3
1 2
3

(Hint: Let z

Prove the commutative law for multiplication:

= x

z z =z z
1 2
2 1

+ iy

etc.)

118

4.3

Complex Plane (Argand Diagram)

A complex number is an ordered pair (x, y) of real numbers, so if we consider (x, y)


to be the cartesian coordinates of a point in a rectangular coordinate system, every
complex number (x, y) then corresponds to some point in the coordinate plane.
Conversely, every point (x, y) corresponds to some complex number x + iy. Thus
there is a one-to-one correspondence between the set of complex numbers and the set
of points in the coordinate plane. The plane is called the COMPLEX PLANE or the
Argand diagram (J. Argand: 1768-1822).

P(x,y)

=x

+ iy

=z

2 t - - - - - - - - , P(3,2) = 3 +2i
y

Fig. 2

Fig. l

z = 3 + 2i is represented by the point P(3,2) in the


The complex number
coordinate plane (Fig. l). The x-axis is called the real axis and the y-axis is called
the imaginary axis. For z = x + i y ,
x = real part of z = Re(z)
y = imaginary part of z = lm(z)
In the above example: Re(z) = 3,

lm(z) = 2,

z = 3 + 2i.

TRIGONOMETRIC FORM OF COMPLEX NUMBERS

A vector is a directed line-segment. We can represent the complex number


z = x + iy by the vector drawn from the origin to the point P(x,y) (see Fig. 2).
The length of the vect0r OP is given by:
r
r
r

OP

is called the modulus of the complex number z = x + iy.


is denoted by lz 1 , so modulus of z is given by:
lz I

(x2:l

The direction of the vector OP is given by the angle 6 which OP makes with the
positive direction of the x-axis. Hence 6 is completely determined by the equations:
x = rcos6

and

rsin6

119

The angle 9 is called an argument of the complex number z = x + iy. The angle 9
is measured in radians, but degrees may be used for convenience. Its value is,
however, not determined uniquely, since
cos9 = cos(9.!. 2kn)

sin9 = sin(9.!. 2kn)

and

where k is an integer. In order to determine 9


that
-n

<9~

uniquely, we impose the restriction

Thus for the complex number z = x + iy, we can write its trigonometric form as:
z = x

iy = r(cos9

isin9),

-n

<9 ~

n.

This is called the modulus-argument form of the complex number z. We abbreviate


this to mod-arg(z) for our convenience. Frequently to save the space, we may write:

For example

r(cos9 + isin9)

5(cos 60

as

rcis9

i sin 60) = 5cis 60.

It is strongly recommended that you draw a diagram depicting the complex number
z = x + i y to determine the angle 9. The formula tan 9 = Y should be used with
X

caution as 9 is defined for -n < 9 ~ n.


Note carefully that arg z is not defined for

z = 0.

We have:
Complex
number

Cartesian
form

Mod-arg form of z

+iy

P(x,y)

= rcos9,

z = r(cos9

y = rsin9

+isin9) -

modz = lz I = r =

rcis9

fx +i
2

arg z = 9
where -n

<9 ~

n
Fig. 3

120
EXAMPLES
(10) Express 3 + 4i in mod-arg form:
SOLUTION:

= /3

+ 4

(Fig. 4)

= 5

Fig.4

~ (8 in the first quadrant)

tan a

a = 53 (to the nearest degree)


:. z = 3 + 4i = 5cis53
X

1 + I in mod-arg form: (Fig. 5)

(11). Express
SOLUTION:

tanS

ff

P(J,I)

I
= 45

Fig.5

z = 1 + i = v2(cos45 + isin45)
cis(11/4)

( 12) Express -2 + 2 v'!i in mod-arg form: (Fig. 6)


2
SOLUTION: r = v1-2)z + (2 /3)
= 4
211
_,..
a = 3 from tan a = - v3
z = -2 + 2 v'3i = 4cis 211/3
(I 3)

Express -2 - 2 .l!i in mod-arg form:


2
2
SOLUTION: r = ~(-2) + (-2 .f3)
= 4
8 = -120 = -211/3 from tanS =

= 4cis

(~

11

P(-2,2 /3) . ,

y
Fig.6

2../3

a
-2

(Fig. 7)

X
X

'1/3

-9
-2 13
Fig.7

P(-2,-2 v'J)
(14) Express 10{3- 10i in mod-arg form: (Fig. 8)
2
2
SOLUTION: r = /( 10 {)) + (-1 o ) = 20
1
9 = -30 from tan9 = - - 10 VJ

Fig.8

-/T

10 = 20cis(-30) or 20cis(-11/6)
P(IO /3,-10)

(I 5)

Express in rr.od-arg form: (Fig. 9)


(a) 2
(b) 2i
(c) -2
(d) -2i
SOLUTION:
(a)
2=2+0.i=2cis0
(b)
2i = 0 + 2i = 2cis(11/2)
2cis11
(c) -2 = -2 + Oi
(d) -2i = 0 - 2i = 2cis (-11 /2)

2 y

-2

-2
(16) Express

(a) 2cis 150

(b)

5cis 21 oo

in Cartesian form:

..rJ

(a)

2cis 150

2 (cos 150 + i sin 150) = 2(--;_- + ~ l = -

(b)

5cis 210

5(cos210 + isin210) =

5 J3

5i

--2--2

13 +

121

Exercise 48

Convert the following to the mod-arg form:


I.

2.

-5

3.

5i

4.

-5i

5.

2 + 2i

6.

2 - 2i

7.

-2 + 2 i

8.

-2 - 2i

9.

-4

I 0. I 0 1J + I 0 i
13.

1+1

II .

-2

..13 .
2 1

IG
14 4 + 3i

12.

II + 4 l'ii
5

513.

-2- -2-1

15. (I - i)

Convert the following to the Cartesian form:


16.

5cis n/2

17.

IOcisO

18.

4cisll

19.

2cis (-n /2)

20.

5cis -n/3

21.

Vi" cis TI/3

22.

l2cis TI/4

23.

5 cis (-5 11/6)

24.

10 cis (-n I 3)

25.

4cis 211/3

122

4.4

Multiplication and Division of Complex Numbers Using Trigonometry

The mod-arg form of complex numbers can be used to advantage in finding products
and quotients. The mod-arg form is useful in finding the roots of complex numbers.
Let z
1
complex numbers, then:

MULTIPLICATION:

z z

= r 1 (cos9 1

+ isin9 ) and z

= r 2 (cos9 2

+ isin9 ) be any two

r r (cos e + i sinS ) (cos e + i sin9 )


2
1
2
1 2
1
r r [(cos9 cos9 - sin9 sin9 ) + i(sin9 cos9 + cose sin9 )]
2
12
1
2
1
1
2
1
2

1 2

r r [cos(9 + e ) + isin(9 + 9 )]
2
1
2
1 2
1

... (J)

Thus to multiply two complex numbers, multiply their moduli


the arguments, 9 and e .
1
2
We can easily generalise:
z z z
zn = r r r
cis(9 + e + e +
2
1 2 3
1 2 3
1
3
Letting z
=
z
=
z
=

= z = rcis9
2
1
3

+ 9n)

rn (cosn9 + isinn9)
DIVISION:

To find the quotient

1
2

and add

(2)

. (3)

r (cos e + i sin e )
1
1
1
r kose + isin9 )
2
2
2

Multiply and divide by the conjugate of cos9


z

and

+ isin9 , i.e. (cos9

- isin9 ), we have:
2

r (cos9 + isin9 ) (cos9 - isin9 )


1
1
1
2
2
r kos9 + isin9 ) kos9 - isin9 )
2
2
2
2
2

Thus the modulus of the quotient of two complex numbers is the quotient of their
respective moduli and the argument of the quotient is the argument of the numerator
minus the argument of the denominator.
From results (I) and (4), we immediately deduce that:

I=

Iz 1 I
= - - , but it is not always correct
r2
I2 2 I
to say that arg(z z ) = e + e , because the sum e + e
may be outside the
2
1
2
1
1 2
domain -11 < 9 ~ 11. It may be necessary to add .::.211 to bring the arg(z z ) or
1 2
arg(z/z ) into this domain. Hence argz z = argz + argz 2:. 211 (if necessary)
1 2
1
2
2
and arg(z /z ) = argz - argz 2:. 211 (if necessary).
2
1 2
1
l 2 12 2l = rlr2 = lz 1 llz 2 1 and

zl

l- 2
2

rI

123

EXAMPLE: (17)

Find the product: 2(cos60 + isin 60) 3 (cos 120 + i sin 120)

SOLUTION: Product= 6[cos(60 + 120) + isin(60 + 120) = 6(cosl80 + isinl80) = -6


EXAMPLE: (18)

Find the quotient:

1O(cos 105

i sin 105) .;- 2 (cos 4-5

i sin 4-5)

SOLUTION:
QUOTIENT

5[cos(l05- 45)

Quotient

512

isin(l05- 45)] = 5 (cos60

isin60)

(5 ljf2)i

+ (1

EXAMPLE: (19)

Find the quotient [-1/2- ( ./312)i]

SOLUTION:

Change each number to the mod-arg form, then:

- i) in mod-arg form.

1 (cos - 120 + isin - 120)


\li(cos - 4-5 + isin - 4-5)

QUOTIENT

(1/ {i) [cos(-120- -45) + isin(-120- -45)


(1 I {'1) (cos - 75 + i sin - 7 5)
(1 I

or

{1.> (cos 7 5 - i sin 7 5)

EXAMPLE: (20)

Solve for r and 9 if r(cos9 + isin9)

SOLUTION:

Numerator = N
N

and

2cis45 + 3cis30
4cis 60 - 3cis 30

Denominator = D

2(cos45 + isin45) + 3(cos30 + isin30)


2cos4-5 + 3cos30 + i(2sin45 + 3sin30)
4.01229

2.914-2 i

Mod-arg form of N = 4-.96 cis (35.99)


D = (4-cos60- 3cos30) + i(4sin60- 3sin30)
= -0.59808

i (1.964-10)

Mod-arg form of D = 2.05cis 106.94- 0


Hence

r(cos9 + isin9) =

4-.96cis 35.99/2.05 cisl06.94


2.4-2 cis (-70.95)

= 2.4-2

and

9 =- 70.95

124

Exercise 4C
Perform the indicated operation and express the result in the form a + i b.
1.

4(cos40 + isin40). 2(cos50 + isin50)

2.

5(cos120 + isinl20). 2(cos60 + isin60)

3.

4(cosl35 + isin135). 3(cos90 + isin90)

4.
6.

5(cos 2n/3 + isin 2n/3). 2(cos n/6 + isin n/6)


2
[2(cos n/3 + isin n/3)]
3
[3(cos n/6 + isin n/6)]

7.

12 (cos0 + isin0) -7 [6 (cos 120 + i sin 120)]

5.

8.

10(cos 2n/3 + isin 2n/3) + [5(cos n/6 + isin n /6]

9.

-10(cos 2n/3 + isin 2n/3)

+ [-5(cos -n/6 + isin -n/6)]


2n/3) + [2(cos- n/3 + isin- n/3)]

10.

6(cos- 2n/3 + isin-

II.

Prove that [r(cos9 + isin9)]

12.

Change each of the following complex numbers to mod-arg form and perform
the indicated operation, giving answers in mod-arg form.

( ) .!.._.!..l
a

13.

If

(b) (-1 + i)
V3 + i

I - i

= 4cisl20

= r

(cos39 + isin39)

(c) (I + i) (l +

v'3 i)

(d) (

V3

(l +

and

= 2cis30,

+ i) (l - i)
v'3 i) ( y'J - i)

find

2
10
and express your answers in
(c) (z > (d) (z )
1
2
the form a + ib.
14.

15.

= I + IJi and z = {J + i in mod-arg form and hence write


2
1
down the following in rcis9 form:
4
5
zl
5
(b) (z )
(a) (z )
(c) (z z )
(d)
(e)
1
2
1 2
z2
(b) z - z
in mod-arg form if
Express (a) z + z
2
1
1
2

Express

(c)

= I + ./3i '

zl + z2
z1 - z2

z =
and

./3

+ i

and hence find

(d)

~1zl -+ z2z~

2
in the form

A+ iB.

125

4.5

Powers of Complex Numbers: De Moivre's Theorem

For any real number x (x > 0), we know that xn = x . x x to n factors, if n is


a positive integer.
The corresponding result for any complex number z = a + ib = r(cos9 + isin9) is of
far-reaching importance and leads to a theorem known as De Moivre's theorem.

z = a + ib = r(cos9 + isin9), then by the product rule of two complex numbers


2
z 2 = r 2 (cos9 + isin9) (cos9 + isin9) = r (cos29 + isin29)
2
3
Then z 3 = z. /
= r(cos9 + isin9). r (cos29 + isin29) = r (cos39 + isin39)

Let

By successive applications of the product rule, we get:


z 4 = r 4 (cos 4 9 + i sin 49) and so on. These results can be summarised in the
following theorem:
If n is any positive integer (n

Theorem:

> 0),

then

[r(cos9 + isin9)]n = rn(cosn9 + isinn9)


For r = I, we obtain De Moivre's theorem:

(cose + isin 9)n = cosne + isinne

De Moivre's theorem can also be proved by the method of mathematical induction.


(This proof is given in a later section.) It also holds for n = 0 and for a negative
integer.
6

EXAMPLE: (21)

Find (I + i)

SOLUTION:

Write z = I + i in the mod-arg form.

z = ../2(cos Tl /4 + isin Tr/4)


:. z 6 = [ v'2 (cos Tr/4 + isin Tr/4) ] 6

6
( Vi.) (cos 6 Tr/4 + isin 6Tr/4)
8 (cos 3 TI/2 + i sin 3TI/2)
-8i

EXAMPLE: (22)

Evaluate in the form a + ib: (I + .fji)

SOLUTION:

We write

+ (-1

/3i)

z = I + vJi = 2(cos60 + isin60)


w = -1 + ni = 2(cos120 + isin120)

2 4 (cos60 + isin60)

Hence?
w

/ (cosl20 + isin120)

~~ ~os (240

16 (cos240 + isin240)

and

128(cos840 + isin840)
128 (cos 120 + i sin 120)

- 120) + i sin (240 - 120)]

(1 /8) (cos 120 + i sin 120),

i.e. -1/16 + i( '113/16)

126

Exercise 40

Evaluate in the form


I.

a + ib, by using De Moivre's theorem:

(cos 12 + i sin 12)

2.
4

3.

(cos nl4 + isin nl4)

5.

[cos(-nl4) + isin(-nl4)] 6

4.
6.

[2 (cos 15 + isin 15) ] 6


[2 (cos nl3 + isin nl3) ] 6
[2(cos2nl3- isin2nl3)] 4
(Hint: bracketed expression = cis - 21113)

Express the following in mod-arg dnd cartesian form


7.

(I

9.

(I

II.

(-1

13.

(I

(3 + 4if 2
10. ( v'j - i)4

+ i)6
- i)4

8.

12. (2 - 2 {jif 4
14. (2 J3 + 2i) 5

- i)IO
+

11'304

Simplify the following in the form rcis9


in the form a+ ib (15 to 20).
15.
17.

(1

- i)4 (I

(I

VJi)3
i)4
-

(I

19.

(2cis nl6)

(7 to 14).

+ i)3

and wherever possible, express the answers

16. (I
18.

20.

VJi)3 (I

(2 + 2i) 4
(I

(4cis nl3) 3

v'Ji)2

(3cis n I 12)

[3cis (-n I 36)] 3

+ i)4

127

4.6

Roots of Complex Numbers

Finding a desired root of a non-negative real number is a simple matter.


example:
The two square roots of
The cube root of

For

4 are .:!:. 2
113
(8)
= 2 and so on.

8 is

But finding the desired roots of complex numbers is not so simple, as we must use De
Moivre's theorem to find the:
three cube roots,
a + ib.

four 4th roots,

five 5th roots and so on of a complex number

EXAMPLE: (23)

Find all the cube roots of

SOLUTION:

We use two methods to find the cube roots.

Method 1:

If z

-1

-I, then

is a cube root of

3
2
z + 1 = 0, hence (z + 1) (z - z + 1) = 0

adding
-l

2kTT to the arg(-1),


COS

(TT + 2kTT)

2
cos 11

-1

We write

Method II:

J3i

1 .:!:_

-1,

The three cube roots are

+ i sinTT

and make the expression general by

hence

. ( l)

+ isin (TT + 2kTT)

Now let the required cube root be


W'cos(TT + 2kTT)

R (cos~ + 1sin~)

+ isin(TT + 2kTT)

= R(cos~

isin~)

We take the cube of both sides:

R 3 (cos3~+ isin3~) =

cos(TT + 2kTT)

+ isin(TT + 2kTT)

Equating the real and imaginary parts:

R 3 cos3~

= cos(TT + 2kTT)

1,

From these:
~

i.e., R =

TT/3

sin ( 11 + 2kTT)

and
and

3~

TT + 2kTT

+ 2kTT/3

Hence the cube roots are given by:


z
where

=
k

(cos~+ isin~)

= 0,

1, 2.

cos(TT/3 + 2kTT/3) + isin(TT/3 + 2kTT/3)

128

For k = 3,
the root for

the angle is n /3 + 2n and the corresponding cube-root is the same as


k = 0. In a similar manner, k = 4 produces the same root as k = 1
1 3
and so on. The three cube roots of (-1) / are:
0,

zl

cos n/3 + isin n/3

1/2 +

I,

z2

cosn + isinn

-I

2,

z3

cos 5n/3 + isin 5n/3

1/2

J3/2 . i

fi/2 i

These three roots are the same as given by Method I.


The modulus of each root is
I, hence these

three distinct cube-roots lie on a circle of


radius I and centre the origin.
They are equally spaced, the angular distance
between any two roots being 360 + 3 = 120"
Definition:

If

n is a positive integer, then R(cos\6 + isin\6) is an nth root of

r(cosa + isina) if and only if Rn(cos\6 + isin\6)n = r(cosa + isina)


From this, and using De Moivre's theorem:
11

R (cosn16 + isinn\6) = r(cos a+ isina)


rcosa = Rncosn\6

and

rsina = Rnsinn\6

By squaring and adding:


Rn = r, hence cosa = cosn\6 gives n\6 = a + 2kn

0=

Hence R = r 1/n ,

a +n 2k 1T '

where k = 0, 1, 2, (n - I)

We conclude that the n nth roots of the complex number r(cosa + isina) have the
1/n
.
a + 2kn
)
modulus r
and arguments are g1ven by --n--' k = 0, I, 2, (n - 1

EXAMPLE: (24)
SOLUTION:

Find the 4th roots of - & + & >/3 i

Write -& + &IJi = 16(cos2n/3 + isin2n/3) = 16(cosl20" + isinl20") = z

.. (2kn + 2n/3)]

[
(2kn + 2n/3)
.. The 1!-th roots are z = 2 cos
+ 1sm
,
4
4

k =

o,

1, 2, 3

The four roots are:

"3

0,

zl

I,

z2

2 (cos 120" + isin 120")

-1 + ,/3i

2,

z3

2 (cos210" + isin210")

-.13-l.i

3,

z4

2 (cos300" + isin300")

I -

2 (cos30" + i sin 30")

+ 1i
X

Fig. II

v'3 i

The points in the Argand diagram that correspond to these four


distinct 4th roots are equally spaced (at an angular distance of 360" .;- 4
on a circle of radius 2 and centre the origin.

90")

129

EXAMPLE: (25)

Find all 5 roots of z


diagram.

SOLUTION:

= 32

- 32 = 0 and show them in an Argand

= 32(cos0 + isinO)

32[cos2k11 + isin2k11]

The five fifth roots are given by


2k11
. . 2k11]
+ I Sin T
z = 2 [cos T

where k = 0, 1, 2, 3, 4.

To clearly visualise the roots, we replace 11 by 180", then the five roots of
z 5 = 32 are:
k

o,

zl

2ciso = 2

!,

z2

2cis72"

2,

z3

2cis 144"

3,

z4

2cis216"

4,

z5

2cis288"

(the only real root)

Note that the argument of successive root increases by 360 "'" 5 i.e. 72". This way
you can quickly write down all the roots.
Observing that:
lz 11 = lz 2 1 = lz 51 = 2, we can show
these roots on a circle of radius 2, centre 0.
From the diagram:

A(z )
1

z5

z2

z4

z3
Fig. 12

EXAMPLE: (26)

Solve z

SOLUTION:

Write

16 .f2 + 16 J2i

1+ i

l'i.cis( 11/4) =

32cis (2k11 + 11/4)

The five 5th roots are given by


z = 2cis(rr/20 + 2krr/5)
The five roots are:

k =

o,

1, 2, 3, 4.

zl

2cis9"

z2

2cis (9" + 72")

z3

2cis(9 + 144") = 2cisl53"

z4

2cis(9" + 216") = 2cis225"

z5

2cis(9" + 288") = 2cis297"

2cis81"

v'2cis(2k11 + 11 /4)

130

Exercise 4E

1.

Find the two square roots of:


(a)

2.

(b)

-16i

4.

(b)

-8

Find the four 4th roots of:


(b) -16
(a) -1

-2/3- 2i

(d)

4 + 3i

(c)

27(~ + ~ 0

(d)

-8i

(c)

-8 - 8 J3i

(d)

2-2

(c)

-2 + 2 I

(d)

32i

V3.1

Find the five roots of:


(a)

5.

(c)

Find all the cube roots of:


(a)

3.

2 + 213i

(b)

32

-32

.fj

1.

Find the solution set for each of the following equations. Express your answers
in the form a+ bi.
(a)
(c)

X
X

(e)

(g)

4
2
4

+ 1

(b)
(d)

X
X

(f)

13.

(h)

21

+ i = 0
1
2 -

+ (3 - i)x
+ 16i = 0
64 = 0
+ i

= 0

3i = 0

131

4. 7

De Moivre's Theorem and Its Applications

We have used De Moivre's theorem in finding the powers and the roots of complex
numbers. We now prove it by the method of induction and consider some further
applications.
De Moivre's theorem is:
For any integer n, (cos9 + isin9)n = cosn9 + isinn9
Proof:

For

= O,

For n = I,

(cos9 + isin9)

= I = cosO

(I)

+ isinO

cos9 + isin9 = cos9 + isin9

So the theorem is true for

=0

and n

= I.

We assume it true for n

=k, a

(cos9 + isin9)k = cosk9 + isink9

positive integer, so:

(2)

Multiply both sides of (2) by cos9 + isin9, then


(cos9 + isin9)k+l = (cosk9 + isink9) (cos9 + isin9)
= (cosk9cos9- sink9sin9)+ i(sink9 cose + icosk9sin9)
= cos(k + I) 9 + isin(k + I) 9

This proves that the theorem is true for n = k + I, if it is true for n = k. Since it
is true for n = I, then it is true for n = 2 and so on for all positive integers n.
The theorem is also true when n is a negative integer.
Let n = -m, where m is a positive integer.
(cos e + i sine )n= (cos e + i sine fm = - - - - - - (cose + isin9)m
by using the theorem for positive integer

cosm9 + isinm9

m.

I
(cosm9- isinm9)
cosme + isinm6 = (cosm9 + isinme) kosm9- isinm9)

Now
Finally

cosme- isinm9
1

cosm9 - isinm9 = cos(-n9)- isin(-n9) = cosn9 + isinn9

Hence the theorem is true for negative integers.


DERIVATION OF TRIGONOMETRIC FORMULA

By application of both De Moivre's theorem and binomial expansion, we can find:


(a)

cos n e and sin n 9 in terms of powers of sine and cos 9

(b)

cosn9 and sin 9 in terms of multiple angles (29, 39, etc.)

11

For (a) and (b) we shall need the following:

132

Let

z
z

= cos9 + isine , then:


-n
cosne + isin ne and z

cosne- isinne.

-n
+z
= 2cos ne
n
-n
Subtracting, z - z
2isin ne
Adding,

1
z

For n = I,

2cos e

Z+-

n = 2,

1
+2
z

(1)

. (2)

'

2cos2e ,

1
z
2

2isine

1
2= 2 isin 2e
z

'

etc.

Express (a) cos4e in terms of cose


(b) sin4e in terms of cose and sine

EXAMPLE: (27)

(c) tan4e in terms of

SOLUTION:

tan e.

We use c = cose and s= sine to simplify our work.

By De Moivre's theorem:
cos49 + isin49 = (cos9 + isin9)
C

= (c + is)

4 + 4c3.IS + 6c2.2
2 4c.3
3 .4 4
1 S +
1 S + 1 S

c4- 6c2s2 + s4 + i(4c3s- 4cs3)


. 1.4 = 1, etc. )
( Note t hat 1.2 = - 1, 1.3 = -1,
Equating real and imaginary parts:
(a)

(b)

4
4
2 2
2
4
cos49 = c - 6c s + s = c - 6c (1 - c 2 ) +(I - c 2) 2 ,
2
.
. 2e
usmg
cos e = 1 - sm
4
2
:. cos49 = 8cos e- 8cos e + 1
sin49 = 4c\- 4cs

3
= 4cos 9sine- 4sin 3 ecose

We note that sin4e cannot be expressed in terms of sine alone.


(c)

an 4

e _ sin4e
- cos4e

4c\ - 4cs3
c4 _ 6c2 5 2 + 5 4

Divide both the numerator and the denominator by c


sin9/cose = tan9,
we have:

tan49

3
4tane - 4tan e
2
4
1 - 6tan e + tan e

remembering

133

EXAMPLE: (28)
(a)

If

z " cos 9 + i sin 9, show that:

(i)

zn

_!_ " 2cos n 9

z
(b)

zn -

(ii)

Express the foUowing in terms of cos n 9


(i)

cos 4 9

or sin n 9

4
sin 9

(ii)

2isin n9

(iii)

3
sin 9

SOLUTION:
(a)

cos9 + isin9

1 = cos9 - isin9

By De Moivre's theorem:
zn " cosn9

isin n9,

z
Adding:

-n

z-n

cos n9 - isin n9
(i)

2cos n9

Subtracting: z

n
z

(b)

(i)

(2cos9) 4
4
16cos 9
4
cos 9

(ii)

.. (ii)

2isin n9

~) 4

(z +

2cos49

= (z

+ -{)

+ 4cos29

4
16sin 9

2cos49 - 8cos29

(2 i sin 9 )3

. 39

4 (z

+ 6

1
+ /) + 6

[in (a)(i) n = 4, 2]

3)

expand and rearrange.

'

1
1
3z ) - 3(z - -z>

2isin 39

~)

[using results (a)(i) n

+ z4 ) -

1 3

(z

Sill

= (z

+ 3)

81 (cos49 - 4cos29
(z - -)

8cos29 + 6

(z - -)

4(z

1
8 (cos49

1 4

4
(2 isin 9 )

4
sin 9
(iii)

6isin9

(3sin9 - sin39)

[in (a) (ii) n

= 3,

1]

= 4,

2]

134

Exercise 4F
I.

Express (a) cos 3 e


in terms of tan e

2.

Express (a) cos 5 e


in terms of tan e

3.

Express (a) cos6e (b) sin6e in terms of cose and sine and hence express
tan 6 e in terms of tan e

4.

Express

5.

Express (a) cos 5 e


(c)

6.

(b) sin 3 e in terms of cos e and sine, hence express tan 3 e


(b) sin 5 e in terms of cos e and sine, hence express tan 5 e

(a) cos 3 e

5
fcos ede

(b) sin e

(b) sin e

in terms of multiples of e and hence integrate

5
(d) fsin ede

6
(a) cos 6 e (b) sin e in terms of multiples of e, and hence
TI /2
Tl /2
6
6
(d)
sin ede
cos ede
integrate (c)
0
0
Express

7.

in terms of multiples of e

Find the constants p, q, r and s if: (a) co/ e = pcos?e + qcos5e + rcos3e + s
Tl /2
7
and hence evaluate: (b)
cos e de
0

8.

Find the constants p, q, r, s if: (a) sin e = psin?e + qsin5e + rsin3e + ssine,
rr/2
7
and hence evaluate: (b)
sin e de
0

9.

Show that 1'1 + cose + isine)n = 2ncos n( 2


e)
is a positive integer

10.

Prove the following:


(a)

cot4e

(b)

cot5e

2
4
- 6 tan e + tan e
3
4tane - 4tan e
4
2
I- 10tan 9 + 5tan 9
3
5
5tan9- 10tan e + tan e

~cos n 2e

n eu
+ 1 sm

, where n

4
2
cot 9- 6cot e + 1
3
4c'ot 9 - 4cot 9

5
3
cot e - I Ocot e + 5cot e
4
5cot e- IOcoie +I

135

4.8

Square Roots of a Complex Number

We usually use De Moivre's theorem to find the roots of a complex number; but there
is also a special requirement to obtain the square roots in the form a + ib.
Two methods of obtaining the square roots are:
2
I.
We convert z = a + ib to mod-arg form and use De Moivre's theorem to solve

l
2.

=a+ ib.

We assume the solution z = x + iy and hence solve two simultaneous equations


involving x and y.
Using De Moivre's theorem.

METHOD 1:

EXAMPLE: (29)

Let

Find the two square roots of 2 + 2/Ji

z 2 = 2 + 2 _,i = 4(cos n/3 + isin n/3) = 4[cos(2kn + n/3) + isin(2kn + n/3)]

By De Moivre's theorem:

~os (

z = 2

2kn ; n/3) + isin ( 2kn ; n/3)

J,

k = 0, I.

The two roots are:

zl

2(cos n/6 + isin n/6)

z2

2(cos7n/6 + isin7n/6) =

Verify that,

tf3 +
-13-

[!. ( J3 + i) ] 2 = 2 + 2/3i

METHOD U:

EXAMPLE: (30)

Let z = x + iy, then

Find the square roots of 7 + 61/ii

SOLUTION: Let (x + iy)

z =a + ib
X2 -

y 2 + 2"IXY = a + 1"b

Equating real and imaginary parts.


x 2 - y 2 = a and 2xy = b, then:
(x2 +/)2 = (x2 -/)2 + 4x2/ = a2 + b2

.22~
22
+ y = '1/a- + b. as x + y

>0

= a and

for x and y.

+ /

= 7 + 6 /2i

:. x - / = 7 and 2xy = 6 v'f


2
2
2
2
2
Now (x +
(x + 4x /

/>

/l + b2

/>

49 + 72

121
X

+y

and x - y

Finally we solve

i -/

=11

(1)

=7

(2)

Solving (1) and (2):


X=!.3,

y=!.fi

We check xy = 6{2 for the proper


combination of x and y.
The required roots are:

136

Exercise 4G
1.

Find the square roots of the following in the form x + iy.


(a)

3+4i

(b)

3-4i

(c)

(e)

5 + 12i

(f)

8 + 6i

(g)

(d)

5+2J6i

(h)

7-6/ii
-8i

2.

Solve the following equations, expressing the answers in the form x + iy.
2
2
2
(b) z = I + {:Ji
(c) z = 2i
(a) z = -15 + 8i

3.

Use the formula z = (-b :!:. lb


equations in x + iy form:

4.

(a)

(b)

(c)

(d)

2
2
2

- (I - 4 i) z - (5 - i)

- 4ac)/2a to express the roots of the following


0

+ (2 + 4i) z - 11 - 2i

+ ( 4 + 2 i) z + (3 + 2 i)
+ (4 - 2i) z + 6

(Hint:

J5:l2i

3 - 2i)

(Hint:

.Js

3+i)

+ 6i

=0

Write each of the following in the form a+ ib.


(Take
(a)

5.

,fZ to mean the square root whose real part


1 +i
./8+6i

(b)

/5 -

l2i
5+12i

Prove the following:


(a)

(b)

..ra+l>1

6.

If X+ iy

7.

Simplify:

va:bT

~
c + !d '
..t5:lii
v57iTi

(c)

(./a

~
I + z + z 2 where z = v&
+ 6i

+b

+a)

h< E:;}_

prove that (X2 + y 2)2


+

~
{5 - 12i

> 0)

a)

2 + b2
2
c + d2
a

137

4.9

Properties of Conjugate Complex Numbers

z= x

- iy

is the conjugate of

If the point

P(x,y) represents

complex plane, then

in the x-axis.

z = x + iy in the

is the reflection of

z = 2x

Fig.l3

arg -z = -argz

(a real number)

5.

Further, for two complex numbers z

6.

7.

9.

10.

z and

(a) z

Let
+ z

= a + ib
and
1
= a + ib + c + id

+ c) -

i (b

= c + id,

then
1

- z

= a + ib - c - id

=(a- c)+ i(b- d)

+ d)

- z

= (a - c) - i (b - d)
= (a - ib) - (c - id)

= zl

=zl

+ z2

= (ac - bd) - i (be + ad)

= (a - ib) (c - id)

- z2

(d)

We have

zI

(ac + bd) - i (ad - be)

z2

(see Section 4.2,


example 5)

c2 + d2

zI
Again, z2

z-2

= z = zz = Izl2

= (a - ib) + (c - id)

= zI

.-~ Q<z>

we prove the following properties:

(b) z

= (a + ib) (c + id)
= (ac - bd) + i(be + ad)
z

''

z-I

2 = (a

(a purely imaginary number)

=(a+ c)+ i(b +d)


~

I
I

: -y

- = x 2 + y 2 = lzl2 -- l-zl2
zz

3.

z = 2iy

8.

lzl

Proof:

..

We have:

2.
z +

Note that x + iy is the

x - iy.

I.

4.

P(z)

Q(x,-y) is the reflection of

P in the x-axis and hence z

conjugate of

z = x + iy.

(a - ib)
(a - ib) (c + id)
_ id) = (c _ id) (c + id)

=(c

(ac + bd) + i(ad - be)


'

( :~)

using property (3)

We easily generalise, but shall not prove that:


II. z I + z 2 + z 3 + + zn
12. ZIWJ + Z2W2+
In particular, if

= z I + z 2 + . + zn

+ZnWn

= zl.wl +Z2

is real, then using

13. z 1w"l + z 2w 2 +

+ znwn

w = w,

W2+

+Zn.wn

we have

w 1z 1 + w 2z 2 +

, ence

+ wnzn

We shall use the properties (9), (10) and (11) in the next chapter on Polynomials.

138

EXAMPLE: (31):

=~

If z

+ 3i, express the following in a + ib form:

(a) z

(b) z + z

(c) z - z

(d) zz

(e)

SOLUTION:

(a)

z =~-

(d) zz

3i

=(~

z =~

(b) z +

+ 3i + ~- 3i

z =~

(c) z-

~ - 3i
~
=z1 =zzz =Izlz2 =2
5 = 25

-1
(e) z

+ 3i)(~ - 3i)

=8

+ 3i- (~ - 3i)

=6i

3.
.
( )
- 25 1, usmg d

= 16 + 9
-1
Note that z

= 25

= (X

If x + 1 y

. )(

+ IY

-z

Using ,

=arg

z .

prove t h at ( x 2 + y 2)2

We square both sides and write

SOLUTION:

= Vr;;-:;Tb
~

EXAMPLE: (32)

-1
z in general, even though arg z

.1-

1 z2

= ca

. )

IY

z1z

+ ib

(I)

+ id

and

(z1)
Z2

w = (X-

. )(

IY

. )

X- IY

a - ib
c - id

(2)

= x2

Multiplying (I) and (2) and using (x + iy) (x- iy)


(x

2
+b
2
c +d

+/,etc.

2
2
2
+ y ) (x + y )

= a2

, hence the required result.

Exercise 4H
I.

Given
o> z

(a) z

= 1 + 2i

(ii) z + z

x + iy
x2 + /

Oii) z-

3.

If a+ ib

(x + i)2
2x - i

+ iy

~
1
'

~-

5.

If a

3 - i, find and sketch the following:


1
ov> z -z <v> lzl <vi> 1zl <vii)

2
(a + ib) , then without finding x and
= (a2 + b2)2

If

-z

2.

If

(b) z

'

prove that a
show that

2 + b2
+y

=cosa

+ isina, b =cos B + isinB, c


1 + c.!. = 0
t hen prove t hat: 1 + b

If (a + ib)l/J = x + iy, prove that:

[Hint: (a + ib)

= (x

(x2 + 1)2

~x 2

+ I

6.

y, prove that

~(x 2

=cosy + isiny and a


<H.mt: -a1 = lal2
a:
- /> =

~X

+ b +c

~y

3
+ iy) , equate real and imaginary parts, substitute

7.

. 3 + 2isin9
Find 9, 0~9~2n, If l- 2 isine

8.

Prove that (I - cos9 + 2isin9)

-1

ispurelyimaginary.
1 - 2icot(9/2)
5 + 3cos9

=0,

etc. >

in~+~]
X

139

4.1 0 The Complex Roots of Unity


The equations of the form zn = a + ib give rise to n roots which are equally
spaced on a circle of radius r, where r is the modulus of any root. In the special
case where

zn = I, then r = I,
2
centre the origin (x + / = 1).

i.e. all of the roots lie on a circle of radius 1,

In the chapter on Polynomials, we shall learn that the complex roots of


(if any) must occur in conjugate pairs.

zn - 1 = 0

We shall also show that if w is a complex root of zn - 1 = 0 with the smallest


positive argument, then the n roots of xn - I = 0 are:
2
1, w, w , ... , wn-J and consequently, the sum of the roots is given by
1 + w + w2 + ... + wn-l = 0
3
Solve z - 1 = 0

EXAMPLE: (33)

3
Factorise z - I = O, then

SOLUTION:

(z - I) ( / + z + 1) = 0. The three roots are:


-1
,f'J.
I
{3.
2
1 = 1 2 2 = 2 + 2 1' 2 3 = - 2- 2 1
Let w = z
2
w =

= -

i-

1
.f'J . h
2 + 2 1, t en

~.

2 .

'?

~ =- ~ -

i = z3
2
3
Thus 1, w and w are the three roots of z - 1 = 0
2
_
. d
_ coefficient of z
Fig. 14
We f m z 1 + z2 + z3 - . .
3 - 0
coefficient of z
0
. I 1
2
1 1 ./3. 1 {j_
Alternative y
+ w + w = z 1 + z 2 + z 3 = - 2 + 21 - 2- 21 =
3
2
The roots of z - I = 0 lie on the circle x + / = 1. The roots are equally spaced
with angular separation between any two roots equal to 360 f 3 = 120. We observe
that z3 = z2

EXAMPLE: (34)
SOLUTION:

i -

Solve z - 1 = 0

(z - 1) (z + 1) ( / + 1) = 0.

The four roots are: 1' - 1' i, - i


Let w = i, then

Again,

=-I
.3
=

Fig.l5

-1

2
3
1 + w + w + w

+i-1-i=O

The roots lie on the circle x 2 + y2 = 1, the angular separation between any two
consecutive roots being 360 -;. 4 = 90.
We also observe that z
= z 2
4

140

Let

SOLUTION:

= l

= cos

2kn + isin2kn

z = r (cos9 + isin9) be a root of /


/

7
x - 1
4
.5

=0, then show that


2
3
6
the other complex roots are w , w , w , w and w Also prove
2
3
4
.5
6
that 1 + w + w + w + w + w + w = 0.

If w is a complex root of the equation

EXAMPLE: (3})

(1)

- I = 0, then by De Moivre's theorem:

= / (cos79 + isin79)

. (2)

From (I) and (2), equating the real and the imaginary parts:
r 7 cos79 =cos(2kn)
and
/ sin79 = sin2kn
14
= 1, hence r = 1 since r > 0
By squaring and adding, r
whence 7 9 = 2kn

cos79 =cos2kn,
2kn
9 = T
, where k = 0, 1, 2, , 6

The seven roots of unity are given by:


z

=cos4n/7
z =cos8n/7
5

+ isin4n/7

z4

=cos2n/7
=cos6n/7

+ isin8n/7

= coslOn/7 + isin lOn/7

=cosO + isinO = 1

2
6

+ isin2n/7
+ i sin 6n /7

z = cos12n/7 + isinl2n/7
7
Now we apply De Moivre's theorem, in reverse order, i.e.
cosn9 + isinn9 = (cos9 + isin9) 7 to each of the above complex roots.
Let

= w =cos2n/7

+ isin2n/7

2
= w
3
3
and so on,
z =cos6n/7 + isin6n/7 = (cos2n/7 + isin2n/7) = w
4
4
.5
6
hence, z = w , z 6 = w , z 7 = <tJ
5
2
6
To prove that 1 + w + w + + w = 0, we note that the sum of the roots of
6
7
-coef. of z
hence
z - 1 = 0 is zero, because: z 1 + z + + z 7 =
2
7
coef. of z
2
6
substituting for zl' z , etc. we have: 1 + w + w + + w =0.
2
z

= cos4n/7 + isin4n/7 = (cos2n/7 + isin2n/7)

Observations:

The roots of zn - 1

= cos ~
n

+ isin

~,
n

=0

are given by:

= 0,

(1)

(2)

If n is even, two roots of zn - 1 = 0 are real, i.e. ~ 1.

(3)

If n is odd, only one root of

(4)

If w is the complex root with the smallest positive root, then the
2
n-1
entire set of roots is given by: 1, w, w , , w

2
n-1
0
.
h
b
.
1 + w + w + + w
= , e1t er y usmg the sum of the roots of

(.5)

zn - 1

=0

.
w1th sum

where k

1, 2, , (n - 1).

zn - 1 = 0, i.e. z

=1

is real.

or summing up by observing that this sum is a geometric series,

1 - wn
=~
=

1- 1
1 _w

0,

since wn

= 1,

w ~ 1.

141

Exercise 41
1.

If w is a complex cube root of unity (i.e. a root of z

2
= 1), prove that w is

also a complex cube root of unity. Further prove that:


(a)

2.

B=

whose roots are given by a = 2 + w and

3.

4.

5.

25

1 + w + w =0 (b) - + - - = 1 (c) (1 + w) = -1 (d) (1 + w ) = -w


1 +w
1 + w2
3
is a complex root of the equation z - 1 = 0. Form a quadratic equation
2 +

d.

If w is the complex cube root of unity, show that:


2
2 3
2 3
a + bw + cw
(a) (1 + w- w ) - (1 - w + w ) = 0
(b)
2
c + aw + bw
2
(c) a + b w + c w = til
2
b+cw+aw
2
2
2
If x = a + b, y = aw + b w , z = aw + bw, where 1, w, w are the cube
roots of unity, prove that:
2
'2
2
2
2
(a)x+y+z=O (b)(a+bw+cw)(a+bw +cw)=a +b +C -ab-bc-ca
2
If 1, w, w are the three cube roots of unity, prove that
3
3
3
2
(a + b +c) (a + bw +
(a + bw + cw) = a + b + c - 3abc.
5
2
3
4
If w is a complex root of z - 1 = 0, show that w , w and w are the

ci>

6.

other complex roots.


(a)
(b)

(c)

4
2
3
1 + w + w + w + w = 0

Prove that

4
Find the quadratic equations whose roots are a = w + w
and
7
6
5
2
3
B = w + w
(Hint: use w = 1 to reduce w and w )
Show the roots of

z 5 - 1 = 0 in an Argand diagram.

(d)

7.

Find the area of the pentagon formed by the roots.


If w is a complex root of z 6 - 1 = 0 with the smallest positive argument, then

show that the other roots are w , w , w , w Prove that:

(a)

1 + w + w + w + w + w

= 0.

(b)

Find all the roots in the form

a + ib

and indicate these roots in an

Argand diagram. Find the area of the hexagon formed by the roots.
(c)

Find the two quadratic equations whose roots are


4
(i) w and
(ii)
and w

(d)

Using part (c), show that


4
2
5
(i)
z 6 - 1 = (z- 1) (z + 1) [(z- w) (z- w )] [(z- w ) (z- w )]
2
2
= (z 2 - 1) (z + z + p (z - z + 1)

(ii)

The roots of

+ z

+ 1

=0

are w, w , w

5
and w

142

8.

Show that if. w is one complex root of the equation zn - I = 0, then


- I) (z - w) (z -

i> ...

(z - wn-l)

zn - I

(b)

Deduce from part (a) that:


zn-l + zn- 2 + + z + I = (z - w) (z - w2) (z - wn-l)
(1 - w) ( I - w2 ) (1 - wn-l) = n

(c)
9.

= (z

(a)

Prove by mathematical induction that for any real 9,


cos ne + isin ne = (cose + isin9)n
(a)
(b)

Find the 6 sixth roots of I, expressing each in the form a + ib.


4
Using part (a), find the four roots of z + z 2 + I = 0 and show their
positions in an Argand diagram.

143

4.11 Miscellaneous: Factorisation over the Complex Field


2
3
3
Consider the factorisation of z - 1 = 0 over C.
z - 1 = (z - 1) (z + z + 1)
2
Now z + z + 1 has no real linear factors, but over C we can write
z 2 + z + 1 = (z - a ) (z - 8 ) where a = -1 +2 t/3i , 8 = - 1 - 2/3i
Thus it appears that we can factorise expressions of the form zn - 1, zn + 1 and
(by extension) zn-l + zn- 2 + + 1, into either: (a) real quadratic factors or
(b) complex linear factors.

Factorise z - 1 into real quadratic and real linear factors, hence


4
2
factorise z + z + 1.
6
We solve the corresponding equation z - 1 = 0

EXAMPLE: (36)

SOLUTION:
z 6 = 1 = cos(2kn) + isin(2kn),
z
z

hence the six sixth roots are given by

= cos(2kn/6) + isin(2kn/6), k = 0, 1, 2, 3, 4, 5.

z
1

i.e.

= cos 0 + i sin 0 = 1

=cos n/3 + isinn /3

z5

=cos4n/3

= cos2n/3 + isin2n/3 ,

= cos5n/3 + isin5n/3 = cosn/3- isinn/3

3
We find that z

4
6

= -1
+

isin4nj3

=cos2n/3-

isin21T/3

z 5 = z 3 , hence:

=z
and
2
6
z2 + z6 = z2 + z2 = 2cosn/3

and

z3 + z5 = z3 + z3 = 2cos2n/3

Also z z = z z = 1 ,
z z =z z = 1
2 2
3 5
3 3
2 6
6
Now z - 1 = (z- z ) (z- z ) (z- z ) (z- z ) (z- z ) (z- z )
4
6
1
2
5
3
= (z- 1) (z + 1) [(z- z ) (z- z )] [(z- z ) (z- z )]
6
3
5
2

= (z

-l)(z + l)[z 2 - (z 2 + z6 )z + z 2z6 ][z 2 - (z 3 + z5)z + z z ]


3 5

2
(z - 1) (z + 1)
2cos(n /3) z + 1] [z - 2cos(2n/3) z + 1]
2
4
2
(z - 1) (z + z + 1), we at once have:

[i -

Since z 6 z

+ z

+ 1

~ =

[z

- 2cos(n/3) z + 1] [z

- 2cos(2n/3)z + 1]

z - 1
EXAMPLE: (37) Solve z

+ 1 = 0. Express the roots in the form a + ib. Show these


6
roots in an Argand diagram. Factorise z + 1 into real quadratic factors.
6
SOLUTION: z = -1 = cos(n + 2kn) + isin(n + 2kn), hence the six sixth roots are given

1T + 2k1T

by z = c1s - -, where k = 0, 1, 2, 3, 4, 5.
6
z = cis7n/4 =- 13/2- i/2
z = cisTT/6 = 13/2 + i/2
4
1
z = cisn/2 = 0 + i
z
= cis3n/2 = 0- i
5
2
z = cis 5n/6 = - /3/2 + i/2 z = cis llTT/2 = .fJ/2 - i/2
6
3
We have z = z l , hence Z l + z 6 = Z l + Zl = V3 , Z l z 6 =
6
and z5 = z2'
z2 + z5 = z2 + z2 = 0
z2z5 = 1
and z4 = z3 '

z2

z3 + z4 = z3 + z3 = - VJ' z3z4 = 1
Fig.l6

144

6
The factors of z + 1 are:
z 6 + 1 = [(z- z ) (z- z )) [(z- z ) (z- z )] [(z- z ) (z- z )]
2
5
3
4
6
1
2
2
2
= [z - (z + z )z + z z ] [z - (z + z )z + z z ] [z - (z + z )z + z z ]
2
5
2 5
6
1 6
4
3 4
1
3
2
2
= (z - .f3 z + 1) (z + 1) <i + 13 z + 1)
2
2
2
= (z + 1) (z - {3 z + 1) (z + .f3 z + 1)
We could have easily arrived at this result by writing:
4 2
2
2
2
2
2
2
z 6 +1 = (z +1Hz -z +1) = (z +l)[(i+0 -3z ] = (z +l)(z -13z+l)(z 2 +13z+l).
So why waste time? This is not so, as the algebraic identities that
established can be used to derive numerous trigonometrical relations.
example.

EXAMPLE: (38) Factorise

+ 1

deduce that 4sin

SOLUTION:

The roots of z

we have just
See the next

into real linear and quadratic factors and hence

to cos J

= -1

= 1

are given by z

71 ; 2k 71 , k

= cis

= 0,

1, 21 3, 4.

The five fifth roots are:


z
z

1
2

z3

z5

= cos71/5

+ isin71/5

Now z

z5

= zl

= z

= cos771/5

+ isin771/5

= cos(371/5)-

isin(371/5)

+ zl = 2cos71/5'
zl z5 = zl zl
+ z = 2cos371/5 ,
z z =z z
hence:
4
2
2 4
2 2
2
z ) (z - z ) (z - z ) (z - z ) (z - z )
3
5
4
1
2
2
2
(z + 1) [z - (zl + zl) z + zlzl] [z - (z2 + z2) z + z2z2]

zl +
z +
2
(z -

+ 1
5

(z + 1) [z

+ 1 = (z + 1) (z

- 2cos(71/5)z + 1] [z
- z

+ z

- 2cos(371/5) z + 1]

(2)
4

now cos 371/5 = sin [; -

4cos71/5 cos371/5 + 1 + 1 = 1
4cos(71/5) cos(3n/5)

= -1

4sin(71/10) cos(71/5)

= 1

(1)

- z + 1)

hence: (z - 2cos(n/5)z + 1] [z - 2cos071/5)z + 1] = z - z + z


2
Compare the coefficient of z on both sides of identity (3), then

~n:l

= -sin

- z + 1

fa,

We can change these into the form zn + I = 0, as follows


n-1

+z

n-2

+z

n-3

zn - I
n-1
n-2
n-3
- z
+ z
- . . - z +
+ + z + I = ----z-:T and z

We solve the equations zn - I = 0 and zn + I = 0, ar.d remove


the real root I from the roots of zn - I = 0 ,

and

the real root -1 from the roots of zn + I = 0 ,


to obtain the roots of equations (l) and (2) respectively.

(n is odd)

(3)

hence:

SOLUTION TO THE EQUATIONS


(1) zn-l + zn- 2 + . + I = 0 and (2) zn-l - zn- 2 + zn-J- ... + - z + I " 0

z1

z 5 = cos971/5 + isin971/5 = cos71/5- isin71/5

= cos371/5 + isin371/5 ;

=-

145

EXAMPLE: (39)
(a)

Solve z

+z

+ z

+ z + l

4 -z 3 +Z 2 -z+1=0

(b)

(b)

We change z - z + z - z + 1 = 0
z5 + 1
to """"Z+T = 0, z -/. -1

SOLUTION:

(a) We change z + z + z
z5 - l
to
0, z -/. l

2+z + l =0

--z:-r ::

5
Hence z - 1 = 0 whose roots
are given by:

The roots of the given equation are


given by:
11 2 11
z = cis [ \ k

z = cis(2k11/5), k = 0, 1, 2, 3, 4, 5.

where k = 0, 1, 3, 4, sihce we must


remove the root z = -1 given by k = 2

We omit the root z = 1, then the


4
3
2
roots of z + z + z + z + 1 = 0
are given by:

z = cis(2k1T/5), k = 1, 2, 3, 4.

Exercise 4J
1.

2.

3.

5
Resolve z - 1 into the real linear and quadratic factors.
1
211
411
cos 5 + cos5
-2.
5
Resolve z + 1 into the real linear and quadratic factors.
11
3lT
1
cos 5 +cos 5 = 2.

Hence prove that


Hence prove that

Find the roots of z 4 + 1 = 0 and show them in an Argand diagram. Resolve


z 4 + 1 into real real quadratic factors and deduce that:
cos29 = 2(cos9 - cosn/4) (cos9 - cos3n/4)

4.

Find the roots of z 6 + l = 0, and hence resolve z

+ 1 into real quadratic

factors; deduce that cos39 = 4(cos9 - cosn/6) (cos9 - cos11/2) (cos9 - cos5n/6)
5.

Show that the roots of (z - 1) 4 + (z + 1)

6.

Show that the roots of (a) (z - 1)

7.

Solve the following equations:

.:!:. i cotn/ 12, .:!:. i cot 5n/12,

(a)

3
4

+ z

2
2

+ z + 1

= 0 are.:!:. icot(n/8) and .:!:.icotOn/8)


6

+ (z + 1)

are

= 0

.:!:. i

=0

(c) z - z + 1 = 0
4 2
2
[Hint: z 6 +1 = (z +1) (z -z +1)]

(b)

(d)

+ z
+ z

+ z

+ z

+ z + 1 = 0

+ 1 = 0

[Hint: z 6 - 1 = (z

- 1) (z

+ z

+ 1)]

146

4.12 Geometric Representation of Complex Numbers


We have remarked that the complex number z = x + iy may be considered an ordered
pair (x, y) of real numbers and further that (x, y) can be regarded as the coordinates
of a point in a coordinate plane.
Conversely a point in the coordinate plane
corresponds to a complex number x + iy.
y
We call such a plane an Argand diagram
or the complex plane.
2
Three complex numbers
z

=3+2i, z =-2+1i and z =0-2i


1
2
3
are represented in the plane by points
P(3,2), Q(-2,1) and R(0,-2) respectively.

-2

-1

-2

Fig.l7

Any relation between points in a plane can be regarded as a relation between


corresponding complex numbers and vice versa. For example, the locus of a set of
2
2
2
.
. f ymg
.
.
. Ie, ra d"1us a, centre at t he ongm
. . )
pomts
sat1s
t he equation
x + y = a (a c1rc
2
can be stated as either zz = a
or Iz I = a where z = x + iy. Many more locus
problems can be solved by the use of complex variable z = x + iy. Here we are
killing two birds with one stone! Instead of two variables x and y we work with
only one variable z. Not only that, but by equating real and imaginary parts of the
equation, we obtain a complete set of solutions.
Addition:
Let the complex numbers z

+ iy and
1
1
be represented by the points A
1

=x

z = x + iy
2
2
2
and B respectively on the Argand plane.
Complete the parallelogram OACB. Then the
mid-points of AB and OC are the same. But
I + X?
y1 + y
the mid-point of AB is P
2 j
0 F--------.._X
so the coordinates of C are (x1 + x 2, Y + Y2>
Fig.I8
1
Thus the point C corr.esponds to the sum of the complex numbers z and z ,
2
1
i.e. z + z = (x + x ) + i(y + y ).
1
1
2
1
2
2

rx
n-=,

21

Subtraction:
We first represent -z

by D, so that AD is
1
bisected at 0. Complete the parallelogram
OBCD, then the point C represents the
complex number z + (-z ) i.e. z - z
2
1
2
1
Since OA = OD = BC and OA II BC,
D(-z 1)
OABC is a parallelogram, and hence OC is
parallel to AB. The coordinates of
C are (x - x , y - y )
2
2
1
1
We also note that the length of the segment AB is given by
AB = lz 2 -z 1 1 = /<x 2 - x / +(y - y /
2
It is advisable to draw a diagram and then calculate arg (z - z )
2
1

A(z I)
X

Fig.I9

147
y
EXAMPLE: (40)

The complex numbers z


z

= 4 + 2i and z2
1
are represented by points A

= 1 + 4i

2
and B respectively in the plane.
zl + z2. = 4 + 2i + 1 + 4i
The end-point

= 5 + 6i

the parallelogram OACB represents

the sum z = zl + z2 = 5 + 6i
c is the point (5,6)

,/

II/ /
~

A(z

I~

From the scaled diagram we lind that

Fi g.20

B(z

3
A(z ) represents z = 2 + i
1
1
B(z ) represents z = 1 + 3i
2
2
D(-z ) represents -z
-1 - 3i
2
2
z = z - z = (2 - 1) + (1 - 3) i = - 2i
1
2
Let C(i,-2) represent z = 1 - 2i

EXAMPLE: (41)

/;
I

-I

D is given

by -z2

-I

is the end-point of the diagonal of


parallelogram OACD where

//
O

/
B/

of the diagonal OC of

/}

C(z

- z2

D( z )3
2

Fi g.21
y

EXAMPLE: (42)

I = -2

and z

= 3 + 2i

2
Find the following:

(b) mod(z -z ) (c) arg(z -z )


(a) z = z - z
2
1
1
2
2
1
SOLUTION:

A~

Let A (-2,0) and B(3,2) represent the


complex numbers zl
respectively; then:
(a)

= z2

- z

and z2

-2

=5

= 3 + 2 i - (-2)

mod(z

(c)

arg (z 2 - z )
1

- z )
1

lz
9

- z

LCOX

LBAX

Fig. 22

+ 2i

C (5, 2) represents the difference z


(b)

~l~

- z

where OC

and = to AB

~2

Fig. 23

= ./29
where tan 9

II

148

EXAMPLE: (43)

Prove that the diagonals of a rhombus intersect at right angles.


SOLUTION:

For simplicity we represent the vertices of


the rhombus by the. complex numbers 0, z , z + z
1 1 2
D(z -z )
2 1
and z 2 where lz 1 1 = lz 2 1 1- 0
C represents z + z and D represents z - z
2
2
1
1
such that OD is II and = to AB. Our
geometrical proposition is equivalent to saying that
z2 - z 1
71

z2 - z 1
arg - - - = ~ - , 1.e. w = - - - is
z +z
2
z +z
2
2
1
1
purely imaginary. This would be so if w +
We find that w +

z2 - zl

z2 - zl
= --- + ---

' ' ',

''

''

''

'

A(z )
1

Fig. 24

0.
2(z2z2- zl zl)
(z2 + z 1) (z2 + z 1)

TRIANGULAR INEQUALITY

We prove the inequality


lz 1 + z 2 1 ~

lz 1 1 + l.z 2 1

Proof: In 60AC, using the fact that

the sum of two sides of a triangle ).


the third side, we have:
OC

OA + AC

lz + z 2 1 ~ lz 11 + lz 2 1,
1
because AC = OB = I z 1 and OA
2
The only time inequality holds is when
argz

= argz

= 9

Fig. 25
C(z +z )
1 2

(Fig. 26)

(z2)
A(z )
1
9
X

Fig. 26

149

Exercise 4K
Perform the indicated operations graphically. Use graph paper.
I.

(2 + 3 i) + (2 + 2 i)

3.

(3)

5.
7.

(3- 2i)
(-1}

+ (-2 - 3i)

O+i)

(-3 + 5i)

2.

(3 - i)

4.

(-1 + 3i)

6.

(6 + 4 i)

8.

(2i)

+ (5 + 2i)
+ (2i)

(3 - 2 i)

(-3 + i)

9.

Given that z = x + iy, ZJ = X] + iy 1> P i~ the point P(x,y) and A is the point
(x 1 ,y 1),
draw the sketch of the directed line segment AP in the Argand
diagram, if z - z 1 is equal to:
(a) z- 3
(b) z + 3
(c) z + 2i
(d) z- I + 2i
(e) z + 2 + 3i

10.

Find the modulus and argument of each of the complex numbers z and w,
1+ i
.n
z = y-:-r and w = T-=-1
Plot the points representing z, w and z + w on an Argand diagram. Deduce
from the diagram that tan

~TT

.f2

+ I

11.

The points P and Q are represented by the complex numbers z = 1 - 3i and


w = -3 + 4i respectively. Find a point R on the real axis such that PRQ is
a right-angled triangle.

12.

The points A, B, C and D in the Argand diagram represent the numbers


1 + 2i, 3, 5 + 2i, 3 + 4i respectively. Prove that ABCD is a square.
the complex number representing the intersection of the diagonals.

Izl

I,

13.

prove that ~ is purely imaginary. By drawing a suitable


z- w
diagram, give a geometrical interpretation of the result.

14.

Prove that for any two complex numbers z

If

lz

+z

= lw

1 ~

lz 1
1

2
1
sign hold?
15.

Provethat

lz

lz 1, assuming lz 1
2
1

and

>

z
2
lz 2 1. When does the equality

2
2
2
2
-z 2 1 + lz 1 +z 2 1 = 2lz 11 + 2lz 2 1

Give a geometrical interpretation of the result.

Find

150

4.13 Product and Quotient: Rotation


p

We now give the graphical representation


of multiplication and division of complex
numbers.
Let

and C

the complex numbers.


z

= r cisS
and
1
1
1

Construct [PO C
Construct

be the points representing

LO CP

Triangles OAB

Let A be the point (1, 0)

= r cisS

2
2
2
equal to 8 l

respectively.

equal to LOAB
and

OCP

are similar,

hence their sides are proportional.


OP
OB

OC

= OA

Fig.27

and this gives OP = r 1 r 2 . Hence point P represents the

complex number r r cis(S +


1 2
1

e2)

which is the product of r cisS and r cisS


1
2
1
2

Although similar graphical representation can be given to division, it is not very


elegant and of no practical use. Let us now turn our attention to a very important
application of i as an operator that rotates a directed line segment.
(a)

ROTATION OF A POINT A(z) THROUGH +90

In the product z z
1 2

e1 =;

= r 1 r 2 [cos(S 1 + e2)+ isin(S 1 + e2)},

and we write

e2 = 8,

then iz

= r[cos(S

The above results mean that when we


multiply z by i, we rotate the point
A(z) through a positive angle of 90 about 0.
If z = x + iy, after rotation through 90 ,
we have iz = i(x + i y) = -y + ix

Note that

lzl

= lizl

let z

= i,

= z,

then

+ ~) + isin(B + ; ) ]

B(iz)

A(z)

= r

Similar interpretations are given to the


C(-z)
powers of i as follows:
Fig.28
2
i corresponds to 180 counterclockwise rotation
3
i corresponds to 270 counterclockwise rotation, or to a 90 clockwise rotation.

151

EXAMPLE: (44)

P is the point
z = 3 + 2i. OP
through + 90.
represented by
the plane.

P(3, 2) in the Argand diagram representing the complex number


is rotated about the origin
Find the complex number
y
the new position of P in

SOLUTION:

3 + 2i

iz = i(3 + 2i) = -2 + 3i

\\

[\

:. New position of P is A(-2,3)


A represents the complex number

z1

lz 11

= -2 + 3 i ,

IzI

v'iJ

-2

-I

\
0

/
2

Fi11;.29

EXAMPLE: (45)

OABC is a square in an Argand diagram. The point A represents the complex


number z = V'J + i. Find the complex numbers represented by B and C in the
form a + ib.
SOLUTION:

o/'J +

A represents z =

i,

represents iz

and B represents z + iz.


Hence C represents i ( v'3 + i), i.e. - I +

V3

B represents

.r5 + i),

+ i + i (

V3 i,

i.e.

o/'j - I + ( v'J + I) i.

A(
I

v'J,J)

Fig.30
EXAMPLE: (46)

P is the point (2, 3) in the Argand diagram representing the complex number

z = 2 + 3i. The segment OP is rotated through + 60, and P now occupies the
position of the point Q. Find the coordinates of Q.
SOLUTION:
We use z z = r r [cos(9 + 9 ) + isin(9 + 9 )]
1 2
2
1 2
1
1
2
= 2 + 3i and z = cos60 + isin60, then:
2
1
multiplying 2 + 3i by cos60 + isin60 is equivalent to rotating the length OP

Let z

through + 60.
Let Q be the point (x, y), then
x + iy

(2 + 3i) (cos60 + isin60)


=

l - - {3
- +
2

Q is the point

(32 ./::;)

~I~

+ v3

_ 3

f3
2

3
, 2

152

Exercise 4L
1.

P is the point in the Argand diagram and represents the complex number
I + 2 i. The segment OP is rotated through (a) + 30
Find the complex number
each

2.

ro~ation

The point

th..tt

(b) + 45

(c) + 90.

represents the point occupied by

after

is completed.

A in an Argand diagram represents the complex number

+ i.

Find

the complex number represented by B if OBA is an equilateral triangle.


3.

OPQR

is a square in an Argand diagram, where 0

represents the complex number

given by

complex numbers represented by Q and


4.

OABC

and

Find the

R.

is a rhombus in an Argand diagram, where

A is (1,2). If [BOA =- 30,

is the origin. The point P

z = r (cos 30 + i sin 30).

B is in the

~econd

is the origin. The point


quadrdnt, find the complex

numbers representing the points B and C.


5.

PQRS

is a square in an Argand diagram, where

numbers

2 + i,

2 i,

rotated anticlockwise about


!h..tt

2 + 3i,

and

2i

through

90.

P, Q, R and S represent the


respectively.

Find the complex numbers

correspond to the new positions occupied by the points

respectively.

This square is
P, Q, R and S

153

4.14 Locus Problems with the Complex Variable z


From our study of the co-ordinate geometry, we know that y = f(x) or f(x,y) = c
represents a locus of a point P(x,y), satisfying a certain condition. Since z = x + iy,
as x and y vary, z varies and hence z may describe a curve in an Argand plane.
CIRCLE x

(a)

Let

+ y

= r2

P(z = x + iy) be on the circle x

Iz I
Iz I

Now

Vx 2 + /

+y

= r 2

= r

= r represent~ a circlt of radius r and


Thus
the centre 0(0,0). Wf:' can also write
zz = r2 as the
2
2
2
equation of the circle x + y = r
(b)

CIRCLE (x- a)

(y- b)

= r2

Fig.31
y

If P(z) is a typical point on the circle,


centre C (a + ib), radius r, then PC = r
i.e. lz- wl= r, where w =a+ ib

Thus Iz - wI = r, represents a circle with


the centre at C (w = a + ib) and radius r.
2
Using the result zz = I zl , we can write
2
this equation as: (z - w) (z - w) = r
- - wz
- - -wz + ww
- = r 2 (Th ere IS
.
i.e. zz
some advantage in writing the equation
of a circle in this manner).

Fig.32

EXAMPLES: (47)

Write the equations of the following circle in the form


(a)

+ y

= 4

(b) (x - 1)

+ (y - 2)

=4

Izl

(c) x

=r
+ /

or

Iz

- wI = r:

- 4x - 6y = 0

SOLUTION:
(a)

2 + y2 = 4

(b)

(x - I ) 2 + (y - 2)
w

lzl = 2

= 4,

I + 2i

We write lz -wl = 2
or
(c)

+ y

(x - 2)

2
2

- 4x - 6y = 0
2
+ (y - 3) = 13 ,

We write

Iz

- (2 + 3i)l =

Iz

- I - 2i

centre is C(2, 3), radius =

Jl3

= 2

liJ

as the equation of this circle.

centre is (I ,2)

154

Exercise 4M
I.

(a) x

2.

+ y

= 1

(b) x

+y

= 9

(c) x

= 25

+ /

(b)

= 64

+ /

= r:

21
+ y =

(d)

2x

+ 2y

- = r 2:
zz

Express the following equations in the form


(a) x

3.

Izl

Express the equations of the circles in the form

(c) ax

+ a/ =

(d)

3x

+ 3y

= 5

Express the equations in cartesian form, stating the radiu<; und the ctntrt of each
circle.
(a)

4.

5.

l2zl = 1

(d) 4zz = 1

(c) zz = 25

Express the equations in the cartesian form, describing each locus.


<a>

Iz

(e)

l2zl=lz-ll

- 21 = 1

<b>

Iz
(f)

+ 21 = 3

Iz

<c>

- 3

i1

(c)

x
X

+ y
+ y

= 2

<d>

Iz

+ 2

i1

= 5

lz-(2+i)l=2

Express the following equations of circles in the form


(a)

6.

(b)

lzl = 4

- 2x - 2y
+ X + 3y = 0

(b)
(d)

2x

+ y

Iz

- wl

= r

+ 2x + 4y + 1 = 0

+ 2y

+ 4x + 3y + 1 = 0

Show that the following equations represent circles in an Argand diagram. State
their centres and radii.
(a)

(z - 2) (z - 2) = 4

(b)

(z - w) (z - w) = 1' where w = 1 + i

(c)

(z - 2 + 3 i) (z - 2 - 3 i) = 4

(d)

(z - 1 - i) (z - 1 + i) = 9

155
4.15 Miscellaneous Locus Problems (Including lnequations)
EXAMPLE: (48)

Draw a neat sketch of points satisfying the following conditions:


(a) Re(z) = 3 (b) Im(z) > 2
y
SOLUTION:
(a)
(a) Re(z)
= 3, z = x + iy
Re(x + iy) = 3
X = 3
2
The locus is the vertical line x = 3
Im (z) > 2 ,
z = x + iy
y >2
The locus is a half-plane above the line
y =2

(b)

EXAMPLE: (49)

Sketch the curve

WJJ!J/111/IIJ/1_ ____.
0

Fig.33
X

lzl

SOLUTION:

Izl is the distance of a point from


the origin, so the locus of I z I = 2
is a circle of radius 2, centre 0(0,0).

The cartesian equation of this circle is


X

= 4
Fig.34

EXAMPLE: (50)

Iz

Describe the locus

- 2 + 3i I

SOLUTION:

Write:

lz- (2- 3i)l = 3

... (I)

We know that the equation of the form


Iz - w I = r, represents a circle.
Hence (I) represents a circle of radius 3 and
centre (2,-3) whose cartesian equation is
(x - 2)

+ (y + 3)

= 9

Fig.35

EXAMPLE: (51)
X

Sketch the region defined by


Iz - 2 + 3 i I ~ 3.
SOLUTION:
From example (50) this region is the set of
points within and on the boundary of the
circle, radius 3, centre (2,-3)

Fig.36

156

EXAMPLE: (52)

P(z)

Describe the loci of z if


<a>
<b>

Iz
Iz

- 2
- 2

I Iz
I < lz

+ 1- i

+ 1- i

I
I
I

I
I

SOLUTION:
(a)

P(z) where z

=x

+ iy

1A

We have:

Iz

- 21

= Iz - <-1

nI ... o>

Let A be (2, 0) and

Fig.37

B be(-1,1)

The relation (1) says that PA = PB for all positions of P and from plane geometry
we know that the locus of P is then the perpendicular bisector of AB.

(b)

lz-21

<

lz-(-l+i)l

represents a set of points closer


to A(2,0) than B(-1,1).
Hence the locus of P(z) is the
region on the right hand side of
the perpendicular bisector of AB,
excluding the bisector itself.

Fig.38

EXAMPLE: (53) (may be deferred until


after Chapter 6)

Describe the locus


lz-21 + lz+21 = 6

SOLUTION:
Here A(2,0), B(-2,0)
we have PA + PB = 6 (given).
This is the condition for the locus of
z to be an ellipse.
The locus of P is an ellipse.
The foci are (.:!: 2, 0).
The centre is o.

Fig.39

Length of semi-major axis is 3


Length of semi-minor axis is .f5

2
Equation of the ellipse is x
9

2
~

157
EXAMPLE: (54)
Sketch the region defined by: (a) 0 ~ argz ~}

(b) lzl ~ 2 and 0 ~ argz ~%

SOLUTION:

(a)

argz =

Locus of

= 60

P(z) satisfying
Fig. 40

0 .{. drgz

n/3 is the set of points


within the angular region
[POX = 60 (shaded) including
the boundaries, excluding the origin.
z =0

argz is not defined for


(b)

Izl

~ 2

and

0 ~ argz ~ n/4

The region shaded is within the circle


of radius 2, centre 0, restricted in
the sector 0 ~ arg z ~ 45,
excluding the origin 0.

Fig. 41

EXAMPLE: (55)
Find the locus of w if w - z z- 1

Izl

given

SOLUTION:

We eliminate z from w
Solve for

z,

lz I
lw -

=z

then zw

II -

z --I
=z
- I

II =

=- I

su z(w - I)

II - wI = Iw - I I

but

wl '

Iz I =

.
by usmg

Hence the locus of w is a circle of radius


2
2
I
of the circle is (x - I) + Y = 4

2
and z =

and

I
r-:-w

lzl

2 ,

then

and centre (1,0). The cartesian equation

EXAMPLE: (56)

z- 2
Find the locus of z if w = - - , given that w is purely imaginary.

SOLUTION:

z- 2
z2z

w
w

--z2
1

zz

2
X

+y

+ iy

_ 2 (x - iy)

2x

+ y

2iy
2
2

+
X

+y

Now if w is purely imaginary, then Re(w) = 0


I

2
2

2x
+ y

x + y - 2x = 0
(x - 1)2 + y2 = I

or
lz-11=1
Hence locus of z is a circle of radius I, centre (1,0).

158

EXAMPLE: (57)

Describe the locus given by the equation

Iz

+ 21

2lz-2+il

SOLUTION:

Let z = x + i y, then the given equation becomes:


lx+iy+21

I<x
or

2lx+iy-2+il

+ 2) + i y I = 21 <x - 2) + Hy + o I
2 ~,_(x---2)-=-2 -+_(_y_+_0~
2

squaring both sides and simplifying:


2
2
Divide by 3
o.
3x + 3y - 20x + 8y + 16
2
2 20
8
16
0. Complete the squares.
X +y -JX+3Y+)
(x _

I~) 2

+ ( y +

jy

1~0

So the locus is a circle, centre (

, -

6:

+ 1; _ 1;
)

radius

EXAMPLE: (58)

Sketch on an Argand diagram the locus given by the equation:


arg (z - 2) - arg (z + 2) =

SOLUTION:

arg(z - 2)

LPBx

a'

B (2, 0)

1$ ' A (-2, 0)
1T
so LAPB =
Now e - 0 =
3
3 '
The locus of z, as represented by the point P,
arg (z + 2)

LPAx
1T

e
Fig. 42

is thus the major arc of the circle with AB as


a chord. This chord AB subtends an angle of

at the circumference. By symmetry, we find the

(Points A and B are excluded


from the locus)

centre on the y-axis. Now angle LACB = 120.


cos30

=~

gives r

CO = 2tan30

= 2sec30

Hence the centre is (0,

*)

J) 2

= 1;

= ~

and radius is

The cartesian equation of the circle is


x2 + ( y _

Fig. 43

159

Exercise 4N
1.

Find the Cartesian equation of the following curves, and sketch and describe
them (Exercises I, m and n may be postponed until after Chapter 6):

<g>

Iz I = 2
Iz - i I = 2
Iz I = Iz - 1 I

(i)

lz+2-3il =

(k)

Iz
Iz

<a>
<d>

(m)*

+ 2 - 3i I
- i

Iz

Iz
Iz
Iz

<b>
<e>
<h>

- 21

+ 2i

Iz
Iz

<c>
<n

+ 21

+ 2 - 3i I

- 21 = Iz + i I

lz+2+il
21 z + 2 + i

3
= 3

+ iI = 4

(j)

lz-2il

(I)*

Iz - II .+ Iz + II =
Iz I + I z - II = 4

(n)*

2lz+ll
4

2.

Sketch the following curves after giving their Cartesian equations. Describe the
curves.

(a)

arg z

n/3

(b)

arg z

(d)

arg z

n/2

(e)

arg z

3.

Sketch the following regions in the complex plane, showing whether the
boundaries are included or not.

<a>
<g>

Iz I < 2
Iz - 2 I < 3
2 ~ Iz + 2 - i I

(j)

- n/2

4.

Sketch the following regions of the complex plane, showing carefully the
boundaries (included or not).

<d>

(a)

<

<b>

~ 4
arg z < n/6

Iz I~ 3 and
Iz I ~ 4

(c)

-n/4

(f)

TT

lz I ~ 3

<e>

Iz

arg z ~ n/3
arg(z + 2)

0 ~ arg z ~ n/3

(b)

and Re (z) >.- I

(d)

Iz

<c>

<h>
(k)

+ 2 + 3i

arg (z + 2) = 5n/6
arg (z + 2i) = n/3

I <
~

n/6

I > 2
< Iz - II
- i

(i)

n/3 ~ argz ~ n/2

o> Iz -

Iz I~ 2 and
Iz I ~ 3

2~

3i

2~

(e)

I~ lzl ~ 3 and 0~ argz~ n/2

5.

For the following, describe the locus of the complex number


restricted as indicated.
z- 2
(b) w
w
z- 2' lz I = 3
lzl
z
z - 2i

(d)

6.

Find the locus of z

(a)

z - I
z

and w is purely real

(b)

z - i
z - 2

and w is purely imaginary

(c)

z - 2
z + 2

and arg w

r:z

lzl

if:

TT

=3

+ 21

and lm (z) >.,. 1

w,

--

(c)

I > Iz

n/6 ~ arg z ~ 2n/3

(c)

(a)

(f)

z - 2 + i
z + 2 - i

'

lzl

where z is

160

Exercise 40 <REVISION)
1.

For the complex number z = x + iy, find the locus of z. Describe it and draw
a neat sketch, if:

z+2

(a) arg (z - 2) = n/3

(b)

lz I= z +

(d) arg [~

(e)

lz+1l+lz-ll=3

= i]=

n /2

(c)

zz- 4 (z + z)

= 10

2.

z , z and z
are three complex numbers that lie on a circle
1
2
3
passing through the origin. Prove that the points which represent 1/z , l/z
1
2
and l/z are collinear.
3

3.

(a)

Express

(b)

Hence or otherwise, find the locus of the point which represents z on the
2
Argand diagram, if. lz - 5 + 12i I = lz- 3 + 2i I

(a)

If

(b)

Show that the locus of the point z = x + i y is

4.

The points

~ in the form a+ ib

i .
x and y are real, solve the equation -...!.L
1 + IX

(i)

a straight

(ii)

a circle if

line if

3y + 4i
3x + y

is purely real

is purely imaginary.

Find the centre and radius of this circle.

5.

Given z
(a)
(b)

= 3 + 4i and z = -3- 4i
1
2
Draw a neat sketch of the locus specified by I z - z 1 = I z - z 1
1
2
Find the Cartesian equation of the locus of z.

Show that the locus of a point represented by the complex number


z = x + i y and obeying the condition I z - z 1 = 31 z - z 1 is a circle.
1
2
Find its centre and radius.

6.

P is a variable point on the line x = 4 and


OPQR is a rectangle in which the length of
OP is twice the length of OR. Find the
locus of S in Cartesian form, where S is the
point of intersection of the diagonals.

7.

Given that (1 + 4i) p - 3q = 3 + 5i, find if:


(a) p and q if p and q are real
(b) p and q if p and q are complex conjugates.

8.

Draw neat sketches of the loci represented by the equations I z - 21 = 2 and


I z - 2 i I = 2. Find the complex numbers for the points represented by the
intersections of these loci.

9.

The point A in an Argand diagram represents the complex number 3 + 4i.


Find the complex number represented by B if OBA is an equilateral triangle.
Also find the point
D
if
OABD
is a rhombus.
(All rotations are
anti-clockwise.)

161
10.

(a)

The complex number z is given by z


(i)

Find
(b)

Im(z)

(iii)

Iz I

(iv) arg z

Iz

- 3 + i

= 2

(ii)

Re (z - i z) ~ 1

Determine the locus of the complex number z if


arg (z - 1)

(d)

(ii)

Draw a neat labelled sketch to indicate the locus of the complex number z.
(i)

(c)

Re(z)

1 + i
= 1 + r:T

i + arg (z

4). Sketch the locus on an Argand diagram.

Shade on the Argand diagram the region for which


2
n < arg z < n
and
lzl ~ 2
3
For part (d), choose a point in the region (above) and label it P. Suppose
P represents the complex number z. Then label clearly the points A, B,
2
C, D and E which represent
-z, iz, z and z + 2.

z,

11.

12.

(a)

OPQR is a square in an Argand diagram where 0 is the origin. The point


P is given by z = r(cos9 + isin9). Find the complex numbers represented,
by Q and R.

(b)

If 9 = 30, r = 1 in part (a) and the square OPQR is now rotated through
60 (anti-clockwise direction) to become OP'Q'R',
find the complex
numbers represented by P', Q' and R' in the mod-arg form.

(a)

Calculate the modulus and argument of the


(i) sum of the roots
(ii) product of the roots
2
of the equation (4- 3i)z - (3- 2i)z ~ 5 + 2i = 0

(b)

Show that the point representing sinn/3 + icosn/3 in the Argand diagram
lies on the circle of radius I and the centre at ( yT, 0).

(c)

13.

14.

2 + it
_ it show that as t
2
circle. Find the radius and the centre.
If t

IS

real and z =

varies 'the locus of z

is a

(a)

Draw the sketch (on an Argand diagram) of the region in which z lies if
both lz- (2 + i) I ~ 4 and n/6 ~ arg(z- 1 - i) ~ n/2 are satisfied.

(b)

= - 13- i, z = fi- i and z = 2i are repre2


1
3
sented by the points P, Q and R respectively. Show that the triangle
PQR is an equilateral triangle.

(a)

Complex numbers z

The complex numbers

real.
(b)

= ~ and z
1
2
Find p and r, if z - z = 4i.
1
2

=1 :

where p and r are

The complex number z satisfies the equation arg (z + 2)


(i)

Sketch the locus of point P which represents z.

(ii)

Find the modulus and argument of z when

(iii) Hence find

Izl

= n /4.

is the minimum.

z in the form x + i y when Iz I is the minimum.

162

15.

16.

(a)

Calculate the modulus and argument of the complex number which


represents the product of the roots of the equation
2
(4 + 3i)z - (1 - i)z + (2 + i)
0.

(b)

Find the two square roots of


diagram.

(a)

Find all six roots of -1, expressing each in the form


them on a circle in an Argand diagram.

(b)

Solve z

+ 16

3 + 4i

and sketch them in an Argand


x + iy

and show

-30i completely.

moves in a straight line parallel to


2
the imaginary axis. Prove that the point Q which represents z moves in a
certain parabola. Find the focus. Also describe the locus of Q when P moves
on the imaginary axis.

18.

(a)

Prove that

(b)

Show that the points represented by the complex numbers z , z


and
1 1
z - 2iz
1
2
_ i
form the vertices of a right-angled triangle.
1 2
z 1 - z3]
n
z - 2iz
(Hint: arg z _ zJ
= 2 , where z 3 = 11 _ 2 i 2 )
[ 2

19.

20.

The point P (which represents z

=x

17.

zl + z2
zl - z2

+ iy)

is purely imaginary if

Iz

2
Let z = x + iy and w = u + iv = (z - 0 + 2 be complex numbers in an
Argand diagram.
Show that as z moves along the y-axis from 0 (0, 0) to
A(0,2) the point w moves along an arc of a certain parabola. Find the
corresponding points on this parabola and the Cartesian equation of the parabola.
(a)

For the complex number z

= r(cosG

+ isinG), given that r

find the modulus and argument of (i) z


(b)

(a)

(ii)

= 4 + Ji and lz 1 = 10, find the


2
1
(i) maximum value
(ii) minimum value of

= 5,

(iii) 1/z

G=

j,

(iv) i z

If z

(iii) express the corresponding values of z


21.

Iz 1

Find the modulus and argument of z

Iz 1 +

2
in the form

= 131 ++ !I

iy

Find the smallest

positive integer, n, such that zn is real; hence find zn for this value of n.
(b)

2
Sketch the circle whose Cartesian equation is (x - 2) + /
= 4
The point A reP.resents the complex number z = r(cosG + isinG).
(i)
Express lzl in terms of G if A lies on the given circle.
(ii)

Find

I~

and arg { ~) if A lies on the circle.

(iii) Show that as A moves on the circle, the point P representing


lies on a certain line.

1/z

163

22.

The complex numbers z = x + i y and w = u + i v are related so that


w

= z - .!z .

Show that:

x X

Find the

+ y

~ocus

and

of w if

+ --::;:-'--y-:;
2
2
X
+ y

lz I

(a)

(b)

lzl = 1.

Describe each locus geometrically.


23.

Find r and 9 if

24.

Solve the equations:

r (cosa + isin9)
3
z +I
(a) - - -

=i

3cis45 + 2cis30
5cis 60 - 4cis 30
(b) z

iz

z - l
25.

The four complex numbers u, v, w and z are represented on the complex plane
by points P, Q, R and S respectively. If u + w = v + z and u - w = i (v - z),
determine the possible shapes of the quadrilateral PQRS.

26.

(a)

9
z - l
6
3
Show that - - - = z + z + I
3
z - l

(b)

Prove that:
z 6 + z 3 + I = (z 2 - 2zcos 2n + I) (z 2 - 2zcos 4n + I) (z 2 - 2zcos 8n + 1)

(c)

Deduce from (b) that:

2cos39 + 1 = 8(cos9- cos


27.

(a)
(b)
(c)

Factorise

3
z + I

2n
4n
8n
9 ) (cosa- cos 9 > (cosa- cos 9 >

into linear

~nd quadratic factors with real coefficients.

5
2
n
2
3n
Prove that z + l = (z + I) (z - 2zcos + I) (z - 2zcos
5
5 + l)
.
15
3 5
By observmg that z
+ 1 = (z ) + l, prove that
15
2
6
3
n
6
3
3n
z
+ 1 = (z + 1) (z - z + I) (z - 2z cos
+ 1) (z - 2z cos 5 + l)

=- ~ ,

3
w

28.

Show that if lw I

29.

1 + sinS + icosa
.
.
Prove that 1 +Sin
. a -lOS
c . 9 = sma + 1COS9 and deduce that

=1

and Re(w)

then

. n
.
n]5 + 1[ 1 + Sin
n]5
. 5n - !COS
.
5
[ 1 + Sin 5 + !COS 5
30.

Express
integers.

.
4k + 1
[ 1 + 1tan ~n

=1

m m
. t he f orm a+ 1. b , where m and k

are

164

CHAPTER 5 POLYNOMIALS
5.1

Introduction

In 3U Mathematics, we learned many important properties of polynomials in the


real variable x. In this chapter we shall study these properties and a few more
theorems about polynomials over the complex field C. Let us first revise some of the
important work on polynomials.
The general nth degree polynomial function cf a real variable x is of the form:
P ( x) " a x

n-1

n- 1

+ a

n-2

n- 2

+ +

x + a

.J 0

where the coefficients a , a , are real, and n is a non-negative integer.


0 1
OPERATIONS ON POLYNOMIALS

When two or more polynomials are added, subtracted or multiplied, the result is
another polynomial. The division of one polynomial P(x) by another polynomial A(x)
may or may not be exact. When P(x) is div1ded by A(x), we can write the identity:
P(x)
where, P(x)
Q(x)
A(x)
R (x)

" Q(x) . A(x) + R(x)


is called the dividend,
is called the quotient,
is called the divisor,
is called the remainder. ..

[deg R (x)

< deg

R (x) " 0, P(x) is exactly divisible by A (x), then Q(x) and


factors of P(x).

If

A (x)]
A (x) are called the

THE REMAINDER AND FACTOR THEOREMS

In the division of P(x) by A(x), if A(x) is a linear polynomial


degree of R (x) must be zero, i.e. R (x) is a constant. We write:

x - a,

thtn the

P(x) " (x - a) Q(x) + R


Substitute x " a,

then

P(a) " 0 . Q(a) + R


R
" P(a) .
So, the remainder theorem is:
If P(x) is divided by (x - a), the remainder is P(a). Further, it P(a) " 0, then x - a is
a factor of P(x), and conversely, if x - a 1s a factor of P(x) then P(a) " 0. The result
is known as the factor theorem. A value of x, such that P(x) " 0, IS called a ZERO of
the polynomial P(x) or a root of equation P(x) = 0. For example, x = 2 is a zero of
3
P(x) = x - 8.

165

EXAMPLE (1)

Find the remainder when P(x) = 2x 3 - 3x


A(x) = X - 2

SOLUTION:

2x
X-

+ x - 2

) 2x 3 - 3x 2 - ~x - 6
3
2
2x - 4x
2
x - 4x

- 2x

~x

- 6 is divided by

If we only want the remainder, we


use:

R = P(2) = 16- 12- 8- 6 = -10


The remainder is -10. We can write
2
P(x) = (x - 2) (2x + x - 2) -10

- 2x - 6
- 2x + 4

- 10
EXAMPLE (2)

Show that x + 3 is a factor of x


other factors.

SOLUTION:

We have P(x) = x
P(-3)

= -27

- 3x + 18 and hence find the

- 3x + 18, x- a = x + 3, so a= -3 and

18

=0

Since P(-3) = 0, x + 3 is a factor of P(x) we divide P(x) by


x + 3 to find other factors.
2
2
Observe that the missing term x
x - 3x + 6
X +

) 3
X

+ 0. X - 3x + 18
+

3/

- 3x
- 3x

2
2

is arranged as 0 . / . This is
important and rtduces the chance

- 3x

of t'rrors.

- 9x
6x + 18
6x + 18
0

over the R-field)

P(x) is /

The other factor of


- 3x + 6. (Irreducible

EXAMPLE (3)

Find the values of the constant m if the polynomial


3
2 2
P(x) = 4x - m x - 4mx + 64 is divisibk by x + 1, hence find the
other fdctors of P(x)

SOLUTION:

P(x) = 4x

2 2
- m x - 4mx + 64 is divisible by x + 1, i.e. x - (-1}
2
P(-1} =- 0 gives - 4 - m + 4m + 64 = 0
2
:. m - 4m - 60 = 0, i.e. (m- 10) (m + 6) = 0

m = 10 or m

= -6

2
3
2
3
For m =- 10, P(x) = 4x - 100x - 40x + 6~ = 4 (x - 25x - lOx + 16)
3
2
Divide
x - 25x - lOx + 16 by x + 1, then
2
P(x) = 4(x +I) (x - 26x + 16), ( / - 26x + 16 is irreducible over R-field)
2
3
3
For m = -6, P(x) = 4x - 36x + 24x + 6~ =- 4(x - 9x + 6x + 16)
3
2
Divide
x - 9x + 6x + 16 by x + 1, then
P(x) = 4(x +I) (x 2 - lOx+ 16) = 4(x + l)(x- 2) (x- 8)

166

EXAMPLE: (4)

Given that x
roots.

=2

is a root of x

- 4x

+ 14x - 20

= C,

find the other

SOLUTION:

2
3
Divide x -4x +14x-20 by x-2
2
2
(x 3 - 4x + 14x - 20) = (x - 2) (x - 2x + 10)
2
The equation is (x - 2) (x - 2x + 10) = 0

l=3b -

2 +
2
.
.
x = 2 (given
root ) or x - 2x + 10 = 0 , I.e.
x -~
- 1 _+ 3.1
Observe that the complex roots 1 + 3i and 1 - 3i are conjugate. We shall later
prove that the complex roots of a real polynomial occur in conjugate pairs.

EXAMPLE: (5)

Solve: x 3 - 3x 2 + 4x - 2 = 0.

SOLUTION:

The constant term 2 = 2 x 1. This suggests we try


2
3
Let P(x) = x - 3x + 4x - 2
P(2) = 8 - 12 + 8 - 2

:1

x =

2,

1 for the roots.

P(l) = 1 - 3 + 4 - 2 = 0

x = 1 is a root of P(x) = 0
Divide P(x) by x - 1
:. P(x) = (x- 1) (x 2 - 2x + 2)
The roots of

P(x) = 0 are

1,

2~

r-4

i.e.

1, 1

Again observe that the complex roots occur in a pair of conjugates.

167

Exercise SA
Perform the following divisions, and check the remainder by using the remainder
theorem.

I.

3.

- 2i + 3x - 1) 7- (x - 2)
4
3
2
2
(x + 2x + 2x - 2x - 3) 7- (x - 1)
(x

In each of the following, decide whether

5.
6.
7.
8.

P(x)=x

2.

- 2x + 3x - 2) 7- (x + 2)
3
2
(2x - x + x - 1) + (x - 1)

(x

4.

A(x) is a factor of

-2x +x +x-6
3
P(x) = 2x - 3x + 1
2
3
P(x) = x + 2x - x + 6
2
3
P(x) = 5x + 7x + 3x - 1

P(x).

A(x)=x-2
A(x) = x + 1
A(x) = x + 3
A(x) =- x + 3

Find the remainder of the following without actually dividing, i.e. use the remainder
theorem.
9.

P(x) = 4x

10. P(x) = 5x

3
4
4

- 3x
- 2x

2
3

+ x + 8

A(x) = x + 3

11. P(x)=2x -4x +5


2
3
- ax + bx + 2

Find values of k such that

A(x) = 2x + 1

A(x) is a factor of

= x4 -

3
3k x + 3x - 1
2
3
14. P(x) = x .:. kx + 4x - 4
2
3
15. P(x) = x - 3x - 6kx + 8k
4
3
16. P(x) = x + kx + 7x + 21

Use the factor theorem to find the value of


and hence find the zeros of P(x).
17. P(x) = 3x

3
3

- 12x

+ 2

A(x)

12. P(x) = x

13. P (x)

=x

- x + 7

- llx- k

18. P(x) = 2x - 6 i + kx + 4
2
3
19. P(x) = k x + x - 8x + 6
4
3
2
20. P(x) = x + x + k x + 4x - 24

A(x)

=x

- 2

A(x)

=x
=X
=x

- 1

P(x).

A(x)
A(x)

- 2

A(x) = x + 3
k

that makes A(x)

A(x)
A(x)
A(x)
A(x)

=X
=X
=X
=X

+ 3

factor of

P(x)

168

5.2

Zeros of a Polynomial/Multiple Roots

Rational or Integral Zeros


We use the formula x

=-

2
b !. /b - lfac
2a

zeros of a second degree polynomial P(x)

(or factorise by inspection) to find the

= ax 2 + bx

+ c, but it is generally very

difficult to find the zeros of higher degree polynomials such as 3xlf + 2x

- x

+ 7.

This is because some, or even aJJ the zeros of a polynomial may not be integers,
2
e.g. x + 2 = 0 has no real zeros, let alone integer ones. The following theorem is
very useful in finding rational zeros (if any), though we shall mainly be concerned
with zeros which are integers.
Theorem:
Let P (x )

= anx n

coefficients. If x

+ an_ x n-1 + + a x + a

=%,

must be a factor of a

Proof:

Since x
n

:. a ( !:.)
n s

be a polynomial with integer

and s must be a factor of a

is a zero of P(x), then P<%>


n-1

where % is in its lowest terms, is a rational zero of P(x),

then r

=%

r
+ + a 1 ( s) + ao

( !:.)
n- 1 s

0::

= 0.

.
Multiplying both sides by s n and rearrangmg:

anr

an-{

Or, r(a r
n

n-1

n-1
+ a

n- 1

n-2

+ air. s

n-1

. s + + a s
1

n-1

- aos
)

n
(1)

The relation ( 1) shows that r is a factor of a sn But r cannot be factor of


0
s

as r and s have no common factor, hence r must be a factor of a In


0

the same manner, we can prove that s is a factor of an. Further, if an


is monic and the zeros of

P(x) are integers as s must be a factor of I.

1, P(x)

169

Find all the zeros of P(x) =

EXAMPLE:

2x

- 3x - 6

SOLUTION:
Since P(x) is monic, the integral zeros must be the factors of the constant term -6.
All the possible factors are .!: I, .:!::2, .!: 3, .!: 6. We find that only x = -2 satisfies
P(x) = 0, so -2 is a zero of P(x).
P(x) is of degree 3 and therefore has 3 zeros.
The other two zeros must be irrational or complex. To find these, we divide P(x)
by X + 2.
2
:. P(x) = (x + 2) (x - 3)
Hence the zeros of P(x) are: -2, .!: Vf.
It is possible that
EXAMPLE:

P(x) has no integral zeros, as the following example shows.

Show that

P(x) = x

- x

- 2x + 6 has no rational zeros.

SOLUTION:
All the possible factors of 6 are: .!: 1, .:!:: 2, .!: 3, .!: 6.
None of these numbers satisfy P(x) = 0, hence P(x) has no integral zeros. Since a
fourth degree polynomial must have 4 zeros (see section 5.3, theorem 3), we conclude
that the zeros of P(x) are either irrational or complex.
EXAMPLE:

Find all the roots of

5x

+ 28x

+ I Ox - 3

=0

SOLUTION:
2
3
Let P(x) = 5x + 28x + lOx - 3 = 0.
To make our work easier, we transform this equation so that the leading term has a
coefficient equal to I.
3
Observing that the coefficient of x
is 5, we multiply the equation by 25 and then
put y = 5x.
(5x)
Or y

3
+

+ 28 (5x)

28y

+ 50

(5x) - 7 5

=0

+ 50y - 75 = 0

(1)

By testing the possible factors of 7 5 i.e . .:!:: I, .!: 3, .!: 5, .!: 15, .!: 25, .!: 7 5, we find that
y = -3 is a root of (1), hence
P(y) = (y + 3)

<l + 25y - 25)

The roots of (I) are:

y = -3,

- 25.!: 5
2

ff9

Using x = ~ , the roots of the given equation are:


-3

X=

S'

-5.:!::

V29

170

Multiple Roots
Consider the polynomial

P(x) = (x - 2)

2 (x 2

5x + 6)

x = 2 is called a double root of the equation P(x) = 0. Similarly for


P(x) = (x - 2) 3 (x + 2), x = 2 is called a triple root of P(x) = 0. In general, we call
b, a multiple root of order r if
Q(b) 1- 0 .

P(x) = (x - b)r Q(x),

We also speak of a root of multiplicity r.


Theorem:

P(x)

= 0,

If

x = b is a root of multiplicity r of the real polynomial equation

then

=b

is also a root of the derived polynomial equation

~~ ~

0 of

multiplicity (r - J ).
Proof:

Let

P(x)

(x - b)r Q(x) , where Q(b) /. 0

dP
dx

r(x - b)

r-l

Q(x) +

dQ
ax
. (x

(x - b{-l [r Q(x) + (x -

- b)

b)~~

(x - b)r-l S(x) , where S(b)

0, S(x) is the polynomial in the bracket.

This clearly shows that x = b is a root of multiplicity (r - I) of the derived equation


dP
dx = O.
EXTENSION:
If

(x - a{

I.

r-1 .
(x - a )
IS a factor of

2.

2p
(x - a)r- 2 is a factor of d
and so on.
2
dx

is a factor of

EXAMPLE: (6)
SOLUTION:

dP
dx

Show that
3(x - 2)

P(x), then
c!P

dx

~:

has a zero of multiplicity 2 if

(x

+ 2) + 2x(x - 2)

2
(x - 2) [3(x 2 + 2) + 2x (x - 2)]
dP has x = 2 as a double root
dx

P(x) = (x - 2)

(x

+ 2)

171
3
Solve the equation x - 3x - 2 = 0 , given that it has a double root.

EXAMPLE: (7)
SOLUTION:

Let

P(x) = x

~~

- 3x - 2

= 3x2 - 3 = 3(x - l) (x + l) = 0

The possibilities for the double root are

gives

x = .:!:. l

and -1

Now P(l) = l - 3 - 2 J 0 and P(-l) = - l + 3 - 2 = 0


3
x = -I is a double root of P(x) = x - 3x - 2 = 0
3
2
Divide out x 3 - 3x - 2 by (x + l ) , i.e. / + 2x + I, then x - 3x - 2 = (x + I f(x - 2)
3
The roots of x - 3x - 2 = 0 are -I, -I, 2
EXAMPLE:(&) Show that:
(a)

(b)

x
x

+ px + q = 0 has a double root, if

+ px + q = 0 has a double root, if

= 4q
3
2
4p + 27q = 0

SOLUTION:
(a)

Let P(x) = x

~~

= 0 gives

dP
then dx = 2x + p

+ px + q,
2x + p = 0,

hence x

= -~

2
is a double root of x + px + q

= 0.

Substituting x = - ~ in the equation,

~-IT+q=o
We

<:..111

whence p

4q

2
2
check this condition for x + px + q = 0 by using fl. = p - 4q = 0 for equal

roots.
(b)

P(x) = x

dP
+ px + q, then dx

3x

+ p = 0 gives x

= -)

For x to be real, p " 0.


To avoid irritating fractional powers, we write the original equation
2
x (x + p) = -q as
x 2 ( x 2 + p )2 -- q 2 , now put x 2 -- - 2
3
- 2 (- 2 + p) 2 = q 2 which gives 4p 3 + 27 q 2 = 0
3 3

172

Exercise 58
Find the real roots of the following equations, given that each equation has a double
root:

2.

3
2
x - 2x - 4x + 8 = 0

4.

I.

3.

5.

Find the real roots of each equation, given that each has a triple root:

- 7x

- 2x

II x - 5 = 0

- 15x + 36 = 0

(a)

P(x) = x

(b)

P(x) = x

4
4

2x

- 6x

- 12x

+ 12x

2
2

x + I = 0

14x- 5 = 0

- lOx + 3 = 0

2
d P
[Hint: - - = 0 and P(x) = 0 have a common root]
2
dx

6.

Find the point where the tangent to the curve


(a)

y = x

(b)

y = x

- 2x

- 2x

2
2

+ 3

at x = 2

+ I

at x =

meets the curve again at Q.


7.

Find the viilues of


root:
(a)

=x3

P(x)

(b)

P(x) = x

+ 3x

9x + k

=0

- 24x + k = 0

P(x) = 0 for each value of k.

Find the roots of


8.

such thdt each of the following equations has a double

+ 6x

Find the coordinates of Q.

Find all the roots of the following equations.


then by division of P(x) find the other roots.
(a) x
(c)

2x

(d) x
(f)

4
4

- x
3

- 4x + 4 = 0
2

- 9x

- 6x

7x

(b) x

6 =0

- x

2x

- 5x - 6

=0

(Hint: Multiply by 4, let y = 2x)

- 7/ - 13x - 6 = 0

First find all the integral roots,

+ 34x + 8 = 0

(e) x

- 4x

x + 6 =0

2
3
(g) 4x - lix + x + I = 0
(Hint: multiply by 16, y = 4x)

3
(h) 4x - 5x - 2 = 0
(Hint: y = 2x)
(i)

9.

4
3
2
(Solve for tan 9 only): tan 9 - 2tan 9 - I 3tan 9 + 14tan9 + 24 = 0

Prove the theorem:


If

x = r is an integral zero of the monic polynomial

P(x) = x

11

n-1

+ + a

, then r

must be a factor of a

173

5.3

Polynomials over the Complex Field

2
A polynomial such as x + 4 has no real factors.
2
x + 4.
2
2
4i = (x - 2i) (x + 2i)
x + 4 =

2
Using i = -I, we can factorise

l -

Thus a real polynomial with no real factors can be factorised over the complex field.
We shall now study several theorems which are essential for the factorisation of
polynomials.
You might wonder whether every polynomial has a zero.
answers this question.
Theor-em 1:

The following theorem

The Fundamental Theorem of Algebra

If P(x) is a polynomial of degree n


complex field.

I, then P(x) has at least one zerc over the

The proof is beyond the scope of this book.


mathematician Gauss in 1799, when he was 22.

It was first proved by the German

From this theorem and the factor theorem, it follows that if P(x) is a polynomial of
degree n ) I, then there is at least one number c
such that:
1
P(x) = (x- c ) Q (x)
1
1
If we repeat the process on the polynomial Q (x),

we get:

P(x) = (x- c ) (x- c ) Q (x)


1
2
2
If we repeat the process

n times, we obtain the following:

If P(x) is a polynomial of degree n


Theor-em 2:
c 1' c 2, , en (not necessarily distinct) such that

I, then there exist n numbers

P(x) = an (x - c ) (x - c ) . (x - en)
1
2
where anf. 0 is the leading coefficient.
Theorem 2 ensures that every polynomial of degree
n ~ I can be completely
factorised into n linear factors. Each of the numbers c , c , . en is a zero of
1 2
P(x) and hence a root of the equation P(x) = 0.
Now let P(c) = 0, then:
P(c) = an (c - c ) (c - c ) (c - en) = 0
1
2
This is possible only if
theorem 2 that:

is one of the numbers c , c , . , en. So it follows from


1 2

Theorem 3: A polynomial P(x) of degree n

1 has at most n distinct zeros.

This ensures that the equation P(x) = 0 has at most n distinct roots.

174

EXAMPLE: (9)

3
The polynomial P(x) = (x - 2) (x - 3) (x + 1) has 5 zeros, 2,2,2,3,-1.
Here 2 is a zero of multiplicity 3.

EXAMPLE: (10) The polynomial


1 -1 +

3
P(x) = x - 1 = (x - 1)

/3i

<i + x +

1) has

3 zeros:

'
2
3
Observe that the complex zeros of the polynomial x - 1 are a pair of conjugates.
An important fact about the complex zeros of P(x) with real coefficients is given in
the foHowing theorem.
Theorem 4: If a + ib is a complex zero of the polynomial P(x)
having real coefficients, then a - ib is also a zero of P(x).

of degree n

1,

It follows that if a + ib is a root of P(x) = O, then so is a - ib.


There are two methods of proving this theorem.
Method 1:

Using the properties of complex conjugates

We shall need the following properties of conjugates.


If z

and z

(a)

(z

+ z )

are two complex numbers and m is a real number, then:

+ z

(b)
(c)

(z l>n

Let z = a + ib be a complex zero of P(x)

a zn
n

a z

= a n xn

ao = 0 ' hence:
0

By property (a):
anz

+ an-l z

n-1

By property (b):
+ alz + ao

By property (c):
- n

an (z)

(- n-1

+ an-1 z)

+ a 1 z + ao

But the L.H.S. of the above equation is P(z)


This completes the proof.
essential.
Verify that x

The requirement that

-2i is a root of

+ 2xi = 0

but x

P(x)

have real coefficients is

2i is not.

175

Alternative Proof:

=a

Let z

+ ib

z =a

We use

be a complex zero of the polynomial P(x) with real coefficients.

- ib, z + z

(x - z) (x - z) = x
If P(x) is divided by

= 2a,

zz

=a2

+b

to form the product:

- (z + z) X + ZZ = x - 2ax + a

+b

2
2
2
x - 2ax + a + b , the remainder must be of degree

< 2.

Hence,
P(x)

[x

- 2ax + a

2
+ b ] Q(x) + mx + n

[x - (a + bi)] [x - (a - bi)] Q(x) + mx + n


Since a + ib is a zero of

P(x), P(a + ib) = 0. Substituting x = a + ib, gives:

0 = 0 + m (a + ib) + n
(am + n) + bm i = 0 = 0 + 0 i
Comparing the real and imaginary parts separately, am + n = 0 and bm = 0 and
hence m

=0

and n

= 0.

(b 1- 0)

P(x) = [x - (a + ib)] [x - (a - ib)] Q(x)

Thus,

Hence P(a - ib)

= 0,

i.e. a - ib is also a zero of P(x).

WORKED EXAMPLES

EXAMPLE: (11)

(a)
SOLUTION:
(a)

Factorise over the complex field

x 2 + 2x + 3

(b)

+ 3x + 2

By completing the square

+ 2x + 3

(x 2 + 2x + I) + 2
(x + I)

(x+l)

(b)

4x

4x 2 + 3x + 2

[i

+ 2

-2i

[x + 1 +

.f2 i] [x

4[ (

i)

4 [x +

+ I -

f2 i]

~~]

'P

i] [x +

j - 'P i]

=-l]

176

Factorise over the complex field

EXAMPLE: (12)
(a)

- 1

(b)

3+

3X

(c) x

- X - 3

- 1

SOLUTION:
(a)

- 1

(x

- 1J tx

+ 1)

(x - 1) (x + 1) (x - i) (x + i)
(b)

Let P(x)

+ 3x

- x - 3. The constant 3

=3 x

1, so we try .!.1, .!. 3

1+ 3 - 1 - 3 =0

P(l)

x - 1 is a factor of P(x)
Divide P(x) by x - 1

P(x)

(x - 1) (x 2 + 4x + 3)

(x - 1) (x + 1) (x + 3)

It must be remembered that R is a subset of C.


(c)

(x - 1) (x

X-

+ x + 1)

(x - l) [( x +

~) 2 + ~]

(X- 1)(X+2+Tl
1 .[3.)(X+2-Tl
1 13.)

Show that x = 1 + i is a zero of the polynomial


2
P(x) = x 3 + 2x - 6x + 8 and hence factorise P(x) completely
over C.

EXAMPLE: (13)

SOLUTION:
P(x)

= x3

2x

- 6x

We know that if 1 + i is a zero of P(x) with real coefficients, then


zero of P(x). We form the product:
(x - z) (x - Z) where z
x

2
2

- (z +

=1

+ i,

=I

- i is also a

- i, z + z = 2, zz = 2

z) X + ZZ

- 2x + 2

2
Divide P(x) by x - 2x
:. P(x)

(x + 4) (x

2
- 2x + 2)

(x + 4) (x - 1 - i) (x - 1 + i)

Note: If z = a + ib is a zero of real P(x), then

z = a - ib

is also a zero of P(x).


2
It is easier to divide P(x) by (x - z) (x - z) = x - 2ax + a 2 + b , than by
x - a - ib.
2

177

Exercise SC
Factorise the following quadratics over the complex field C.
1.
2

4.

X +X +

7.

4x

- x + 1

5.

8.

2x

One zero of the polynomial


field C.

9.

2.

3.

+ 3x + 5

6.

+ 1

P(x)

3
2
x - 3x + 4x - 4 ;

P(x)

is given, find the other zeros over the complex


X =2
X =2

10.

P(x)

2x

11.

P(x)

3
2
X +X - 7x + 65 ;

12.

P(x)

13.

P(x)

4
3
2
x - 2x - 7x + 26x - 20 ;

X =2 + i

P(x)

4
3
2
X + 3x - x - 13x - 10 ;

X=-2-i

14.
15.

2x

P(x)

2x

- 2x

- 5x

+ 1lx

- x - 6 ;

X = 2 - 3i

+ 6x - 2 ;

+ 20x

X=1 - i

X =i - 2

+ 7x - 10 ;

Factorise completely over the complex field:


4

[Hint: P(x)

16.

17.

4
X + 16

18.

19.

20.

Evaluate the following in the form a + ib.

4
6

(a)

+ 3x

+X

+4

+ 1

- 1

P(x)

+x

(x

[Hint: P(x)

(x

[Hint: P(x)

(x

[Hint: P(x)

(x

+x + 1

for

2
2
+ 2) - x ]

2
2
+ 4) - 8x ]

2
2
+ 1) - x ]

=1 -

- 1) (x

+ x 2 + 1)]

2
[Hint: Write P(x) = x (x + l) + (x + 1) and substitute]
(b)

P(x)

2
4
3
x - 2x + x - 3x + 2

[Hint: P(x)

=x 2 (x 2 -

for x

2
2x) + x - 3x + 2]

=1 + i

178

5.4

Relation between Roots and Coefficients of P(x) = 0

In 3U Mathematics we have studied the relations between the roots and coefficients of
polynomials equations P(x) = 0 of degrees 2, 3 and 4.
1.

ax

2.

ax

2
3

+ bx + c = 0
+ bx

+ ex + d = 0

'

S = --ba

b
+ s +y = -a

aS

ax

+ bx

+ex

c
a

+Sy+ya

--ad

aSy
3.

c
a

+ dx + e = 0

b
a + s +y+o=-a:
ras

= ~a '

raSy=-

d
3,

aS yo

The following two theorems are used in establishing the relation between the
coefficients and the roots of the polynomial equation.
P(x) = anx

+an_ x
1

n-1

+ ... +a 1x+a =0.


0

Theocem (5):
If a polynomial P(x) of degree n ~ 1 vanishes for more than
values of x, it is the zero polynomial, i.e. an = an-l = = a = 0
0

Referring to the theorem (3), i.e. a polynomial of degree n cannot have more than n
distinct zeros, we find that the assumption that P(x) is not the zero polynomial
contradicts the fact that the polynomial vanishes for more than n values of x.
Hence P(x) is the zero polynomial.
EXAMPLE: (14)

Show that a = b = c = 0 if P(x) = ax


x = m, n, r (3 distinct values)

+ bx + c

SOLUTION:
We have: am
an

a/

+ bm + c = 0

.. (1)

+ bn + c = 0

. (2)

+ br

c= 0

Subtracting, we get: a(m


Similarly

(3)
2
- n ) + b(m - n) = 0 ,

a(m + n) + b = 0

. (4)

+b =0

(5)

a(n + r)

Subtracting (5) from (4) : a(m - r) = 0 ,

m 1- n

m 1- r

:. a = 0 , then from ( 4) : b = 0, then from (1) : c = 0

vanishes for

179

lbeorem (6):

If two polynomials, each of the degree


n

Q(x) = Ebnx
an = bn '

n, P(x) = Ea xn and
n

have equal values for more than n values of x, then:

an-1 = bn-1' ao = bo

Proof: We wr:ite R (x) = P(x) - Q(x) = 1: (a - b ) xn


n

Now R (x) is a polynomial of degree


x, hence by the theorem (3):
an- bn = 0 '
an-1 - bn-1 '
This proves the theorem.

n and it vanishes for more than n values of


ao- bo = 0.

RELATION BETWEEN ROOTS OF P(x) = 0


Let

x , x , , xn be the n roots of P(x) = 0. We divide out P(x) by the leading


1 2
coefficient an and write the identity:
X

an-1
a

+--X

n-1

an-2 n-2
+--X
+
an

R.H.S. = xn - ( Ex )x n-1 + (Ex x )x n-2 + .. + (-1} n x x xn.


1 2
1
1 2
comparing coefficients on each side.
(1)

(2)

sl = Sum of the roots= XI + x2 +

an-I
+X=--n
an

an-2
s2 = Sum of products =XI x2 +XI x3 + = +-aof roots taken
n
two at a time
and so on.

(n)

sn = Product of all
roots

These relations also help us to write down the equation whose roots
are given.
Xn - S l

n-1 + S X n-2 + + (-1 )nSn = 0


2

The relation for


equations.
S

xl' x 2, , xn

is frequently needed in many problems of solving polynomial

= Sum of the roots = x

+x

n-1
coeff.
of x _
= _..;;;...;.
____

+X

coeff. of xn

180
WORKED EXAMPLES

If a, 13, y are the roots of x - 2x

EXAMPLE: (15)

(a)

+ 13

+ y

(b)

a + 13

+ x + 3 = 0, evaluate:
+ y

(c)

4
4
4
a+f3+y

SOLUTION:

a + .8 + y = 2 ,

We have:
2

+ 13

aS + 13y + ya = 1 ,

2
+ l=(a+ 13+y) -2(af3+8y+ya)=4-2=2

(a)

(b)

A formula to convert l:a


a simple trick:

Re-write the equation as x


a

af3y = - 3

a3 +

= 2 (a

in terms of l:a and l:a8


3

= 2x

is very difficult, so we try


.

- x - 3 and substitute x = a, B , y

and add:

2
2
+ B + y >- (a + B + y ) - 3 - 3 - 3

=2x2-2-9
= -7

By using the formula, we can verify that


ra

2
= (l:a). (l:a -l:a8) + 3a8y = 2 (2- 1)- 9 = -7

..

There is no known formula that relates l:a


follows:

(c)

Multiply the given equation by x and write: x

with l:a etc. so we proceed as


4

= 2x 3

2
- x - 3x

Substitute a, B, y and add:


4
3
:. ra = 21:a - r i - 31:a =2. (-7)- 2- 3 (2) = -22
4
3
2
Solve P(x) = x - 3x - 6x + 28x- 24 = 0, given that P(x)= 0 has a
root of multiplicity 3.

EXAMPLE: (16)
SOLUTION:

2
dP
d P
Since P(x) has a root of multiplicity 3, P(x), dx and - - have a common zero.
2
dx
2
Now dP = 4x 3 - 9x 2 - 12x + 28 and d p = 12x 2 - 18x- 12
dx .
dx 2
2
Solving 12x - 18x - 12 = 0 + (x - 2) (2x + I) = 0
X

=2

Since P(-

or

4> /.

-zI

O, P(2) = 0, x = 2 is the triple root of

P(x)

Now the sum of the roots is:


2 + 2 + 2 + x = 3, so x = -3, hence the roots are:
4
4
2, 2, 2 and -3

181
Transformation of Polynomial Equations:
I.

The equation whose roots are the reciprocals of the roots of a given equation, is
obtained by putting x = ! in the given equation.
y

2.

The equation whose roots are those of a given equation multiplied by a constant
m, is obtained by putting x = X. in the given equation.
m

3.

The equation whose roots differ by a constant k from the roots of a given
equation is obtained by putting x = y + k (k > 0 or k < 0) in the given
equation.

4.

The equation whose roots are the squares of the roots of a given equation is

.JY

obtained by putting x =

in the given equation.

If a, S, y are the roots of


EXAMPLE:
whose roots are:
(a)

a'e':Y

(b) 2a, 28, 2y

(d)

a - 2, B - 2, y - 2

(e)

2x

+ 3x

+ x -

2 + a, 2 + B, 2 + y

(c)

2 , B2 , y 2

5 = 0, form the equation

(f) -a , - B , -y

SOLUTION:
(a)

Let y

I
=x
,

x = -I (f rom
y

P ut

.. 32
y

2
y

I
+ y

5 =0

5x

=0

2 + 3y + y 2 - 5 y 3 = 0

..
givmg
- x

Let y = 2x, since x = a, B, y


From

Y=a.sy

. t he given
.
.
2 x 3 + 3x 2 + x - 5
m
equation

= -XI )

The required equation is:


(b)

since x = a, B, y ,

- 3x - 2 = 0.
y = 2a, 2B, 2y

. t h e given
.
.
2x 3 + 3x 2 + x - 5 = 0
equatiOn,
2x = y, we put x = ~ m

2 3
3 2
.t.... +2.L +t -5=0
8
4
2
Multiplying by 4,

+ 3/ + 2y - 20 = 0

So, the required equation is x


(c)

3x

2x - 20 = 0.

Let y = 2 + x, since x = a, B, y,

y = 2 + a, 2 + B , 2 + y
3
2
Substituting x = y - 2 in the given equation, 2x + 3x + x - 5

=0

2(y- 2) 3 + 3(y- 2) 2 + (y- 2)- 5 = 0


This simplifies to 2y 3
9/ + 13y - II
2
3
2x - 9x + 13x - II = 0

0, hence the required equation is:

182

(d)

Let y = x - 2, then substituting x = y + 2 in the given equation:


2 (y + 2) 3 + 3 (y + 2) 2 + (y + 2) - 5
2y 3 + l5y

+ 37y + 25

=0

Let y = x 2 then putting x =

2
Cl. '

s'

which simplifies to:

=0

:. 2x 3 + l5x 2 + 37x + 25
(e)

=0

is the required equation.

JY,

we obtain the equation whose roots are

'Y

3 2
:. 2y ' + 3y + Vy- 5 = 0
To remove the radical signs, we write:
3y - 5 = -

JY (2y

(y3/2 = y

+ 1) '

.JY)

Squaring both sides:


9/ - 30y + 25

=y

(4/ + 4y + l), which simplifies to: 4y

4x 3 - 5x 2 + 3lx - 25
(f)

Let y

=-x,

=0

- 5/ + 3ly - 25 = 0

is the required equation.

then putting x = -y we obtain the equation whose roots are

-a, -S, -y.


- 2y 3 + 3y 2 - y - 5
2x 3 - 3x

+ x + 5

=0.

=O,

hence the required equation is


(Compare this with the original equation)

183

Exercise 50
1.

If a, a, y are the roots of x


(a)

2.

If

+ a

+ y

If

a, a, y are the roots of

(a) a-l + a-l + y-l


3.

(b)

(b)

a5 +

- 2x

+ a
3
-

+ y

5x - 1

a, B, y, o are the roots of x

= o,

(c)

=0,

a5 + y 5

evaluate:
a

a4 +

evaluate

(c) (a+ B-y) (B +Y -a) (y +a- B)


(Hint: a + B + y = 0)
2

+ x + 2x

+ 3x + 4

=0,

evaluate

(b) aB + cry + ao +By + BcS +yo

(a) a + B + y + o

4.

- 3x + 1

3
2
Solve the equation x - 3x - 6x + 8 = 0, given that the roots are in geometric
progression.
[Hint: Let the roots be ~ , a, ar)

5.

4
3
Solve the equation x + 2x - 21/ - 22x + 40 = 0, given that the roots are in
arithmetic progression. [Hint: Let the roots be a - 3d, a - d, a + d, a + 3d)

6.

If

7.

Find the condition satisfied by


are in arithmetic progression.

8.

Solve the equation


its roots is -2.

9.

The equation
completely.

10.

The roots

4
3
2
2
2
x + 4x + mx + nx + 9' = (x + bx + c)
for all values of x, find all
possible values of b and c and the corresponding values of m and n.

a-l,

+ 2x

if the roots of

+ px

+ qx + r = 0

3 - 17x 2 - 5x + 6 = 0, given that the product of two of

- 3/ - 4x + 4

=0

has a double root. Solve the equation

3
2
a, B, y of the equation x + bx + 12x + 4 = 0 are such that

B-l, y-l

[Hint: Let

6x

p, q, r

are in arithmetic
=

, then 4 u

progression. Find b and solve the equation.

+ 12 /

which are in arithmetic progression)

+ b u + 1 = 0 has roots

a,

and y

184

11.

a,

If CX,
y
roots are:
(a)

12.

are the roOtS Of X

+ 2x

a s Y

(b)

2ex, 2B, 2y

(d) ex - 2, B - 2, y - 2

(e)

ex ,

If ex, B, y are the roots of


roots are:
(a) ex + 2,

a + 2,

2x

y + 2 (b)

+ 3x

1
ex + 2,

form the equation whose

B+

(c) ex + 1,

62, y2
3

= 0,

- 2x + 3

(f)

- x - 1 = O,
1

8'+2 , y

1
+ 2

1, y + 1

, }

form the equation whose


(c)

2
ex '

(Hint: use (a) )


13.

If ex,
are:
(a)

a, y

2ex,

are the roots of x

28,

= 0,

+ bx + c

form the equation whose roots

(b)B+y-ex, Y+ex-B,ex+B -y

(c) ~ ,
14.

2y

8+Y ,

(Hint:ex+B+Y=O)

1
(Hint: let y = ~ = - - = ex+p
-y

1
Y +ex

.2

.!.X

etc.)

If ex, B, y are the roots of x + bx +ex + d = O, form the equation whose


-1
-1
-1

(a) -1
-1
-1
ex , B , y
and hence evaluate
ex + B + y

roots are
(b)

ci'l

81

8 1 .y1

+ .y1 ci1

, (c) . ex-2 + 8-2 + Y-.2


2

[Hint: p, + q

15.

+ r

= (p + q +

rl - 2 (pq + qr

4
2
If ex, 8, y, 0 are the roots-:i:.x .- x + 2x + 3 = 0, form the equation whose
roots are:
2
2
2
2
-1
(b ) ex-1 , B-i, -1
(c) ex , 8 , y ,
(a) 2ex, 2 8, 2y, 2o

y, o

16.

+ rp)]

ex, B, y are the roots of

- 2x+ 3 = 0

2 . 2 2
8 , y

(a)

Form the equation whose roots are ex ,

(b)

Using the result of (a), now form the equation with roots

ex

+ l,

8 + 1, y

(c) Hence evaluate

+ l.

<i

+ l) (8

+ l)

<l + l)

185

5.5

Miscellaneous (Worked Examples)

EXAMPLE: {18)

For what real values of c is z - c i a factor of


4
2
3
P(z) = z - z + 9z - 4z + 20? Hence solve P(z) = 0, c

By the factor theorem:

SOLUTION:

.
P(c 1") = 0 g1ves
c 4 + c 3.1 - 9 c 2 - 4c.1 + 20
:. c

-9c

1-0.

+20

+i(c

=0

-4c)=0

Equating the real and imaginary parts:

c
i.e.

(c

Now c

- 9c

- 5) (c

We divide

= (z

(z +

+ 4) (z

(z - 2i) , i.e.

(l

a By

= -c

x2 = 82,

'

51

Ex

52

Ex x

= a

1 2

x3 = y

2
+ 82 + y

2 2
= Ea 8

+5 x-5 =0,
2
3

I+/T9i
2

then
(Ea)

(Ea8)

= xlx2x3 = a2 s2i
53
The required equation is

+ 4

+ 8+ y

aB +By + ya
2

a, 8 and y are the roots.. of x


2
2 .
2
whose roots are a , 8 and y

We have

(l

If

METHOD I

XI

2i~

- z + 5)

SOLUTION:

-5 x
1

=0

- 4c

.
P(z) = 0 are: _+ 2 1 ,

EXAMPLE: (19)

+ 2 satisfies both
i.e. c = _

by

P(z)

The roots of

'
on l y c 2 = <t,

Let

and

- 4) and c (c - 2) (c + 2) = 0
2
0 and c = 5 satisfy one but not both equations

P(z)

=0

+ 20

i.e.

(Ea8)
b2
2

- 2La8

2
2

2
- 2La8 y
- 2a8y Ea

- 2b

..

+ bx + c = 0,

find the equation

186

SOLUTION:
Let

y=

METHOD II

2
This transformation will give us an equation whose roots are a ,

s2

and y

To minimise the calculation, we write:


x(x

IY

= -c

+b)

Substitute x =

/Y
-c .

(y + b) . =

Squaring

</ + 2by + b >


2

c2

2 2
whose roots are a , B ,

i.

3
2
2
2
y +2by +by-e =0

or

is the equation

This method is much superior to and easier than the other

method.
When a polynomial P(x) is divided by x - I the remainder is 2,
when divided by x - 2 the remainder is 3. Find the remainder
when P(x) is divided by (x - I) (x - 2).

EXAMPLE: (20)

SOLUTION:
The degree of the remainder is less than that of the divisor. When the divisor is
(x - 1) (x - 2), the remainder must be of the form ax + b, hence
Let P(x) = (x - I) (x - 2) Q(x) + ax + b
Given that P(l)

=2,

EXAMPLE: (21)

=3, we
= 1, b =1,

P(2)

Solving these, we get a

now have:

2=a+b

and

3=2a+b

hence the remainder is x + 1

=cose

+ isine, prove that zn + z-n = 2cosne, hence solve


4
2
3
the equation 3z - z + 4z - z + 3 = 0
If

(I)

SOLUTION:
z

= cose

+ isine

cos(ne) + isin(ne) and z-n

cos(ne)- isin(ne)

1
(1)
zn + - - = 2cos(ne)
zn
The equation is: ( rearrange)
2
4
3
2
3(z + 1) - (z + z) + 4z = 0 . Divide by z ' then: 3(z 2 + I
z
Using (1) with n =2 and n = 1, we now have:
2
6cos2e - 2cose + 4 = 0 ,
using cos2e = 2cos e 2
2
= 2c - 1
6(2c - I) - 2c + 4 =0
2
6c - c - 1 = 0
1
1
(2c - 1) (3c + I) = 0 /jves c = 2 or - 3
1
, - n <e ~ n
If cose =
, sine = .!.

-z> - (z + z>1 + 4 =0

and cose

=- ~,

The roots are:

=.!.I;}
(1.!. v'Ji),

sine

(-1.!.

.{ii)

187

Exercise SE (REVISION AND MISCELLANEOUS)


1.

3
(a) X
2.

3.

4.

(a) R

Factorise the following polynomials over

4
(b) X

16

Factorise the following polynomials over c.


(b) X2
(a) X2 + 4
2
(c)
(d) X3
X - 4x + 6
(f) X3
(e) X4 + 4x 2 + 3

(b) C
3
(c) x + 1

+
+

8x
3x
2

4x

12

- 4x - 24

+X

For each of the following, find a polynomial of lowest degree, having the given
zeros.
(a)

2 + i, 2 - i

(c)

-2, 2 + i, 2 - i

(e)

l -

12,

l +

3 + 4 i, 3 - 4 i

(b)

12,

+ i

I +i

(d)

3,

(f)

4 - i,

3+ i

For the following polynomials, one zero is given. Find the remaining zeros.
(a)

P(x)

(b)

P(x)

(c)

P(x)

(d)

P(x)

3 - 7x 2 + l7x - 15,
3
2
x - 7x + 17x - 15,
x

=3

=2

- i

x = l - 3i

x - 3x + 6x + 2x - 60,
2
3
2x - l3x + 32x - 13,

=3 -

2i

5.

Factorise the following over C, by using the factor theorem:


2
2
3
(a) x 3 + x + x + 1
(b) x - x - 4x - 6
2
3
2
3
(c) x - 3x + 7x - 5
(d) 2x - 2x - x - 6

6.

Solve the following over C. (Factor theorem is useful)


3
4
(a) x + I = 0
(b) X - 16 = 0
4
2
4
(c) X - 4 = 0
(d) x + 3x + 2 = 0
4
2
3
(e) x - 7x + 18x - 22x + 12 = 0

7.

4
Given P(x) = x - 5x 3
the equation completely.

8.

2
Given 1 - i is a root of the equation z - (c - 2i) z + 3 + ib = 0 where b
and c are real. Find b and c. [Hint: a+ 13 = - b/a, and use aS = c/a]

9.

If z

= cos9

+ isin9,

zn + z-n

4x 2

10.

If mx

+ nx

[Use P(x) =

3x

9 = 0

has a root of multiplicity 2, solve

prove (by using De Moivre's theorem) that:

2cosn9
4
3
2
Hence solve 2z + 3z + 5z
3

+ p

~~

=0

3z

2
has a double root, prove that 27m p

=0

have a common root]

188

11.

(a)
(b)
(c)

12.

13.

Use Mathematical Induction to prove that


(cose + isin9)n = cosn9 + isinne, for positive integers n.
Solve x 6 = -1, giving answers in A + iB form.
4
2
Find the four roots of x - x + I = 0 and show them on an Argand
diagram.

(a)

Show that x - 3 + 2i is a factor of the polynomial


2
3
P(x) = x - 9x + 3Ix - 39 and hence decompose P(x) over
(i) field R
(ii) field C.

(b)

P(x) = 2x - 13x
find the roots.

24x

8x

- 64x

48 has a root of multiplicity 4,

and a and 8 are the roots of z

Given that cotS = y + I

( Y + a )n-( Y + 8 )n

prove that:
14.

- 2z + 2 = O,

sin ne
(sine )n

a - 8

5
Solve x = I by De Moivre's theorem dnd indicate roots on a circle of radius
in an Argand diagram. Express the roots in the cis8 form.

Find the area of the regular pentagon formed by the five points representing
these roots.
4
2
3
Solve the equation x + x + x + x + I = 0
211
411
2
Deduce that cos
and cos
are the roots of 4x + 2x - 1 = 0

15.

Given that 2- i is a root of the equation 3x


!he equation completely.

16.

Solve z

17.

Find K if

18.

+ z

+ 1 = 0,

One root of x

- 10x

+ 7x + 10 = 0,

giving all the four roots in rcis e

= x3

- 6x

+ (I -

i) x

P(x)

K = 0 is

(a)

Solve tan49 = 1,

0 ~ 9 ~ 11

(b)

Prove that tan49

in the form

from

the

(cis9)

expansion)

Using parts (a) and (b) and z = tan 9 , find the roots of
4
3
2
+ 4z - 6z - 4z + 1 = 0.

z
(d)

Using (c) find:


(i)

tan

(ii)

tan

11

16
2 11

T6

tan
+

51!

T6

tan

<e ~

3i .

3
4 tane - 4 tan 9
4
2
I - 6 tan 9 + tan 9

(Hint: Find sin48 and cos49


(c)

form, -11

9x + K has a root of multiplicity 2.

Find the other root of this equation and the value of K


19.

solve

2 511

T6

911
tanu;

tan

2 911

T6

1311
tan T6

+
+

tan

2 1311

T6

ib

11.

189

20.

Using the method of finding the square-roots of a complex number, find the
roots of the following equations in the form A + i B, where A, B are real.
2
2
(a) z +3iz+2=0
(b) z +(2+3i)z-~-2i=0
(c)

21.

- (4 + 2i) z + 6 + 8i = 0

(d)

+(4+2i)z+3+2i=0

(f)

(e)

(a)

Show that the roots of z

+ 2z + 3 = 0 lie outside the circle

(b)

22.

Show that the roots of z + z + 1 = 0 lie on the circle


(Hint: Consider the modulus of each root)
4
2
3
Prove that z = 1 - i is the root of P(z) = z - 6z + 15z Find the other roots.
[Hint: Dividing P(z) by z - (l - i), you can show that the
but this involves very awkward division, so a better method is
2
2
2
[ (z - 1) + i] [ (z - 1) - i] = (z - 1) - i = z - 2z + 2 .
This not only proves that z = 1 - i
roots in one simple division.]

23.

1z 1

lz 1 = 1
18z + 10 = 0.
remainder is zero,
to divide by

is a root, but also gives all the remaining

Solve
(a)
(c)

24.

z 2 = 3 - 4i
2
z -(3+i)z+4+3i=0

2
- 2z + 4 = 0
2
2z = 1 + i

Solve x

(b)
(d)

6 + (x + 1) 6 = 0
4
2
z - 30z + 289 " 0

(x - 1)

+ 1 = 0 , express each root in the form rcis9 and then A + iB.


8
+ 1 into real quadratic factors and deduce that

Decompose x

cos49 = 8[cos9 -cos

i J[cose -cos 38

11
]

511
[cos e -cos 8

7
[cose -cos ;

25.

2
3
z = 1 - i is a root of z + az + bz + 6 = 0, where a and b are real.
Find a and b and all the roots of the equation.

26.

z =-

27.

Find m and n such that x = 3 is a double root of


4
2
3
x + mx + 13x + nx - 36 = 0

28.

Show that the polynomial x

~-

(
29.

"J" is a root of z 4 + 2z 3 + z 2 - 1 = 0.

!:!!)n
n

11

Find all the other roots.

+ mx - b = 0 has a multiple root provided

+ (-b
)n-1
_
11
1

= O.

Find this root.

3
If the equation x + 3mx + n = 0 has a double root, then prove that
2
3
n = -4m , and that this root is - ~

30.

Solve x - 6x 3 + 12i - 1Ox + 3 = 0 given that the equation has a root of


multiplicity 3.

31.

Solve z

+ 2rf2 iz + 20 i = 0; express the roots in the form

A + iB.

190

32.

If the polynomial x

(mn- r) 2

4(m

3 + 3mi + 3nx + r

- n) (n

33.

Given that the roots of 4x


find them.

34.

If

- 24x

r:a.

l+B
~

!...:!:....r.
1-y

has a double root, prove that:

- mr).
3

+ 45x - 26

a, B, y are the roots of x - x - 1

l+a

35.

=0

=0

[Hint: y =

=0

are in arithmetic progression,

find the equation whose roots are


1
+x
1 - X

where x = a , B, y]

When a polynomial P(x) is divided by (x - 3), the remainder is 5 and when


divided by x - 4, the remainder is 9. Find the remainder when P(x) is divided
by (x - 3) (x - 4)

191

CHAPTER 6 CONIC SECTIONS


6.1

Introduction

The subject of conic sections had been studied by ancient Greek mathematicians (200
B.C.). In recent times, it has acquired an added importance in space explorations.
The safest and the most economical path to the planet Mars is an elliptical orbit,
called 'the Hohmann ellipse'. (The Hohmann ellipse is an ideal trajectory requiring a
minimum of energy for a journey between any two planets, named after the German
engineer who calculated it in 1925.)
The conic sections are a family of curves obtained by cutting a right circular cone by
a plane at various inclinations to the axis of the cone.

Two
Branches of
Hyperbola

Fig. I

Fig. 2

If the slice is parallel to the base, the curve is a circle, if the slice is parallel to the
slant side of cone, the curve is a parabola; between these two, we have an ellipse.
Finally, any other cut (intersecting also the mirror image of the cone) produces a
hyperbola.

192
We shall only study the CENTRAL CONICS ellipse and hyperbola and their properties.
The case of the parabola has been studied previously (3U Mathematics).

Definition:
A conic is the locus of a point P which moves in a plane so that its distance from a
given point S and its distance from a fixed line,d, always are in the same ratio, i.e.
PS
PM

The fixed point, S, is called the focus


The fixed line, d, is called the directrix
The ratio, e, is called the eccentricity
If e = I,

e
e

< 1,
> 1,

PS = PM,
PS
PS

< PM,
> PM,

the conic is a parabola


the i::'onic is an ellipse
d

the conic is a hyperbola.

Fig. 3

6.2

Ellipse (e < 1): (Focus and Directrix Definition)

Let P (x, y) be any point on ellipse.

S is the focus, ZM is the directrix.

P(x, y)

Draw SZ 1 the directrix , and


mark the points A and A' on this
line so that
SA
AZ

SA'
A'Z

e,

e
A'

Fig. 4
By definition, A and A' are on the ellipse. 0 is the mid-point of AA'.
rectangular axes at 0, as shown.
S (c, 0),
then
Let AA' = 2a;
SA'
eA'Z
SA
e. AZ
and
SA'
and

SA'

SA

A'A = 2a

SA

e (A'Z - AZ)

(l)

2ae

(2)

Adding and subtracting:


SA = a (I - e)
Then

c = OS = OA

SA'

a (1 + e)

SA= a - a(l-e)

ae

Take the

193

and

oz =
s
z_

Now

OA + AZ

is (c, 0)

(ae, 0)

(x - ae)
(x - ae)
(I -

e 2~

OD

a
--x
e

(PM )

+ /

+ /

Let

(by definition)

oz

PM

a
e

a + .!. (a - ae)
e

is (~ 0)
e'

PS
PM
SP

= e

(~e

- x) 2

2
= (a - ex)
2
2
+ /
a (1 - e )

a (1-e)

(3)

Equation (3) is the equation of the ellipse in standard form.


The point S is called the focus of the ellipse and the line x = ~ is the directrix. By
e
symmetry of the equation of ellipse, we infer that it has two foci S(ae, O) and
S' (- ae, O) and two directrices (See Fig. 5)

d'

Z'

B'
B'

d'

Fig. 5

Fig. 6

194

Important Features of the Ellipse:

2
2
The eJlipse x + ~ = 1 has the following properties. (Fig. 5)
2
a
b
1.

The curve is symmetric about both axes.

2.

The major axis AA'

2a

3.

The minor axis BB'

2b

Note that since b

A'(-

A(a, 0)

B'(O,- b)

B(O, b)

(l - e ) , e

< 1,

then b

4.

The centre of the ellipse is at 0 (0, 0)

5.

Foci: S (ae, 0) ,

6.

Equation of two directrices are x

a, 0)

< a.

S' (-ae, 0)

a
e

lf the foci are at S (0, be) and S (0, -be), the standard form of the equation of the
2
2
2
2
2
ellipse is x + ~
1,
with a = b (l - e ) , a < b
2
a
b

and its properties are: (see Fig. 6)


l.

The major axis is along the y-axis,

BB' = 2b

2.

The minor axis is along the x-axis,

AA' = 2a

3.

The foci are given by (0, . be) with centre 0 (0, 0)

4.

The directrices are y

..

eb

WORKED EXAMPLES
Find the eccentricity, foci, directrices of the following ellipses:

(l)

~5

Solution:
a
b

~ = 1

(see Fig. 5)

= 5,
2

b
2

=3
2

=a 0 - e )

9 = 25 (I - e )
e =

195
The foci are (! ae, 0) = ( 4, 0)
a
The directrices are X =
= . 425

2
(2)

x9

Solution:

(see Fig. 6)

For this ellipse, the major axis is along the y-axis.

a = 3,

b = 5

a2 = b2 (l - e2)
2
9 = 25 (l - e ) gives e

The foci are (0, . be) = (0, . 4)


The directrices are:

= eb = . 425
Exercise 6A

Find:

(a) the centre


(b) the eccentricity
(c) the foci
(d) the lengths of axes
(e) the equations of directrices
for the following ellipses:
2

1.

3.

5.

7.

4/ + y

9.

16

L
25

9 =

36

9/

4.

T6

10
2

4/

36 = 0

2.

25

8.

4x
X

16

48

36

T2

10.

6.

9/ =
2/

1 = 0

Find the equations of the ellipses described as follows.

=4

11.

Foci on the x-axis, centre (0, 0) , a

12.

Foci on the y-axis, centre (0, 0) , a = 2 , b = 5

13.

Foci ( 4, 0) , b = 3

14.

Foci (0, .

15.

Centre (0, 0), the length of major axis = 10 and length of the minor axis = 6

16.

Centre (0, 0), minor axis = 12, focus (0, 4).

17.

Eccentricity e

/5),

, b = 2

a = 3

=~,major

axis= 12

196

6.3

The Hyperbola

Starting with the definition

~~

= e , e

> I for the hyperbola and using the

following diagram, you can easily derive the standard equation of the hyperbola as:

2
a

where b

'1

- Lb2
2

= a

(e

- I)
A'

Fig. 7

The hyperbola x
L
a2 - b2

Y.

has the following properties (see Fig. 8)

Fig. 8
I.

It is symmetric about both axes.

2.

The transverse axis is the line segment joining the vertices A (a, 0), A'(- a, 0) and
AA' - 2a.

3.

The foci are S (ae, 0), A' (-ae, 0),' centre 0 (0, 0)

4.

The conjugate axis is the segment BB', where B (0, b) and B' (0, -b)

5.

The equations of two directrices are x

6.

The asymptotes are:

+ ~

The rectangle through the vertices A, A', B and B', is very useful in drawing the graph
b
of the hyperbola. Also draw the asymptotes y =
X
through the corners of this

rectangle. The hyperbola fits nicely between the two asymptotes.

197
2
x2
The hyperbola given by L - a2
b2
details.

I is shown in Fig. 9 with all the important

I.

It is symmetric with respect to both


axes.

2.

The foci are on the y-axis, S (0, ae),


S' (0, - ae).

3.

The transverse axis AA' = 2a,


A (0, a), A' (0, - a).

4.

The conjugate axis BB' = 2b,


B (b, 0), B' (-b, 0).

5.

The directrices are

6.

The asymptotes are: y = ;!;.

y = . ~
e

6x

Fig. 9
WORKED EXAMPLES
I.

Find the foci, directrices, asymptotes and vertices of the following hyperbolas and
hence sketch them.
y
2
2
~-L(a)
(See Fig. 10)
16
25 -

(i)

a = 4, b = 5
c2 = a2e2 = a2 +b2

c =

.J4J

or c = -

ll4i.
X

The foci are ( ;t v'iii, 0)


(ii)

The directrices are

But

a
-e

= +-

c = ae
.Jii1=4.eore=

V4l
4

.
.
16
t h e d trectnces are x = .~
v41

Fig. 10

198

(iii)

The vertices are


A (4, 0), A' (-4, 0),

(iv)

The asymptotes are y =


(b)

45 x

(See Fig.

a = 3,

b =4

c2 = a2 + b2 = a2e2
5

:. c = 5,

e =3

If we put y = 0, we have x

-16
and this shows that the foci lie on
the y-axis
. .

the foci are (0, 5)

and vertices (!: 0, 3)


The directrices are

y = .:!:.

The asymptotes are


+ 3
y = - 7f

Fig. II

Exercise 68
1.

For the following hyperbolas, find the foci, directrices and vertices and sketch them:
(a)

(b)

25

;
T6

16

(c)

x2
25

(d)

4/

4/ =
- y2

4
2

2.
3.

Find the angle between the asymptotes of the hyperbola x


a2
2
2
Draw an accurate sketch of x - y = a 2
Find the angle between the asymptotes.
2
[The curve given by x - /

= a

is called a RECTANGULAR HYPERBOLA]

199

6.4

Shape of the Conics

We can study the variation of the shapes of conics


in general by varying the eccentricity e or changing

the ratio b/a.

=0
=0

'

Eccentricity

< e < I,

For 0

where c

c =

the conic is an ellipse and

is the focal distance given by

~.

a>b>O.

If we keep a fixed and vary c

0~ c

over the interval

a, the resulting ellipses vary in shape,

For e = 0, i.e. b = a, the shape is circular.

and when e is nearly I, the ellipse reduces


towards a line-segment.

S,S'

"oE ~
0

as shown in the diagram.

As e increases, the shape becomes flatter

\_

e"O\C
e

= 0.97

::>

e~l

S'

The orbit of a planet around the sun is elliptical


in shape, but the orbits of most of the planets

Planet

Neptune

0.01

Earth

0.02

Jupiter

0.05

Mercury

0.21

are nearly circular, as can be seen from the table


of eccentricities.

The varying shape of an ellipse can also be examined by changing the ratio b : a,
as shown below.
(cont)

200

E---+---3>
the variation of the shape of conics as e

vanes from zero to a

ver
e

>1

hyperbola~

e
circles

1.1

O<e<l

c.~.
------

1.40.95

E___J

= 0.97

parabolas

e .....

oo

Note the relative positions of the foci

201

6.5

Parametric Equations of the Circle and the Ellipse

Consider a circle with centre at the origin and radius


Let P(x,y) be a pomt on the circle and

L POx

a.

= e.

We have the relations:

= a COS e

=a sin e

The point (a cos e, a sine) lies on the circle


x

+ y

=a

for all values of e. We restrict e

m the range 0 ,< e

< 2n,

m order to have one-one

correspondence between a point and its parameter.


x = a cos 9,

y = a sine are called the parametric

equations of the circle.

Parametric Equations of an Ellipse: Auxiliary Circle


Consider now the equations:

=a COS e ,

= b sin e .

It is easy to show that the pomt

(a cos 9, b sin 9) lies on the ellipse

;- +

a
0

'

for all values of

a,

i.e.

< 2n

2
a 2cos e
+
a2

or

I, which is true.

x = acose , y = bsin9 are then the


2

parametric equations of the ellipse ~ + L


2
a
b
and e

1s called the eccentric angle and it has a

geometriC meamng as shown m the diagram.

It can be shown that the ellipse can be

constructed, pomt by point, by usmg the parametric equations as follows:


we draw two concentric circles w1th radii a and b, a

> b.

These are

called the major and minor AUXILIARY circles, respectively.


P on the ellipse, draw a lme through 0
X-dXIS.

To determine a point

making any angle e with the positiVe

202

Let

and

be the points of intersection of this line with the auxiliary circles.

Through A draw a line parallel to the y-axis, and through B draw a line parallel to
the x-axis. These lines intersect at P where
X

ON

0Acos9

acosa

bsin9
0Bsin9
= NP
The point P(acosa, bsin9) is usually referred to as P(9), or 'the point 9',

ellipse.
. wort h notmg
.
It 1s
t hat PN
AN

bsin9

b
a,

on the

a constant for all positions of P. This

fact alone is sufficient to determine a point on the ellipse by reducing each


y-coordinate of the auxiliary circle x

+ /

=a

in the ratio b : a.

203

6.6

Equations of the Chord, Tangent and Normal to the Ellipse


y

We consider the ellipse with general point


(acos9, bsin9). The gradient of the line
joining the points r/J and 9 is

bsinr/J - bsin9
acos\6 - acos9

2bcos(~) sin(
-2asin(\6 ;

Y.>

) sin(\6

Z9)

q, + 9

-bcos(~)
. m=
q,
asin(+)

~ cot( 9 + r/J)
a
2

or

The equation of the chord is then:


- bcos(r/J + 9)
2
y-bsin9=
.(x-acos9)
16 9
asin<-f->
or

i cos(

; r/J) +

sin(

This simplifies to: icos(

; r/J) = cos9cos(

; r/J) +

sin(

; r/J) + sin9sin(

; r/J) = cos(

; r/J)

2r/J)

The equation of the tangent at 9 is obtained by letting r/J = 9 in the equation


above, giving:
i cos 9 + ~ sin 9
Alternative Method:
The gradient of the tangent at the point P(x , y ) is obtained by differentiating
1 1
2
2
~ + L
= 1
2
2
b
a
2
-b x
2x
b: ~
~ = - 2 -1
0
and
at P.
dx
a2 + b2 dx
a yl

The equation of the tangent is y - y

2
-b XI
- 2 - (x a yl

X I)

2
XI
-2a

I, as P(x , y ) lies on the ellipse.


1 1
(cont)

204

So, the equation of the tangent is:

Substituting x
xcose
a

=acos9,

=bsin9,

+ ysin9
b

we have

as the equation of the tangent at P(9), in


the form obtained above.

Equation of the Normal

We have ~
dx

-b XI

, so the equation of the normal at P(x , y ) is


1 1

a Yl

!..... (x - x 1) , which simplifies to:.


XI

Finally, substituting x

=acos9,

=bsin9,

this equation becomes:

or

Note:

The corresponding results for the circle

+ /

are easily obtained,

=a in the above results or deriving them as above.


2
2
2
We have: for the circle x + y = a , at P(x = acose, y = asin9).
1
1
The equation of the tangent:

either by substituting b

x. 1 yy 1 a

<eoB y;no a

The equation of the normal at P(xl'yl) or P(acos9, asin9):


xsin9 - ycosa

205

6.7

Parametric Equations of the Hyperbola

It 1s easy to venfy that x = a sec 9,

y = btan9 are the parametric equations

of the hyperbola ~ - ~
a
b

= 1.

We can also

derive the parametric equations of the


hyperbola by using the auxiliary circle

. t he a d"Jacent
x2 + y 2 = a 2 , as s hown m

diagram.
The point T lies on the circle, where

LTOx = 9 and the tangent to the circle


at T meets the x-axis at N. PN l_ Ox,
with P(x, y) on the hyperbola.
OT
Now x = ON and from the right-angled llOTN, cos 9 = ON
So, ON = OT sec 9 = a sec 9.
x=asec9

2
I , we find sec 9 -

By substituting x = a sec 9 into !.__ - L


a2

2
2
2
y = b (sec 9 - 1).
y = b tan 9 since y
We note that:
and
1.

=; < 9 <;

>0

b2

7b 2 = 1,

i.e.

in the first quadrant.

for the right-hand branch

.!!.2 < 9 < 311


2 for the left-hand branch.

The equation of the chord joining the points P(asec9, btan9) and
Q(asec0, btan0) is

i cos j (9- 0)
2.

~sin~

(9 + 0) =

cos~ (9

+ 0)

The equation of the tangent at P (x , y ) is:


1 1
XXI
yyl
a2 = I ,

bl

or at P(asec9, btan9) is: xsec9 _ ytan9 =


a
b
3.

The equation of the normal at P(x y ) is:


1 1

2
xa
XI

2
yb
2
2
+ - - = a +b
y1

or, at P(asec9, btan9) is: xacos9 + ybcot9 = a

2 + b2

206

A Speciai(Rectangular) Hyperbola xy =

6.8

c2
y

= ct,

xy

=c

=~

We have

are the parametric equations of

= -X

2
- c
-2x

~
dx
SubstitutE;

x = ct

~-.:...!
dx -

t2

Hence, the equation of the tangent at


P(ct,

I)

is given by:
y -

c- = - - 1 ( x - ct)
t
t2

or
The tangent has gradient -

-1 ,

and so the gradient of the normal is

equation of the normal is y -

I= t 2 (x - ct),

i.e.

t3 X

ty = c{t 4 - 1)

The

c
The equation of the chord PQ with P(cp, ~) and Q(cq, q)
is given by
y - ~

c
P

c
9 (x - cp).

cp- cq

This simplifies to

x + pqy

=c (p

+ q)

By solving the equations of two tangents at P and Q, namely: x + p 2 y

= 2pc

and

x + q y = 2qc, we find the point of intersection of these to be


R

p2~p%

/~ q )

The following results are left as an exercise to the reader.


1.

The equation of the chord joining P(x , y ) and Q(x , y ) is:


1 1
2 2
2
y- yl = ~ (x- xl)
x 1 x2

2.

The point of intersection of the normals at P(cp, ~) and Q(cq, ~) is given by:
p

X-

cqp(p2+l+rq)+C
pq (p + q

y =

cp3q3+C(p2+~q+l)
pq (p + q

Though the coordinates look formidable, it is a basic and interesting exercise in


;,IPPhr;,ir m;,ninulation!

207

EXAMPLES:
Find, the equations of the tangents arid the normals to the following curves at the given
points:
(1)

2
+

(3)

h=l,
2y

=l

(2)

x = 2cose, y = 3sin9 at 9=4

P(-3,2)

(4)

x = V2sec9, y = 2tan9 at e

(1) 3x

+ y

= 12

6x + 2y

(3)

~ =0
~

P(-1,3),

At

and y - 3

X +

= -(x

1, i.e. y =
+ 1),

X +

=-

4 (T)

i.e. y = -x

y - 2

2 (N)

i.e.

(4)

P(9) = (2cos9, 3sin9)

.".

3x

y +

~ =- ~

2y = 6

and y 4x - 6y

3/

.j2), i.e.

V2

...

72. = 3 (x - 112),
= - 5 v'2

(T)

2
~ =

1 at

P(

= 34

(x + 3)

3x + 4y + 1

=ofi sec 9,

=0
=0

/i., /2)

(T)

(N)

y = 2tan9, 9 =%

P(xl'yl) = P(2,2) for

9 =%

2
2sec e
dx - 0/isece tanS

{isece

At

tanS

e = ~, ~ = 2

The required equations are:


y - 2 = 2(x - 2)

i.e.
(N)

We can find these by using the


cartesian equation of the ellipse

at P

~-

/i)
(x -

~ =0

4x - 3y + 18

For e = ~ , ~ = -~.
The required equations are:

.[2,

P(-3,2)

1 ,

The required equations are:


3
y - 2 = - 7i (x + 3) and

(2 ) ~ _ dy~de _ 3cos9
dx - dx dB - -2sin9

P(n/4) = (

2y 2

. 2x - 4y

The equations of the tangent


and the normal are:

y - 3 =

=4

N = normal)

SOLUTION: (T = tangent,

P(-l ,3)

and y - 2

= - ~ (x

- 2), i.e.
(T)

y = 2x - 2
and x + 2y

=6

(N)

We can also find these equations


from the cartesian equation of the
hyperbola

x - \- = 1 at P(2,2)
2

208

Exercise 6C
Find the equations of the tangents and the normals to the following curves at the
points indicated:
1.

2x

+y

2
=6
2

+ 2y

=5

=3sin9,

p (1,2)

2.

3x

3.

=2cos9

P (9

4.

7.

x =4cos9, y =3sin9
2
3x - 4/ =24
2
x - 2/ = 2
2
3x - 2/ = 1

p (9

8.

5.

6.

9.

= 2sec 9,

p (-1,1)

p (4, - ~)
p (-2,1)
p (l,-1)

=3tan9

P (9 = n/4)

x = sec9, y = tan9
=c 2 (c is constant)

P (9 = n/3)

10.

xy

11.

Find the (two) equations of the tangents of gradient

12.

The point P(x

=3,

=n/3)
=T! /4)

-------EJct
- . -~ )
t

> 0)

~ to ~h; ellipse l
2

lies on the ellipse ~ 5 + ~

= 1.

+ 6/

= 15.

Find the equations of

the tangent and the normal at P. Also find the coordinates of the point in which
this tangent intersects the directrix corresponding to the focus S.
13.

Find the equations of the tangent and the normal at P(9,-3) on the hyperbola

4 - fg

= 1.

The normal meets the curve again in Q. Find the coordinates of the

point of intersection of the tangents at P and Q.


14.

=2x

-y2

- 4 intersects the curve x


=1 at P and Q. Find the
3
coordinates of the point of intersection R of the tangents at P and Q. What
The line y

is the angle between the normals at P and Q?


15.

Prove that the line x + y

=5

is a tangent to the el!ipse 9x

2 + 16y 2

144.

Find the coordinates of the point of contact.


16.

Show that the line 3x + 4y = 10 is a normal to the hyperbola 2x 2 - 3/ = 5


and find the point at which the line is normal.

17.

Find the equations of the tangents to the hyperbola 2x


parallel to the line x

= 0.

2 - 3y 2

Also find the points of contact.

=6

which are

209

18.

(a)

2
hyperbola X
2a

ellipse

(b)

19.

=0

Show that the line lx + my + n

2
a

if a212 + b2m2

I '

b2

touches the

2
I'

b2

= n2

if a212 - b2m2

= n2

x2
2
The tangent at P (xI' y 1) on the hyperbola a - ~ = 1, x .> 0, intersects
2
1
the directrix at

Q.

is the focus

(ae,O).

Prove that

PSQ

is a right

angle.
20.

Find the equations of the four tangents common to the hyperbola


2
x - 2/ = 4 and the circle
+ / = 1. Find the points of contact of these

tangents with the circle.


[Hint: Let xx

6.9
1.

+ yy

=1

be a tangent to x

2 + y2

1 at P(xl'yl)]

Miscellaneous Problems on Conics


x2
2
Find the equation of the tangent at P(x , y ) to the ellipse a
+ ~
1 1
2

Find the point Q(X, 0) where this tangent meets the x-axis, and prove that Xx

=
1

1.

= a 2

Find the point R (0, Y) where the tangent meets the y-axis. Show that the locus of a
a2
b2
point T (X, Y) is given by the equation
2X + 2y = 1.
y

Solution:
The equation of the tangent at P (x , y ) is
1

YYt

XXI

-2- + -2- = I.
a
b

(Prove it)
X

Substituting for Q (X, O) ,

Xx

= a

or

.... (l)

210

(2)

Similarly

To find the locus of T (X, Y), we eliminate x and y by using:


1
1
2
Xl
-2
a

yl
+

17

If

blf

a
+
x2a2

Y2b2

Hence the locus of R is given by the equation


1

2.

Find the equations of the tangent and normal to the hyperbola 3x 2 - y 2

at P (If, 3{5).

The tangent meets the x-axis in M and the normal meets the y-axis in

N, find the length MN.

Solution:
There is no need to remember the eqJations of the tangent and normal.
3x

6x

- y

2y

At P(lf, 3/5),

~
dx

Differentiating

3
or

_ 3x
dx - y

If

v'5

:. Equation of the tangent at P (If, 3/5) is


or

r5y

lfx -

(I)

For M, put y = 0
:.

M is

<i ,

0)

The equation of the normal is


y

or

r5x

=3

3 f5
+

lfy

- 415
16f5

(x - If)

(2)

y - 3/5

If

f5

(x - If)

211

For N, put x = 0
N is (0, 4/5)

MN
3.

/-1~

{ii8i

-4-

80

Find the equation of the normal at P (a sec 9, btan9) to the hyperbola

2
2
x2-~=l.
a
b
This normal intersects the x and y axes at Q and R respectively. M (X, Y) is the
mid-point of QR. Find the equation of the locus of M as P varies on the hyperbola.
y

Solution:

The equation of the normal at P is

axsin9 + by

Put

=0

(a

+ b ) tan9. (Prove it)

at Q
(a

2 + b 2) tan9

asin9

Again at R, put x = 0
y

(a

+ b ) tan9

M (X, Y), being the mid-point of RQ, is


X

a (a

+ b ) sec 9 ,

1
2
2
b (a + b ) tan 9
2

Eliminate 9 to find the locus of M

or

(a

2 2

b )

is the equation of the locus of M. (Another

hyperbola)
Note:

If b = a, then the original curve is

also x - y = a

2
x - /

i.e. the same curve!

=a

and the locus of M is

212
2

The tangent at P(asece. btan9) on the hyperbola x - ~ = 1 intersects the


2
a
b
axes Ox and Oy at M and N respectively. Prove that as P varies on the
a2 b2
hyperbola, the locus of a point Q is given by
:: 1. where OMQN is

4.

2X

a rectangle.

2y

SOLUTION:

The equation of the tangent at


P(asece. btan9) is ~ sec9 - ~ tane ::
at N. x

==

o.

==

-bcote,

at M. y

==

O, x

==

acose

:. Q(x,y) is (acose. -bcot9)

:. The equation of the locus of Q. by


eliminating e. is:
X ::

aCOS9t y :: -bcot9

~y

~ = sece
X

==

-tane

2
b
2
2
=sec 9-tan 9::
2
2
X
y

.
a2
1.e. 2

-2 = 1

5.

The tangent to the hyperbola xy

b2

=c 2 at

the point P(ct.

in Q and R and the normal at P intersects the line


that PQ = PR

~) intersects the axes

y =x

in

s.

Prove

= PS.

SOLUTION:

The equation of the tangent at P(ct, ~) is


x + yt 2. = 2 ct

Q is (2ct, 0)
R is (0, ~c)

We find that P is the mid-point of RQ.


:. PR = PQ

(1)

Now the equation of the normal at P is


c
2
Y - t = t (x - ct)
Or t \ - ty = c{t 4 - l)

.. (2)

(Solution continued next page)

213

Solving y
x(t

=x
3

and equation (2) for S, we have

- t)

= c(t

- 1)

c~ 2

S is

c(t - 1)
t3 - t

1)

[~ (t 2 + 1), ~ (t 2 + l)J
~ (t2

+ l) -

ct] 2

SP=~

(3)

(ct - 2ct) 2 + ( ct
/ 2t2
.jc

+-

From (1), (3) and (4):

PQ

:. PQ

~ (t2

,\ 2
- q.,

c2
t2

(4)

= PR = PS

+ l) -

~2

214

Exercise 60
Find the equations of the (a) tangent and (b) normal to the following curves at the
points given:

1.

l._

1,

2.5 + 16

P(~,

2.

1 , at

3.

2 , at x

u'3>
2

2
1T

4.

4cos9 , y

3sin 9 , 9

.5.

x = .5 sec 9 , y

4 tan 9 , 9

6.

Show that the equation of the normal to the ellipse x


L
a2 + b2

= 41T
2

(a)

(b)
7.

2
at P(acos9, bsin9) is axsin9- bycos9 = (a - b 2) sin9cos9.

Show that the equation of the tangent to the hyperbola


2
x

- L
b

2
2

yy 1
- -

XXI

1 at P (x , y ) is 1
1
2

and at the point (asec9, btan9) is ~sec a

&tan9

8.

Show that the equation of the normal to the hyperbola ~ - L


a2
b2

(a)

(b)

at P(asec9, btan9) is axcos9

9.

P is any point on the ellipse -X + l._


a2
b2
PS + PS'

= 2a .

bycot9
1, with foci S and S', prove that

----------

215

10.

P (x, y) is any point on the hyperbola


prove that

11.

12.

I PS'

x2

2a - Lb2

= 1 with foci S and S',

- PS I = 2a

Find the equations of the tangent and normal to


(a)

2
2
the ellipse x + ~ =
9

(b)

the hyperbola x

- /

at x
=

at x = 3.

Find the equations of two tangents to the ellipse 16x

+ 25/

~00

which are parallel to the line y = x + 2.


13.

The tangent to the hyperbola ~

- ~ = 1 at P(4 Ji., 3) meets the

asymptotes of the hyperbola at A and B. Show that P is the mid-point


of AB. Find the length of AB in the exact form.
1~.

Find the equation of the tangent to the curve whose parametric equations are
x = 2cos9 and y = 3sin9 at a=
A

15.

J . This tangent meets the x-axis in

and y-axis in B. Find the length AB.

2
2
Show that the equation of the normal to the ellipse x
L =
a2 + b2
at P(acosa, bsin9) is given by axsin9 - bycosa = (a

2
- b ) sin9cos9.

The normal at P meets the x-axis at M and N is the foot of the


2
b cosa
perpendicular PN to the x-axis. Prove that MN
a
16.

x2
L2
The tangent to the hyperbola = 1 at P(a sect, b tant) meets the
a2
b2
asymptotes in A and B. Prove that P is the mid-point of AB.

17.

The chord through the focus S(ae, 0) of the ellipse x + ~ = 1,


a2
b
at right angles to the x-axis meets the ellipse at P(acos9, bsin9). The
normal at P

passes through the end-point B' of the minor axis, of the

ellipse. Prove that:

sin9=~

(a)

cos a= e

(b)

2
2
equation of the normal at P is axsin9 - bycosa = (a - b )sin9 cosa

(c)

2
e 4 + e -1

and

= 0.

Hence, find e in the exact form.

216

18.

2
2
The ellipse x + L ::
a2
b2

1 intersects the x-axis at A and A'. Find the

co-ordinates of A and A'. Write down the equations of the tangents at


A, A' and

P (x , y ). Let the tangents at A and P intersect in Q and


1 1
those at A' and P at Q'. Prove that the product AQ.A'Q' is independent
of the position of P.
19.

Show that the condition for the line y = mx + c to be a tangent to the


2
.
x2
.
2
2 2
2
elhpse
Prove that the pair of
2 + ~ :: 1 ts c :: a m + b
a
b
2
2
tangents from the point P (4, .5) to the ellipse ;.5 +
= 1 are at right

tr

angles to one another.


20.

x2
v2
+ J- Show that the tangent to the ellipse a2
b2 .

XXI

equat10n

a2

yyl
b2

= 1.

1 at P (x

1'

y ) has the
1

This tangent meets the x-axis at T.

PN

is perpendicular to the x-axis, and the normal at P


2
meets the x-axis at G. Show that OT x NG = b , where 0

is the

centre of the ellipse.


21.

The line y

= mx

Show that c

+ c

x2
is a tangent to the hyperbola 25 -

= 2.5m 2 -

f-6

16. The tangents from

P (x , y ) to this hyperbola
1 1
2
meet at right angles. Prove that the locus of P is the circle x +
= 9.

22.

P(asece, btan9) and Q(aseccj, btancj) are two points on the hyperbola

2
X

- ~ ::

1 , SUCh that 9 + f$ :: 90.

Find the co-ordinates of the mid-point R of PQ and hence show that the
2

locus of R is given by

a2

23.

b2

x2
P (x , y ) is a point on the hyperbola 25 1 1

f-6

= 1

Prove that the equation of the tangent at P is 16xx


(a)

2.5yy

= 400

Find the co-ordinates of the point G at which this tangent cuts the
x-axis.

217

(b)

~;p_ = ~~

Hence prove that

where S and S'

are the foci of the

hyperbola.
24.

2
2
Show that the normal to the ellipse ~ + L
a2
b2
(a

(a)

- b ) xI y I

(b)

=e

PS and

PS' , where S and S' are the foci of the ellipse.

Hence prove that

PS

GS

= GS'

Write down the equation of the normal at P (5cos9, 3sin 9) to the ellipse

~5

t-

= I.

This normal cuts the x-axis and the y-axis at


that the locus of the mid-point of GH
eccentricity as the first.
26.

=e

This normal meets the x-axis at G. Prove that GS


GS'

25.

I at P (x , y ) is given by
1 1

G and H respectively.

is another ellipse with the same

Sketch both ellipses on the same co-ordinate axes.

Show that the gradient of the line joining the points P (ct , ~ ) and
1

Q(ct ,
2

) on the hyperbola xy = c
2

is

t-~

. The points

P, Q, R lie on

I 2

this hyperbola.

The line through P perpendicular to QR

through Q perpendicular to PR
2
xy = c

at M. Prove that

meets the line

lies on the hyperbola

2
27.

Show

Show that the line y = mx + c

is a tangent to the ellipse x + L


a2
b2

= I

2 2
2
a m
+ b . Hence obtain the quadratic equation satisfied by m

is the gradient of the tangent from the external point P (x , y ).


1 1
Find the locus of P if the two tangents from P are at right angles.

where

28.

Find the equation of the normal at

P (x , y ) to the hyperbola ~ - L
1
1
aa b2

PN is perpendicular to the x-axis, and this normal meets the x-axis at G.


Show that

NG : ON = b

: a

, where 0 (0, 0).

= I.

218

29.

Show that the equations of the tangent and the normal to the hyperbola

x - ~ = 1 at P (asece, btan9) are respectively:


2
a
b
(a)

bx sece- ay tane = ab ,

(b)

by sece +ax tane = (a

and

2
+ b ) sec9tan9.

The tangent and the normal cut the y-axis at M and N respectively.
Show that the circle on MN as diameter passes throu3h the foci of the
hyperbola.

30.

Show that ab

= 2c 2

(a)

if the ellipse x
L
a2 + b2
2
touches the hyperbola xy = c

(b)

P(x , y ) moves on the line y = mx and Q(x , y ) moves on the line


2 2
1 1
y = - mx. Find the co-ordinates of R, the mid-point of PQ, in terms
of x

PQ
31.

= 2K,

, x and m. Show that the locus of R is a certain ellipse, if


2
1
where K is a constant.

Show that for all values of 9, the point P(4cos9, 3sin9) lies on the ellipse
and find the equation of this ellipse.
(a)

Find the equations of the tangents at the points P and Q(-4sin9, 3cos9 ).

(b)

Find the point of intersection, T, of these tangents and show that as


9 varies, the locus of T is the ellipse 9x

32.

+ 16/

288

The ordinate at P(asece, btan9) meets the asymptote of the hyperbola

2
2
x - ~
2
a
b

1 at Q. The normal at P meets the x-axis at G. Prove that

GQ is perpendicular to the asymptote.


(Hint: axtane + bysec9 = (a

2
+ b ) sec9tan9 is the equation of the

normal.)
33.

2
Prove that the equation of the tangent to the hyperbola x - y 2
P (x , y ) is xx - yy
1 1
1
1

=c

. This tangent meets the lines y

=c 2

=x

at

and

y = -x at Q and R respectively. Prove that area of t.OQR is constant.

219
3~.

Show that P(acos9, asin9) lies on the circle x


and Q({,6) are two points on the circle x

locus of the mid-point of PQ is the line

+ /

a 2 If P(9)

2
a , prove that the

= V'Jx

given that 9 +

1,6

= 23n

for all positions of P and Q


35.

Simplify cos ( 9Q [r 2 cos ( 9

21
r

i)

+;), r

and sin( 9

2 sin ( 9

21

21 - 21 ,

+;).

Show that if P(r cos9, r sin9) and


1
1

where 0

1 , then

is the centre. Deduce that if OP is

perpendicular to OQ, then r

l 2 + r 2. 2 is independent of the positions of P and Q.


2

36.

:! -~ =

+;)] lie on the hyperbola

The normal at P(asec9, btan9) to the hyperbola x

- ~

=1

meets the

x-axis at G, and PN is perpendicular to the x-axis. Prove that OG : ON


where 0

37.

= e2 ,

is (0, 0).

2
2
P (x , y ) is any point on the ellipse ~ + L = 1 Find the equation of
1 1
a2
b2
the tangent at P. A line drawn from the centre 0 (0, 0) parallel to the
tangent at P,

meets the ellipse at Q. Prove that the area of t.OPQ is

independent of the position of P. Find the area of t.OPQ.

38.

Find the area of the largest rectangle that can be inscribed in the ellipse
9x

39.

+ 25/

225.

A conic is a rectangular hyperbola with eccentriCity

.f2,

directrix x = 1. Prove that the equation of this hyperbola is x

focus (2, 0) and


2
- / = 2.

Sketch the hyperbola with its asymptotes.


(a)

Find the equation of the normal to this hyperbola at a point

(b)

P(x 1 , y ) = (/2seci,6, .fitanr/J).


1
This normal meets the x-axis at Q (X, 0) and the y-axis at R (0, Y).
2
Show that the locus of a point M (X, Y) is given by x - /

~0.

2
The tangent at P(acos9, bsin9) to the ellipse x

a
at M and N. Show that M and

2
+ ~

= 8,

cuts the axes

N are (asec9, 0) and (0, bcosec9)

respectively. Find the minimum value of the area of t.OMN and the
corresponding co-ordinates of P.

as P varies.

220

41.

2
AB is a chord of the curve xy = c , where A is (cp,
Find the equation of the chord

AB.

AB

~)
and B is (cq, ~ ).
p
q

meets the coordinate axes in

and N and R is the mid-point of AB. Show that OR = MR = NR.


42.

Show that the locus of the foot of the perpendicular drawn from the origin to
2
the tangent to the curve xy = c
at the point P(ct, ~) is given by
(x2 + />2 = 4c2xy.

43.

Find the locus of the mid-points of chords of the curve

xy

drawn

parallel to the line lx + my = 0.


44.

A tangent at P(ct,

~)

to the hyperbola

xy = c

intersects the axes in

and B and 0

is the origin. Prove that the area of triangle OAB is


2
independent of the position of P on the curve xy = c

4.5.

The tangents to xy

=c 2

at A(cp,

p)

If the chord AB touches the curve xy


2

and B(cq, ~) intersect at R.


q

2
4c , show that the locus of

R is

given by 4xy = c
46.

47.

A variable chord PQ of the hyperbola xy = c 2

where P is (x , y ) and Q
1 1
is (x , y ), is such that Ix - x I = 2c. Prove that the locus of the mid-point
1
2
2 2
2
2
of PQ is given by the equation x y = c (x + y).
2
P(cp, ~) , Q(cq, ~) are two points on the conic xy = c Show that the

gradient of PQ is

~~

. If PQ subtends a right angle at a third point

R (cr, ~) on the conic, prove that the tangent at R is perpendicular to


48.

PQ.

P(cp, ) and Q(cq, ) are variable points on the conic xy = c 2. Prove that
p

the tangents at P and Q intersect at T ( ~ , ~ ). Hence prove that if


p+q p+q
pq

=k ,

origin.

a constant, then the locus of T is a straight line passing through the

221

49.

PQ

is

is ( 4p,

~)

variable

chord

and Q is 4q,

*).

of

the

xy

hyperbola

= 16,

where P

Prove the following:


(a)
(b)

The equation of the chord PQ is x + pqy = 4 (p + q)


2
The equation of the tangent at P is x + p y = 8p

(c)

8
- -)
The point of intersection of the tangents at P and Q is T(_!Es_
p+q'p+q

(d)

If the chord PQ passes through the point R (0, 8), show that the locus of

T is a straight line, x
50.

=4.
2

P(x 1' y ) is a point on the rectangular hyperbola x - y


1

the foci. Prove the following:


(a)

The eccentricity e

(b)

IPS I= a- .fix 1

(c)

PS PS'

=OP 2,

= Vi

ls'PI

where 0

=a+

.fix 1

is the origin.

= a S and S' are

222

6.10 Tangents and the Chord of Contact


Many properties of the two central conics, the ellipse and hyperbola, are so common
that it is convenient and instructive to treat them as one by writing the equation in
the form

2
~

1r-

=a 2

where A

= 1

(1)

and B

=b2

The tangent at P (x 1' y 1)

.
IS

for the ellipse but B


xx1

=- b2

for the hyperbola.

yy 1

---a-

(2)

= 1

To find the condition for a line to touch the conic, the line:

=0

lx +my + n

must have the form of the tangent (1). Comparing coefficients:


X/A

-,- =

y1/B

---m-

-AI

= -n-

xl

- 1

--n

-Bm
-n

'

But P (x , y ) lies on the conic (1 ), so that:


1 1
2
2
AI
Bm
- 2 + -2n
n
A1

Bm

1 , which simplifies to:

This is the condition for the line lx + my + n

(2)

=0

to be a tangent to the conic

+~=1.

XA

If the equation of the tangent is expressed in the form

= mx

+ b, the condition

becomes:
Am

B = b

(3)

This is left as an exercise to the student.

Chord of Contact:
The chord

PQ,

jommg the points of the contact of tangents drawn to the conic from

an external point T(x , y ), is called the chord of contact.


1 1

223
y

Let P(x ,y ) and Q(x ,y ) be the points of


3 3
2 2
contact of tangents drawn to the conic from
T(x l' y ). The tangent at (x , y ) to the conic
1
2 2
x2

1f-

xx 2

yy 2
+ B"" '"'

"A"

= 1 is

l and

T (x l' y ) lies on it, so


1
xlx2
Y1Y2
-A-+ - B -

(1)

Similarly, for the tangent at Q(x ,y ),


3 3
xlx3
Y1Y3
-A-+-B-=l
(1) and (2) clearly indicate that the points
XX

(2)

P and Q lie on the line

yy 1

r=

(3)

which is therefore the equation of the chord of contact of T (x , y ).


1 1
Though the equation of the C.O.C. is the same form as that of a tangent, it must be
remembered that T(x l' y ) does not lie on the conic.
1

The method shown above can be used to find the equation of a chord of contact to
2
2
. 1.e.
.
. Ie x 2 + y 2 = a 2, a h yper bl
any come,
a para bl
o a x = 4 ay, a Clrc
o a xy = c
etc.

It is left as an exercise for the student to prove the following:

(Chord of

contact = COC)
1.

The equation of the COC, from


xx

2.

+y

=a

is

.
P(x I' y ), to a parabola x
1

= 4ay is

= 2a(y + y )
1

2 2
2
The equation of the COC, from P(xl'y ), to a rectangular hyperbola x -y = a is
XX l

4.

=a

The equation of the COC, from


xx

3.

+ yy

P(x I' y ), to a circle x


1

- yy} = a

The equation of the COC, from P(x I' y I), to a (special) rectangular hyperbola
xy = c

2 IS
. xx I + yy = 2c 2
1

224

6.11 Geometric Properties of the Ellipse


The standard equation of the ellipse
2
2
is ; - +
a
b

I; Centre 0.

Foci: S(ae,O), S'(-ae, 0)

Directrices: x = .! ~
e
Length of semi-major axis
Length of semi-minor axis
Vertices: (.!a, 0), (0,.! b)

=a
=b

B'

-a

X=-

The following properties of the ellipse are proved here:


I.

The sum of the focal lengths is a constant, i.e. SP

PS
We have PM

(Definition)

=e

and

PS'
PM'

=e

S' P = 2a

.". PS + PS' = e (PM + PM') = e MM'


But MM' = The distance between two directrices

= 2ea

2
a = 2a , which is a constant
e
This fact gives us a fast, accurate (and inexpensive)

:. PS + PS' = e .

method of drawing

the ellipse.
A thread of length 2a, fastened at S and S' is kept tightly stretched by a
pencil at P. As the pencil moves, it traces out an ellipse. (See the diagram
above)

2.

(a) The normal at P bisects the angle between SP and S'P.


(b) The tangent at P is equally inclined to SP and S'P.
These properties are equivalent and we only have to prove one of them.

purely algebraic method is too laborious, so we use the following geometric


result. (The proof is given in the appendix)
If

AM is the internal (external) bisector of LBAC of l1ABC, then

AB
BM
AC = MC

BM'

( = M'C

)
D

...........
'

..... ...

..... .......

...
M'

a
e

225

The normal at P(xl'yl) is

2
2
~ - l2._ = a 2 - b 2 = a 2 e 2 , and meets
X1

y1

the x-axis, where y

= 0,

2
in G(e x
2

Then, SG = OS - OG = ae - e x
SP
Now PM

,o)

(definition)

SP = e PM = e(NM - NP) = e(~- x )


1

=a

SP

= e (a -ex )

- ex

X=-

(Note: this result is very


1
useful)

L-----------------------

SG =e. SP

=e

Similarly S'G
SG
S'G

S'P

SP
S'P

PG bisects the LSPS'


Thus, the normal at P bisects the angle between SP and S'P.
(b)

Since the normal PG J. the tangent T'PT,


LGPT'

(both are 90)

LGPT

LGPT'

LS'PG

LT'PS'

= LTPS

LGPT

and LS'PG

LGPS

LGPS

Hence, the tangent at P is equally inclined to SP and S'P.


This is the reflecting property of the ellipse.

A ray of light or a sound wave

originating from the focus S, will be reflected through the other focus S'.

3.

The chord of contact from a point on a directrix is a focal chord.


Proof:
Let T(xl'yl) be a point on the directrix,
SO

that

Xl

a
=e
.
y

The equation of the chord of contact


is

XXI
yy 1
-2- + -2- =I.

For PQ, this bec.omes ea

yy 1
b2 -

= 0,

i.e.

+ -- - 1

PQ meets the x-axis where y

x = ae, which is the focus S(ae, 0).


Hence the result.

226
II.

That part of the tangent between the point of contact and the directrix subtends
a right angle at the corresponding focus.
Proof:
Using the diagram from property 3, we have to prove

l PST = 90

2
2
xx
m

Th e tangent at p( x ,y 2) IS
a 2 + yy = 1 an d t h.IS meets t he d.1rectnx
2

T( ~, k), where
e

(1)

=x

The gradient of SP is m
The gradient of ST is m

mm'

ke

k
a
-e ae

:. LPST

=90

ke
a(l - e 2 )
aeky

a(l - e )
Using (1), we have, mm'

y2
- ae
2

= 2
a (1

2
- e ) (x - ae)

=- 1

and this proves the result.

The ellipse possesses a wealth of useful and interesting properties.

Some of these

follow simply from the definition and. others can be proved by co-ordinate geometry
and plane geometry. The reader who masters the general techniques of proving these
properties will have no trouble in proving the same properties when particular values
of a and b are used.

227

6.12 Geometric Properties of the Hyperbola


y

d'

-a
e

X=-

= -ea

Many properties of the hyperbola are similar to those of the ellipse, so to avoid
repetition, the properties are stated without proof.

lt would be instructive for the

reader to supply the proofs referring, if necessary, to the corresponding results for
the ellipse.
1.

The difference of focal distances is constant.


5'P- 5P
5P - 5'P

i.e.
2.

= 2a,
= 2a,

if

P is on the branch near 5, and


P is on the other branch

= 2a

I5P- 5'PI

The tangent at

if

bisects the angle

5P5'

internally and hence the normal at

P bisects L5P5' externally.


(Using the equation of the tangent at
5T
5P
prove that ,T = ,p )
5
5

P, find the co-ordinate of T and hence

This is the reflection property of the hyperbola.


towards the focus

A ray L P

of light

directed

of a hyperbolic mirror, is reflected towards the other

focus 5'.
3.

The chord of contact from a point on the directrix is a focal chord.


In the figure, RP, RQ are tangents from

4.

R on the directrix.

That part of the tangent between the point of contact and the directrix subtends
a right angle at the corresponding focus ( L P5R = 90).

228

6.13 Properties of the Rectangular Hyperbola


y
y

=c

=y

=a

Fig. 2
In this section we investigate the geometric properties of two special hyperbolas
2
2
2
2
and xy =c .
x - y =a
2
2
2
2
2
= I reduces to x - y = a
and the
If b = a, then the equation ~ - L
a2
b2
asymptotes are y = .:!: x,
each

other.

i.e.

hyperbola

x + y = 0 and x - y = 0, which are perpendicular to


whose

asymptotes

are

perpendicular

is called

RECTANGULAR hyperbola (Fig. 2).


The reader is familiar with the equation y =

i.e.

hyperbola (Fig. I) with the axes as asymptotes.

xy = k, which represents a
=c 2 is also a rectangular

Thus xy

hyperbola, but the reader who is not satisfied by this reasoning should refer to two
2
a
explanations given in the appendix, where it is shown that the hyperbola xy =
is

obtained by rotating the hyperbola x


.
2
a2
.
.
wnte c =
our work.
, to Simplify

- y

The eccentricity of both hyperbolas is given by b

through 45 about the origin. We

= a 2 (e 2

- I) and with b

the

same,

~- L
a2
b2
2
but a few peculiar to xy = c are

this gives e = {2. Obviously most of the properties of hyperbolas


2
2
2
2
x - y = a
and xy = c
are

= a,
I,

given below with proofs. In science and engineering, it is the form xy = k which is
2
2
2
useful rather than x - y = a The law connecting the pressure and volume of a
perfect gas under constant temperature is

pv = k,

and in electricity the law

connecting the current C, the resistance R and the E.M.F. V is given by CR = V.

229

6.14 Geometric Properties of the Rectangular Hyperbola xy = c2


1.

The area of the triangle bounded by a tangent and


the asymptotes is a constant.
y

Proof:
The equation of the tangent at any point
2
P(ct,
on the conic xy = c
is:

r>

x + t y = 2ct

This meets the axes where


OA = x = 2ct , OB = y =
Area of the t.OAB =
1

=2 .
2.

2ct .

2c

t2c
0

OA OB

= 2c = a

constant.

The length of the intercept, cut off from a tangent by the asymptotes, equals
twice the distance of the point of contact from the intersection of the
asymptotes.
Proof:
In the diagram, we want to prove AB = 20P. Using the previous example:
P{ct,

r) ,

2
A(2ct, 0), B(O, tc )
... (I)

(2)

From (I) and (2):


AB

= 40P

which gives AB = 20P.

230

Worked Examples: Geometric Properties


EXAMPLE: ( 1)
Find the equation of the tangent to the ellipse 5x

j>.

P(2,

+ 9y

= 45 at the point

Find the coordinates of the foci S and S'. SV and S'V' are the

perpendiculars to the tangent at P. Prove that:


2
2
x + y = 9 and SV S'V' = 5.

V and V' lie on the circle

SOLUTION:

The equation of the ellipse is

+
2

The equation of the tangent at P(2,


2x

j>

1.
is

2l

l ( .
XX 1
yy 1
15 = usmg -2- + -2- = I]
a
b

or

2x + 3y

=9

(1)

We have a = 9, b = 5, then
2 2
2
2 .
a e = a - b g1ves ae = 2, so the foci are: S(2, 0) and S' (- 2, 0)
SV J. to the tangent (1), so the equation of SV is
Solving (l) and (2), we find
S'V'

J. to

V(

~;

3x - 2y " 6

(2)

, : ~)

the tangent (l) and its equation is 3x - 2y = -6

(3)

Solving (l) and (3): V' (0, 3)


V' (0, 3) obviously satisfies the equation x
Substituting for

+ y

= 9.

V in the L.H.S. of this equation:


2

2
13

hence both V and


Now SV
and

V' lie on the auxiliary circle x

( ~; - 2)2 + ( : ~ )2 = ( : ~ )2 + ( : ~ )2 =

sv 2
SV

3 X 13
----,~-

L.H.S. =

(- 2) 2 + (3)

S'V'

325
.
169

= 9 = R.H.S., and

= 9.

~~~

= 13

13

25 g1vmg SV S'V'

This example is a particular case of the general property of the ellipse

2
Y

7+~
SV . S'V'

i.e.

= I,

= b

V and V'

lie on the auxiliary circle x

For this example a

= 9, b

= 5.

+ y

= a

and

231

EXAMPLE: (2): Prove that the portion of


the tangent at P (x 1' y ) to the hyperbola
1
2
2
~ = 1 intercepted by the

7a - b

asymptotes is bisected at the point of


contact.
SOLUTION:
X

The equation of the tangent at P(x , y ) is


1

XXI
yyl
-2--2=
a
b

... (l)

Let R and Q be the intersections of the


tangent with the asymptotes.
We find the quadratic equation which gives
the ordinates of R and Q. The equation
of the asymptotes is

2X

... (2)

= 0.

From (I)

or

Substituting into (2): a

(l

2 2
y yl

2yyl

+7+~

2 2
2 a YI
I
Re-arranging: y ( 4 b2 - -b
2)

XI

2
2
2YY Ia
a
b 2 2 + -2

XI

XI

This equation gives the ordinates of R and Q.


Let M(X, Y) be the mid-point of RQ.
If the roots are

and y , then:
2

2
2 2
22
- a b y1
1 - b x1 ~
(
42
/
22
22
bx
ay
-bx
XI
1
1
1
22
2 2
2 2
Since P(x , y ) lies on the conic, a y - b x
-a b
1
1
1 1
:. y = yl
2
Xxl
yl
Then using (I), since M(X, Y) lies on the tangent, we have - 2 - -y=
a
b
2
2
2
Xx
XI
yI
XI
1
But - - - - I so - - = - giving X= x
1
2
2
a2
b2
a
a
2

yla
- -b2
2 -

a y

Thus M (X, Y) = (x , y > which proves the result.


1 1

232

Exercise 6E:
I.

PROPERTIES OF CONICS

2
2
144 meets the directrices
The tangent at P (xI' y ) to the ellipse 9x + 16y
1
at T and T' respectively. S and S' are the foci. Prove that L PST and

LPS'T' are both right angles.


2.

Show that the equation of the normal to the ellipse

point P(x ,y ) is 25xy - 16x 1y


1
1 1
(a)

3.

9x 1y1

=e

25/

=e

400 at the

. PS and

. PS', where S and S' are the foci.

PS
Hence prove that PS'

GS
= GS'

.
PG btsects the

and that

V and V' are the feet of the perpendiculars from


the tangent at P(x I' y ) to the ellipse 4x
1

4.

The normal meets the x-axis at G. Prove that GS


GS'

(b)

16x

=4

(a)

SV S'V'

(b)

V and V' lie on the auxiliary circle x

S and S'

9/ = 36.

+ y

LS'PS.
respectively to

Prove that:

= 9.

M is the mid-point of a variable chord PQ of the ellipse

16x

+ 25y

400,

where P is (x , y ) and Q is (x , y ). Prove that the product of the


1 1
2 2
gradients of PQ and OM is constant.
5.

6.

22
22
22
NP is the ordinate of a point P(x , y ) on the ellipse b x + a y = a b .
1 1
2
The tangent at P meets the x-axis at A. Prove that ON . OA = a , where 0 is
the origin.
2
2
P(x , y ) is a point on the ellipse ; 1 1

1e
ctrc

+ y

=a 2

~ = 1 and Q is the point on the


b

.
P
avmg
t he same a b
sctssa.
roveh
t at t he tangents at

and Q meet on the x-axis.


7.

2
2
P(x I' y ) on the hyperbola 16x - 25y = 400 meets
1
the directrix at T. Show that L PST = 90, where S is the corresponding

The tangent at a point


focus.

8.

NP is the ordinate of a point P(x I' y ) on the hyperbola


1
The tangent at P
0

meets the x-axis at L

P(xl'y 1> is a point on the hyperbola }


If

= I.

2
Prove that ON . OT = a , where

is the origin.
2

9.

7 -~

2
v
- ~ = I with the focus at S.

PS is parallel to the asymptote, prove that the directrix, the asymptote

and the tangent at

P are concurrent.

233
PQ is a chord of a hyperbola 9x 2 - 16y 2 = 144 passing through s. The
tangents at P and Q intersect at T. Prove that T lies on the directrix
corresponding to the focus S.
11. Prove that the point of intersection T of the tangents at P(cp, ~) and
10.

Q(cq' q ) on the hyperbola xy = c 2 is given by T ( ~ , .1._) OT


p+q
p+q
produced meets the chord PQ at R. Prove that PQ is bisected at R.
12.

Prove that the portion of the tangent at P(l, -1) to the hyperbola
2
Jx - 2y 2 = 1 intercepted between the asymptotes is bisected at the point of
contact.
(Hint: Use the equations of asymptotes Jx 2 - 2i = 0 and the tangent
Jx + 2y = 1 to find a quadratic in either x or y.)

234

CHAPTER 7 ELEMENTARY PARTICLE

DYNAMICS
7.1

Introduction

Dynamics is the branch of Mechanics (Physics) that deals with the conditions
under which bodies move. The other branch of mechanics is called
STATICS, which deals with bodies at rest or under equilibrium under the action of some
forces.
Two branches of Dynamics are called:
1.

KINETICS: Kinetics is the study of effects produced by forces acting on the


bOdies in motion.

2.

KINEMATICS: This deals with the motion of the body without regard to the
cause, effect or result of the motion. So far in our work, we have done just
that, i.e. the motion of a particle in straight line, the motion of a projectile,
the SHM etc. We discussed the motion in terms of the position, velocity, time,
acceleration. It did not matter which forces caused this motion.

We shall now introduce "the elements of KINETICS" which relate the forces with the
motion of the body. This not only enhances our knowledge of the subject, but we can
now solve a wide variety of motion problems, such as the motion in a RESISTING
medium, the motion in a circle etc.
Remembering that the 4- Unit Mathematics syllabus requires us to study not only the
harder new topics, but also 3U-Maths harCier motion problems, we shall first completely
summarize the previous work, then revtse with harder 3U problems, and then extend to
the required new topics.

7.2

Laws of Motion - Force

In everyday life we use force to pull or push an object. In this chapter we study the
cause-effect relation between the observed motion and the system of forces.
Newton (1642-1727), one of the most famous and greatest scientists, formulated laws of
motion after studying the motion problems which involve application of natural
(gravitational) or mechanical (push, pull, friction) forces.
Newton's First Law of Motion. (Inertia)
A body remains in a state of rest, or of uniform motion in a straiRht line (a
constant velocity, no external force) in the absence of a force.

235

A force is an invisible entity, it is recognised only by its effect, so it is a CONCEPT.


A heavier object requires a greater force to move than a lighter object, hence we say
a heavier object has greater inertia.
The First Law introduces us to the idea of a Force and mass (inertia), i.e. the definition
of what is a force.
Newton's Second Law of Motion
This law relates the change in velocity i.e. acceleration with the magnitude of the
force that produced the motion. It states that:
A force acting on a body produces an acceleration which is proportional to the
magnitude of the force and this acceleration is in the direction of the force.
The mathematical form of the second law is:
Force
Mass x Acceleration
F
= m a
The mass m is a measure of the amount of material and hence the inertia of the body.
In the Sl units:
Mass is in kilograms (kg)
Acceleration is in m.s- 2
Thus a force of 1 N acts on an object of mass of 1 kg, the object accelerates by
1 m.s-2

Mass and Weight of a Body


The weight is the force acting on an object of mass m, due to gravity.
The value of g, the acceleration due to gravity is 9.8 m.s- 2 (at earth's surface).
Weight = m x g
W (newtons) = m(kg) x g (m.s- 2)
A weight of 1 kg is equal to 9.8 N.
Never confuse the weight with the mass.
Weight is a force and hence a vector, but mass is a scalar quantity.

236

Newton's Third Law of Motion:


This law states that the force exerted by one body on another body i.e. action force, is
equal to the force exerted by the second body on the first, the reaction force, and they
are opposite in direction.
N

When you kick a f.oot ball, you are


applying a force on the foot ball, and
at the same time, the ball's reaction
applies an equal force to your foot,
it hurts!

mg
Fig. I

A body of mass m lying on a horizontal smooth surface is pressing the table with a
downward force of its weight, but at the same time the equal and opposite force N
acts on the body. This force is the reaction in the direction at right angles to the
surface.

WORKED EXAMPLES

EXAMPLE 1:
A body of mass 10 kg is suspended by a string from a ceiling.
Find the tension in the string.

SOLUTION:
You need not show the entire system, only the forces acting on the particle.

The two forces acting on the particle are the


tension T, upwards
and the gravitational
force mg , downwards.

We use Newton's second law:


mg

The resultant force = mass x acceleration.

Fig. 2

T - mg = m x O, because the system is at rest, hence,


acceleration = 0
T = mg

9.8
= 98 kg. m.s-2

m = 10 kg,

= 98 N

(I N

= 10

=I

= 9.8

kg. m.s- 2 )

m.s-2

237

EXAMPLE 2:

A particle of mass 20 kg is suspended by two strings. Calculate the tensions in the


strings, shown in the diagram. (Acceleration due to gravity g = 10 m.s-2).
SOLUTION:
y

The forces acting on the particle P


are the two tensions T and T
1
2
and the gravitational force mg.

Fig. 3

We select the rectangular coordinate system at P and decompose (resolve) the system
of forces into the horizontal components Fx and the vertical components, F Y
l:Fx = T 1 cos 4.5 - T2 cos 60
lF y = T 1 sin 4.5 + T2 sin 60 - mg
The upward components are considered positive and downward components negative.
Since there is no acceleration in any direction, we have:
l:Fx

0 and lFY

T l cos 4.5 - T2 cos 60 = 0

and

T 1 sin 4.5 + T2 sin 60 - mg = 0


Remembering, cos 4.5 = sin 4.5, subtract (1) from (2)
T (sin 60 + cos 60) = mg
2
substitute m = 20, g = 10
T2

200
+ cos 60

= sin 60

146 N

From (1):

...

T I cos 4.5 = T2 cos 60


Tl =

146 cos 60
cos 49

= 104 N

(1)

(2)

238

Note: It is very important that:


1.

A free-body diagram is drawn, showing all the forces on the particle.

2.

The proper resolution of each force into two components at right angles to
each other is shown.
The two perpendicular directions need not always be the HORIZONTAL and the
VERTICAL directions,

3.

If there is no acceleration, i.e. the system is at rest, only then is:

IF X = 0 and IF y = 0.
Normally write, F x = m.ax and F y = m.ay' and then substitute for ax
and a , for each problem.
y
EXAMPLE 3:
A truck of mass 3 tonnes is descending an inclined plane at a speed of 20 m/s. Find
the retarding force R, necessary to stop the truck in 30 m. (Angle of the incline is
10).
SOLUTION:
The forces on the truck are:
X

Normal reaction N, perpendicular to plane.


Retardation force R, along the incline.
mg, force due to gravity.
Angle of incline = 10
Resolving the forces along and at right-angles to the plane:

and

R - mg sin 10

(1)

N- mg cos 10

(2)

Now the net force on the truck is along the incline, given by m.a , where
retardation to be calculated using:
2
u + 2ax
2

given u

20

2
-2
-6 J m.s

IF x

rna = 3000 x

From(l), 20000

+ 30 x 2a

= R-

20

20 m/s
30m

= 20000 N

3000 x 10 sinl0, giving R 25200 N.

is the

239

Exercise 7A
In the following examples, take g :: 10 m.s 2
1.

2.

A particle of mass 10 kg is suspended by two strings of length 3 m and 4 m


attached to two points at the same level S m apart. Find the tensions in the
strings.
2
The combined air and road resistance of a car in motion is proportional to v ,
where v is its speed. When the engine is disengaged the car moves down an
incline making an angle sin- I (l/30) with the horizontal, with a velocity of 30
m/s. Find the force required to drive the car up the incline with a steady speed
of 24 m/s, g!ven that the mass of the car is 1200 kg.

3.

A truck of mass M is driven up a road inclined at an angle 9 to the horizontal.


After its speed has reached u m/s the engine continues to exert a constant force
of F newtons. If the resistance R is constant, find the time taken to reach the
velocity v m/s.

4.

A car of mass 1SOO kg is moving at 60 km/h; when the brakes are applied with a
braking force of 12000 N.
Find:

(a)
(b)
(c)

the acceleration
the time taken by the car to stop
the distance travelled before coming to rest.

s.

A body of mass m is pulled up a smooth incline making an angle 9


with the
horizontal, and has an acceleration f. Find the force F that pulls the body.

6.

A mass of 10 kg is pulled along the horizontal by a chain making an angle of 30


with the horizontal. If the tension in the chain is SON, find the acceleration of
the body and the magnitude of the normal reaction.

7.

A smooth block of mass 2 kg slides down an incline making an angle of tan- I (3/4)
with the horizontal. Find the acceleration and the magnitude of the normal
reaction.

8.

A truck of mass 2000 kg starts to climb an incline of angle given by


1
9 = sin 0/JO). The total resistive force is 2000 N.
Find the retardation it experiences.

9.

Find the magnitude of the braking force to stop a car of mass 1200 kg in 20 m
when it is travelling at 60 km/h (a) on a horizontal road (b) down an incline of
an angle
sin-l (1/40).

240

CHAPTER 8 MOTION PROBLEMS IN


TWO DIMENSIONS
We shall first completely summarise the results of various types of motion studied so far
in 3U Mathematics, then revise the harder problems of simple harmonic motion and
projectile motion, then embark on 4U Motion.

8.1 Introduction: Motion In a Straight Une

=
=

Displacement
Velocity

x
dx
dt :

(x, t, v)
X

Acceleration a

= x

..

Fig. 1

dx = dlv)
'd;2
dx \ 2
=

dv

dx

For constant acceleration ONLY, the equations of motion are:


u

initial velocity

constant acceleration

I.

2.

3.

= u + at
2
= u + 2ax
2
= ut + !2 at

Do not use these formulas for the variable acceleration

Method of Solution (variable acceleration)

I.

Given

2.

Given

x =

I
I

f(t), integrate

f(t)dt

g(t) + c

g(t)dt + ct

f(x),

use

Time = t

~x (

2
T)
= f(x) and integrate.

241

3.

.
x = f (v ), use v dv
.
G1ven
dx = f (v ), and mtegrate

y
For VERTICAL MOTION under gravity only replace a, by, g
acceleration due to gravity. x may be replaced by y.
I.

2.

3.

ut u
2

2 gt

gt

2gy

2
p

mg
Fig. 2
0

For downward vertical motion under gravity, assuming the


object falls from the rest, we have: (u = 0)
1

2 gt

gt

8.2

Fig. 3

2gy

Simple Harmonic Motion (Revision)

Definition:
A particle M on a straight line is said to perform a SHM if its displacement
satisfies the differential equation
y
2
d x
2
-n x
2
dt

A'

Fig. 4
The acceleration is always directed towards, and proportional to the displacement from
the centre.
1.

ix

The general solutiOn of 2


dt
x = a cos (nt + a), where a is the amplitude of the SHM,
n and

are constants.

242

2.

If the motion starts at A, where x

substitute in

a cos (nt + ex)

a cos ex
giving

cos ex

0.

ex = 0

a cos nt

x
3.

+a,

A cos (nt) + B sin (nt) is also a general solution.

Details of the motion:


Let x = a cos (nt + ex) be the solution of x
(i)

-n

x.

The greatest displacement given by the function


x = a cos (nt. + ex) is

Ix I

a and is called the amplitude of SHM.

period of SHM is T

(ii)

The

(iii)

dx
The velocity v = dt

2n
n

-an sin (nt + ex)

The maximum velocity is v = an


v

v
(iv)

an

for the particle moving to the right.

an for the particle moving to the left.

We have:

2
X

2
a

Fig. 5

or v

Since v

n2 (a2 - x2)
2

then

3> 0,

lx I ~ a

The graph of v shows that v is greatest


at x = 0 and v = 0 at x = a.
Hence the particle oscillates between two extreme positions
why it is also called an oscillatory motion (-a< x <: a).

a.

This is

243

(v.)

It is instructive to see that the SHM is closely related to a circular motion of a


point P(x,y), centreO(O,O), radius a. (See Fig. 4)

We have:

nt

9
X

a cos nt

a sin nt

-na sin nt

an cos nt

2
-an cos nt

-an

2
-n y

-n x

sin nt

Both components of vector OP perform S.H.M. We study more of this in the


chapter on CIRCULAR MOTION.
As the point P moves on a circle from initial position at A(x = a), its
projection M on the x-axis moves toward 0.
M is at A' (x = -a) when P is halfway round the circle. As P continues to
describe the negative half of the circle, our point M retraces its path towards
the initial point A. Thus as P moves on a circle, its foot of perpendicular
PM, i.e. M describes to and fro motion about the centre of attraction, 0.

EXAMPLE: (I)
A particle P is in the x - y plane and its coordinates satisfy the equations
2
d x

d/

2
d 2v
-n x and :::.....L
2
dt

= -n y

dx
dt

When P is at (5,0),

= 0 '

Prove that the locus of

P is the ellipse

0 '

= 4n

dt
2

25

16

Sketch the graph of the locus.


SOLUTION:

2
d x
2
dt

2
-n x

d~ ( ; )=
v

2
-n x.

- 2I n 2 x 2

+ c

at x = 5 ,

~~

lntegrat ing

= -n;:;;-7,

Fig.6

of negative sign because at A velocity is towards 0.

244
dt
n dx

-J
cos

For

- I

=
X

nt + a

t = 0 and x = 5 , we find a = 0

y = 4 Sill nt.

x = 5 cos nt Similarly,

we write these as: cos nt = x/5


Sill nt = y/4

25

16 = I

Hence the locus of P is an elhpse. (Shown in Fig. 6) with centre at (0,0) and semi-axes
5 and 4.

Exercise SA

I.

The rise and fall of the tide at a certaill port may be considered as simple
harmomc, the time difference betwf:'en successive high tides being I 0 hours. The
harbour entrance has a depth of 20 m at high tide and 8 m at low tide.
If the low tide occurs at 10.00 a.m. on a certain day, filld the earliest time that
a cargo ship requirmg a minimum df:'pth of 15 m of water can pass through the
entrance.

2.

The acceleration of a body movmg along the x-ax1s


-x for

given by

x -$ I

2 - x for
(a)

IS

>I

Show that the quantity

~~)

f(x) is a constant of motion, where

f(x) =

I
2
x
2 or 2 (x - 2) accordillg as x-$ I or x > I

(b)

When t = 0 , x = 0 and the initial velocity of the body is 1/3 in the


positive direction.
Determme the extreme points of the suosequent
motion, hence prove that the motion is S.H.M.

(c)

If in Exercise (b),

v = 2, find the extreme points of the motion.


whether the motion now is SHM or not.

Explain

245

3.

Show that x

=r

cos (wt +

rJ)

where r , w , rJ are constants, is a solution of

2
d x
2
= -wx.
2
dt
A small naval target rises and falls with SHM of period 10 seconds; the height
of the waves from the crest to trough is 2 m.
At a horizontal range of 2000 m a gun is fired so that the target would be hit
provided it remains stationary in its highest position. The horizontal component
of velocity is 1000 m/s. Show that the target would be missed by a vertical
height of approximately 0.69 m.
4.

A particle moves in a straight line Ox and its equation of motion is


2
d x
= 9- 9x.
2
dt
Prove that the displacement at time t
x = 1 +

~cos(3t-

is given by

127), given that v = 4 m/s, when x = t

= 0.

What is the centre of oscillation?


Find the amplitude and the maximum velocity.
5.

The velocity v of a particle moving along the x-axis is given by


v

= 12

8x - 4x

, where x is the distance from the origin 0.

Prove that the motion is SHM and find

6.

(a)

the amplitude

(b)

the centre of oscillation

(c)

the period.

P, Q and R are three points on the x-axis such that PQ = QR = 2 m. A


particle performs a SHM along the x-axis and is observed to have the velocities
of 12, 10 and 6 m/s at P, Q and R respectively. Taking the origin at the
centre of oscillation, find
(a)

the constant n in the equation

(b)

the distance of

(c)

the amplitude.

x = -n 2x

P from the centre

246

8.3

Motion of a Projectile (Revision)

A particle IS projected from a point 0


in a vertical plane, at an angie of a
to the horizontal, with velocity v.

The only acceleration acting on the


particle is due to gravity, hence
the equations of the motion are:

0
Initially,

and

= 0 = y = t

'

2
d v
= -g
2
dt

=-.L

Fig. 7

VI

v cos

a (horizontal

v2

v sin

a (vertical

~
dt

v sin

a-

gt

(v sin

a)

t -

component)

component)

Integration gives
dx
dt

v cos

(v cos

Eliminate

a)

(I)

gl

(2)

from (2):

The path of the projectile is a parabola

gx sec
2
2v

ay = xt a n

(3)

From (2) , the time of flight when


T = 2v sin
g
From

x
R

(v cos a) T,
v cos

y = 0, t = T

a . 2v

(4)

the horizontal range


sin
g

sin 29
g

R = OA , is

(5)

247

v2
The range is the maximum when 9 = 45 , R
=
'
max
g
At the highest point H , velocity

= 0 .

sin 9
v - - , hence the maximum height from (2) is
g
2
v sin2 9

(
from 1) t
h

From (5) :

~(vertical)

(6)

2g

R = v

(7)

sin 29

Since sin 2(90 - 9) = sin 29, there are two angles of projection 9 and 90 - 9, which
give the same horizontal range R. It is easy to see that these two directions are
equally inclined to the direction given by 9 = 45.
y

Fig. 8
There is no need to memorise the above results.
Start from :

= 0 and

= -g

The formulas of motion are different from those given above when the initial
conditions are different. For example, the projectile may have been fired at a height,
say, a, above the horizontal ground, in that case the component y is given by:
y =

a+(vsin9)t-~gl

EXAMPLE: (I)

A particle is projected in a vertical plane at right angles to a wall of height, h,


standing on horizontal ground at a distance, c, from the point of projection 0. It just
clears the wall at the highest point of its path. Prove that:
(a)

the speed v of projection is given by

(b)

the angle 9 of projection is tan-

1 2h

-j. (c 2 + 4h 2)

248

SOLUTION:
Starting with axes at 0 and
2
2
d x
d
-=Oand~=-g
2
2
dt
dt
We can show that
(1)

x = (v cos a) t

= v sin

a-

(2)

gt

t2
y = (v sin a) t - ~-

y= 0

At the highest point,


Substitute t = v s!n
:. c = v cos

a .

(3)

Fig. 9
, hence t = v sin
g

=v

sin

a .

(4)

in (1) at P(c, h).

v s!n

2
= vg sin

Substitute y = h and t = v sin


g

v sin
g

a - ....2 (
2h

Divide (6) by (5) : tan

or

a cos a

(5)

in

(3)

v sin
g

a)

tan- 1 2h

21

v
. 2a
sm
g

From (5) :
c

2g

cos

. 2
a sm
a

v; (1 - sin
g

2-

2 a)

sin 2 a

. 2a
sm

v4(1-~)~
2
2
2

=~]
2

This simplifies to v 2

...&

2h (c

+ 4h )

(6)

249

Exercise 88
1.

A stone of mass m is projected with velocity 30 m/s from a point at the foot
of an inclined plane making an angle of 30 to the horizontal. The path of the
projectile can be assumed to be in the vertical plane containing the line of the
greatest slope of the inclined plane. If the angle of projection a > 30, for what
values of a will the stone strike the inclined plane:
(a) horizontally?
(b) at right angles?

2.

Prove that the range R on a horizontal plane of a particle projected at an angle


9 to the horizontal and velocity v is given by :
where g is the acceleration due to gravity in m.s-

2
v sin 29
g

R
2

Also prove that the equation of the path of the projectile can be written as:
2

2
tan 9 -

2
2
2
2v x tan 9 + ( 2v y + gx ) = 0

(a)

gx

(b)

1
At what angle must a body be projected with a speed of 50 m.sto just
clear a wall 10 m high at a distance of 60 m from the point of projection.
(Hint: use part (a) with v = 50, y = 10, x = 60, g = 10. Find two answers)

3.

A stone is projected with an initial velocity v

in a vertical plane at an angle 9


1
to the horizontal and hits the ground at a point A. Another stone is projected
at the same angle of projection, but v > v
2
2
1
Let B (x 2, y > and C (xI' y ) be two points on the respective paths of flights
2
1
at the same time t. Show that the gradient of segment BC is independent of
time t.
with an initial velocity v

When the second stone just clears a wall of height h, the first stone hits the
ground at A. If the wall stands at point D on the level ground, prove that
AD = h cot 9.
Further show that
tan (-r.f)

tan 9 -

g T
, where r,f is the angle made by the downward
v cos 9
2

flight of the faster stone with the horizontal, and


the slower stone.
Hence show that v

(tan9 + tanr.f)

2v

tan9,

is the time of flignt for

250

4.

The nozzle of a water hose is at a point 0


on the horizontal ground. The
water comes out of the nozzle with speed
U m/s. Neglecting the airresistance, prove that the water can reach the wall at a distance d from O, if
U

> gd,

where g is the acceleration due to gravity.

If U 2 = 4gd, also prove that the maximum height that can be reached by the
15d
. on t h"1s wa 11 IS
. g1ven
.
by 8
Jet
.

5.

A missile is fired from 0 with initial velocity U at an angle


horizontal. Prove that it describes a parabola of focal length

with

the

u 2 cos 2 a
2g
4
2
Also prove that any point P(x,y) within and on the circle x + /
= v
1s m
2
2
danger of being hit by the missile. (g m.s- is the acceleration due tog gravity).

6.

A stone is projected upwards at an angle 9 to the horizontal. Find an expression


for the velocity v at time t in terms of g, t and the initial velocity U.
If the stone at time

velocity, show that t


v
7.

= U cot

= -u
g

is moving in a direction perpendicular to the initial


cosec 9 and that the stone's speed is given by

A body is projected with speed U from a height h, above a horizontal plane,


at an angle 9 to the horizontal. Show that the range R on the horizontal is
given by
2
2
2
2
gR sec 9 - 2U R tan 9 - 2hU

=0

Further show that the maximum range R

is given by

and the corresponding angle of projection is given by


tan9

u2

=
gR1

Hence prove that h

= R 1 cot

2 9.

251

8.

A particle is projected from a point 0 with velocity v at an angle 9 to the


horizontal. It passes through the point P (x,y) in the vertical plane through 0
where (x,y) are the co-ordinates of P with respect to the rectangular axes at 0.
Prove that .y

=. 20

=x

tan 9 - gx se;: 9
2v

-2

m, g = 10 m.s , v = 20 m/s, find the two values of tan 9


.
. 2 9 =- 2t
. o f two ranges
and usmg,
t = tan 9 an d sm
prove t h at t he ratto
2
. 5
I + t
IS j .
If x

9.

= 10

m, y

A particle is projected with velocity v at an angle 9 to the horizontal. Show


that by SUitable choice of axes, the equation of the path of the projectile is
y =

tan 9 - gx sec B
2
2v

Prove the following:


(a)

There are two possible directions of projection given by tan 9


and
1
tan 9 for a given range R.
2

(b)

tan 9 1 + tan 9 2

(c)

Let T

2
2v
gR

Tl

sin 9

T2

sin 9

(d)

From (b) prove tha1 9

(e)

(f)

and tan 9

tan 9

I.

and T be the times of flights corresponding to angles of


2
1
projections 9
and 9
1
2
Prove that

10.

1
2

+ 9

90.

sin 29 , where R is the maximum range.


1
2

T'

then

R'

[Hint: sin29

= ~]
I + t

Two particles P and Q are projected from the same point 0 with the same
velocity 25 m/s. They both strike the horizontal plane through 0 at the point
A, 60 m from 0. P reaches A before Q. Show that:

(a)

the angle of projection of P is tan-!

(b)

the time of flight of P is 3 seconds.

(c)

the distance between P and Q at the instant P reaches A is

(d)

and that of Q is tan-!

the ratio of the maximum heights reached by P and Q is

15

/2

16

m.

252

8.4

Resisted Motion: Other Laws of Motion

In our previous studies of motion, we neglected the effect of air-resistance or air-

friction. A body moving in a fluid experiences a resistance which tends to stop the
motion. In many cases of motion, the resistance is an important consideration. Cars,
planes and boats are streamlined so as to reduce the frictional drag and improve fuel
economy.
The air or fluid resistance on an object depends on its:
(i) shape

(ii) size (exposed area)

(iii) speed.

For example, a sky-diver with an unopened parachute falls quite rapidly, but the
descent is slowed when the parachute opens.
The parachute encounters greater
resistance due to its shape and size. A sky-diver can enjoy a free fall (without an open
parachute) by employing a spread-eagle position to increase the air resistance and
prolong the time of fall.
Air or fluid resistance also depends on the speed of the object. The greater the speed,
the greater the air resistance. We shall mostly be concerned with motion for which
the resistance is proportional to the speed v or v 2
Contrary to our perception of resistance, it is quite beneficial to us. Actually it is the
road friction that makes car driving possible! Sky-diving is
pleasant and possible
because the air-resistance helps to slow the descent. Though streamlined cars cost
more, they are at least pleasing to our eyes, if not to our purse!
EXAMPLE : (1)

A particle of mass m moves along the x-axis. It experiences a resistive force R


given by R = kv, where k is a constant and v is the velocity at any time t and
position x. Discuss the motion.
SOLUTION:
(x,t,v)
p

The equation of motion at time t is given by


dv
m. dt

-kv

(F

= ma)

We attach a negative sign because the resistance opposes the motion.


k
dv
dt = .v

Integrating
loge v

I~v

f~ .dt

kt
A
m +

or v = B e-kt/m ' where B is a constant.

Fig. 10

253

If v = v

at t = 0, then B = v

-kt/m
v = voe
As t ...

the function e-t ... 0 and hence v ... 0

co

dx
-kt/m
Further v = dt = v0 e
The distance travelled in time t
X

- Jt

0
or x

jp-

vO

is

v e-kt/m dt

[I -e-kt/mJ
mvo

Again as t

-+- "' x-+-

i.e. it moves with decreasing velocity towards the

limiting position xt

EXAMPLE : (2)

A particle of mass m falls under gravity from rest in a medium with the resistive
force given by R (v) = kv. Discuss the motion.
SOLUTION:
0

We take the initial position as the origin and x-axis


along the direction of motion. The equation of
motion at time t is:
dv
m dt
dv

mg -

mkv
p

dt = g - kv

or

dt

- kI [log (g- kv)- log (g)]

Fig. 11

- k1 loge ( 1 _kgv)

254

We can solve this equation for

;kt = 1 _ kv
g

f (1

- e-kt)

(1)

As t-+oo,

Fig. 12
vT

=f

is called the terminal velocity and the particle continues to travel with

constant velocity vr

This happens when the resisting force

mkv

balances the

gravitational force on the particle, i.e.


mkv

mg

or

gk

[Have you ever wondered how a team of sky-divers frolic (in the sky) with their
parachutes not open! From above you see that a sky-diver should enjoy a free fall
until his weight balances the resistive force, thereafter his parachute must open, and
with good luck the diver should then enjoy his fall with the reduced terminal velocity.
Terminal velocity before the parachute opens is about 200 km/h and it is 40 km/h after
the parachute opens.]
Further

dx

When t

hence

from (1)

dt

= O,
x

=0,

this gives c

255

EXAMPLE : (3)

A particle of unit mass is thrown vertically upwards with a velocity of U into


2
the air and encounters a resistance kv . Find the greatest height h achieved by
the particle and the corresponding time.
SOLUTION:

Equation of motion is
dv
v dx

-g - kv

vdv
g + kv2

dx

- 2k loge (g + kv ) + c

When x

= 0,

= U,

this gives c
2
g + kU
2
g + kv

x = 2k loge

= 21k

2
loge (g + kU )

(1)

For the greatest height, x = h, v = 0


1

h = 2k log ( 1

k 2
g
U )

. (2)

For the corresponding time, we use:

2
dv
dt = - g - kv

-dv
g + kv2

Fig. 13
dt
dv

or

-1
g+kv2

=- k

1
.fiJk

r"

Ltan

1/fkg . v

]o

256

EXAMPLE : (4)

A particle is thrown vertically upwards with speed U in a medium with resistance


R = mkv, where m is the mass of the particle and k is a constant. Find the
greatest height h reached and the corresponding time.
SOLUTION:

Selecting the origin as the point of projection,


the equation of motion is
dv
m.v. dx.

- mg- mkv
p

_I ~

= -~+~

When x

= 0,

k1

(g + kv) g + kv

= U,

mkv
dv

loge (g + kv) + c
hence 0

=-

log (g + kU) + c

k
(1)

(U - v)

At the greatest height, v


h

= .k!d. -

= 0,

=h

&... log ( 1 + kgU )

(2)

k2

To find the time, we use


dv
dt

-g - kv

o
t--

U g + kv

or
1

k log

dt
.dv

( 1 +

~u)

-1
g + kv

(3)

Fig. 14

257

EXAMPLE : (5)
A body is projected vertically upwards from the earth's surface with velocity U. The
2
acceleration of a particle in space is given by kx , towards the centre of the earth,
where x is the distance of the body from the centre of the earth.
Given that the acceleration is g at the earth's surface, prove that the velocity v at
time t is given by
v

= u

2
- 2gR (

~-~

where R is the radius of the earth.


If u

~~

= 2gR, find

in terms of x and show that the body will reach a distance &R

from the earth's surface in 2.72 hours.


Also find the velocity of ESCAPE, i.e. the velocity
never returns to the earth.

U of projection so that the body

(R = 6400 km, g = 10 m.s-2 ).


SOLUTION:

Selecting the axes as shown, the equation


of motion is
X

dv
m.v. dx
and

mk

(l)

-7
...

k
(given) at x = R
;2
2

k = gR '

hence

dv
vdx

R2

(2)

mk
x2

-&y
X

x=R
Integrating ,
At

= R, v = U,

gR2

u2
so C=y - gR

v 2 = U 2 - 2gR 2(

R.

Fig. 15
(3)

258

If U

2gR then from (3), v

= dx
dt

For

2 R
T

i 112 (sign

+ v'22gR.

IR

9R

xl/2 d

ffgR

+ as v t from

= 6.4

x 10 m, g

= 10

-2 , t

m.s

Finally, the particle never returns if x

The body never returns if v

(27 R/R - R {R)

H.

= 2.72
-+

hours

oo,

(since

- 2gR

/ZgR

5: .

0)

2 [ x3/2] 9R

ffgR .

=3

For R

112
x
dt
dx
Rffg
8R + R = 9R (from the centre) time taken is

hence from (3)

.!X

-+

0)

u > 2gR
U

> /2gR

U must be slightly greater than


i.e.

> 11300

> 11.3

m/s

km/s

The escape velocity

12 km/s

h x 10 x 6.4 x 106

259

Exercise SC
I.

A particle of. unit mass falls vertically from rest in a medium with resiStive
force R = kv, where v is the velocity of the particle at time t. (k is a
constant).
Find the velocity v and hence show that the terminal velocity is given by

2.

f.

2
A particle of unit mass falls from rest in a medium with resistive force R = kv ,
where k is a constant. Prove that the distance x fallen when the velocity is
v, is given by:
1
= 2k

loge (

g 2 )
g- kv

Find the distance fallen when it reaches half of its terminal velocity.
2
= 0, i.e. v = .fijk).

(Hint: The terminal velocity is given by g - kv


3.

An object is projected vertically upwards with initial velocity U from the


earth's surface. The acceleration obeys the law given by
2
d x
dt 2

k
- x2

where x is the distance of the particle from the centre of the earth whose
radius is R.

= R,

Given the acceleration is g when x


is given by:
v

= u

Hence show that

2gR

show that velocity v in any position

U = 12 km/s

for the escape velocity.


not return to the earth). (R = 6400 km, g = 10 m.s- 2 )

(Hint: x
4.

+ oo

for escape from the earth and v

(i.e. the object does

> 0)

A particle moves under gravity in a resistive medium with the resistance


= kv, where v is the velocity in any position and k is a constant. The

particle is projected vertically upwards with velocity U


time the expressions for the velocity v and position
-kt

v = U \2e

- l)

and

x =

ku (2

-kt - kt ) .
- 2e

Find the greatest height achieved by the particle.


(Hint: v = 0 for the greatest height).

=f .

Show that at any

are given by:

260

5.

A particle of mass m is projected vertically upwards under gravity with velocity


2
at any time t and position x.

U m/s. The resistive force is R (v) = mkv


Show that the expression for x is given by:
x = 2k1 loge

[g

+ kU2

g + kv

Show that the greatest height H is given by:

H= ....!.

2k

6.

log

[1

2
kU ]
g

A body of mass m falls from rest in a medium with resistive force R = kv,
where k is the coefficient of air resistance and v is the speed of the object.
(k is a constant.) Prove that the distance x fallen when the velocity is v, is
given by:

= - mkv -

m2 g log
k2
e

[I -

kv ]
mg

Find the terminal velocity for a falling 70 kg sky-diver, if k = 14 and


g
7.

= 10

m.s

Express your answer in km/h.

A ball of mass

is thrown vertically upwards with velocity

resistance is per unit mass f(v)


(i)

(ii)
(iii)

2~

loge

[1

Draw a neat sketch of the downward motion of this ball after it reaches
the greatest height H. Show that the distance y fallen when velocity is
W, is given by

= 2~

loge [

g- kW

2]

Deduce from (iv) that the terminal velocity V is given by


that H can also be given by:
H =

2~

loge [

g
g- kW

W .is the velocity when y


(vi)

The air

Show that the greatest height achieved is

Y
(v)

U.

, at a distance x when velocity is v.

Draw a neat sketch of the motion, showing the forces acting at a distance
x from the point of projection.
2
Show that x = kI loge [ g + kU ]
2
2
g + kv

(iv)

= kv 2

2]

=H

From (iii) and (v) deduce that

where
for downward motion.
I

u2

::::2

v2

f , and

261

8.

9.

A particle is thrown vertically upwards where the air resistance is given by


2
R = 0.01 mv If the velocity of projection is 60 m/s, find the
(a)

time to reach the highest point

(b)

greatest height H.

2
A particle falling from rest in a vertical line in a medium with resistance kv
per unit mass, k is a constant, v is the velocity at any time and position x,
prove that it acquires a speed

[/I -

e-2kh

J vr

in falling through a distance h, where VT is the terminal velocity given by

ff

A particle projected upwards in the same medium with initial speed


returns to the point of projection with speed VR .

VI

and

Prove that
10.

An object of mass 20 kg experiences a resiStive force, in newtons, of one-fifth


of the square of its velocity in m/s, when it moves through the air.
This object is projected verticaHy upwards from a point
30 m/s and reaches the highest point H in time T.

.
Given g

11.

= 10

with velocity of

-2
m.s , find

(a)

the time T

(b)

the distance OH.

A particle of unit mass moves in a straight line against a resistance R given by


= v(l + v 2), where v is the velocity of the particle at a distance x metres

from the origin. Prove that x = tan-l v

- tan-l v,

velocity at the origin. Use the formula

-1
-1
-1 [A
- B
tan A - tan B = tan
I + ABj to show that:
v

v - tan x
0
1 + v tan x
0

where

is the initial

262

12.

A particle of unit mass travels in a straight line against the resisting force
f(v) = v(l + v 2 ). Its initial velocity is c m/s at the origin. Show that the time
t, when velocity is v, is given by:
=

2]
1
'
[1
Iog
_
2
2
+ v-

1 +

2
Find v as a function of t and hence the limiting value of v at t
13.

14.

+ co

A particle of mass
m moves along a straight line under the action of a
constant (propelling) force P, and a resistive force mkv, where k is a
constant, v is the speed at any time t. Show that if the speed increases from
2 m/s to 4 m/s over a time interval of 5 seconds,
= 2km

rL :~

(a)

2e
e

- I] .
- 1

(b)

Find the corresponding distance moved.

(c)

Find the propelling force P for k

= 0.5.

The acceleration due to gravity at a distance

from

centre is directed towards the centre and equal to !5. when r


2

the earth's

> R,

equal to

kr when r < R and equal to g when r = R. Imagine a narrow tunnel along


a diameter XY of the earth and the particle is projected from X with initial
velocity U towards Y. (R = radius of earth).
If U

< 2gR,

prove that the motion is oscillatory and the amplitude is given by

15.

A particle is projected vertically upwards from the surface of the earth with
initial velocity U. The acceleration due to gravity at a distance x from the
centre of the earth is given by k , directed towards the centre. Prove that
2
X

2
the rocket will escape from the earth provided u ? 2gR, where g is the
acceleration due to gravity at earth's surface and R is the earth's radius.
2
Further, if u = 2gR, show that the time to achieve the height R above the
2
earth's surface is approximately equal to 0.273 fR , given g = 10 m.s-

263

16.

A body of mass m is released from a height h above the ground and it


experiences a resistive force R given by R = 0.1 mv 2, where v is tile velocity
achieved by the body at time t.
2
If the object falls from
rest under gravity (g = l 0 m.s- )
Find: (i)

the terminal velocity U

(ii) the height h


the ground.
17.

if the velocity is

0.5U

+ m ).

just before the body strikes

A ball of mass, m, is descending vertically in a tank of fluid (under constant


gravity). The resistive force is kv per unit mass, where v is the speed and
k, a constant.
(i)

Draw a motion diagram at time t and write down the equation of motion
dv
using F = m dt

(ii)

Write down an expression for time t taken by the ball to acquire the
velocity v from rest and hence show that:
v

(iii)
18.

(i.e. velocity as t

= ~

(l - e-kt )

Find the terminal velocity vT. If g


of v against t.

= 10

and k

= 0.2,

draw the graph

A particle P of unit mass is projected from a point 0 with velocity U at an


angle 9
to the horizontal.
The particle moves under gravity and each
component of its velocity experiences a resistance k times the magnitude of
the component. By considering the rectangular axes Ox and Oy and the particle
has components v and v of the velocity v at time t:
X
y

(a)

Draw a diagram of the motion of P

(b)

Write down the equations of motion in the form:


and ~ (v )
dt y

(c)

Prove that v

(d)

Prove: x

= Ucos9e-kt

u
(1 = kcos9

y =

e-kt)

(kUsin9 +g)
2

and

=
. ) e-kt - &
k1 (g + kUsme
k

264

19.

A projectile is fired vertically upwards from the earth's surface with velocity
U m/s. The retardation due to gravity is given by the law k
X

where x is the

distance of the projectile from the centre of the earth, and k is a constant. The
acceleration due to gravity on the earth's surface is g. The earth's radius is R.
2

Neglecting the air resistance, show that if u


reaches the height R above the earth's surface.
for this journey is
[

20.

= gR,

then the projectile


Show further that the time

1] A"

An object of mass
mv

resistance R

=k ,

is thrown vertically upwards in a medium with a

where v is the velocity of the object and k is a constant.

Given t = 0 = x and the initial velocity is k(c - g), where c is a constant


and g is the acceleration due to gravity, x is the distance travelled in time t.
(a)

Draw a neat sketch of the motion and the forces acting at a point P,
distance x from the origin. Hence write down the equation of the motion.

(b)

Find the time taken by the object to reach the highest point H and find
the height of H above the point of projection.

(c)

The object falls to its original position with the same law of resistance.
Will the time of descent be the same as that of ascent? Give your
reasons.

265

CHAPTER 9 CIRCULAR MOTION


9.1

Introduction

The study of circular motion is of great importance in science and engineering. The
orbit of the earth around the sun, or the moon around the earth, can be considered
circular for practical calculations. The safe speed on a circular section of a
highway or railway track is governed by the laws of circular motion. We shall also
study the problems related to circular motion, such as conical pendulum and
banked tracks.
In solving the motion problems, we often require resolution of the forces in two
perpendicular directions OX and OY. These directions need not always be the
horizontal and the vertical. Study the following examples of two resolved parts of the
force F.

y
y

Fig. 1
We shall study the dynamical problems, in which Newton's laws have to be used,
namely

(1)

(2)

Force of action

rv

ma (mx or my)

= Force

of reaction

To solve any dynamical problem,


1.

Resolve all the forces acting on the


system in two perpendicular directions.

2.

Then using F = mx and F = my :


mx

3.

= .EX

- .EFx and my

= .EY

.EF

.EX

- .EFy

Substitute known values in (2) and


solve for the unknown (velocity, force
etc.)

.EF

Fig. 2

266

9.2

Angular Velocity: Period

For a point moving in a straight line, its velocity is defined as the rate of change of
its displacement.

(x, t)
p

Fig. 3
v

= ~~

in m/s where x

= f(t)
2
d x

-2

Linear acceleration of a particle is given by a = dt 2 in m.s

When a point moves on a curve, we talk about its angular velocity, i.e. the rate of
change of its angular displacement, as defined below.
ANGULAR VELOCITY of a point about a given point
Let 0 be the given point, and OX a
line through 0 of fixed direction.
Suppose P moves in the plane containing
OX and LPOX = 9 at any time t.
The anticlockwise rotation is considered
positive and the clockwise rotation negative.
The angular velocity w (omega) of the moving
point P about 0 is defined as the rate
of change of 9, i.e.
X

Fig. 4

The unit of angular velocity is the radian/s and is abbreviated as rad/s. Angular
velocity is a vector and when the direction is not significant we speak of angular
speed. It must be remembered that as defined above w is a variable, i.e. a function
of t. In most of our applications in circular motion, w is a constant, i.e. a uniform
angular velocity. In a later section, we shall talk about the angular acceleration of a
point, about a given point.

267
EXAMPLE: (I)

A point P moves on a circle with uniform angular velocity of

rad/s. Show the

positions of P for t = 0, 1, 2, 3, 4, 5, 6. What is the time taken by the point P to


describe the circle once completely?
ll
3 = 60 .

SOLUTION:

ll

4n

5n

We have:
ll

2n

2n

t=3

t=O
t=6

Time to describe the circle once completely is 6 s.


T = Angular displacement
Angular speed

2n

1iT3

= 6 s
Fig. 5

9.3

Circular Motion: Tangential Velocity

Suppose a particle moves in a circle of radius


r (anticlockwise) and sweeps out angle 9 in
time t.
Let it describe a small arc

PQ in time 8t.

arc PQ = r x 89
Then v,

the tangential velocity at P is given by:

d
d
d9
v = dt (arcPQ)=dt (r.89)=rdt asllt~O
d9
Now the angular velocity is w = dt , hence:
Fig. 6
. (l)

The angular velocity w is usually defined in radians per second, but if the radius of
the orbit is extremely large as in planetary motion, w may be defined in radians per
hour or a day or even a year.
It should be remembered that w is a variable in formula (1), but most of our work
deals with the constant angular velocity and hence constant tangential velocity. In
that case; at time t,
9
1 9

angular velocity x time


=

wt 1

... (2)

It is usual to describe w as revolutions per second or minute or hour etc.

268

EXAMPLE: (2)

Convert the angular velocity w


radians per second.

SOLUTION:

Since 1 rev

50 revolutions per second to

2n radians

50 rps

50 x 2n

lOOn rad/s

EXAMPLE: (J)

A satellite moves in a circular orbit with 20 revolutions per day.


Describe w in rad/s.

SOLUTION:

(J)

20

20 rpd

11
36 X 60

rad/s

11
2160

rad/s

51!
60

24

211 radians per hour

rad/min

EXAMPLE: (4)

A wheel of radius 2 m revolves at 1200 rounds per minute. Find:


(a) its angular velocity
(b) the tangential velocity of a point on the wheel.

SOLUTION:

(a)

(b)

1200 rpm

=2x

rw

1200 X 211
rad/s
60

40n

SOn

= 40rt rad/s.

251 m/s.

THE PERIOD:
The period T of a circular motion with constant angular velocity
the time for one revolution.
T

EXAMPLE: (.5)

w is defined to be

angular displacement in one revolution


angular velocity

An artificial satellite travels in a circular orbit of radius 9000 km. If


the period is 90 minutes, find the angular and the tangential velocity
in km/h.

SOLUTION:
T
(a)

(J)

(b)

211

given T

(J)

211

=r

x w

211

T.5

9000

rad/h
X

4.19

90 min

= 4.19 rad/h
= 37700 km/h

1.5 h

269

Exercise 9A
1.

A particle on a disc rotating with a uniform angular speed of 4 revolutions per


second is 0.2 m from the centre of the disc. Find:
(a)
(b)

the tangential speed v of the particle


the angle through which it rotates in 0.4 s.

2.

A car travels halfway around a circular track in 12 s. What is the angular


speed of the car? If the radius of the track is 100 m, find the velocity of the
car in km/h, to the nearest km /h.

3.

A motor shaft has a speed of 240 revolutions per minute.


speed in rad/s.

4.

Find the average angular speed in rad/s of the earth's


(a)
(b)

Find its angular

rotation about its axis. (Hint: period is 24 h)


revolution about the sun. (Assuming a circular orbit and a period of
I year = 365.25 days)

5.

Find the tangential speed of the earth at the equator due to its rotation about
its axis, given that the radius of the equatorial circle is 6440 km.
[Use exercise 4 (a)]

6.

Find the tangential speed of the earth due to its revolution about the sun,
given that the radius of the earth's orbit around the sun is 1.5 x 108 km.
[Hint: use exercise 4(b)]

7.

What is the angular speed of the particle in a circular path of radius 5 m and
a tangential velocity of 100 km/h?

8.

A belt is wrapped around a pulley that is 0.4 m in diameter. If the pulley


rotates at 240 revolutions per minute, what is the linear velocity in m/s of the
belt?

9.

An aeroplane propeller
revolutions per minute.
(a)
(b)
(c)

whose blades are

1.5 m long is rotating at 2400

Express the angular speed in rad/s


Find the angular displacement in 4 s
Find the linear speed of a point on the end of a blade.

10.

An artificial satellite travels in a circular orbit of radius 10000 km. If the


period is 2 h, find the angular speed in rad/h and the tangential speed in km/h
of the satellite.

11.

The average distance of the moon from the centre of the earth is 3.85 x I o km
and the period of the moon's revolution about the earth is 27.3 days. Find the
angular speed in rad/h and the linear speed of the moon in km/h.

270

9.4

Acceleration of a Particle Rotating in a Circle


y

The diagram shows a particle P moving


anticlockwise on a circle.
At time t,

P is (x, y)

LPOX
w

angular velocity (constant)

tangential velocity

Cit = a constant, so dt

de

dw

The horizontal com onent


X

The vertical com onent

= rcose

rsine

The horizontal velocity

The vertical velocity

dx
. e de
. e
dt = -rsm dt = -rw sm

dv
dt

The horizontal acceleration

The vertical acceleration

de = -rw 2 cos a
-d2x
2- = -rw cose dt
dt
2
=-rw cose

d9
= rcose dt

= rwcose

dt

:. x

. e de
2 . e
2 = -rwsm dt = -rw sm
2
= -rw sine

The resultant acceleration

fx2 + -y2
2

1<-rw cose)
rw
rw

+ (-rw

sine) 2

/(cos e + sin 2 e)
0

The direction is towards the centre 0, along the radius PO of the circle.

Fig. 8

Thus the acceleration of a point moving in a circle with angular velocity w is


2
a = rw
2
2
2
v
v
But v = rw, so
a= rw = r.

2r

rw

v
r

At a later stage, we shall derive the expression for acceleration when w is a variable"

271

9.5

Uniform Circular Motion

As proved before, the acceleration of a particle


moving on a circle is given by:
a

rv

2
or a= rw '

directed along the radius, towards the centre.


By Newton's first two laws, there must be a resultant force that causes this mot1on
and its magnitude must be given by
or

mrw

where m is the mass of the particle and this force must act along the radius directed
towards the centre. This force is called the CENTRIPETAL FORCE (centre seeking
force).
EXAMPLE: (6)

Find the tension in the string when a stone of mass 5 kg is rotating at 50 rpm, the
stone is tied at one end of the string and the other end is fixed at point 0. The length
of the string is 2m. (rpm = revolu~ions per minute)
SOLUTION:
w

50 X 211
- -- - rad/s,
60

50 rpm

511

5 kg, r

2 m

rad/s.

The only force acting on the particle is the tension


T along the string, hence:

centripetal force

mrw

2
25n /9

Fig. 10

274 N
EXAMPLE: ( 7)

A string of length 2 m is attached at one end to a fixed point 0 on a horizontal table,


while the other end is attached to a heavy particle of mass m kg. Find the speed v
in m/s of the rotation of the particle, given that the tension in the string is n times
the weight of the particle.
SOLUTION:

The reaction
Tension T
mv

N = mg is of no concern to us.
centripetal force

n x mg

(given)

rmg

ffng

m/s

=2
Fig. II

272

Exercise 98
(Take g

=9.8

m.s-2 , for the following problems)

I.

A particle moves on a horizontal circle of radius 2 m with uniform linear speed


of 10 m/s. Find the centripetal acceleration required for this motion.

2.

A ball of mass 0.5 kg is rotated at the end of an inelastic string in a horizontal


circle at 5 m/s. If the length of the string is 2 m, find the centripetal force
exerted by the string on the ball.

3.

Find the centripetal force on a car of mass 1500 kg travelling around a circular
track of radius 7 5.0 m at a speed of 45.0 km/h.

4.

A flywheel of radius 0.4 m rotates with the uniform angular speed of 40n rad/s.
Find the centripetal acceleration of a point on the rim of the wheel.

5.

The earth rotates on its axis at an angular speed of 1 revolution per 24 h.

Find:

(a)

the linear velocity in m/s of a point on the equator where the radius of the
earth is 6400 km.

(b)

the centripetal acceleration of a point on the equator.

6.

A curve of radius 250 m is planned in a highway. The expected legal limit is


90 km/h. Find the centripetal force of the road on the car of mass 1200 kg.

7.

An earth's satellite at a distance of 36000 km above the earth's surface is called


synchronous satellite because it and the earth have the same period. Find:

8.

(a)

the angular speed of the satellite in rad/s and rad/h

(b)

the linear speed of the satellite in km/h if the radius of the earth is
6400 km

(c)

the centripetal force required to keep the satellite in the orbit if it has a
mass of 1000 kg.

A communication satellite in a circular orbit at an altitude of 500 km above the


earth's surface makes one revolution in 95 minutes. Given that the radius of the
earth is 6400 km, find the centripetal acceleration of the satellite.

273

9.6

Conical Pendulum
0

A particle of mass m is attached to one end


of a light inelastic string of length l, the other
fixed at 0.
The particle P moves in a horizontal circle,
so that the string describes a cone whose
vertical axis passes through the centre C of
the circle.

mg
Suppose that the particle moves with velocity
v, then the resultant force is along PC and
resolving the forces as shown, we now have
2
Tsin9 = centripetal force = ~
(l)
r
and

v2
Divide (l) and (2): tan9 =
rg
r
and v = rw
Now tan9
Ti

Tcos9

... (2)

Tcos9 = mg

2 2
r w
rg

Fig. 12

f---- mv 2 /r

... (3)

Tsin9

Ti

mg

=ft

... (4)

Hence the period T =

~11

= 211

JI . .

Fig. 13

(5)

The period is independent of the mass m and depends only on h, the vertical distance
of the circle of rotation below the point of suspension 0.
Note that w

depends only on h.

SUMMARY (CONICAL PENDULUM)

1.

tan9

3.

2
v
rg

2.

211

4.

= rg tan9

=If

274

EXAMPLE: {8)

A disc of radius 2 m rotates in a horizontal circle about a vertical axis AB. A light
string PQ of length I m tied at the rim of the disc at P, and carries a mass of
2 kg at Q. The disc rotates uniformly such that PQ is inclined at an angle of 45
to the vertical. Find:
(a)

the speed of the mass at Q

(b)

the tension in the string.

2
(g = 9.8 m.s )
A

SOLUTION:
The forces acting at Q are:
Tension T in the string
Weight m acting vertically downwards
Resolving at Q along the horizontal and the
vertical directions:

(I)

Tcose = mg

mg

Fig. 14

The unbalanced component must supply the


2
necessary centripetal force mv
for the
r

circular motion of the mass at Q, where


v is the tangential speed of Q and

Tsin6

r = BQ = 2 + I . sin 9
2

Tsin6 = ~

(2)

mg

Divide (2) by (I): then


tane
Given e
v

Fig. 15

gr
45, r

2 + sin 45

gr tanS

9.8 x (2 + sin 45) tan 45

(a)

v = 5.15 m/s

(b)

Using Tcose = mg
T = mg sece

'

2 x 9.8 x sec 45

g = 9.8

27.7 N

275

Exercise 9C
-2

(Take g = 9.8 m.s , for the following problems)


I.

A mass of 3 kg is attached to one end of a light inelastic string of length I m,


the other end of which is tied to a fixed point 0. If the mass is rotated in a
horizontal circle and the string makes an angle of 25 with the downward
direction of the vertical, find
(a)
(b)

2.

A mass of 0.5 kg is suspended from a fixed point 0 by means of a light rod of


length I m. The mass 1s rotated in a horizontal circle and the rod makes an angle
of 30 with the downward direction of the vertical. Find:

(a)
(b)
(c)
3.

A
lm

Fig. 16

the speed of the mass at Q in m/s


the tension in the rod.

A model plane of mass 5 kg attached to


the end of an inelastic wire of length 20 m
flies in a horizontal circle of elevation 30,
while the other end of the wire is held
fixed. If the lift (force) L acts at right
angles to the wire and L ts twice the
wetght of the plane, find:
(a)
(b)

5.

the linear velocity of the mass


the period
the tension in the rod.

A honzontal disc of radius I m is free


to rotate about a vertical axis AB as
shown in the figure. PQ is a light rod
of length 1.2 m pivoted to the rim of
the disc at P and carrying a mass of
3 kg at Q. The disc is rotated uniformly,
so that PQ is inclined at an angle 30
to the vertical. Calculate:
(a)
(b)

4.

the speed v of the mass in m/s


the tension T of the string in newtons.

the speed of the plane in m/s


the tenswn in the wire in newtons.

A train is moving around a circular track of radius 800 m at a uniform speed of


80 km/h. A light inelastic wire is attached to the roof of the carriage and has a
small package attached at the other end. Find the angle of inclination of the
wire with the vertical. (Hint: The radius of the circle described by the package
is approximately the same as that of the circular track.)

276

9.1

Banked Tracks

The tendency of a vehicle to skid outwards (along the tangential path) as it rounds the
curve on a horizontal roadway is opposed by the friction between the road and the
tyres (see Fig. 18). The friction is the only force providing the centripetal acceleration
as the vehicle travels along the circular path. This frictional force is never sufficient
to keep the vehicle in the circular path. 1f the road is suitably banked, the upward
thrust of the road on the vehicle then provides the required centripetal force. Banking
means the outer edge is raised above the level of the inner edge of the road (see
Fig. 19).

mv

2P\

' r
\

0..----2

mv
r

Fig. 19
We shall now show that it is possible to choose a banking angle e such that the lateral
friction force is zero.
In Fig. 19, R is the reaction of the road on the vehicle of mass m, as it travels along
the curve of radius r at a speed v. We resolve R into two components:
the friction force F (side thrust), parallel to the slope
(ii) force N, normal to the road surface.
(i)

The resulting acceleration of the vehicle has a magnitude ~ and is directed towards
0, the centre of the circular path.
r
By resolving the forces at P, along and perpendicular to the slope, we have:
force along the slope

mass x acceleration along the slope.


mv2
F + mgsine = -- cose
r
2
F = ~ cos e - mg sin e

Resolving along the perpendicular to the slope, N

=mgcose

- m;

sine.

By choosing 9 such that the side thrust F is zero, we eliminate the tendency of the
vehicle to skid either up or down the slope.
The proper angle of banking for speed v is obtained from
2
m; cos e - mg sine = 0
i.e.l..._ta_n_e___

;_:---1

The frictional force acting parallel to the slope is called the lateral thrust. For a
railway track the use of the correct banking angle 9 (obtained by raising the outer rail
above the inner one) ensures that there is no lateral thrust between the rail and the
wheel flanges, thus reducing the wear on both (rails and wheels). Even when F = 0,
the friction force between the tyres and the road (acting parallel to the direction of
motion) provides the tractive force on the vehicle.

277

WORKED EXAMPLES ON BANKED TRACKS


EXAMPLE: ( 9)

A section of a road is in the form of an arc of a circle of radius 1000 m. Find the
banking angle 8 if the road is designed to carry traffic at a speed of 25 m/s.
(g = 9.8 m.s-2 )
SOLUTION:

The forces on the vehicle are its weight


mg and the reaction N (exerted by the
road) at right angles to OP. The
2
acceleration of the vehicle is vr ,

Ncose
4

Nsin9

mg

horizontally towards the centre of the


curve.

Fig. 21

Fig. 20

Resolving along OP:


2
mv cos a

mg sine

Substituting v

= 25

m/s, g

= 9.8

-2
m.s

r ~ 1000 m ,

2
. tane = v
rg

(1)

tan9

625
1000 X 98

gives 9

= 3.65

Alternatively we can resolve along the horizontal PX and the vertical PY, then:
2
v2
N sin 9 = !!!.!.._ and Ncos 9 = mg,
hence tan 9 =
r
rg
EXAMPLE: (I 0)

A train is travelling around a horizontal curve with uniform speed of 60 km/h. The
radius of the curve is 400 m. The rails are 1.5 m apart. Find the elevation h of the
outer above the inner rail if there is no lateral force on the rails.
SOLUTION:
B

Using the same explanation as in the


previous example, resolving along AB,
we have:
mgsin8 = mv

v2

cosQ, hence tan9 = rg

Now v = 60 l<m I h = 50 mI s, r = 400 m, g = 9.8 m.s-2 , we find 8


3
T

Then h

= ABsin8 = 1.5

sin 4.05

0.106 m

Fig. 22

= 4.05

278

Exercise 90

1.

A section of the highway is in the form of an arc of a circle of radius 250 m.


Assuming the average speed of cars on the highway to be 60 km/h, find the angle
at which the highway should be banked.
(g = 9.8 m.s- 2 )

2.

A section of the road is in the form of an arc of a circle of radius r. The road
is banked at an angle 9, so that a car of mass m travelling at speed v has
no tendency to side slip.
(a)
(b)

Show that the resultant force on the car is given by mgtan9.


2
Show that v = rg tanS and hence calculate the banking angle 9, given
v = 80 km/h, g = 9.8 m.s- 2 and r = 500 m. Give your answer in degrees
with one decimal place.
(g = 9.8 m.s- 2 )

3.

A car is travelling around a circular section of the highway which is banked at


an angle of 12. The radius of the curve is 200 m. Find the maximum safe
velocity v of the car, so that it does not slip. Express your answer to the
nearest km/h.
(g = 9.8 m.s-2)

4.

A train is travelling around a circular r:urve of radius 200 m with the proper
safe speed of 50 km/h. The rails are 1 m apart. Find the banking angle 9 of
the curve and the elevation h of the outer rail above the inner rail, if there is
-2
no side force on the rails. (g = 9.8 m.s )

5.

A cycling track is in the form of a circle of radius 100 m. The proper safe
speed for this track is 40 km/h. Find the banking angle to 2 decimal places.
(g = 9.8 m.s-2)

6.

A bobsled track has a hairpin curve of radius 16 m. The curve is banked at 72.
What is the maximum safe speed in km/h for the curve, assuming no frictional
forces?
(g = 9.8 m.s-2)

279
9.8

Components of Acceleration (Variable Angular Velocity)

2 2
2
Suppose a particle P moves on a circle x + y = r
and at time t
LPOX = e '
w = d9/dt '
P(x, y)
:. x = rcose
and
y = rsin9
Differentiating wi~h respect !o time t:

.
de
.

de
x :: - r sm 9 dt = - rwsm 9 , y = rcos 9 dt

velocity

.2

y
d9
.
dw
-rwcos e dt - rsm 9 dt
v

let:

rwcose

(I)

rw
.
9
= -rw 2 cos 9 - rw sm

(2)

.
de
2 .
dw

- r wsm 9 dt + r cos 9 dt = - rw sm 9 + r w cos 9

(3)

2
rw
cos9 - ; t p
2 .
rw sme
p

rwsin9

Fig. 24
Fig. 23

Fig. 25

From equations (2) and (3) and the diagrams showing the resolved parts of two
2
accelerations rw and r w, we infer that the acceleration of a point moving on a
circle has two components:
The tangential component

rw
2
2
rw or v
(towards the centre)

The normal component


EXAMPLE: (I I)

p moves on
' a c1rc
Ie x 2 + y 2 = 16 w1t
. h um.f orm angu Iar
A pomt
velocity of 2 rad/s. A is the point (2,0). If LPOA = 9, LPAX = <$
find the angular speed of P about A when 9 = 0 and when 9 =

1T

2.

SOLUTION: Noting that LOPA = <$ - 9,


<PAO = 180 - rJ, using the sine rule in 6POA:
sin(<$ - 9)
sin(i80 - 0)
2

2 sin(<$ - 9) = sin<$
Differentiating with respect to time t:
2 cos(<$ - e) . (0 -

e)

Angular velocity of

= (cos<$) 0

P about A is

. (I)

0.

Fig. 26
(cont. on the next page)

280

When 9

= O,

Ql = 4 rad/s.
Again, when 9

= O,

=~ , ~

= 2,

then from (1): using cosO

is obtuse, P is at B(0,4), tan!,4

= 1,

= {4

2(~ - 2)

= -; = -2

or

Using equation (2):

= ~ cos!,4
Now cos(IS - 90) = sin~, hence dividing by costS,
2taniS (~ - 2) = Ql
Put tan IS = - 2 and solve for Q1 :
Ql = 1.6 rad/s
:. Angular velocity of P about A when 9 = ~ is
2cos(~ - 90) (~ :.. 2)

1.6 rad/s.

Fig. 27

EXAMPLE: (12)
A car is travelling at 60 km/h and its wheel has a radius of 0.25 m.
u m/s of the highest point of the wheel.

Find the speed

SOLUTION:
-1

-1

v = 60 km h = 16.7 m.s
The angular velocity of a point on the rim about
the centre is given by
_
1
w = v/r = 16.7/0.25 = 66.7 rad h
The situation at time t is shown in the diagram
with P(x,y) on the circle. Assuming P to be
at 0 at t = 0, the distance travelled by the
wheel is OB and this is equal to the arc BP,
i.e. OB = r9.

= OB- AB = r9x = r(e- 9cos9) =


x

rsin9,

rw(l - cos9),

BC- NC

y =

=r

Fig. 28

- rcos9

rwsin9

velocity u of P at any time t is given by:


2 + y2 = r 2 w2 [(1- cos9) 2 + sin 2 9]
u2
2
Now 1 - cos9 = 2sin ~

2 2
2r w 0- cos9)

u = 2rwsm
. 9

= =
w = 66.7,

The direction of the velocity of P if desired is given by taniS


Substituting for the velocity of the highest point, i.e. 9

=n

required velocity u = 2 x 0.25 x 66.7 x sin~ = 33.4 m/s.

sin9
1-cosa
r

= 0.25,

the

281

Exercise 9E

(ANGULAR VELOCITY)

1.

A point is moving in a circle. Prove that its angular velocity about the centr~
of the circle is double its angular velocity about any fixed point on the
circumference.

2.

A point P is moving in a circle of centre 0, with uniform angular velocity w.


N is the foot of the perpendicular to a fixed diameter of the circle. Show that
2
the acceleration of N is given by w ON and is directed towards the centre.

3.

A point P moves with uniform velocity u


along Ox. A is a point on the line y = b,
and PA = r. Show that the angular velocity
of P about A is given by

b~
r

dX
d9
[Hint: X =PM = bcot9, dt = u, dt = w
0

4.

A point P is moving (anti-clockwise) in a


. Ie g1ven
.
b y x 2 + y 2 = a 2 , w1t
. h um.f orm
c1rc
angular velocity w. A is the point (r, O),
r >a. If <POX = 9, <PAX = rf,,
(a)

prove that rsinr/1 = asin(r/1 - 9)

(b)

prove that the angular velocity of P


about A is given by:
dr/1
awcos(rf, - e)
dt = acos(fi - e) - rcosfi

Hence find the angular velocity of P about A, when P is at B(a, 0)

and

C(O,a)

5.

2
2
A point P moves on the circle x + y = 25 with uniform angular velocity of
1 rad/s. A is the point (3, 0). If <POA = 9, find the angular speed of P
11

about A when 9 = 0 and when 9 = "2

6.

A car is travelling at 80 km/h and its wheel has a radius of 0.3 m.


speed of the highest point of the wheel.

Find the

7.

A train is travelling at 60 km/h and a wheel has a radius of 0.4 m. Find the
magnitude and direction of the velocity of a point 0.2 m above the rail.

282

9.9

Miscellaneous (Worked Examples on Circular Motion)

EXAMPLE: (13)

A 15 em record rotates at an angular speed of 33 rev/min.


(a)
(b)
(c)

Find

its angular velocity in rad/s


the speed of a point P on the rim of this record
the acceleration (centripetal) of the point P.

SOLUTION:

33

2TI

II TI
= TO rad/s = 3.46 rad/s

(a)

33 rpm

(b)

r x w = 0.15 x lln/10

(c)

rw

60

11 TI 2

0.15 x ( lO)

0.518 m/s
-2
1.79 m.s

EXAMPLE: (14)

A particle of mass 5 g is rotating in a circular path of radius 1 m with a speed of


2 m/s. Find:

(a)
(b)
(c)

the angular velocity, w, in rad/s


_
2
the centripetal acceleration, a, in m.s
the tension in the string, in newtons.

SOLUTION:

(a)

(b)

(c)

The tension T in the string supplies the


centripetal force.
2
T = m rw = 0.005 x 1 x /
= 0.020 N

2 rad/s

1 x 2 2 = 4 m.s- 2

Fig. 31

EXAMPLE: (15)

The force of attraction between the earth and an artificial satellite in circular orbit
around the earth is given by
F

_ GMm
- (R + h) 2
where G is the gravitational constant, M is the mass of the earth, m is the mass
of satellite, R is the radius of earth, and h = the height of the orbit above the earth's
surface.

A 1000 kg satellite is circling the earth at 1000 km above the surface of the earth
(g = 10 m.s-2 at the earth's surface). Find:
(a)
(b)
(c)

the gravitational force acting on the satellite.


the speed (constant) v of the satellite
the period (given R = 6400 km)

283

SOLUTION:

s
r.adius of earth

6400 km

satelJite's height
above the earth

1000 km

10 m.s- 2 at the earth's surface

Given R

7400 km

R + h

GMm

We have: F

Fig. 32

(R+t-i
(a) The gravitational force on the
satelJite is
1000 X 7.48

For h = 0, F = mg

7GMm
GM

mg

gR

7480 N

(I)

When at height h,
let F

(b) We must have

mgh
GMm
(R +h)

where r

~2

=R

= 7400

{6400

(7400

+ h

= rgh = (R

(R + h)

10

mv
.
1 f orce = -r= centnpeta

mgh

7.48 m.s-

32
10 )

32
10 )
(c)

= 7.44

The period T =

= 2nx

h) gh
X

10

x 10

2n
w

7400 x 10
7440

6250 s

1.74 h

7.48

m/s

2nr
v
3

284

EXAMPLE: (16)

A stone is rotated in a vertical circle of radius 2 m at a constant angular velocity of


30 rad/s, by a string about a pivot 4 m above horizontal ground. Consider the rotation
to be clockwise.
The string breaks at the highest point of the circle. Find the
distance from the pivot, A, to the point B, where the stone hits the ground.

(g = 10 m.s-2 )

Fig. 33

SOLUTION:
Given: w = 30 rad/s
Radius
CD

AO

AD

oc

2m

2 + 2 + 2

6m

2m

The stone flies off horizontally at 0, with velocity


v

r x w

2 x 30

60 m/s.

Thereafter it travels freely under gravity, so describes a parabolic arc OPB


Equations of motion are

At B, y
t

= 60 t

-6

= 1.2

and
or

I
2
2 gt (Axes at 0, as shown)

= {l.is

Now substitute in x = 60 t, x = BC

vf.i'

BC

60 x

AB

65.8 m

-2

10 m.s

285

Exercise 9F
1.

A satellite moves in a circular orbit of radius 8000 km, making 12 revolutions


per day. Find:
(a)
(b)

2.

the velocity v in km/h.


the centripetal force acting on the satellite, if the mass of the satellite is
500 kg.

A flywheel of radius I m makes 500 revolutions per minute. Find:


(a)
(b)
(c)

the angular velocity, w, in rad/s.


the tangential velocity, v, in m/s of a point on the rim.
the acceleration of a point on the rim.

3.

An artificial satellite of the earth travels in a circular orbit of radius 36000 km.
If the period is 12 hours, find the angular velocity, in rad/h, and the speed v in
km/h.

4.

Find the tension in the string when a stone of mass 10 kg is rotating at 60 rpm,
the stone is tied at one end of the string and the other end is fixed at point 0.
The length of the string is 1 m. (rpm -= revolutions per minute)

5.

A string of length 1 m is attached at one end to a fixed point 0 on a table,


while the other end is attached to a heavy particle of mass 2 kg. Find the speed
v in m/s of the rotation of the particle, given that the tension in the string is 4
times the weight of the particle. (g = 10 m.s-2)

6.

The force of attraction between the earth and its artificial satellite in circular
orbit is given by F

= G~m ,

where x is the distance of the satellite from the

earth's centre, G is a constant, M is the earth's mass, m is the satellite's


mass. A 500 kg satellite is circling the earth at 4000 km above the surface of
the earth. If the radius of the earth is 6400 km, find:
(a )
(b)
(c)

the gravitational force acting on the satellite given g


earth's surface
the velocity v in km/h of the satellite
the period in hours.

= 10

m.s-2

~t

the

7.

A mass of 10 kg is rotated in a horizontal circle of radius 0.25 m on the end of


a string 2 m long. Find the greatest number of rev/min, if the string can just
sustain a tensile force of 5000 N.
(g = 10 m.s-2 )

8.

A smooth circular disc of radius 0.25 m rotates in a horizontal plane with angular
velocity w = 10 rad/s about a vertical axis through the centre 0. A particle of
mass m is attached at P by light inelastic strings to points M and N on
the ends of a diameter of the disc, such that MP = 0.4 m, NP = 0.3 m and
both strings remain taut. Find the tension in the string NP, and the mass m
if the tension in MP is 20 N.

286

9.

A particle of mass m is attached to a fixed point 0 by a string of length I


metre and by another string of the same length to a small ring of mass M
which can slide on a smooth vertical wire through 0. If m describes a
horizontal circle with constant angular velocity w, prove that its depth below 0
is

10.

A particle of mass 5 kg is whirled around at the end of a string I m long. It


describes a horizontal circle with 36 rev/min. Find the tension in the string and
the angle of inclination of the string with the vertical. (g = 10 m.s- 2 )

II.

The (water) drops shaken off the rim of a rotating umbrella meet the ground in a
circle of radius r metres. The rim is a circle of diameter I m and is 1.5 m
above the ground. It is rotating with an angular velocity of 411 rad/s. Find the
radius r.
(g = 10 m.s-2)

12.

One end of a light inelastic string of length I m is attached to the rim of a


horizontal disk of radius 2 m and the other end carries a mass of 5 kg. If the
maximum tension in the string may not exceed 300 N, find the maximum angular
velocity w in rad/s and the corresponding inclination of the string to the vertical.
(g = 10 m.s- 2 )

13.

A train travelling at 80 km/h is rounding a curve of radius 500 m. Determine


the banking angle, a, of the track with the horizontal, so that the sideways
thrust on the flanges of the wheels would not derail the train. (g = 10 m.s-2)

14.

A stone is rotated with an angular velocity of 5 rad/s in a vertical circle of


radius 4 m and centre 10 m above the ground. It breaks off the string when it
makes an angle of 30 with the vertical in the positive quadrant. If the motion
is clockwise, find the distance from the centre to the point where the stone hits
the ground.

15.

A truck is travelling around a section of a circular track which is banked at an


angle of l6,andhasradius80m.Find the velocity, so that the truck does not slip.
(g = 10 m.s- 2 )

16.

Two particles are connected by a string passing through a hole in a smooth table,
one particle being on the table, the other underneath. If the masses of both
particles are equal, find the velocity v of the particle on the table moving on a
circle of radius 2 m, so that the other remains at rest.
(g = I 0 m.s-2)

17.

An object of mass I kg is attached to a string of length 8 m to a fixed point P.


v in a horizontal circle of radius r. Given that
the centre is 5 m vertically beneath P, find r and v
(given g = 10 m.s-2)

It is set rotating with velocity

18.

The orbit of the earth around the sun can be assumed to be a circle of radius
11
1.5 x 10
m described with uniform angular velocity w radians per second.
Given that 1 year = 365.25 days, find
(a)
(b)

19.

w, in radians per second


the earth's acceleration towards the sun in m.s- 2

The moon makes a complete revolution of the earth in 27.3 days with a nearly
circular orbit of radius 3.85 x 108 m. Find:
(a) the acceleration of the moon towards the earth
(b) the linear velocity, v m/s, of the moon.

287

20.

A mass of 12 kg at C is connected by light rods to sleeves P and Q which


revolve freely about the vertical axis PQ, such that L P = 60 and L Q = 30.
(a)
(b)

21.

Given that PQ = 4 m, show that the radius of the circular path of rotation
of C is .f3 m.
Find the tensions in the rods PC and QC when the mass makes 120
revolutions per minute.

A satellite of mass m revolves in a circular orbit of radius r about a spherical


planet of mass M and radius R. It is known that force of attraction between
. o f masses m an d m
- given
b y F = Gm I m2 , wh ere x IS
. t he
two b od 1es
IS
1
2
2
X

distance between the centres of the bodies. Show that the period T of revolution
of the satellite is given by 2~r

["f ,

where k is the force per unit mass at

the planet's surface due to its own gravity.


22.

A light inextensible string of length 3 metres is threaded through a smooth


vertical ring which is free to turn. The string carries a particle at each end.
One particle, P, of mass m is at rest at a distance of 1 metre below the ring,
while the other particle B of mass M is rotating in a horizontal circle whose
centre is P. Show that the velocity, v, of B in terms of m and M, is given by

23.

A satellite S is circling the earth at a height 35800 km above the earth's surface,
with a period of 1 day. Calculate orbital speed of this satellite. (R = 6400 km,
the radius of the earth). Give your answer in km/h.

24.

A point P is moving in a circle with an angular velocity w in an anti-clockwise


direction. If the radius of the circle is r metres,
(a) show that the speed of P is given by rw
(b)
M and N are the points of projection of P on the coordinate axes with 0
as the origin. Prove that both M and N execute simple harmonic motion as
P rotates about 0.

25.

A massless rod of length L is hung from


pivot A, attached to a rim of a horizontal
circular disc of radius r.
An object of mass m is attached at the
other end B of the rod. AB makes an angle
9 with the vertical when the disc rotates
with constant angular velocity w about the
vertical axis through 0.

(a)

Show that BD = R = r

(b)

2
Prove that (r + L sin 9 )w = g tan 9, where
g m.s- 2 is the acceleration due to gravity.

- - - - - _J:::J __ _
D
C
B
~-

L sinS

- - - - - --- ->
R

(c)

Prove that the speed, v, of the object B is given by


2
= g tan 9 (r + L sin 9)
v

(d)

Find the speed of B when 9 = 45, g = 10 m.s


L = 1 m, r = 5 m.

-2

Fig. 34

288

Chapter 10 Harder 3 Unit Topics

10.1 Harder Trigonometry


We shall need the following formulas:

=sinA
A) =-cosA

1.

sin (A' .:!: B) = sin A cos B .:!: cos A sin B

8.

sin(l80 - A)

2.

cos(A +B)

=cosAcosB

9.

cos(l80 -

10.

If sin x

sinAsinB

3.

tan(A.:!: B) = tan A.. tanB


I+ tanAtanB

4.

. A + sm
. B
sm

. (A+B) cos (A-B)


=2 sm
2

5.

. A - sm
. B
sm

. (A-B)
=2cos (A+B)
- - sm
- 2
2

6.

cosA + cosB = 2cos ( -A+B)


- cos (A-B)
- 2
2

7.

cosB- cosA = 2sin(A;B) sin(AiB>

Prove that:

-r

sinB + sinC
sinB - sinC

Solution:

. <B-r+ c> cos <Bc>


~

2s1n

L.H.S.

B+C. B-C
2cos(-- ) sm ( -- )
2
2
tan ( B ; C) cot ( B

R.H.S.

then

x = nn + (-l)na

Worked Examples

1.

=sina,

i C)

11.

If cosx =cosa, then


x = 2nn.:!: a

12.

=2sinxcosx
2
cos2x = 2cos x- 1
= 1 - 2sin 2 x

13.

sin2x

. 2

=cos x- sm x

289
2.

If A + B + C

= 180,

sin2A + sin2B + sin2C

prove that:

=4sinAsinBsinC

Solution:
L.H.S. = 2sin(A + B) cos (A - B) + 2sinCcosC
Now A+ B + C

180, so sin(A +B)= sin(l80- C)= sinC

and

cos(l80 - A - B) = -cos (A + B)

cosC

:. L.H.S.

2sinC cos (A - B) - 2sinC cos (A + B)

= 2sinC [cos (A - B) - cos (A + B)]


2sinC (2sinAsinB)
4sinAsinBsinC
3.

6
6
Show that cos 9- sin 9
Solution:
3
3
Using a - b

=cos29 (1

-! sin2 29)

2
2
2
2
- b) (a + ab + b ), a = sin &, b = cos 9
2
2
4
4
2
2
L.H.S. = (cos 9- sin 9) (cos 9 + cos 9 sin 9 + sin 9)
2
2
Now cos a - sin a = cos 29
. 2a + sm
. 4a = <cos 29 + sm
. 29)2 - cos 2a sm
. 29
an d cos 4e + cos 29 sm
2
2
:. LH.S.
cos29 (1- cos 9sin 9)
1 2 2a
a cos a , so sm
2 a cos 2 a = 4sm

2a = 2 sm
Agam,
sm
2
:. L.H.S. = cos29 (1
sin 29) = R.H.S.

= (a

. 2a
=1 - cos 2a sm

-!

4.

(b)

Solve the equation sin29 =cos39 , 0 ,< 9 .t. 360


3
Show that cos39 = 4cos a- 3cos9

(c)

Using the parts (a) and (b), show that sin 18

(a)

= ./5-4 1

Solution:
(a)

= sin29 = cos(90 - 29)


3 9 = 360 n .:!: (90 - 2 a)
(1)
59 = 360 n + 90
or
9 = 360 n - 90
(2)
For (1), n = 0,1,2,3,4 (0~ 59~ 5. 360)

cos39

Dividing by 5,
9

= 72.
= (18,

n + 18, n = 0,1,2,3,4

(b)

We use:
cos(A+B) = cosAcosB - sinAsinB
Put A = 9, B = 29, then:
cos39
= cos9cos29 - sinS sin29
=cos9(2cos 29-1) - 2sin9. sin9cos9
= 2cos39 - cos 9 - 2cos a (1 - cos2 9)
= 4cos 3 9- 3cos9

90, 162, 234, 306)

From (2), 9 = 270 for n = 1.


Hence the complete solution is:
9

(18, 90, 162, 234, 270, 306)


(cont)

290

(c)

5.

Using (b), the given equation becomes:


4cos 3 9- 3cos9 = 2sin9cos9
Now 9 = I go is a root of this equation, so cos 9 f 0
2
and hence: 4cos 9- 3 = 2sin9
2
4 (l - sin 9)- 3 = 2sin9
2
! /50r 4sin 9 + 2sin9 - 1 = 0 which gives sin9 =
4
{)- l
Since sin 18 > 0, we have: sin 18 =
4
2
2
Note: We can evaluate cosl8 by using cos 18 + sin l8
(a)

Using tan (A + B)

tanA + tanB
prove that
1 - tanAtanB'

tan (x + y + z)
(b)

Hence show that

(c)

Show that tan

(d)

Solve the equation

n
<n>

is a root of the equation t


t

2
- 3t - 3t + 1

= tanx
3

- 3t

- 3t + I

=0

Solution:
(a)

Put A

= x,

=y

+ z

tan (x + y + z)

tanA + tanB
1 - tan A tanB

in the formula tan (A + B)


tanx

tanx + tan(~ + z)
1 - tanxtan y + z)

tany + tanz
1 - tanytanz
tanx(tany + tanz)
1 - tanytanz

This simplifies to:


tan (x + y + z)
(b)

Put z = y

=x

tan 3x

3tanx - tan x
2
- 3tan x

(c)

Put x = Ti
Now tan

(d)

<i>

t = tan x
n

1.e. 3x = ij. , then:

3
3t - t
tan (!!. ) = - -2
4
I - 3t

= I, so this simplifies to:

- 3t

- 3t + I

(l)

Hence t " tan ( ~) is a root of the equation (l ).


1
The constant term I suggests we try t = ! 1 as the possible roots of (1).
t = - 1 satisfies the equation, hence either by inspection or long division, we have:
2
2
3
t - 3t - 3t + I = (t + 1) (t - 4t + 1) = 0
So, the roots are t

= -1,

< I,

< tan (

> = tan I 5

4! {i2

i.e. -1, 2!

1/3.

we have tan I 5 = 2 -

/3.

Observing that

291

Exercise 1OA
For questions 1 to 5 prove that:
2
2
1.
(a)
8sin ecos e = 1 - cos49, hence
2
4
(b)
32cos 9sin e = 2 -cos29- 2cos49 +Cos69
8
8
2
2.
(a)
cos e- sin e = cos29 (1 - ~ sin 29)
(b)

6
6
cos e + sin e =

(1 + 3cos 2 29)

jl1 _sine
+ sine

3.

n e)
tan ( 4 + 2

4.
5.

(2cos9 + 1) (2cos9- J) (2cos29- J) (2cos49- J) = 2cos89 + 1


9
cosec 9 + cosec 29 +cosec 49 + cot49 = cot < >

6.

if A + B + C

= seca +tanG

(a)
(b)

7.

= 180,

prove that:

cos2A + cos2B + cos2C = -1 - 4cosAcosBcosC


2
2
2
cos A + cos B- cos c
1- 2sinAsinBcosC

1f A + B + C

(a)

= 90,

prove that:

sin2A + sin2B + sin2C


. 2B +Sin
. 2C
. 2A +Sin

(b)

4cosAcosBcosC

Sin

1 - 2sinAsinB sinC

8.

Solve the following equations: 0 ~ 9 ,< 2n

9.

Solve the following equations: 0 .$- 9 .$- 360

(a)

sin4a = cos29

(b)

sin39 = sin29

4 sin 2 a - 3cos 2 9 = 1

(a)

sin59- sin39 + sin9 = 0


= 4cos 3 9 - 3cos9
3
Put x = 2cos9 in the equation x - 3x

(b)

10.

Show that cos39

reduces to cos 3 a
(a)

= V2

and show that the equation

= V2
2

Hence solve the equation x - 3x -

.f2 =

0, giving your answer in the

exact form, x = 2cos9


(b)

n +cos (9rr
17n
Prove that: cos (TI)
TI) + cos(TI)

(Do not use a calculator)

11.

(a)

Show that sin (A + B) - sin (A - B) = 2cos A sin B

(b)

Show that 2 sin x (cos 2x + cos 4x + cos 6x) = sin 7x - sin x

(c)

6n)
Deduce that cos ( 2n ) + cos (411) +cos <7
7
7

(d)

11 +cos ( 7311) + cos (7


5n)
Prove that cos<-,>

= 21

2-1

292

10.2 3 Unit Co-ordinate Geometry: Circles(Harder Problems)


No new theories or ideas are required to solve problems on
circles. The following basic properties of circles will be
very useful.
(i)

A tangent and the radius at the point of contact are at


right angles.

(ii)
(iii)

An angle in a semi-circle is a right angle.


A line is a tangent if its distance from the centre of
the circle is equal to the radius.

I.

Equation of a Circle
(x - h)

+ (y - k)

= r

(1)

is the standard form of the equation of a circle centred at the point


C (h, k) and radius r.
2
2
2
x + y =r

(2)

is the equation of a circle centred at the origin and radius r.


2.

The General Equation


x

+ y

+ 2gx + 2fy + c

(3)

where g, f and c are constants, is called the general equation. By completing


the squares in x and y, we have:
(x + g)2 + (y + f)2 = g2 + f2 - c
This equation has the standard form of the circle with centre at (-g, -f) and
radius

~2

+ f

- c.

Finally the equation:


Ax

+ Ay

+ Bx + Cy + D

(4)

when divided by A, reduces to the general form (3), so, (4) is also a general
equation of a circle.

293

EXAMPLE: (1)

Find the centre and the radius of the circles:


2
2
2 . 2
x + y - ax - by = 0
(b) 3x + 3y + 5x + 12y = 0

(a)

SOLUTION:

(a)

Completing the squares, we have:


(x _

~ )2

+ (y _

~ )2

=a

(b)

b
2 , 2)

a
So, the centre is C <
and the radius is

/a 2

Dividing the equation by 3,

+ y

(x +

+ b

5 2

6)

5x
T

4y

+ (y + 2)

=0
2

169
= 36

. (- 5 ,
Th e centre 1s

and the radius is

2)

EXAMPLE: (2)

Find the equation of the circle through the points (0, 0), (4, 8), (9, 9).
SOLUTION:

The given points must satisfy the equation of the circle:

x + y

+ 2gx + 2fy + c

= 0,

=0

16 + 64 + 8g + 16f

These simplify to: g + 2f


Solving for

f and g:

= -10
= -8,

(l)

=0

and
g+ f

and
f

81 + 81 + l8g + l8f = 0

= -9

= -1

Hence the required equation is x + y - l6x - 2y

=0

EXAMPLE: (3)

Find the equation of the circle which touches both axes and passes through the point
A (l, 2).

SOLUTION:

Let (x - h)

+ (y - k)

=r2

(l)

be the equation of the circle.


A(l,2) satisfies this equation, so
2
2
2
+ (2 - k) = r

( l - h)

.,. (2)

Since the circle is tangential to both axes, the centre


(h, k) must be at a distance equal to the radius from
each axis.
h = k = r and from (2):
2
2
2
2
(l - r) + (2 - r) = r , which simplifies to: r - 6r
r

=l

+ 5

= 0.

or 5

Thus, there are two circles, satisfying the given conditions, i.e.
2
2
2
2
(x - 1) + (y - l > = l
and
(x - 5) + (y - 5) = 25

294

3.

Equation of a Circle having AB as Diameter

Let A be (h, k) and B be (h', k'), and P(x, y) be


any point on the required circle. We use the fact that
the angle APB in the semicircle is a right angle.

m'

gradient of AP
= gradient of

BP =

Since AP J..BP, mm'

= -1

~
X.-

v - k'
~

and this gives:

(x - h) (x - h') + (y - k) (y - k') = 0

as the equation of the required circle.


0

EXAMPLE: (4)
Find the equation of the circle on AB as diameter where A is (2, 3) and B is (4, 5).
SOLUTION:
The required equation is obtained by substituting for

A (2, 3) and B ( 4, 5) in the

equation above, i.e.


(x - 2) (x - 4) + (y - 3) (y - 5)

or

+ y

- 6x - 8y + 23

= 0

As a check, we find that the centre (3, 4) is the mid-point of AB.

295
T

4.

Tangents to a Circle

We use the fact that the tangent is perpendicular to the


radius at the point of contact.
Consider the equation of the circle
x

+ y

+ 2 gx + 2 fy + c

= 0.

. (I)

The centre is (- g, -f).


If P(x , y ) is a point on the circle, then m
1

of CP

Y1+f

=-xl+g

= gradient

and the gradient of the tangent PT is then m'

The equation of the tangent is y - y 1


Or (y - y ) (y + f)
1
1

= - (x 1 + g)

=-

-J

=-m =-

X}+g
--yl+f

XI + g
'Yi"+T
(x - x 1)

(x - x )
1

Now P(x I' y ) lies on the circle (1),


1
Hence the equation of the tangent is

The reader is advised not to memorise this equation, but derive the result for each
question.
It will be instructive to prove the following results:
1.

2.

2
2
2
The equation of the tangent at P(x I' y ) to the circle x + y = a is
1
2
xx + yy = a and at P(acos9, asin9) is xcos9 + ysin9 =a.
1
1
Prove that

- ; :

from the equation (I) and hence derive the equation

2
of the tangent at P(xl'yl) to the circle x + /

+ 2gx + 2fy + c

= 0.

EXAMPLE: (5)
Find the equation of the tangent at A(-1, 3) to the circle x
. C ( 4, T
-7).
SOLUTION: The centre IS

+/

- 8x + 7y - 39

= 0.

A is (-1, 3)

.
3 + 7/2 = lO
- 13
Th e grad 1ent
o f CA = :-y-:t;""

The tangent is perpendicular to the radius, so the slope of the tangent is

10
13

and

hence its equation is y - 3 = : ~ (x + I) , which simplifies to lOx - 13y + 49 = 0.


Alternative method:
We find the gradient of the tangent by differentiating the given equation:
10
:. 2x + 2y ~- 8 + 7 ~ = 0. At (-1, 3), ~ =
etc.
dx
dx
dx
13

296

Exercise 1OB
1.

Find the equation of the circle passing through the points:


(a)

2.

(1,0), (0,-1),(0,0)

(b)

(-1,3),(2,2),(1,4)

Find the centre and the radius of the following circles:


2
2
2
2
x + y + 2x - 4y + 1 = 0
(b) 8x + 8y - 12x + 20y

(a)

3.

Find the equation of the circle whose centre is on the x-axis and which passes
through the points (0, 3) and (4, 1).

4.

Find the equation of the circle whose centre is at the point C (-3, -4) and
which is tangential to the line 3x + 4y = 20.

5.

Find the equation of the circle through the point A (-1, 2) and which is
tangential to both axes.

6.

Find the equation of the circle centred on the line y = 2x, which passes
through the point A (-2, 4) and is tangential to the x-axis.

7.

Show that the chord whose equation is x - 3y + 8 = 0 subtends a right angle at


2
the centre of the circle 9x + 9/ - 18x + 6y = 170.
2
Show that A (1, 1) lies on the circle x + / + 4x + 6y - 12 = 0. Find the
co-ordinates of B, if AB is a diameter of the circle.

8.
9.

Find the equation of the circle passing through the origin and making intercepts
a and b on the x- and y-axis respectively.

10.

Find the intercept made on the x-axis by the circle which has AB as diameter,
where A is (0, -1) and B is (2, 3).

11.

Find the equations of the tangents to the following circles at the points indicated:
2
(a)
x + / - 6x - 2y - 3 = O,
(5, 4)
(b)
(c)

(x - 1)

+ y

+ (y + 2)

- 4x + 2y

= 5,
= 20,

= mx

(3, -3)

(5, 3)

= a 2,

13.

2
Find the equations of the two tangents to the circle x + / - 2x - 6y + 6 = 0
which pass through the point P (-1. 2). Use lx + my + n = 0. (Warning: do not
use y = mx +b)

14.

Write down the equations of the circles on AB as diameter where:

15.

(a)

A (4, -8), B (3, 5)

(c)

A (a, a), B (-a, -a)

+y

Show that the line y


2
b =~a /1 + m

+ b

is a tangent to the circle x

12.

if

(b) A (a, b), B (b, a)

2
2
2
2
Show that the circles x + y = 4 and x + y + 6x - 8y + 16 = 0 touch
externally.

297

10.3 Plane Geometry: Clrcles(Harder Problems)


A

EXAMPLE: (l)
ABC is an equilateral triangle, inscribed in a
circle. X is a point on the minor arc BC.
Prove that: (i)

t.BDX

(ii)

Ill

XB + XC

t.ACX

XA

SOLUTION:
Given:

t.ABC is equilateral

Prove:

(i)

Ill

(ii)

t.ACX

XB + XC

XA

We have,

Proof:
(i)

t.BDX

=z =60

x =z

(t.ABC is equilateral)

and y = u ( Ls

in the same segments on arcs AB and AC respectively)

Then, in t.BDX and t.ACX:


X

= y = 60

=q

L BDX

(ii)

XB
XA

( L s in the same segment, arc CX)

= LACX
t.BDX Ill t.ACX
BD
AC

giving

( L sum of a t. being I 80)

XB AC = XA BD

Similarly we can prove that t.CDX


XC AB

= XA

Ill

t.ABX and the corresponding result

CD

(2)

Adding ( 1) and (2):


XB . AC

XC . AB = XA (BD

CD)

Now t.ABC being equilateral, AB


length, we have the required result:
XB

CX = XA.

(I)

= BC

XA BC

= CA, so, removing the common

298

EXAMPLE: (2)
In the diagram, AD and BE are perpendicular
to BC and AC respectively. Prove that:
(a)

HDCE is a cyclic quadrilateral on HC as diameter

(b)

AH

= AK

Given:
Prove:

ADlBC, BEJ.AC
(a) HDCE is a cyclic quadrilateral
(b)

AH

Proof:

LHEC

(a)

...

LHDC

AK

= 90
= 90

(BE lAC)
(AD .i BC)

E and D are on the circle with HC as diameter.

Hence, HDCE is a cyclic quadrilateral on HC as diameter.


y =z

(b)

=z
X :

(Ext. L of eye. quad. HDCE = Internal opp. L)

(L s in the segment, arc AB)


y

t.AHK is isosceles
AH

AK

EXAMPLE: (3)
In the diagram, BC is a fixed chord of a circle,
A is a variable point on the major arc on the
chord BC.
BD J.AC and CE
(a)

j_ AB.

Prove that:

BCDE is a cyclic quadrilateral on a circle


with BC as diameter.

(b)

as A varies, the segment ED has


constant length.

(c)

the locus of the mid-point of ED is


a circle whose centre is the mid-point
of BC.

299

SOLUTION:
Given:

BC is a fixed chord, BD .1 AC, CE .1 AB

Prove:

(a)

BCDE is a cyclic quadrilateral

(b)

ED has constant length

(c)

Locus of mid-point of ED is a circle.

(a)

LBEC and LBDC are both 90.

Proof:

(BD .i AC, CE .lAB)

:. E and D lie on the circle whose diameter is BC.


:. BCDE is a cyclic quadrilateral.
(b)

Since chord

BC

is of constant length, it subtends a constant angle,

say a, at the circumference of the given circle.


Now m =

L ABO

= 90 - a

(BD J_AD)

Since a is constant, m is also constant.


Using the fact that equal chords subtend equal angles at the circumference of a circle, we conclude that for various positions of chord
ED

on the fixed circle, (on

BC

as diameter)

ED

must be of

constant length.
(c)

Let P and M be the mid-points of ED and BC respectively.


Join MP and MD.
M is the centre of the circle BCDE.
MP J_ED
MP:2

MD

- PD

Now r = MD= MC =

= r 2 - s2

1
2 sc = a constant

s = PO = ED which is a constant
2
MP
is a constant
Hence, the locus of P is a circle with the centre
at the mid-point of BC and the radius

300

Exercise 10C:
1.

PLANE GEOMETRY: CIRCLES (HARDER PROBLEMS)

In Fig. 1, if PQ is parallel to RS, prove


that PQ = RS.

2.

In Fig. 2, P is any point on a diameter


AB of a circle; QPR is a chord such that

L APQ = 4.5, prove that


2
2
2
AB :: 2PQ + 2PR
3.

In Fig. 3, 0

is the centre of the circle,

prove that a + b
4.

=c.

Two lines OAB, OCD cut a circle at


A, B and C, D. If OB = BD, prove
OC =CA

.5.

(Fig.2)

ABCD is a quadrilateral inscribed in


a circle. BA and CD when produced
meet at P. 0

is the centre of the

circle PAC. Prove that BD is


perpendicular to OP. (See Fig. 4)
(Fig.3)
6.

Two circles ARPB, AQSB intersect


at A and B. PAQ and RAS are
straight lines. PR and SQ are
produced to meet at M. Prove that
MPBQ is a cyclic quadrilateral.
(See Fig .5)

7.

AB is a chord of a circle. The tangents


at A and B meet at T. AP is
drawn perpendicular to AB. TP is
drawn perpendicular to TA. Prove that
PT is equal to the radius of the circle.
(See Fig. 6)

8.

Two circles of radii 3 em and 12 em


touch each other externally. Find the
length of their common tangents.

(Fig.6)

301
9.

In Fig. 7, two tangents from T (to the


circle) meet the two tangents from D (to the
T

same circle) at A and C, as shown.


Prove that AT - CD
10.

= TC

- AD.

The altitudes PM and QN of an acuteangled triangle PQR meet at H.


PM produced cuts the circle PQR at A.
Prove that HM

= MA.

(Hint: Join AQ)

(See Fig. 8)
11.

AD is an altitude of the triangle ABC,


inscribed in a circle. DP is drawn
parallel to BA and meets the tangent at
A at P. Prove l CPA = 90. (See Fig. 9)
\Hint: Show that x

12.

= y = z)

AB is a diameter of a circle, AC is any


chord. M is the mid-point of the arc BC.
Prove that AC is perpendicular to the
tangent at M.

13.

In Fig. 10, three circles intersect at P.


Prove that AB is parallel to DC.
(Hint: Join PF and PE)

14.

In Fig. 11, 0

is the centre, TP is the

tangent and TC bisects LOTP, prove


that [TCP

= 45.

(Hint: Join AP)

(Fig.10)

(Fig. II)

302

15.

Prove that the quadrilateral formed by the


angle bisectors of a cyclic quadrilateral
is also cyclic. (Fig. 11)

16.

If the two non-parallel sides of a trapezium

are equal, prove that the trapezium is cyclic


(Fig.l2)

(Fig. 12)

17.

Prove that the angle bisectors of the angles


formed by producing opposite sides of a cyclic
quadrilateral intersect at right angles. (Fig. 13)
(We have to prove L FMP = 90
Hint: Produce FM as shown and prove
AFDE

18.

Ill

AFBN

The bisectors of the opposite angles LP


and L R of a cyclic quadrilateral meet
the circle at A and B as shown. Prove
that AB is a diameter of the circle.
(Fig. 14)

(Fig.l4)
19.

Two circles intersect at A and B.

A straight line PAQ cuts the circles at


P and Q. The tangents at P and Q
intersect at T. Prove that PBQT is a
cyclic quadrilateral. (See Fig. 15)
(Hint: Join AB, x = y etc.)

{Fig.l5)

303
20.

AB and CD are parallel tangents to a circle,


centre 0. APC is another tangent at P.
Prove that

LAOC is a right angle.

(See Fig. 16)

21.

(a)

In Fig. 17 CP is a tangent and CAB


is any secant.
CP

Prove that

=CA. CB

(Hint: Join AP, show that t.PCA,

t. BCP are similar)

(b)

In Fig. 18 PQ is a common tangent to


the two circles, and CAB is a common
secant. Prove that CP = CQ.
[Hint: Use the result (a) ]

(Fig.18)
22.

PQ and RS are the common tangents to two


circles intersecting at A and

B. AB produced

(Fig.l9)

either way meets the tangents in M and N as


shown. (Fig. 19)

23.

Prove:

= MA MB and RN

(a)

MP

(b)

PM = MQ

= NB. NA

(See question 21)

(c)

PQ = RS

(Hint:

t.OEC : t.OFC)

(d)

MA = BN

(Hint:

MP = RN)

P is any point on the circle ABC. PL, PM, PN


are the perpendiculars to the sides BC, CA and AB
respectively of t.ABC.

Prove that the points

L, M, N are collinear. (Fig. 20)

(Fig.20)

304
p

24.

In the diagram, two fixed circles intersect


at A and B. Prove that

L PMQ is of

constant size for all positions of Q.


(Fig. 21)

25.

ABCD is a quadrilateral inscribed in a circle.

R
(Fig.2l)

X, Y, Z, W are the mid-points of the arcs


AB, BC) CD and DA respectively. Prove that
XZ

26.

.L YW.

In Fig. 22, two fixed circles intersect at A

and B. P is a variable point on one circle.


PA and PB when produced meet the other
circle at M and N respectively. Prove
that MN is of constant length.

(Fig.22)

27.

In the diagram (Fig. 23), 6ABC is inscribed


in a circle. AD, BE, CF are the altitudes of
6ABC.

H, the point of intersection of

the altitudes,is called the ORTHOCENTRE of


the triangle. Prove that:
(a)
(b)

l BHF = LBAC
l BHC + LBAC

Ill
Ill

180

(c)

6AEF

(d)

6BDF

6ABC

(e)

AD bisects L FOE

(f)

l EDF = 180 - 2 l BAC

(g)

If BC is fixed and A varies remaining

6EDC

on the major arc on BC, find the locus


of H.
(h)

[Hint: Use part (b) ]

BFEC is a cyclic quadrilateral lying on


the circle on BC as diameter.

(Fig.23)

305

10.4 Inequalities
Definition:

>y

if and only if x - y

> 0.

On a real number line this means x lies to the right of y.

Worked Examples

1.

Prove that a ; b

Solution:

b are positive real numbers.

Consider

a+b2
1 2
2
1
2
Now ( 2-) - ab = 7i (a + 2ab + b ) - ab = 7i (a - b) ~ 0

~ -Jib,

a ; b

2.

> 0,

From example (1), a+b'l2


1

=i ,

Using (1),

From (2),

3.

, b

Given that x + y = c, prove that

Solution:
Put

>0

for a

=y ,

then:

VxY ,<

VxY ,<

-y1

1
4
i1 + y
'l c
for x > 0 , y > 0

vas

(1)

~ _2_

1
-~
VxY

or
~

= b.

(2)

VXY

and putting x+y = c

and equality when a

If x, y, z are positive real numbers, prove the following:

(a)

+ y 'l2xy

(d)

x +) +z

'l

(b) - + L. 'l2
y X

(c)

+y

+z

3xyz

(xyz)l/3
(cont)

306

Solution:
(a)

Consider
X

(b)

+y

+ y

+ y

~ 0

- 2xy

(x - y)

- 2xy

~ 2xy

Divide the result in part (a) by

xy,

then:

~+x.:V2

(c)

We have

From (a),
x

+ y
x

+ y

(x + y) (x
~

- xy

+ y

- xy)

xy

3 + y 3 >.,- (x + y) xy
~ xyz ( ~ + ~)

>,.xyz(~+i),

y3+z3

Similarly

Adding these:
2 (x

+ y

3
+ z )

XV

VZ

ZX

~ xyz [ (- + .._)

+ ( L + -)

+ (- + -) ]

>.,.. xyz [ ( ~ + X.)

+ (X. + ~)

+ ( ~ + ~) ]

y
Z

Now using (b):


2 (x

+ y

Hence
(d)

3
x

3
+ z )
3

+ y

>, xyz (2 + 2 + 2)

+ z

From (c), we have:


Put

=x

x+y+z
Hence

=y

3xyz
+ b

, c

then:

+c

=z

3abc

'-v 3 x 1/3 .y 1/3 .z 1/3

x +

+ z

(xyz)

113

The general result for a set of

equality for

Arithmetic Mean

= y = z.

n positive numbers

x 1 + x2 + . + xn

1.e.

>, Geometric Mean

x , x , , xn
1 2

is

307

-Exercise 100
1.

(a)

Show that, for


a

+ b

+ c

>0

a
~

, c

> 0,

ab + be + ca.

Hence show that a 2 + b

(b)

>0

, b

c 2 >,... 3 (abc) 2/ 3

[Hint: Use worked example 3 (d) ]

2.

Prove that for


(Hint:

3.

(a + b + c) 2 >.... 3 (ab + be + ca)

Prove that

(c)

:r

x, y, z

0,

x + y + z

:r

[3 (xy + yz + zx) ] l/

Use question 1c)

Prove that for

a, b, c, d >,... 0, a + b : c + d >,... (abcd)l/ 4

.
a+b
c+d
(Hmt: Let x = - - , y = - - , then use worked example
2
2
4.

A rectangular box of sides a, b, c

formulas S

= 2 (ab

(ii)

x, y

prove that ~ (x 3 + y 3) ~ (x ; y)3

> 0,

/> (z 2 + i>

>,

(xz + yu)

(Hint: Show that L.H.S. - R.H.S.


7.

~ 0)

For any real numbers x, y, z, u, prove that:


(x

>,. 0)

For any real numbers x, y, z, a, b, c, prove that:


(x

+ y

+ z ) (a

+ b

+ c )

Using the

(volume), prove that:

V is a maximum when the box is a cube.

(Hint: Show that L.H.S. - R.H.S.


6.

= abc

Let x = ab, y = be, z = ca, then use worked example 3d)

(Hint:
If

repeatedly)

has a constant surface area S.

and V

+ be + ca)

(i)

5.

>, (ax

+ by + cz)

308

10.5 Method of Mathematical Induction


Worked Examples
I.

(i)

Show that for each positive integer n, there are unique positive integers
a

(ii)

and b

= a n + /3.

such that (1 + Jj)n

-n

Hence show that a 2 - 3b 2 = 2n (-l)n


n
n

Solution:
(i)

.{3;

+ .fj. b
1
1
= 1 which are unique integers.

For n = I, L.H.S. = 1 +
a1

= I,

1
Assume that (1 + o/'J)k

R.H.S. = a

= ak

o/1.

bk , where k is a positive integer.

(l + v'j) (1 + o/3)k

Then (1 + v'J)k+l

= (l + Jj) (ak +

.r::3)k+l ::: (
3b )
r::3 (
b )
(1 + '"
~ +
k + v" ak + k

VJ.

.. (1)

bk), using (1).

ak+l +

and
Since ak and bk are unique integers, so are ak+l and bk+l . Thus the
statement is true for n = k + I.
Since true when n
Since true when n

= 1,
=2,

the statement is also true for n

= 2.

the statement is also true for n :: 3, and so on, the

statement is true for all positive integral values of n.


Rather than repeating the foregoing three lines, in the subsequent examples we
will say "by the principle of induction, the statement is true for each positive
integer n".
(ii)

To prove: a 2 - 3 b 2
n

= (- l)n

=a 12 -

2n

= I - 3 = -2, using part (i).


1
Also R.H.S. = -I 2 = -2, so the statement is true for n = 1. Assuming it to

For n = I, L.H.S.
be true for n

=k,

3b

we have:

a / - 3 b/ = (-l)k 2k
Then,

a~+l

- 3 b~+l

This simplifies to - 2

(2)

= (ak + 3bk)

(a~

a~+l-3b~+l =

3b~)

- 3 (ak + bk)

, using part (i).

and using (2), we have:

-2.(-l)k.2k

Hence the statement is true for

=k

= 2k+l(-l)k+l
+ 1 and, by the principle of induction,

the statement is true for each positive integer n.

309
2.

(b)

Hence, by induction prove that E


1

2n + 3

(2n + 3)

4n

>

2 ( .fi1:;l - 1)

1
.;J

+ +

For n = 1, L.H.S.

=1

and

1+

1
./2

Assume it is true for n

We add

+ 12n + 9 - 4 (n

1::'"7""1
v'il1 > 2 ( -vn
+ ,

+ 3n + 2)

>0

Vi<1 >

+ +

-~

=k,
2(

(1)

- 1)

R.H.S. = 2 ( {'1. - 1) ~ 0.8

So, the statement is true for n

ff1

> 4 (n + 1) (n + 2)
> 2 J(n + 1) (n + 2)
+

1 +

r.:vr

We have:

2
(2n + 3) - 4 (n + 1) (n + 2)

S(n)

.J(n + I) (n + 2)

(2n + 3)

(b)

>2

Show that for n

Solution:
(a)

> 0,

(a)

= I.

where k is a positive integer, then:


~

vk

+ 1 - 1)

on both sides, then

V" + 1

s (k

+ 1)

>

2 v'i('"":;'"1 +

v1<TJ
>

Using part (a), S(k + I)

This simplifies to S(k + 1)

- 2 = 2k + 3 - 2

v'i<Ti

v1<+T

2 y(k + 1) (k + 2) - 2 ~

v'i(;1
2 ( ~- I)

>

So, the statement is true for n

=k

2 ( ./(k + I) + I - 1)

+ 1 and by the principle of induction, it is

true for aU positive integers.

3.

Prove by induction that:


sinx + sin2x + sin3x + +sin nx

Solution:

For n

1, L.H.S.

= sinx,

So the statement is true for n


Assume it is true for n
S(k)

= k,

R.H.S.

= 1.

. <"x).
(n+l) x
sm
2 sm - 2sin ( ~)

sin

<1-> sin x
sin ( ~)

sin x

where k is a positive integer, then:

sin x + sin2x + + sinkx

. ( kx ) . sm
. ( -k +- 1 ) x
sm
2
2
sin ( ~)

Adding the next term sin(k + 1) x,


(cont)

310
. ( -k +- 1 ) x. sm
. (T
kx )
sm
2

S(k + 1) =

sin ( ~)

= sin (

:. S (k + 1)

"(k+1)
2sm
- x. cos (k+1)
- 2x
2

Now sin(k + l)x

+ sin (k + 1) x

+)
k

. ( kx)
sm T

[ sin ( ~)

k +- I) x
+ 2cos(2

( 1)

Bracketed expression

. (T
kx ) + 2 cos ( -k +- 1 ) x sm
. x
sm
2
2
sin ( ~)
We use

. ( kx )
. ( k + 2)
. ( kx )
sm T + sm - 2x - sm T

2cosA sinS

sin ( ~)

=sin(A +B)

- sin(A- B)

. (k+2)
sm
- 2x

sin ( ~)

s (k

sin(~)x. sin(~)x

+ 1) =

sin ( ~)

Thus the statement is true for

n = k + 1 and, by the principle of induction, it is

true for all positive integers.

Exercise 1OE
Use mathematical induction to prove the results in questions 1 to 5.
1.

xn - 1
x:-r

2.

r:-3 + ""2':4" + 3."5 +

n-1

n-2

+ X

+ 1,

2n
+ + 1+2+3+ +n = n+T

4.

A sequence {tn} is defined so that

2 3

and

5.

1 .2+2

.2

2n + 3
2(n + 1)(n + 2)

1
3
+ -n(;-n--:'-+-;:2~) 4

Prove that t

JI

2,

t2

=6.

5n-l + 1.

+3

.2

+ +n

.2

(n 2 - 2n + 3) 2n+l - 6
(cont)

311
I
Show that ;; -

6(i)(a)

I
n-+1
=

<

I
and
1
n (n + l)
(n + 1)2

n (n + I)

Using (a) and the method of induction, prove that:


n
I
1
Sn = t - 2 ,< 2 - for n I
I
r
n

(b)

>

Show 1.4.5 ~

(c)

s99

1.99

(ii)Using the method of induction, prove that for each positive integer n there are
unique positive integers a and b such that
n

(l+f2)n=a

.fi.b.

n
n
2
2
Prove that a
- 2b
n
n

= (- l)n

7.

Prove by induction that (l + x)n ~ I + nx, x

8.

If u 1

= 2,

u 2 = 3 and u
n+ 2

= 3un+ 1 -

>- I

2u , prove by induction that


n

n-1
un = 2
+ I.
9.

I o + c I~ Ibl

Assuming the triangular inequality

+ lei,

prove by induction that:

lx 1 + x 2 + x 3 + + xnl ~ I x 11 + lx 2 1 + + lxnl
10.

Prove by induction that xn + yn is divisible by x + y for all odd positive


integers n.

11.

3
2
Prove by induction that 2n - 3n +

12.

Using the results nc

= nc

r\

+ 31

and

~ 0.
n+lc
r

nc

+ nc
r-1

, prove by
r

induction the binomial theorem:


(I + X)n

13.

= nC

+ nC X + nC X
1
2
. ( 3x)
. ( x)
smy-sm
2

(a)

Show that

(b)

Prove by induction that:

2sin ( ~)

+ + nc x

+ +

nc x

cosx

cosx + cos2x + cos3x + + cosnx

. (2n+l)
x
sm - -22sin ( ~)

- 2

312
10.6 Properties of Integrals
y

1.

Piecewise Functions

Let f (x)

=
=

g (x), if a

h(x), if c

<x ~

Jb f(x)dx

Then

/1

Jb h(x)dx

g(x)dx +

I
I

I
I

l
0

EXAMPLE: (1)
Evaluate

J-11 f(x)dx, where f(x) = lxl, -1 ~ x ~ 1.

SOLUTION:
f (x)
f(x)

= Ix I, - l ~ x ~ 1
= -x for -1 ~ x ~
=

x for 0

-1

f(x)dx

-1

f(x)

=
=

sinx for

(-x)dx +

EXAMPLE: (2)
If

<x ~

for

-n"
0

x~ 0

< x 'n

fn f (x) dx

(a)

Evaluate

(b)

Find the area between the curve y

-n

= f(x)

and the x-axis, -n ~ x" n

SOLUTION:
(a)

As an integral

= I_:sin x dx

I:f(x) dx

[-cosx]
-'11

'1l

+ [~ ]

x dx

'1l

-'11

'1l

-2 + T
y = sinx

(b)

We observe that part of the required area is


below the x-axis, so we proceed as follows:
0
The required area A = sin X dx +

-n

Using part (a), A

=2

+ ~

Sq. u.

X dx

313
CXAMPLE: (3)

The function F(x) is defined as .F(x) =


where f(t) = 2 - t, 0 ~ t ~ 4

=t

- 6,

<t

r:

f (t)dt

(a)

Sketch the graph of the function f(t)

(b)

Find and determine the nature of the stationary points of the curve y

(c)

Sketch the curve y

=F(x);

hence find the area between the curve y

= F(x)
=F(x)

and the x-axis.


(a)

F(6) =

4
O (2 - t) dt +

4
t2 4
t2
6
[2t - - ]
+ [ - - 6t]
2
2

F(6)

16 (t - 6) dt

0 + -2

= -2
10

A is (2, 2) , B is (6, -2)


(c)

(b)

F(x) =

J0 t(t)dt,

:. F'(x)

=f(x)

O~t~8

.". 2 -

X ::

0, 0 ~

= 0,

X ~

<x ~

I:

F 1 (x) = 2x -

=0

F(x)

F(2)

f(t)dt =

=[2t

<t ~

I:

y , 0'

x~ 4

=y- 6x + 16,

< x~

gives the intersections with the

X ::

0, 4, 8

diagram.

Hence A(2, F(2)) is a maximum


point and B(2, F(6)) is a
minimum turning point.

I:

(t - 6) dt, 4

The sketch of y = F(x) is as shown in the

= f' (x)

Now f'(x) = -1, 0-' x-' 4


and f' (x) = 1, 4 < x ' 8 .
. F"(2) < 0 and F"(6) > 0

F(2) =

F 2 (x)

~ t~

(2 - t) dt ,

x-axis, i.e.

So the stationary points


are where x = 2 and x = 6.
From (1 ), F"(x)

(1)

The stationary points of F(x) are


given by f(x) = 0
or x - 6

F(x) =

(2- t)dt
B

2 2

T] 0

The required area


A

(2x - y)dx +-

f (y- 6x + 16)dx
4

16 -(-16)

32 sq.u.

314

2.

Inequalities involving Integrals

Many important inequalities such as:


1

l.

n1 ~

logen

n
13 + 2 3 + + n 3 ~ T

2.

property of the integral

etc. can be proved by using the following

f(x) dx.

y
y

f(l)

f(J)

3 n-1 n

n-1

Decreasing function

Increasing function

Using the diagrams we obtain the following inequalities:


I.

For f (x) increasing:

,<

Area of inner rectangles

r
r

f(x)dx

Area of outer rectangles

f(l) + f(x) + + f(n - 1)

2.

f(x)dx

f(2) + f(3) + + f(n)

For f(x) decreasing:


f(x)

f(J) + f(2) + + f(n- I)

Prove the inequality:

f(2) + f(3) + + f(n)

EXAMPLE: (I)
SOLUTION:

i
I

- dx

= logen,

:. f (2) + f(3) + + f(n) ~

+ +

f(x)

then using:

Let f(x) = - , 1 ~ x ~ n

The L.H.S. of the inequality is a lower sum


for the integral
n 1

= .!.X is

decreasing function
f(x) dx

I
n ~ logen.

Note that the function f(x) is chosen by


replacing n by x in the general term of the
given series.

3 n-1

n x

315

Exercise 1OF
Find the integral of the function f(x) on the stated interval:
f(x) = x + 2 if 0 ~ x ~ 2 and f(x) = 4, 2

1.

On [0, 5] ,

2.

On [0, 2] , f(x) = Ix -

3.

On [- 11 , 11] , f (x) = Isin x I

4.

On [ -11, 11] , f(x) = cos x,

=
5.

<x ~

II
-11 ~

X,

x ~ 0,

(X~ 11

The graph of the function


f(t) = 1 - t, 0 ~ t ~ 2
= t - 3, 2

<t

is shown in the diagram.


t

The function h (x) is defined as:

f:

~ x~

-1

h(x)

(a)

Find the function h(x) over the

f(t)dt, 0

intervals 0

2 and 2

<x ~

(b)

Find all the turning points of the function h(x) and hence sketch it.

(c)

Find

h(x)dx

Find the area between the curve y = h(x), the x-axis for 0

(d)

~ 5

Sketch the function f(x) and prove the following inequalities:


1
1 1
1
6. f(x) = - : 1 + -2 + - + + - - ~ log n
x
n- 1
e
3

7.

f(x)

8.

f(x)

9.

f(x)

1
= _/'::':

.Pr
VL

vx

rx : .fi

= x 2 : 12 +
=--1- 2 :

_rrf"

+ + - ~

Vz

+ +

f:

Vo ~

...... + (n - 1)

rn- 1)

2 (

vn

VJ

X dx

n
T

~ 1

+ 2

+ + n

Subdivide the interval [ 0, 1] into n equal parts. By


1 + X
considering the values of f(x) at x = O,
~
~
~ prove that:

10. f(x)

n r = I

*, ,. . , ,. . , ,

+ r

(Hint:
n

+ r

2 ::

1
+ (r/n)

316

Appendix 1
Bisector of an Angle of a Triangle
Theorem: The bisector (internal or external) of an angle of a triangle divides the
opposite side (internally or externally) in the ratio of the sides containing
the angle bisecte_?_. p

A,"
,

,'

I
I

Lhl

(Fig. 1)

(Fig. 2)

Given: AD bisects L BAC, internally, in Fig. (1), externally in Fig. (2), and cuts
BC (produced) at D.
Prove:

BD
AB
DC = AC

Construction: Through C draw CP parallel to DA to cut BA (produced) at P.


Proof: Take any point E on AB in Fig. (1) and on BA produced in Fig. (2).
LEAD = LAPC (AD // PC, corresponding L s )
LDAC = LACP (AD // PC, alt. Ls)

Then:

But, LEAD = LDAC (given)


LAPC = LACP, so AP = AC
BD
BA
Since AD 1/ PC, DC = AP
But

AP : AC,

:.

~~

BA
= i\C

Corollary: If the base BC is divided internally (or externally) at D in the ratio


AB : AC, then AD bisects the angle SAC internally (or externally).
The proof is similar to that of the theorem above.

317

Appendix 2

=c2

Rectangular Hyperbola: xy

''

''

''

(Fig. 2)

(Fig. 1)

Consider the rectangular hyperbola:

- y

=a

(l)

Its asymptotes are the two perpendicular lines


y =0

X -

Let X

= PM

and

and Y

+ y =0

= PN

(Fig. 1)

be the perpendicular distances of any point P(x, y) on

the conic, from these asymptotes.


X

!...:....l

XY =

.fi
2

Putting c

-2 ~ 2

and

<]

[ , ax 1 +by 1
2
2
fa + b

= ~

Vi

as P(x, y) is on the conic.

a
= y,

(2)

the equation of the rectangular hyperbola x - y

. asymptotes as co-or d"mate axes IS


. XY
to 1ts

=a 2

referred

=c 2

Alternatively, using complex numbers and taking P(x,y) as any point on the conic
2

x - y

=a 2,

Iet

P' (X , Y) a f ter OP 1s
. rotate d
X + 1. Y represent t he pomt
.

anticlockwise through 4 5.
X + i Y = cis(i->
X =

(x

+ iy) =

~(I

+ i) (x + iy) = x~-:/

!...:....l and Y = ~

.fi

.fi

The effect of the rotation used in this method is shown in Fig. 2 - where the
orientation of the hyperbola and its asymptotes is that of the familiar type
2

xy = c

318

Answers
(All answers are given to three significant figures)

319

Chapter 1: Curve Sketching Exercise 1A


I (a)

(b)

y = sin x + cos x

,/
/

"'

_.,

"'
,~----

-X

y,v.

(c)

y
(d)

=X

+COS X

//

\ /'

' ,><.... ......


--

........-1

/\

/A\

~
2 (a)

_--

y =X+-

~\

(b)

= sin x -

I
I

__,

cos x I

/I

I
X

(d)
(c)

--

= X -COS X

= X - -XI

320
(b)

3 (a)

l
.I

(c)

"-----r

,_

4 (a)

-ll

(b)

y
y = cos(x-1)
/

"

. . . -r

Y=e

X+l

(b)

....
X

, ,0

./

-ll

. .

Mammum at

6 (a)

"

,"

7 (a)

sin x
Y =X-

,~

X -

= iC-+"2
I

(b)

'

--)

""

1
e

-,

""

1
---~--

\
\

\
\

I
I

I
X

y = (log x)

321

8 (a)

8 (b)

.Jsin x

y =

,
y =

3/2

2n

9 (a)

-x
,Y
..... = e
-n

3n

-4

o/f, 2)
Min (- If. 2)

Max, (
-2

-~

.
= e-x smx

9 (b)

1\

2
'

o o O , . , . , , \oo o

, -

/4
I

Ooo oo , / . o -

.5
o

:v,

. , o - oo

= I'}I - sin (x - I >I

9 (d)

I
I

9 (c)

,,

"

'

\
\
\

I
I

Min. A (3.4, 5.8)

1 Max.B (0.6, 0.2)


I
X

2 -x
y =x e

I
I

2
I

y :

2 (2 +X)
2-x

I
I

y = (x - 2)

-7

+ I

')i

______ 1. ____ 3
I

_ _!L

II (a)
y

= 2lx - II 8

I
I

-)

3 4

2
3x - 7
y = (x- 2){x + 3)

I
I
I
I

(b)

=-3 ,
I

(c ) Max. ( J ,

x~O

= 2

5) ,

Mm. (7,

(d) See the graph above

IY I .(.

(e)

(f)

2.66

n /2

14
5 )

or

x?4

323

11 (b)

12-------:~r-~--------~--~--------------------

( ) E.!1
a

(b) 471
3

15

13

(c) 1.6

14

(d) 1.66

Yt

15

I
I

(b) A

= 1,

(c) 1oge6

16

Root: 0.6, 0.57

-1

-:
)

I
I
I

-I

-2

3x + 2

y = (2x - I Hx + 3)

324

Exercise 18

1.

(a)

~-

(b)

2~

(c)

""'

-1

-2

!/

IV
1.5

.. X

3
y

(d)

3'\

X
2

""' ""-

-2
-4

-,L

-11

(e)

~
3

2.

(a)

y
1

0
- 1

-2

- 1

1~

- 1

Y.l'

- 1

1/

2L

1'

""

"-~2

v /I
~

Y.i-

-1

(b)

I'
c)

'/

(d)

""-2

I""
v

/1
l

"'0
-I

X'

'

325

(e)

{f)

I/

"

"'

1/
0

1"'

~
-2

/1

-1 -1 1

v
/

'\~2

(g)

(h)

"'

"'"

0
- 1

I
3.
y

e--C

__.

'

...-o'
I

-2

' I

-1

3
X

..-o

/2
I 1
-211

"'0

-3

...

.-2 1\.
~

-2

-~

)(

(q

(P)

(::>)

('e)

/
X (

z-

(q) 9
X

!;""0

(l!)

x,

'e

zq + z-e
q-

VN

= -edJ\1

327

8 (a)

(b)

Max. A (- J , 5.6)

(c)

(d)

-
(e)

-1

9 (a)

~)_/;'
_____ I_ 2_ ..
I
I

12

)X

(d)

-~.=..L

__ _

0
_j

\j

(!)

v.

r
(4)
!.

1I

,....1,....

//

~I

329
10 (c)

"T
X

------~--~-~~-

(a)

- - --2 -

See Section 1.5, page 13, Example (c)


(c)

(b)

(d)

(f)

2n

ll

3n

4n

f(x) = f (-x) = si~l x , the


graph is symmetric about the y-axis.
(h)

(i)

Y n/2

-n/2

331
12 (k)

12 (m)

13 (a)

(b)

-1
X

(d)

(c)

X
(e)

(f)

(1nl4, 0.003)

I
(3m4,- 0.067)

-x
y = e COS X

~ X~

211

y = 0 for x = 11/2, 311/2


Minimum at (311/4, - 0.067)
Maximum at (711/4, 0.003)

2n
6 .

:)

(d)

i\__/
A

I
X

Min. at A (e-1 , -e-1)

Min. at

X
A (e, e)

(f)

Max. at

1
A (e, e)

e)

2 2
Max. at A (e ,

(d)

-----,--1

~~

333

16.

<X

y'

I ,

( ) Minimum A (

-1.39
1
2 , - 1.39)

Asymptptes at x = 0 and
X : 1

(d)

17 (c)

c 1 (x)

~Jog
I X

-I

(a)

1 , -1

(b)

All real x ,

(c)

Inflexion at x

18. (e)

Iy I < 1
= 0.

Y
(a)

IX I '

(a)

~-~ + sin- 1 <i)

(d)

Inflexion at x = 0

(b)

dx- ~
I -X

does not exist at x=!l

I
'- _.,.'

X
(c)
y

I2Yl ~

(c)

19~-

1'

(d)

~I

is

n: :J

c)

Max. at

(b)

Asymptote at x = 0

eI )

(a)

-I

A (e,

as x 4

co.

y -+-co as x -+ 0 .

1.

--------I
1

~),----A

Intersections with axes.

C (0,

B(4, 0),

1 ) y -+

Min. at

A (I .8, 0.7)

Max. at

P (6.6, 0.2)

3J
I

I
I
I
I

Asymptote at x = 2
Min. at A (0,

0
~3.

I
I

I
I
I

\i

4)

I1

24.

I
Min. at 0 (0, 0)
Max. at
y

A(2, z)
e

as

y -+ co

as

-+

+CO

.
0 as x + .:!: co

335

~-I,

Y.

-----~---------

:~

-~-------

27.

Y.

26.

p (1, 6)

Q(-1, 6)
B(-1, -6)

28.

A (1, I)

Min. A (0.5, 0.67]

-In,
0 :2

B(-1, -I)

Max. B (0.5, -0.6

I
I
29.

30.

------2_.L_

----- --ry

31 (a)

I
I
I

t2- - - - -

Q_ ,_ _ _ _ !__

-'i/

Asymptotes x = .!. 2 (vertical)

31 (b)

31 (c)

y = .!. I (horizontal)

0 (0, 0) is the point on the graph

12

'(\:
t.

'

337

(b)

II
I
I

A is (-1, -2)

CHAPTER 2 INTEGRATION

Exercise 2A (constant omitted)

1.

2.

3.

4.

(a)

(x2 + 1)4
4

(b)

(d)

_ (x4 + 0 -1

(e)

_.! (I

(g)

Ft

(h)

- 4 /1-

(a)

3
cos x
--3-

(b)

4Sln

(d)

- 5I cot 5 x

(e)

-I
cosx

(g)

5
I
5 sec x

(h)

3 (1

(a)

l X
3e

(b)

(d)

log (ex+

(a)

I
2
2(logex)

(d)
5.

6.

(I + logx)

loge (x

(f)

-1 2
tan (x )

(c)

5
I
5 tan x

l . 4
X

- 5I (l

- tan- I (e-x)

(c)

(b)

3I (log x) 3

(c)

- I (logx) -2
2

(b)

tan (cosx)

(c)

sin x
2
log 2
e

+ tanx)

+ cosx)

3
. -1
sm x

-2

(d)

1 l o[ex
- 1
g2
ex + I

-16 4
0.367

+ I)

rx

-2

_.! ecos 2x

(d)

_ x2)3/2

(c)

~)

(a)

(a)

(f)

- (1 - l>5

-1

1
(b)
(e)

16
0.(,2

(c)
(f)

338

CHAPTER 2 INTEGRATION

Exercise 2A (constant omitted)

I.

2.

(a)

(x2 + 1)4
4

(b)

25
-{l-x)
5

(c)

loge (x

(d)

_ (x4 +

(e)

_!

(f)

tan

(g)

./T"".

(h)

- 4

(a)

3
cos x
--3-

(b)

I . 4
ij SIO X

(c)

(d)

- 5I cot 5 x

(e)

5
sec x

(h)

J1 (I

(b)

(g)

3.

4.

5.

6.

0 -1

1 X
Je

(d)

Jog (ex+
e

(a)

2 (logex)

(d)

(I + 1ogx)
-2

(a)

_!

(d)

I
-I I og [ex
- -2
ex + 1

(a)
(d)

+ I)

2
(x )

rx
I

5
tan x

- 5I

- tar\ 1 (e-x)

(c)

(b)

3I (logx) 3

(c)

- 2I

(b)

-1
.
tan (cosx)

(c)

sin x
2
Jog 2
e

-1
cosx
+ tanx)

(I + cosx)

. -1
sm x

~)

(1ogx)-2

-2

ecos 2x

1
1

ii

(b)

T6

0.367

(e)

0.62

-16

/1-

-1

(f)

(a)

(I _ x2)3/2

(c)
(f)

337

(b)

34.

I,
I
I

-1

I
I
A is (-1' -2)

IIA

fl

339

Exercise 28

1.

(a)

e-x (-x - 1)

2.

()
a

21 x

(c)

(a)

x tanx + log cosx

(c)

-x cotx + log sinx

(b)

(constant omitted)

ex (x

"2 x + 1 cos 2x
sm

(b)

4X

sinx + 2x cosx - 2 sinx

3/2

(a)

5.

(a)

X COS-I X-

(c)

2x

(a)

x log (x

(c)

2x

7.

(a)

x tanx + log cosx - x


2

8.

(a)

~~x /xG) +Jog (x + {xL + 1

9.

(a)

(b)

-1

tan

2x

-1

2 tan

logx +

2
3>

11

10.

(a)

12.

(a)

.{3 -

11

log 2

8I COS

x [ (log x) 2 - 2 Jog x + 2]

(c)

3
TI

(b)

or

[ (Jog x)

2
(b)

r;:-)J ()
b

-2x .
2
e
(sm 3x cos 3x)

2 - log x + I

(e)

. -1 2
I x {4r---7
x
x"- + 2 sm

(b)

/2n -

(b)

I
log 10

or

(e 11/2 - I)

2]

i sin 2x + i cos 2x

(b)

4 - 2

X -

(d)

x tan-1 x - I log (x 2 + I)

[~ : ~]
x

- 4X Sill

(Jogx-

(b)

- I) - 2x - Jog

(logx)

;;-7

x -

(sinx - cosx)

- x cosx + 2x sinx + 2cosx

4.

6.

(b)

3.

2e

(c)

- 2x + 2)

log I Oe

(c)

4 +4

(f)

112 + lJ
_1_6_

(c)

-2

(d)

(g)

(e)

~ Oe 2

(h)

- 4e

-3

- I)

(f)

3 -2
( i) _ iie

+4

340

1r

(j)

11

(m)

(k)

4 - 2

TT

(I)

(1 - log 2)

2 -

+ 411 - 8

64

Exercise 2C
2

1.

7.

sin 4x
- ----rr-

2.

5.

COS

311
+ 32

u3
3

8.

9.

2
cot x
--2-

I 0.

21 (log

11.

-u - u 3
3

13.

1
2
I
4 tan 2x + 2 log

15.

_ ~ u5/3

u5

log sinx

c:os 2x

~ cos

14. -

.1.

~ ull/3 _
u17/3
+ 11
17
'

u7

(U = sinx)

19.

log (sec x + tan x)

22.

11/3

24.

- Jog (2 + cosx)

'

tan

12. 21 SIO
. 2X

(U = cotx)

16.

2U

17.

23.

25.

6.

Tu - -2u
5-

D,

X
2 -

u
3 , (U = sin x)

(U = cosx)

cosec x cot x)

.
- (; SIO

3 2
X

2x +

cos

2x

(U = cosx)

- 31 cot 3 x

18.

1
U+D

(U = cosx)

'

2
- tan (x/2) or tanx + secx 21.

20.

13

4 - 3
7

5
3
X
COS X
-5- - -3-

2 u7/2, (U = sinx)

11

3.

log [ t + I - /'i
t+l+n

t =

13-

tan[~ J

1
4 log 3

Exercise 20

1.

e X x ll - n 1

11

4.

(2n - 2)

_.1

2 , x

n -2x n
e
+ 2 1n-1

[ (x 2 xJ 2)n- I + ( 2n - 3) 1n - I ]

3.

x (log x)

11

n I

n- 1

341

5
- 5u

(ij)

5.

(i)

6.

(a)

c 5s

-6-

3
5C S

--vi'"

15CS

48

3
Ju -

15x

48

x ,

(U = cotx)

S = sinx)

(C = cosx,

3511

(b)

256

7.

(a)

3511
256

8.

(a)

4 - 2 -

9.

(a)

10.

32
-m

12.

e (x

t4

311

(b)

32 - 4

t2

log cosx ,

11

<b> ii -

(t = tanx)

76
m

n .
n-1
In= U smU + nU
cosU- n(n-1) ln_ , where x = sinU
2

{b)

11

8- 311 .,. 6

II.

- 4x

+ 12x

28

13.

- 24x + 24)

TI

Exercise 2E

+~)

I.

log(x

4.

6.

8.

~ ~-

10.

12.

-2

.-lx

/lJ

2.5

. -1
sm

5X

a22 log(x +

25

J 2.5 -

13.

;;-7) +
r--2)
I+
X

19.
21.

- log

Y'l- X
~

~ ~ + 2 log (x
I

sec- I

X~
+ x'

2V4
~

2 log (x +

(4 + x 2) -l/ 2

1~.

~X

I - x

v,;r::-;)

18.
20.

~log (JT:9

a . . ;~

. -1 x

sm

r---2
VI
- x~

"(x)

~ ~

7.

II.

I log (
or

,---z
- X~

9.

I 5.

17.

~)

11 [;;;-::;
2]
x
og

3 - 2 V 25

~)

21og(x+

. -1
SH\

. -1 (X) X r---'f
8 Sin
7i - 2 V 16 - X~

3.

9x

.5.

- Sin

2.

+ x)

~
X

logj.;:z:: +

xl

v::-:;,z)

342

Exercise 2F

I.

2x -

J4

x4

x3

x2

2.

-4 - 3 - 2 -

4.

- x +

18

log (3x + 2)

log (x - I)

x -

3.

T-

5.

ax
c + be - ad log (ex + d)
2

6.

7.

V3 an-1

8.

+ log(x+l)

1 (3x 2 + 2x) +

9.

2 t

(2x +

1)

\7)":

x +a

-1

Vb - a

;:==:=;;

tan

if b - a

log (l - 2x)

V32 tan-1

> 0,

1 log (3x - 1)

27

(2x- 1)

----v)

-1

x +a

and

-1
tan (3x - l)

12.

1
2
1
-1 (2x - 1'I
1
2
1
r~ + 1 tog(x - x + 1) +73tan
~/ 13. 2log(x + 2x- 2)- Wlogl:X +I+
2

14.

x+1og(x

16.
19.

I I.

2 -x+1) +

2log(x

-1

17

1
-Vi

2x - 1

-1
tan

.J3
\lj

15.

1!
+613

18.

-I

log 2 - 5 tan

1)

+ 1 (a 2 - b 2) + (a - b) + log ( ia):l

20.

log 2

+ 2x + 3)

73 tan

1 1 (3 + "'->2
{'i og
7

ex\1'21)

10.

Exercise 2G

log

2.

[<x - 2); J
(x - I)

3.

x + 6 log (x - 2) - 3 log (x - I)

4.

- 9 log (x - 3) + II log (x - 4)

6.

3I

5.

2X -

7.

8.

x + 2 log x - log (x

I 0.

12.

x +

5 log x - 4 log (x + 1)

log (x + 1)

-TO

log (x
2

+ 4)

-1

+TO tan

+ I) - tari

log (x - I) -

log (x

9.

'

+ x + I)

log (x + 3)

II.

2 log (x - 2) - 2 log (x + I) - 3
4( x-::-2/
I '\

+J

log (x - 2) - log (x + 2) +

V3

-1
tan

4I

log (x

2x + I

Vf

log

(-x ) X - I

- I) -

log (x

+ I)

343

13.

161

15.

log

18. -

20.

log

22.

5
2
log (x + x + 2) +
16

log (x - 2 ) +

16.

1
1
2
8 (x + 2) - 32 log (x + 4)

log (x + 2)

fi

-1

..12 tan f l

2x + 1

-1
tan

ff

(j)

3
Jog 2

-1
14. tan

41

Jog

45

1
-1 I
2 tan 2

{j)

kJog

2 I.

1/X - 2 log

log f1
\'

19.

(:

~ :)

1
-)

+ -

V2

2 (x

- I)

.fi)

(1 +

Exercise 2H

tan-!(~)
V3
'1/3

J.

- 1-

3.

x - log (x 2 + 2x + 4) - - I tan-l(x+l)
--

5.

Vx

7.

2
-t
a n-1 4x + 1

9.

r---"2
- V6x
- x' + 3

11.

~log

1 3.

- 2

ru

16.

211
T

19.

11'3

(x

si.n-l x +

-Jc

Sin

-X- -

lx

17.

{t7

20.

11

Vll:J> ,

U = x + I

~ Jog (x + ~ + lx 2 + x

+ I)

1)

Sin

- /1 - 2x- x

12.

V3

. -1 (2x +
\;:-3-

I o.

U = x 2 + 2x + 3

sin 1 (~1 )

15.

tan-

3- tan-

r--2 3 sin-1 ( 2x-1 )


-ojx-x-+

18.

v'3

tar1 1 (~)

sin- 1 (x-l)

6.

+x + 1 +

3)
Vtl) '

+ 2x + 4)
4. log (U +

+2x+4]

fx 4 - 3x2 +I)

+ 3 log (x + 1 +
log

8.

v'39

v'3

+2x+4-log[x+I+Vx

V39

log (x

2.

2
/x +

log (x + 1 + .r;;r-;;x)

2x -

Exercise 21
I.

(a)

(e)

2
11

2.

(a)

3.

(a)

- 112

(b)

1/2

(c)

(f)

(g)

(b)

I
840

(b)

-1
.;ab tan

(d)

(h)

(d)

(c)

Ifa

11

96

11

(e)

344

Exercise 2J
I.

32 (x - 2) 3/2

+ 4 (x - 2)

4. -0.059
7.

1/2
I

5.

2b

1og (a

8.

2 tan

-1

tan-! (.JX)

16

2.

+c

2 . 2

+ b sm x)

3.

6.

b tanx~
I tan-1 ( a
ab

9.

x - log (I + ex)

- e + log (I + e)

10. x - 2Jog (I - ex)


13.

(! + x)5/2

-l

II.
(I +

3.3

)3/2

,;;-:-;

14.

12.

X
3 Jog x- 9

15.

-I
sec X

-1

1i tan

19.

(x +

2)

(x + ~)

25.

-1 [4
2
3 tan

27.

a sm

29.
31.

. -1

+ 5 ~an x/2

- /i-

V5 tan x- I
I
--log
\1'5 tan x + I
2./5
- X + 5 tan-J (2 tan~)

33.

3 log (x - 2) - log (x + I)

35.

I
2 log

[~]
3
X

26.

12x
TI
- TI5 log ( 3 cos x

28.

2x - I
-4-

30.

- (4 tan x + 6f 1

32.

2Vx sin

36.

I
2 log [-)

log (I + sin x) - log (2 + sin x)

40.

I
6 log

(I - cos x) +

4 3.

8 - 4

(I + cos x) -

44.

11

- I
83 log (-2x
-2-

VP)
p ' P =

+ I

rx + 2cos rx

39.

34.

2.I log

.;pP

+ 2 sm x)

+ 3x + 8 log (x - 2) - Jog (x - I)

~:21
I

38.

2 1og

2
3 log

(I + 2 cos x)

(x + 1) -

1i log (x

+ 1) +

-1

tan

345

CHAPTER 3 VOLUMES

Exercise 3A
20611

(1)

211

(2)

15

(4)

(5)

11

(7)

51211

IT

(8)

16

(IO)

(II)

(13)

15

(16)

(a)

144

(b)

36

(d)

72

(e)

36

(17)

11

-2
e

[e: - 2

+ 2]

51211

11

- 211
8

411 ab

11
30

(6)

11
11 (1- 4)

(9)

211 [(log 2) 2 - 21og 2

(12)

12811
-5

5611

(14)

(15)

411log2- 1.511

v'3

2
(18)

---r-

(3)

{c)

112

1811

40011
-3-

Exercise 38
(I)

108811
-15-

(2)

16211
-5-

(3)

2711

4011

(4)

(5)

(6)

(7)

811

(8)

6411

(9)

(10)

211

(II)

11

5
1611
-3811

11 (e 8 +4e 4 +3 )

Exercise 3C
(2)

3211
(b)

(6)

(3)

81Th

15

81!

(c)

3
(8)

311

(4)

TO

6411

3211

(5)

4811

(9)

3
32na
_!_5_

!3.25 m

I]

346

10.

2 2
2n a c

11.

16.

15. 4113
19.

.5na

(a)

J
16na
-.5-

4na

---r

(b)

J20n

14.

117n
17. -5-

51211

15

-r

13.

20.

200n

8
;

18.
112n

15

CHAPTER 4 COMPLEX NUMBERS

Exercise 4A
).

5 - i

2.

4.

4i

5.

III

6.

II - 23 i

7.

3 + 6i

8.

7 - 24 i

9.

2i

10.

2
- 25

II.

5i

12.

14.

19 + 4 i
_2_9_

15.

-5

17.

18.

=3 , y =- 2

20.

14
23
= - 25 ' y = - 25

3 .

+ 50 I

13.
16.

- 2 + i

19.

(x

21.

2 + II i

(b)

TO

= I,

V3.
10 1

19 + 4 i
29

(b)

25.

(a)

+y
I

5
4 , y =2

y -= -I) or (x : -1, y ' I)

(c)

3.
+

3 .
2-2 1

13

2' y =- 5 2

22.

23.

(a)

(d)

I + IIi

(c)

10 + 3 i

+ ~

(x - 1)

(b)

7 .

9
2+2 1

-15 + i

3.

51
I

25

-2

(b)

2 + 2i

24.

(a)

1 - 2~i
2
+ y

(d)
(c)

25 .,fj i
2

(a)

iy

- y
2.

51

- 1 + 2xyi

347

~i

26.

25.

(d)

(c)

4-

(b)

(e)

5i - (12 + 4 i) x + 5

3 +
2

./'li

- 2x + 2 = 0

2
x - 4x + 13

(c)

30.

1.

=0

+ I

28. (a) z = 2, w = 3

=8 + 4

5- 5 1

(b)

3 + 4i

(e)

2 + 11 i

(f)

- 7 + 24 i

Va . Vb = ..raE

=8 - 4

=I

(d)

4.
3
25- 25 1

(c )

(a )

- 6, Rule

2
x -(4+4i)x+l0i=0

(d)

v'Ji

.:!:

(b)

27. (a)

(c)

v'Ji

(d)

-4-

(b)

29

_!2

(a)

+ i,

= -1

78 96.
25 + 25 1

(d)

is not defined for the imaginary numbers.

Exercise 48
5cis 0

I.

2.
5.
8.

5cis n
2 Jicis (n /4)

3.

4.

5 cis (- n/2)

7.

I 0.

20 cis ( n/6)

11.

cis (2n/3)

13.

14.

2cis( 9) where

15.

2 cis(- n/2)

16.

5i

v"i CIS (3 n/4)


cis(-n/4)

18.

-4

21.

..ti(~

24.

5 (I - .fj i)

.Ji cis(-

5cis n/2

6.

2 .ffcis (-11/4)

9.

8 cis (3 n/4)

12.

5 cis(- 2 n/3)

3n/4)

tanS=-~
17.

and 9 in 4th quadrant


10
5

5..;3.

19.

-2i

20.

2--2-'

22.

23.

25.

2 (-I

+
+

~~

~v

.f3i)

Exercise 4C
1.

8i

5.

-2 + 2

9.

- .,fj + i

(c)

2 V'I cis (

(c)

- 8- 8

14. z
(c)

./Ji
~;)

VJi

15. (a) z
(c)

-10

3.

-6

6.

27 i

7.

-1-

10.

~(I

(d)

"; cis (-n/4)

(d)

512-512Vli

= 2cis(n/3), z

1024cis(n/2)
1

z2

- (2 +

- >/3i)

= 2cis(n/6)

V2 (1
(d)

+ v'3) cis (n/4)


- 7 - 4

+ i)

.f3i

(a)

'+

32 cis(- n/3)

- 5

8.

2i

t/3 +

Si

1 . 7n
(b ) \i'iClS T2

12. (a) cis ( n/2)


1.3. (a) - 4 V3

4.

4i

(b)

2i

(b)

16 cis (2n /3)

(e) cis(- n/6)

(d) cis(n/6)

113) i

.f2(1

2.

V3

(b)

- z

Vi ( '113 -

1) cis (3n/4)

348

Exercise 40
J.

2.

64i

3.

-I

5.

6.

-8-8VJi

7.

8cis(-11/2),-8i

8.

I cis9, w here
25

9.

4cis( 11),- 4

12.

2 ~6

tan 9

64

-24 (9 m
. 3r d qua drant ) - 7
"--::'7
- 24 1.
625
625

10. 16cis(-211/3),- 8-8 t/3


II. 32cis(11/2), 32i
1
cis(-211/3),
(- I - ~i)
13. 16cis(-211/3),- 8- 8 VJi
5 12

V3

14. 512 (-

18. 16cis(-11/3), 8-8 .IJi

17. 2cis 0, 2

i cis (-11/ 3), i

v'3 i)

(I -

V'i cis (-11/4), 8- 8i

15. 8

+ i), 1024 cis(511/6)

16. 32cis0,32
19.

4.

20. 9 cis ( 11/2), 9 i

Exercise 4E
J.

(a)

(d)

{2(1- i)

. 2

3.

4.

5.

:t

-f

cv'J + i)

(c)

2cis(- 511/12)'

/5cis (n + 9/2), where tan 9

.f5cis(9/2),

vr3

2.

(b)

.
+ ~

2cis (711/12)

= ~ , Cartesian

v2

- i,

(c)

3cis

(a)

_I_ (+ l + .)

(d)

cis(k;- ; ) . k = 0,1,2,3-+ cis (-15, 75, 165, -105)


1

(a)

z = 2cis

(b )

2CJS - -- ; ,
5

(c)

o.

cisQ;

(d)

2
z = 2cis( ;

(a)

(b)

- 0.820 + J.02i, 0.472- l.22i


2 11
- 3, i
(c) cis n ~ k

(d)

2cis ( k; -

(f)

.:!: 2, (.:!: 1 .:!: .fS i) (six roots)

2k11 11
- - +
9
3

.J2-

II 3

.
Cis

11
-

11

VJ - i

, I +

J3 i,

i),

.fJ +

.r::

v3- i

i, - 1 - ,fj i

to) , k = 0,1,2,3,4
r7"

where r - v 5, t<m9 =

= 0,1,2,3

I
2 , the roots are 1.29

= 0' 1' 2' 3


(e)

-+

2 1/4

n) . .

.:!:

( 1 - i)

+ 0.201 i,

..fi (.:!: l .:!: i)


. (

CIS

n g
7n
g,

k = 0,1,2,3 .... cis(-15, 75, 165, -105)

3
cis (- .!J , 4

t.

k = 0,1,2,3,4

2k 9
-3

~ 2}

(c)

2i,

k = 0 , 1,2, 3,4

~).

(. I .. i)

(d)

k = 0,1,2,3,4-+ z = 2cis(o, 72, 144, 216, 288)

. (11 + 2k11"\

(g) cis( k2
(h)

v'2

(b)

11

11

I .!

k" 0,1,2--+ 3cis(2o, 140, 260)

_l

2
;

- 2 ,

(3;;. i)

V3 i

(a)

(b)

form .:!:

,k

0,1,2,3

349

Exercise 4F
(c = cos 9' s = sin 9' t = tan a)

I.

(a)

4c

3 - 3c

(b)

2.

3.

4 2
2 4
6
- 1.5c s + 1.5c s - s

4.

I
3
(a) 'ii cos 39 + 'ii cos a

5.

(a)

(b)

39 + 'ii3 Sin
a
(b) - 'ii1 Sin

1 ~ (cos 59+ 5cos39 + 10cos9) (b) 1 ~ (sin 59- 5sin39 + 10sin9)


I I
5
1 ~ (~sin 59+ j sin39 + 10sin9) (d) - T6 <3 cos 59- 3 cos 39 + JOcosa)

(c)

(a)

I
(cos69 + 6cos49 + 15cos29 + 10)
32

(b)

7.

(a)

8.

(a)

+ 5s

5
3
6t - 20t + 6t
4
6
2
I - I 5t + I 5t - t

(c)

6.

16s - 20s

(b)

(a) c

3
(c) l.!._:_!_
2
I - 3t

3s - 45 3

32

(cos69- 6cos49

I
64'
I

- 64 '

7
64 '

21
64 '

(c) 5TT
32

15cos 29- 10)

35

(b)

64

7
21
35
64 ' - 64 ' 64

(b)

(d)

5TT
32

16

35
16
35

Exercise 4G
(a)

(2 + i)

(b)

!.. (2 - i)

(c) . (

(e)

.:!:. (3 + 2 i)

(f)

.O+il

(g)

2.

(a)

.!. (I

3.

(a)

2 - 3 i, - I - i

(d)

- I - i, - 3 + 3 i

4.

(a)

7.

--y

].

3j

+ 4i)

(2 + i)

(b)

-+ ...L(

12

(b)

(b)

.fS' + i)

(d)

i)

.;1;

(3 -

Vi

(h) . (2 - 2 i)
+-h(l f i)
2
(c) .!. (I + i)

2 - i, - 4 - 3 i

1~

116 ..

(5- 12i)

(c)

(c)

-l,-3-2i

12 + 7 i

j)

350

Exercise 4H
1.

(a) (i)
(b)

7.

1 - 2i

(i) 3 + i
211

11

(ii) 2
(ii)

411

(iii) 4 i

(iv) 5

(v)

J5

(vi)

v5

(vii)

~(1-2i)

(iii) - 2i

(iv)

(v)

v'iO

(vi)

JJO

(vii)

1 (3 + i)

10

511

3 ' 3' 3' 3


Exercise 41
2

2.

7.

(b)

9.

(a)

~.

+- . 21'
!: 1' -2

(c) (i)

(b) z 2 +Z-1=0

6.

- 3z + 3 = 0

Fig. 1 (d) 2.38

3v'3
see Fig. 2, area = -2(ii) z 2 + z + 1 = 0

- z + 1=0
1

(c)

VJ.

VJ.
(b) .:!:21 .21.
See Fig. 3.

.:!: 1, .2.:!:2 1

z5

Fig. 1

Fig. 2

Fig. 3

Exercise 4J

3.

Jz <

7.

(a)

1 i),

- 1, i

.,f'j

1.
(c) .2.:!:21

see Fig. I
1
V3.1
(b) - 1'.2.2

V3.1
1
(d) .22

351

Exercise 4K
(I)

P(z) = (4, 5)

(2)

X
X

-2

(5)

,,

-z2

zl
(7)

-3

-2

-1

-2

-3

-2

-s

-22
P(z) ~ (2, -5)

(8)
X

zl

352
9.

(c)

(b)

(a)

y
P(z)

P(z)

A (I, .:2)
(d)

A(-2,-3)

10.

Iz I = 1,

arg z

11. R(3,0) or

=; , lwl

= I, arg w =

1T

Ii

R(-5,0)

12. z = 3 + 2 i

13. The diagonals of a rhombus intersect


at right angles.
I 4. Equality sign holds when arg z

= arg z

+ n
X

15. The sum of squares of the diagonals of a


parallelogram is equal to the sum of squares
of the four sides.

Exercise 4L

+G +V3)i

1.

(a)

2.

--2- + - - 2 - .

3.

Q(z ), z =
1
1

..f2 r

R(z2)' z2 =

-2 +-2-.

B(z 1), z 1 =

~- V3 +(

4.

5.

z =

1-\13

- I

v'3+1.

(b) z = -

VJ+I

(cos 75" + i sin75)


.

1-\.13.

- - 2 - + --2-.

or

v'Jr

3)

- I + 2 i, - 2 + 31, - 3 + 2 I, - 2 + i .

~i

(c) z = - 2 + i

353

Exercise 4M
t.

<a> Iz I = t

2.

(a)

3.

(a) x

zz = 2.5

(c) x

4.

(b) (x
(c) x

2)

lzl=~

(d)

I zl =

zz = 64

(c)

zz = -1
a

(d)

zz =

(b)

4x

(d)

4x

(b)

= 2.5, 0(0,0), r = .5

+/

= 9, a circle, r = 3, C (-2, 0)

+ 3y

= 1, a circle, r = 1, C(2,0)

= 4, a circle, r = 2, C (0, 3)

= 2.5, a circle, r = .5, C (0, -2)

+ (y + 2)

+ /

+ (y - 3)

(f) (x - 2)

6.

+ y

(e) 3x

.5.

(c)

2 + y 2 = 16, 0(0,0), r = 4

(a) (x- 2)

(d)

<b> IzI = 3

+ 2x - 1 = 0, a circle,

+ (y -

- 1 - iI =

1)2 = 4,

Iz

<d>

Iz + 1 + i1

(a)

C (2, 0), r = 2

(b) C (1, 1), r

(c)

C (2, -3), r = 2

(d)

32 ,

+ 4y

C (-

3.5

= 1, 0(0,0), r =
= 1, 0(0,0), r =

21
1

1
3,
0)

a Circle, r = 2, C(2, 1)

<a>

V2

r =

+ 4y

"!

<b> Iz

+ 1 + 2i

VJ7
=1

C (1, 1), r = 3

I= 2

v'iO

-2

354

Exercise 4N
(b)

The circle x2 + y2 = 4
r = 2, C(O, 0)

The circle
(x - 2)2 + y2 = 4
C(2,0), r = 3

(c)

(d)

The circle
(x + 2)2 + y2 = 9
C(-2, 0), r = 3

2
2
The circle x + (y - I > ,
C(O, 1), r = 2

(e)

Locus of z is the
perpendicular
bisector of OA,
A (I, 0), 2x = I

The circle
x2 + (y + 2)2 = 9
C(O, -2), r = 3

,,A

(i)
I

Locus of z is the
perpendicular bisector of
AB, A(2, 0), B(O, -1),
4x + 2y = 3
(I)

P(z) /

x'
Locus of z is the
perpendicular bisector
of AB, A (-2, 3),
B(-2,-1). y = I

2
2
x4 + ~ = I

The locus is a circle Locus, a circle


(x + 4/3)2 + (y + 2/3)2 (x + 2)2 + (y + 7/3)2 Locus is the ellipse
= 20/9. C(-4/3, -2/3), = 64/9, C(-2, -7/3),
Centre (0, 0), Foci (~ I, 0)

V25/3

(m)

= 8/3

~y

(n)

Ellipse
Centre (0, 0)
Foci (0, ."!:I)

P(z)

2.

!:(,)

(a)

(
0

(x - 1/2)
4

2
_y__
+15/4=

~~
(c)

(b)

Foci (0, O), (1, 0)

~ + L
3
4

Ellipse
1
Centre < , 0)

/3
.. X

Locus is half ray along


the line y = VJx excluding
the point 0(0, 0), y > 0

5n/6

A\-2,0)
0
Locus is the half ray Locus is half ray AP
along the line y = -x where A (-2, 0) and
excluding 0(0, 0) and y > 0 excluding A
y < o.
< PAx = 5n/6
< POX = -n/4
x+y J3+2=0

Locus is half-ray from


0(0, 0), y ) 0, X = 0
excluding 0.

355

.t~

(e)

~ox

(f)

Locus is half-ray
OP, X< 0, y = 0

}(z)

)f~~--

Locus is the half-ray


AP, A (0, -2),
excluding A.
Y= JJx-2

3.
(d)

(a)

~T~h~e~se~t~o~f~p~:~i~:~ts~ ~The:~ ~i":' ~"'ide'

c
-~

The set of points


of
The set of points within
outside the circle of outside the circle
the circle, C (2, 0),
the circle of radius 2,
radius 3, not including
centre 0(0,0) excluding radius 3, centre 0(0,0) C(O,J), radius 2, not
including the boundary including the boundary the bounCiary.
the boundar
(e)

The set of points inside


the circle, C(-2,-3),
radius 2, excluding
the boundary

The set of points


The set of points
between two conbetween two concentric circles of
centric circles, centre
radii 2 and 3, centre C(-2, 1), radii 2 and
0(0, 0), including the 4, including the
boundary of the larger boundaries
circle, but excluding
the boundary of the
smaller circle.

The set of points within


the angular region
<POx = n/3, including
the boundary

(i)

-~ ......
X

The set of points within


the angular region
<AOP, including the
boundaries, where
<AOX = n/2 and
<POx = n/3 excluding
point 0

A(-2,0), <PAX = n/6.


The ' set of points
in the angular region The set of points in the
<AOB, not including angular region, including
the boundaries
the boundaries
<AOx = n/6,
<xOB = -n/2

' ''
The set of points in half
of the complex plane
divided by the perpendicular bisector of AB,
where A is (0, 3) and
13(-2,0), not including
the boundary

356
4.

(c)

(b)

(a)

<POX = 60

<ROx = 30, <POx = 120


y

(d)

(e)

Fig. for 6(c)


A and B are excluded.

5.

(a)

Locus of w is a circle of radius 3, centre (-2, 0),


equation (x + 2l + y

(b)

6.

Locus of w

= 9.

is a circle, radius 2, C(l, 0), equation (x - 1)

C (\. a c1rc
. Ie, ra d"IUS -..[26
w IS
3- ,

(c )

Locus o f

(d)

Locus of w is a circle, r =

(a)

The x-axis, y = 0, excluding 0

(b)

Locus is a circle, r =
diameter

(c)

"!,

'{5,
C (I,

+ /

2)

3'3

C (-

~ , o)

~),

excluding A (2, 0), AB is the

B(O, l)

Locus of z is the major arc of the circle on chord AB containing the angle
angle of 60,

A(2, 0), B(-2, 0), excluding A and B.

Equation of the circle is /


(see the last fig. above)

+ /

-jf - 4 = 0,

centre ( 0,

J-r),

r =

J-r

357
Exercise 40
1. (a)

1. (b)

Fig. 2

Fig. 3

Locus of z is a circle

j , o)

2
2
x + y - 8x - 10 = O,
C(4, 0), r = v'26

Locus is a half-ray
AP, A(2, 0), excluding A
<PAx =60

A hyperbola
2
2
3x - y + 8x + "4 = 0
centre (-

l.(d)

l.(e)

Y / A(3,4)

Fig. 5

Fig. 6

Locus is an ellipse.
Foci (.! 1, 0), centre (0, 0)
Centre (0, 0)
Equation

A semi circle on AB
as diameter, excluding
A and B

20x

36y

Fig. 7

= 45

3. (a) 3 - 2 i

(b) Locus is either a point (3, -2) or the


2
2
circle (x - 3) + (y + 2) = 1

=3/2,

4. (a) x

y = 2 or x = -3/2, y = -2

'( + 2y + 2 = 0

(b) (i)

2
Circle x + /

(ii)

+ 2x + y = 0, C(-1,-1/2), r =" [

5. (a) Locus of z is the perpendicular bisector of AB,


A(3, 4), B(-3, -4), its equation 3x + 4y = 0 (see fig. 6)
.

(b) C1rcle x + y
C (-

I~

'

-5) '

15

r =

x + JOy + 25 = 0

@('
X

Fig. 8

I~

6. A straight line 2x + y = 5
7. (a ) p = /i5 , q = -

T27

(b) p = 4 - 11.1, q = 4 + II I.
4
4

8. Centres (2, 0), (0, 2), radii 2, 2,


the points of intersection are z = 0 or z = 2 + 2 i
(sec Fig. 7)
9.

B(z) =
D(z) =

3- 4 V3
2

-3-4./3
2

4+3v'3.
2
I

+
+

3113-4.
2
I

Fig. 11

358

10. (a) (i) 1

(ii) 1

v'i

(iii)

(iv) n/4

(b) (i) A circle of radius 2, C(3, -1) (see Fig. 8)


(ii) The half-plane x + y ~ I
(c)

(see Fig. 9)

Locus is the major arc of a circle on

AB as a chord,
X

A(-4, 0), B(l, 0), with P(z) on the circle, <APB = 60,
excluding A and

(see Fig. 10)

(d) See Fig. 11

= Vi

II. (a) Q(z)


(b)

cis(e + 45),

12. (a) (i)

= cis 180 = -

"?,

e = 3.18
2

(c) The circle x

+ /

13. See Fig. 12.


(ii)

1/2,

t/5/5,

1
(ii)

~,

e = 58.67

1, r = 1, C(O, 0)

is a ray AP, A(-2, 0), <..PAX = 45

3n/4

, _+

(iii) - 1 + i

(see Fig. 13)

6 = - I 0.3

16 (a ) -+ 2v'3 _
+2
i

if

1.

(b) .!. (2 + i).


(see F.1g. 15)

(b) .!.(3-5i)

Fig. 14

P moves on the y-axis.


/

= 4 (3- x)

I
(ii) 5,- n3
(iii)
20. (a) (i) 25 2n
5'
' 3
(c) 8 .. 6i
(ii) 5
(b) (i) 15

(c)

(see Fig. 14)

2
= 4a 2 (a - x), focus (0, 0). The locus of Q is the x-axis,

19. (3, 0), (-I, -4),

21. (a)

+ n/2)

14. (a) p = - 20, r = - 8

14. (b) (i) Locus of z

17. /

= r cis(e

P' (z) = cis 90 = i, Q' (z) = "'- cis 135 = - I + i

R' (z)

I 5. (a)

R (z)

1/2, - 15,

= 12,

-64

(iv)

-3

5, 5n
6

(d) - 8- 6i
(ii) _ I _ -9
4cos9 '

(b) (i) 4cose

z5

4x = I

2
2
22. (a) The ellipse I OOx + 36y = 225
23. r = 1.97,

9 = -73.5

25. a square

27.

<a>

(b)

The y-axis, excluding the origin.

24. (a) .:!: '(-

(z+l)(z

-z+l)

Fig. 15

4,

30.

(b)

z = 0, -i,

1 f , ;../3 t

<-Oksece\~ 1 n)m

;j.>

359

CHAPTER 5 POLYNOMIALS
Exercise SA
(Q = quotient,

I.

Q =x

4.

Q = 2x

9.

-35

R = remainder)

2. Q = x

R = 5

+ 3,

5.

+ x + 2, R = 0

15. -I

- 2x

Yes

464

II.

33/8

16.

17.

k =

19.

k = I,

No

12.

7.

Yes

+ 2x + 3, R = 0

8.

10- 4a + 2b

No

13.

14.

- 3.!6 ~i

20, X = 5,
X

3. Q = x

+ 2x - I, R = 0

6.

10.

I ;t t/5
18. k = 2, X = 2, - -2

fi

= 1, - 1 .!

20. k = - 2; X = 2, - 3, .:!: 2 i

Exercise 58
1.

I, I, 5

5.

(a)

7.

8.

I, I, I, - 5

2.

2, 2, -2

3.

(b)

I, 1, I, 3

6. (a) (- 2, - 13)

3, 3, -4

(a)

k=4,x=-1,-1,-4; k=O,x=0,-3,-3

(b)

k=28,x=2,2,-7; k=-80,x=-4,-4,5

(a)

1, 2, -2

(b) -1, 2, -3

(e)

-1, 2, 3

(f)

4.

(c) -~ , 2, 3

r:
-2, 4, 2 .:!: v5

-1 3
(g) li,

-1' - 1

(b) (0, I)

(d) -1, -1, -2, 3

.:!: fl

(h) -1

1 .:!:

Vi7

tanG= -1, 2, -3, 4

(i)

Exercise 5C
l.

(x + 2 i) (x - 2 i)

3.

(x +

5.

(X +

v'5 i)

(x -

v'5 i)

7.

v'li.I)
z3 - -21
m.
1 m.
- - - 1)
4 (x + - - 1) (x -

9.

~(I

Jii.

+ -2- I) ( X +

.:!:

../?i)

13. 2 - i, - I .:!:

.J5

2.

(x + c i) (x - c i)

4.

1
v'3.)(
1 ../3.)
(X+2+2
1
X+2-2 1

6.

3 (x +

8.

2 (x + j_ ) (x - j_ )

~ i) (x

../2

./5i)

11.

- 5, 2 + 3 i

14.

- I, 2, - 2 + i

15.

, - 2, - 2 - i

16. (x- x ) (x- x ) (x- x ) (x- x ) where (x , x , x , x )


4
1 2 3 4
1
2
3
17. With form used in ex. 16 above, (x , x , x , x )
1 2 3 4

1- "'[

i)

../2

10.

(- 1.:!:

= .:!: ~.!

21 t

12

"!

+I

.:!: Vi.:!: Vii

+1+v'3.
18. With form used in ex. 16 above, (x , x , x , x 4) = 2-2 I
1 2 3

19. (x + I) (x- I) (x- x ) (x- x ) (x- x ) (x- x ) where (x , x , x , x ) as in ex. 18


4
1 2 3 4
2
3
1
(b) - I - 5 i
20. (a) - 5 i

Exercise 50
(c)

(b) 25

(c) -8

(a)

3.

(a)
(c)
(b)
4. 4, - 2, 1
5. - 5, - 2, 1, 4
-3
-1
2
(b = 2, c = 3, m = 10, n = 12) or (b = 2, c " - 3, m = - 2, n = - 12)
2 I
3
9. I, I, -2,-2
8. - 3' 2' 3
2p + 27r = 9pq

6.
7.

(b)

10

I 0. b = 9,

23

= - I' - 4 . 2 ,fj

74

2. (a) - 5

l.

360

11.

(a) 3x3 - 4x2 + 8x + 8 = 0 (b) x3 + 4x2 - 8x + 24 = 0 (c) x3 - x2 - 3x + 6 = 0


3
2
3
(d) x3 + 8x2 + 18x + 1.5 = 0 (e) x - 8x - 8x - 9 = 0 (f) 8x +
4x + 3 = 0

8i -

12.
13

(a) x3 + 4bx +

dx 3 +

14.
1.5.

= 0 (b)

(a) 2x3 - 9x2 + 11x - 3

16.

8c = 0

ci + bx + 1 = 0

(a) x 4 - 4x
(c) x

3x3 - 11x2 + 9x - 2
3
(b) x + 4bx - 8c = 0

- 2x

(a)

+ 2x

- x

- bx

+ 7x - 1 = 0

=0

- 1

- 2bd

d2

(c)

13~2

4x: -

(c) ex

(b)

+ 16x + 48 = 0 (b) 3x

= 0 (c)

+ 1 =0

7i - 1Ox + 9 = 0

(a) x3 _ 4x 2 + 4x- 9 = 0

(b) x

- 7x

+ 1.5x- 18 = 0 (c)

18

Exercise 5E
I.

2,

3.

(a)

(x - 2) (x

(b)

(x - 2) (x + 2)

(c)

(x + 1) (x

+ 2x + 4)

; (x - 2) (x

ci + 4)

(x - 2) (x + 2) (x + 2 i) (x - 2 i)

- x + I)

(x + I) (x -

2I + T"J.t)

(x -

Vs)

(x + 4 +

(x + 3) (x + 2 i) (x - 2 i)

v'3 i)

(f)

(x - 3) (x + 2 + 2 i) (x + 2 - 2 i)

(b)

P(x) = K(x

(x - 2 +

(e)

(x + Vj i) (x -

(a)

P (x) = K

(x + i) (x - i)

Ci - 4x + .5)

=K(x

Vs )

(b)

(d)

(x + 2 i) (x - 2 i)

2I - T./3.t)

(x - 2 - Vii)

(a)

(c)

-./2 i)

v'3 i)

I + Jj i) (x + I -

2
2

(x + 4 -

- 6x + 25)

(c)

P(x)

(e)

P(x) = K[x

4.

(a)

2 .:!:. i

5.

(a)

(x + I) (x + i) (x - i)

(b)

(x - 3) (x + I + i) (x + I - i)

(c)

(x - I) (x - I + 2 i) (x - I - 2 i)

(d)

2 (x - 2) (x +

(a)

-I ,

(b)

.: 2, .: 2 i

(d)

-+"t,

(e)

2, 3, I .!. i

6.

- 2x

- (3 + i)x

(b)

+(I + 2i)x + I + i]

3, 2 + i

8.

i
+2

12.

14.

(a) (i) (x- 3) (x

5
x

- 6x + 13)

P (x) = K [x

(d)

- 3x + 10)

= - 5,

13

'- T

(f)P(x) = K(l- 7x + 13 + i)

-2, 3, I + 3 i

(c)

- (4 + i) x + 3 + 3 i]

(c)

:.

=5

9.

i
.{3 i
- 2 ' - t, T - 2

(d)

VJ

~+

~,

3 + 2i

~-

i) (x +

i)

Vi, ! Vii
2 V3i ,

-1!

-1 :. v'i1i
4

()+.[j+i

(ii) (x- 3) (x- 3 + 2i) (x- 3- 2i)

T -2
(b)

4 roots

2, 2, 2, 2,-3/2

-s- ,

. 2~w
= 1 has 5 roots c1s
K = 0, 1, 2, 3, 4, 5. Area is 2.38.
4
3
2
2~w
Roots of x + x + x + x + 1 = 0 are: cis
K = 1, 2, 3, 4

-s- ,

15.

2.!.i,-~

16. cis(.!.j), cis<.:

211
3

17. K=0,-4

18. -3-2i,K=-13i

361
19

'

( ) n .5n 9n
a 16' 16' 16
(d)

(i) -4

(b)

z1

13n

'16
(ii) 28

= ~[-2

i,

=3 -

23.

(a)

.!

(c)

.! (x

cis

-2k1r8 + ll ,

24.

+ i

tan ( ~ ),

(- 3

(a)

!:

{rt> i
,----

(-3- y)]

=1 + 3 i

i, z 2

22.

20.

where

z1

tan ( :~ ),

+ x + i(-3 + y)]

z 2 = 2 f-2 (c)

r6 ),

(c) tan (

/19 - 3 '
-v'-'2
-

(d) .! (2 - i)

f/409 + 3

y -

V_;,

(e) -1, -3 - 2 i

2 - i, 1 + 2 i

(f)

2 .! i

A /I
.!

i y) ,

(b)

i.

.! i,

where x = ~ ./ "i + 1 ,

k =

o,

J7 +

.! (

/4i)i

=~ J fl-

.! (

./7-

Wfi)i

(d) !: (4 !: i)

1,

1, 2, 3, 4, .5, 6, 7

These can be expressed in the form A

i B by evaluating ratios of .! 22.5,

! 112.5, !: 1.57.5. A great deal of time can be saved by noting that the roots are
symmetrically placed with respect to both axes.

= 0.924 + 0.382 i ,
=-0.924 + 0.382 i ,
= 1, b = -4; 1 -+ 1, - 3

z1

z
2
z,

z4
2'

28.

m(nb~ l)

33.

2, 2 .!:

30.

21.

Uo

= 0.382
=

z,

1, 1, I, 3 131.

"!

34.

0.924 i ,

z6

= '23,

T.tJ 1,

-1 .!

: -0.382 + 0.924i ,
3
z7 = %2,
z8 = zl

.r;

I - i ( V'j +

y 3 + 7y 2 - y + I

22
3 , n = 12
1IJ - '2)

27. m = -

v'i),

-1 + j (

=0

4x - 7

3.5.

CHAPI'ER 6 CONIC SECTIONS


Exercise 8A
I.

(a)

0 (0, 0)

(b)

:!/-

(c)

( !.

2.

(a)

0 (0, 0)

(b)

-{1

(c)

(0, !.

3.

(a)

0 (0, 0)

(b)

(c)

(.!

4.

(a)

0 (0, 0)

(b)

(c)

(0, .!

.5.

(a )

()(b)
00,0

~
J

(c)

(.!

6.

(a)

0 (0, O)

(b)

(c)

7.

(a)

0 (0, 0)

(b)

(c)

(.!

"2

= .! 16

0)

(d)

12, 10

(e)

=.! 36
Iii

ffo)

(d)

12, 8

(e)

.! 18
Y=-

(e)

(e)

=.! 2.5
tiU
=.! 8

(e)

=.! 4

4, 2

(c)

0 (0, 0)

(e)

(d)

(a)

8, 6

(O, !. Jj)

(b)

10.

(d)

(0, .!

0 (0, 0)

o/7)

(d)

(c)

(a)

=.! 16

(0, !. 6)

"!

9.

10, 2

(b)

(b)

(e)

<d>

(+

0 (0, 0)

8, 6

.111,

(c)

(a)

(d)

Jii,

:q.

8.

O)

t/7,

0)

"'3,

"7

..,

t/S

ViO
4

v'3

v'j

(d)

1,

(e)

.!3
x =-

15>

(d)

6, 4

(e)

.!9y=

- ,

(d)

2,

(e)

.g
-T

0)

0)

1/2

2v'5

"'

=.!t/2

362

11.
14.
16.

x2

-n

>7r

x2
+

9
X

1.5.

J6

16 = 1

or

x2

17.

13.

= 1

2.5

14 =

12.

36

~ = 1

or

9
X

20

x2

23

+ \-

2.5
2

Exercise &B
1.
Foci

Directrices

(.!.

Viii, 0)

x=.!.~

(.!. .5, 0)
(See Fig. 1)

(o, .:!: Jiii)

16
Y=.!.--

( 0, .:!: 4)
(See Fig. 2)

(.!.:!f.. 0)

:.!. ..1_

rs

(.!. 1, 0)
(See Fig. 1)

(.!.

X=.!.-1-

(.!. 1, 0)
(See Fig. I)

2.

v'4i

Vii1

-.15,

t an-1c

0)

V5

2ab )
""'"2--:-2
a

Vertices

3.

Fig. I
y

90

- b

Fig. 2

Exercise 6C
(Note: T = tangent, N = normal)
I.

T:

4.

T: 3x + 4y = 12 ..fi
N: V'1 (4x - 3y) = 7
T: 3x + 2y = 1
N: 2x-3y:5

7.

y =3
N: x-y=-1
X +

2
10. T: x + t y = 2ct

T:
N:
T:
N:
T:
N:

T: 2 v'!x + 3y = 12
N: 6x - 4 VJy = .5 fi
.5.
6. T: X+y+1=0
3x + V6y = 6
fix- 3y = 7V6
N: x-y+3=0
8.
3Vi'x - 2y = 6
9. T: 2x - 1/Jy = 1
2x+3fi.y:13V2
N: ..13x + 2y = 4 fi
12. 9x + 20y 75
11. x-2y=.!.5
lOOx - 45y 192
P(25/4, 15/16)
2.

N: t 3x - ty = c(t 4 - 1)
13. T: X + y : 6, N: X - y = 12, (
1.5. ( 1.56

20. y = .:!: x .!.

j)
Vi

3x - 2y + 5 = 0
2x + 3y = 1

~ ,~)

16. (2,1)
(four tangents) ;

(+
1 + I )
_V'i,_Y'l

3.

14.

P(~, ~),

63.43

17. X + y : .!. 1, (3, -2), (-3, 2)


(four points)

363

Exercise 60
1.
2.
4.
5.

.f3

4x + 5 /)y = 40
1. (b) 10 v'3x - 8y = 9
2. (a)
(b) 8 .f3x !. 4y = 3/3 3. (a) y = !. (x - 1)
3. (b)
(a) 3x + 4y = 12
4 (b) 4 .f2x - 3 J2y = 7
5. (a)
(b) 5x + 4 ./iy = 41 v'2
11. (a) Tangents are: x !. 3 /2y = 9
(b) Tangents are: 3x
Normals are: 3y !. 9/2x = i 5.{2
Normals are: 3y
(a)

=x

12.

17.

(e)

!.

v'4i

13. AB

j<V5 i

= ..li"36 = 2 J34

22.

1)

[~

23.

27.
28.

31.

25

!. .[5y = 4
!. f5x = 6{5

14. 3x + 2 .JJy = 12, AB =

(sece +cosec e) ,

~ (tane

1 + -1~
[-a(
cosa sin a- '
2 -

or

x!.2hy=2
y = !. (x - 3)
4 .fix - 5y = 20

+cote)

ff8

b
]
2 cos a sin a

Normal is (5sin9) x- (3cos9) y = 16cose sinS


2
Locus is 25x + 9/ = 64, e = ~ (see the sketch below)
2
22
2
2
2
2
2
2
(x - a ) m - 2x y m + y - b = 0, x + y = a + b
1
1 1
1
2
2
2
2
(xl + x2
a y x + b x 1 y = x y (a + b )
30. R - -,
1
1 1
2
.
4 2
2
2 2
Locus of R 1s m x + y = K m
25.

3x cos9

(a)

r(-

(b)

4y sine= 12 and 3x sinS- 4ycos9 = -12.

4(sin9- cos9),

33.

Area = cL

38.

30

40.

P(

41.

x + pqy = c(p + q)

3(sin9 + cosa>)

37. Area =

5 X

1
2 ab

2
2
.~
39. x - y = 2 (a) x tanc,6 + y secc.6 = 2 v2 secc,6 tanc,6

*,

~),

(Fig. for ex. 2 5)

Minimum area is ab.


43.

lx- my = 0.

CHAPTER 7 ELEMENTARY PARTICLE


DYNAMICS
Exercise 7A
I.

T I = 80 N, T 2 = 60 N

4.

(a)

5.

F = m (f + sin 9)

7.

-2
a = -6 m.s

9.

(a)

-2
-8 m.s

'

8330 N

2.

656N

(b)

Reaction 16 N.
(b)

2.08 s

3.

M (v - u)
F- R - 10Msin9

(c)

17.4 m

6.

-2
a = 4.33 m.s

8.

-2
-2 m.s

8030 N

'

Reaction 75 N.

364

CHAPTER 8 MOTION PROBLEMS IN


'IWO DIMENSIONS

Exercise 8A
2.

1.

12.46 p.m

4.

Centre at x = 1 ,

(b) x = .:

Amplitude =

5.

(a)

(b)

6.

(a)

II

(b)

j,

(c)

2 .: 2 ../'1., motion is not S.H.M.

Max. velocity 5 m/s

(c)

11

3.4

(c)

8.32

Exercise 88
2 )
tan-1 ( .J3

(b) tan- 1 (

1.

(a)

6.

. 9 + g 22
v 2 = u 2 - 2 ug sm
t

5 )
7j

2. (b)

82.9 or

16.6

Exercise 8C
1.

&
-kt
v=k (1-e)

(2)

2k

6.

180 km/h.

8.

(a)

10.

(a)

(b)

32.1 m

13.

(b)

17.

(ii)

20.

2.40 s
[ -2 + -!rclog
=

-~loge

~;

c: :~~~~
(1-

=- mg-

~v)

~v

loge (4/3)

3.43 s

k(l -

logi)

(b)

76.3 m

12.

1
(1 + c -2) e 2t - 1

2.1 m N 16. (i) 10 m/s

(c)
(iii)

(b)

4.

50 m/s

18. (b) -kvx

(ii)

and

1.44 m
-g- kvy

t = kloge(c/g), h = k (c- g)+ gk loge (g/c)

(a)

(c)

The time of descent < the time of ascent, since the gravity opposes the
upward motion of the object.

CHAPTER 9 CIRCULAR MOTION

Exercise 9A
I.

4.

5.03 m/s, 10.1 rad


2.
0.262 rad/s, 94 km/h
3. 25.1 rad/s
5
7
7.27 x 10- rad/s, 1.99 x 10- rad/s
5. 468 m/s (1690 km/h)

6.

29.9 km/s (107000 km/h)

9.

(a)

10.

11

251 rad/s

rad/h, 31400 km/h

7.

5.56 rad/s

8.

5.03 m/s

(c)

377 m/s

(b)

1005 rad

II.

9.59 x 10- 3 rad/h, 3690 km/h

365

Exercise 98
I.

50 m.s- 2

5.

{a)

465 m/s

7.

{a)

0.262 rad/h (b)

2.
(b)

6.25 N

3.

3130 N

0.0338 m.s- 2

6.

3000 N

(c)

11100 km/h

6320 m.s- 2

4.

2
8. 8.38 m.s-

224 N

Exercise 9C
I.

(a)

1.39 m/s

(b)

32.4 N

2. (a)

1.68 m/s

(b)

1.87 s

3.

(a)

3.01 m/s

{b)

33.9 N

4. {a)

19.6 m/s

(b)

71.7 N

(c)

5.66 N

5. 3.60

Exercise 90
2.

5.8

3.

73 km/h

4.

5. 7.18

5.62, 0.0979 m

6.

Exercise 9E
4.
6.

(a)

wa
(b) -2--2

- aw
r - a

5.

2.5 rad/s, 0.735 rad/s

7.

60 km/h, 60

+a

160 km/h

Exercise 9F
1.

(a)

25110 km/h

2.

(a)

52.4 rad/s

(b)
(b)

3.

Angular velocity = 0.524 rad/h

4.

395 N

6.

(a)

7.

I 5 I revolutions per minute

10.

Tension

I 3.

5.63

16.

4.47 m/s

52.4 m/s

I 890 N

(b)

71.1 N,

10- 7 rad/s

(a)

1.99

19.

(a)

2.73 x 10- 3 m.s.- 2

~M
+ m)
-m
- g

2740 m.s- 2

6.32 m/s

22600 km/h

8.

9 = 45.3

(c)

Speed = I 8800 km/h

5.

18.

22.

F = 3050 N

11.

(c)

2.89 h

I 2.

1+.1+5 rad/s, 80.1+ 0

I 5.

15.1 m/s

15 N, I kg
3.48 m

14.

20.6 m

I 7.

r=V39m, v = ff8 m/s

{b)

5.94 x 10- 3 m.s- 2

(b)

I 024 m/s

20.

(b) 2900 N, 1540 N

23.

I IIOOkm/h

25.

(d) 7.55 m/s

79 km/h

366

Exercise 1OA

8.

9.

11

311

(b)

o,

11, 211,

(a)

24.2,

511

711

11

11

311

711

511

1311

1711

WlTTTITTIrrrr
911

'3 ' 5 ' 5 , 5

102.7,

204.2,

282.7

(b) (in degrees) 0, 30, 60, 120, 150, 180, 210, 240, 300, 330, 360

10.

9rn 1'Tht
<u ' u'
u)
11

(a) 2 cos

Exercise 1OB
2

1.

(a)

2.

(a) (-1, 2), 2

3.

5.
6.

2
2

+y
+y
+ y

+ y

2
2

-2x-9=0

- 4x - 8y + 4 = 0 ,

11.

(a) 2x + 3y = 22

13.

x + 1 = 0 ,
(a) x
(c)

x
x

+ y

+ y
+y

x +y

9.

+ y

4.

+2x-2y+l=0,

(-5, -7)

14.

(b)

+y

5y + 4

X -

=0

4)' 2
+ y

+ 6x + 8y - 56 = 0

+ lOx - lOy + 25 = 0

- 20x - 40y + 100 = 0

- ax - by = 0

y = 2x - 9

(b)

-5

3
(b) ( ii,

8.

=0

+ y

10. 4
(c)

3x + 4y = 27

3x + 4y = 5

2
2

(b) x

- 7x + 3y = 28 = 0

= 2a

+ y

- (a + b) x - (a + b) y + 2ab = 0

Exercise 1OC
8.

12cm.

Exercise 1OF

1.
15.

18

2.

(a) h(x)

3.

=x

T,

(b)

1 B ( 3, "2
-1
A ( 1, 2),

(c)

3'

(d)

4.
X

1T

- 3x + 4

)
X

You might also like